MBBS De-Code Semi Solved Series 2019-2007 1st Prof 2ed (Pb 2020) 8194578388, 9788194578383

279 39 487MB

English Pages [731] Year 2020

Report DMCA / Copyright

DOWNLOAD FILE

Polecaj historie

MBBS De-Code Semi Solved Series 2019-2007 1st Prof 2ed (Pb 2020)
 8194578388, 9788194578383

Citation preview

=—_~

Ist

Prof

})

DE-CODE

2 Edition

Semi-solved Series 2019-2007 > Year-wise cum

oe

- Extra Gage Section



Toptc-wise approach

~ Colored Spotters

Exam Rs (Long

— mportant tables

§ Short Ans)

~ MC@s

|



- Clinical Briage

ne,

Q

-ClLntnal Pattern MCRS

=

Includes Model

Gobind ——©—,!,

Papers

of

Guru

=

———

Singh Indraprastha University (2017-2004)

—== —

First Prof 2019-2007 Covering

12 Years Question

Papers

with

Explanations

&

References from Latest Editions of Standard Textbooks (BD Chaurasia's Human Anatomy 7th ef 8th ed.; IB Singh's Human Histology 8th ed.; Human Neuroanatomy 10th ed. ef Human Embryology 11th ed.; Langman's Medical Embryology 13th ed.; SD Gangane, Human Genetic 3rd ed.; AK Jain, Physiology 7th ed.; Ganong's Medical Physiology 25th ed.; Biochemistry, Satyanarayana Sth ed.)

Subjects Covered

Contributors

¢ Anatomy

¢ Physiology ¢ Biochemistry

Suchismita Ghosh

Edited by Sudhir Kumar Singh

Anupinder Thind Dhaliwal Neha Sethi

CBS UG Exam Series 2020 Where Kuouledge meets the Tunovatiou

CBS

Decode your University Exams with CBS MBBS DE-CODE Semi-solved Series (Delhi University)

508-2018

covering 10

208% (weld Unsionversity) xplansations prof pird Pre |— gard Texxtbooks Ber vestsion from la test Bai

Request to Readers

Dear Readers, ‘No work is complete without the support of our readers’. The book "MBBS De-Code Semi-Solved Series, 2nd edition, is an outcome of our readers' demand and their

support by sharing question papers and updates related to the exams. We take the pride privilege to thank all those students without whom this endeavor couldn't be achieved. And hope this book will help you to develop the approach and skills of answer writing. In the pursuit of providing the forthcoming titles of semi-solved series for the undergraduates, we request our readers to share the recent questions of upcoming exams or any previous exam papers which are not covered in this edition at [email protected] or WhatsApp the questions on + 91-9555590180.

Your contribution will be highly appreciated and acknowledged in the next relevant upcoming title(s) of this series. Any student who shares the latest question paper or the additional papers first with us will be entitled for the complimentary copy of MBBS DE-CODE Semi-solved Series IIIrd Prof/Final Prof book along with our other CBS books (optional).

Publishers |

Join our facebook page (CBS MBBS

CBS Publishers & Distributors Pvt. Ltd. v Delhi ¢ Bengalur

Above

hocks

* Kochi * Kolkata * Mun

available

at All

* Hyderabad * Nagpur ¢ Patna ¢Vijay

Medical Book

Stores

of India

De-Code Series) and get a chance to win a complimentary copy

wy ovine BS wwrw.chepd.co.in

Pi amazonin

prepladder.com

© idutancte Parasredkart.com

For any availability issue please contact : +91-9555590180

Contents CLINICAL PATTERN Multiple Choice Questions

Physiology a”

4 bo

-| ©) —) d

RP-87

PAPe rl woe cccesseesesesesceceessseseecececssesssseeesceensessseseeeeeees RP-96

Anatomy

MICOS

Biochemistry eeesssssntstntsntntintstiseneintntntintntntntieiesnntstssnsnse Xxi-Xxiii

en

4.10)bn

Physiology MICOS

of | ©) ~) Gol PR ee

RP-107

age] ©) =) al | eee

RP-118

eccscsssssntntintntststiseteiatntntistntntnsseieasntntstnisss XXiV-XV

Biochemistry eae’

MICOS

MODEL PAPERS (Guru Gobind Singh Indraprastha University)

—§ Model Papers

A." AM

eke) OL ad | eee

11

od | ©) ~) Cl Peete

19

ee) OL) ad | eee

28

4.0)0 RR of | ©) =) Cl Be a=) ©) Gal | eee

ea

40 Fo

1 CS

oou.ececcccccssessssessssessseececsesssssssneeneees RP-3-RP-19

a

M—

oo..eeeeccccccsssecscsessesesesesescsneseseseencees RP-20-RP-36

Biochemistry

-| ©) =) Ca Pe

— Model Papers

a

fea”

oon. cccccsssecscsesnesesesesesesnessseseeneees RP-37-RP-56

RECENT PAPERS 2019 (Delhi University) Anatomy

34 40 47

age) ©) Gal | eee

57

oc] ©) Ca Bee

63

40 0

QOT2

40 Ot I

no -| ©] Ca Pee

76

ake | © =) Gad | eee

81

PaperH ceccessesesesesesesesssnseseesesssssssssescsssesessssseeseeeeasaeees 86 ad: | OL =) Ged | ee

92

od | © ~) Col Bee

99

ake) ©) } Gal | ee

106

40 1) Rt oc | © =) Co Be

110

eke) OL = ad | eee

>

3

PAPe rel ve eccesesessssseesecestsesesecesesesessseseeeseesecsssesesceeeeeenenseees 70

Physiology —§ Model Papers

en

ea”

Anatomy

>

of | ©] =) Cl Bee

erssssssstctntsntntntntineiesennnanintntntntnesessianennisss Xvi-xxvii en

mm

401;

aM

ead

4")| > BRR of | ©) —] Gol Oe

122

eke) OL a | ee

eee 128

4")|); Sn od | © =) Gol Oe

133

ake) OL =) a | eee a

116

4") A

139

of | ©) —) Col Oe

ee 145

age) ©) bal | ee

151

PHYSIOLOGY ee

30:

od | ©] —] Ge ee

E] a

4") 7 AA

a

4.0)Ot IE od | © ~) Gl DO

boner

i

eae?

4.000 |

oc | ©) —) Gol Dn

476

159

kc) OL) Gal | ee ee

ES Users ceenrersceceercecceneereercarcercescerne perenne nore 165

of | ©) =) Gal DE

2009

30

od | ©] —] Ge ne

2008

206

4!) A

mpl ne

i.

oc | ©] Ga Be

229

40 RS

4" 0

of | ©) =] Cal Be

PAULL

LOT

> — 2009 M2008

A") |) AA

of | ©) =] Gol Bee ae

B

-

Physiology

i) 010) 8 C=) 6

539

280

Biochemistry

296

310

SPOTL S ....asesssssssssssssssssssnnsnnssssscssesennssssssssnnnsssssosssseessssssssssssnsssssooossseaee 545

Anatomy

ee 317 355

ST RY

335

>

IMP Ortant Tables... ecsesessesecesssesseseseeesesseseeesscssessseseeenseneneen 550

MIM

POrtant MCQS.u ne eeesssseseeeseeeesssseseceseeeseseseseececeeessesesceeeeeeeneees 554

>

Clinical Bridge to ANatoMy ou... ccceecesessesesesecessesnesesesseeeencsnenees 558

ee

=>.4 ©) [-] 1-14 (0) 8 | Se

559

Physiology

cocnucsanacsunnccocmieamioaineccenisonic 346

IMPoOrtAant Tables .......cccsecccssssssssssescssssessssseccsssescsssssessseeeesssessssseeesessess 561 Important MCQS.vsssscsssesssnesesessineteteeinaatesnnsen 565

ee) ©)

ade | OL =) ad | eee

383

of | ©] =) Gal Be

392

ake | OL =) fad | ee

400

2OTH — PAPPHL n. eee esceseesestessssestesessesnssessseeseseessssseesteessssssseeeneeeess 409

Biochemistry

=m

PAPDEI-II .rcssssssssssesnorsnerssnesenesvsneesensseevernveesenssenesonesensntennevenorenes A24 40 BS

7

Cm

H E M

531

355,

nr a

ee 520

4." 7 AAI of | ©) =) Cal Dee

Ce




=



O

Li —™

CO



w

lliotibial tract 142 Indirect inguinal hernia 75 Inferior cerebellar peduncle 20 Inguinal canal 44, 131 Inter-atrial septum 22, 108 Inter condylar 31 Intercostal nerve, typical, branches 123 Internal capsule 26 Interossei of hand 51 Interventricular septum of heart 29 Inversion Ischiorectal fossa 44, 85 lV ventricle, boundaries 36

O Karyotyping 131 Knee joint 94 locking 153 unlocking 153

Openings in the diaphragm Otic ganglion 113 Ovary, microanatomy 13

L Lacrimal apparatus 25 Larynx 102 Lateral medullary syndrome 39 rotators of hip joint 13 wall of nose, openings 5, 48 Left atrium of heart 93 Left kidney, anterior relations 13, 14 posterior relations 30 Left lung, medastinal surface, relations 43, 44 Leg, posterior compartment, muscles 129 Lesser omentum, contents 71 Structures 107 Little’s area in the nasal septum 64 Liver biopsy 143 Lower limb, lymphatic drainage 94 varicose veins 85 Lumbar puncture 97 Lungs, pre and post natal development 153 Lymph node 117, 123

M Mammary gland lymphatic drainage 6 Maxillary artery, branches 35 Meckel's diverticulum 109 Medial cord of brachial plexus 48 Meniscus of knee joint 75 Median nerve, 4 Medulla oblongata 77 Meningocele 27 Middle ear 104 Mixed salivary gland, 5 Myocardial Infarction 121

Nasal septum, nerve supply and blood supply 36 Neck posterior triangle 90 Neural crest 48, 77 Neural crest tube defect 115

33

P Palatine tonsil 78 Pancreas 119 posterior relations 140 microanatomy 129 paramesonephric duct 12, 41, 71 Parietal pleura 58 Parotid gland 24 transverse section RP-64 Patent ductus arteriosus 18 Pelvic diaphragm 31 perineal membrane 117 Peritoneal recesses 155

reflection in the pelvis 42 Pharyngeal arch first 123 pouches 135 Pharynx constrictors 37 Pituitary gland microscopic structure 21 Placental barrier structures forming 82 Poliomyelitis 80 Polycystic kidney 75 Pons, transverse section level of facial colliculus RP-64 Popliteal fossa, boundaries and contents 31 Popliteus muscle 84 Portal acinus 117,118 lobule 117,118 vein 74 Portocaval anastomosis 62 Posterior mediastinum, contents 129 Prostate 120 Prostatic urethra 109 Pterygopalatine ganglion RP-70, 135

Pulp space infection 69

Rectus sheath contents 152 formation 130 Recurrent laryngeal nerve 126 Referred pain in myocardial infarction 18 of acute myocardial infarction 62 of cholecystitis 46 around umbilical region in appendicitis 85 Retina microanatomy 100

S Scalenus anterior muscle 138 Second pharyngeal arch 35 Secretomotor pathway 148 Sensory supply of the face 22 Sex linked inheritance 45 Shoulder joint 125 abduction 36 Skeletal muscle histological structure 64 Soft palate 89 Sole of foot nerve supply 94 Spermatic cord 13 Sphygmomanometer cuff 149 Spina bifida 9 Spinal cord ascending tract 123 microanatomy 35 Spinothalamic tract 79 Spleen, 13 Splenic artery, 140 Stomach 32 arterial supply 108 bed, structures 83 blood supply 17 lymphatic drainage 97 Styloid process of skull, muscles attached 48 Subclavian artery 77 branches 77 Submandibular ganglion 9

Suboccipital triangle 134 Subsartorial canal 152 Superficial palmar arch 49 Superior orbital fissure, 64, 134 Suprarenal gland, 42 Synovial joint 48

T Temporo-mandibular joint 51, 68 Thoracic duct 126 splanchnic nerves 130 Thoraco-abdominal diaphragm 96 Thymus, 93 Thyroglossal cyst 55 Thyroid gland 7 histological structure 64 Tongue 37,52 microanatomy AP-63 Tonsillectomy excessive bleeding after 91 Tonsil microstructure 36 Trachea microanatomy 58 Turner's syndrome 61 Tympanic membrane 65 Typical intercostal nerve 74

muscles innervated by deep branch RP-62 Upper lip 124 Ureter 30,118 Urinary bladder 16 males, posterior relations 94 nerve supply 73 Uterine tube 142 Uterus 60 lymphatic drainage 83 Supports 120

V Vagus nerve functional column nuclei RP-62 Varicose veins

Ventricular septal defects 98 Venous anastomosis

111, 137, 146

85

Vermiform appendix 45, 141 Visual reflexes RP-69

W White fibers of brain 87 Wrist drop 27 Wrist joint, movements

U Ulnar nerve,

121

147

X XY female

127

A Absolute refractory period in cardiac muscle 284 Accommodation reflex 226, 282 Acromegaly 327 Action potential of the ventricular muscle 223 in ventricular muscle, long duration 220 Activity of a muscle spindle afferent 306 Adequate stimulus 315 Adrenaline 286 Adrenal gland hormones secreted by 224 insufficiency 184 After-Load 160 Aganglionic megacolon 261 Aldosterone 268 escape 331 5 alpha reductase, deficiency 198 Anemia &P-94 Anemic Hypoxia 319 Anion gap 308 Anterior pituitary gland 264

hormones secreted 236 Anovulatory cycle 298 Apical alveoli have higher PO, than basal alveoli 278, 294 Apnea occurs during deglutition 324 Apoptosis 164 Aqueous humor functions 314 Arterial blood pressure 331 Ascending tracts 207 Aspirin in ischemic heart disease 324 Audiogram sensorineural deafness of left ear 184 Audiometry RP-104 Auerbach’s plexus 276 A-V PO, difference in stagnant hypoxia 177

Babinski’s sign 269 Bainbridge reflex 176

Basal body temperature 286 Basic pathways in medullary control 274 Bilateral destruction of primary visual cortex 285 lesion in the ventromedial nuclei 224 Bleeding and clotting time in hepatocellular diseases 308 tendency in vitamin C deficiency 323 Blood clotting intrinsic mechanism 202 Blood pressure 258 after clamping of renal artery 191 during isometric exercise 322 short term regulation 176 when intracranial pressure rises 163 Blood testis barrier 227 blood transfusion complications of 276 Bone density is reduced after menopause 270

SUBJECT-WISE CUM TOPIC-WISE CONTENT LIST

PHYSIOLOGY

Bradykinesia in Parkinson's disease 316 Brodmann's area, lesion in RP-702 Brown sequard syndrome 284, 331

SUBJECT-WISE CUM TOPIC-WISE CONTENT LIST

C Ca** deficiency 278 Caisson’s disease 260 Calcium stabilizes the cell membrane 315 Carbon monoxide poisoning 323 Cardiac cycle left ventricular pressure 175 volume changes 175 Cardiac cycle blood flow in coronary vessels 203 Cardiac failure central venous pressure 292 Cardiac output 201 factors affecting 230 methods to measure 230 Cardio-respiratory changes in exercise 163 Carrier mediated transport 192 Cellular Immunity 324 Central and peripheral chemoreceptors 290 retina 312 Central inhibition of pain 235 Central venous pressure 307 Cerebellum functions 222 functional division 222 afferent connections 222 Cheyne stroke respiration 307 Cell mediated immune responses 256 Choleretics 322 Chvostek’s sign 270 Circadian rhythm 186 Coagulation in vitamin K deficiency 204 Color blindness 332 Compliance of lungs alone 321 Components of stored blood, if not stored at proper temperature RP-92 Compound action potential RP-98 Conditioned reflex 301, 315 Conduction velocity in Purkinje’s fiber 291 of Aafibers 267, 329 Contraceptive pills 226 Contractile response of the ventricular muscle 223 Counter-current mechanism 217, 259 Counter irritant creams 270 C—reactive proteins 291

Cross-matching 178 Cushing's syndrome 224, 251 Cyanosis 323 Cystometrogram 161, 203, 276

D Dark adaptation ineye 252 Decerebrate rigidity 330 Decompression sickness 204 Decorticate 282 Deficiency of clotting factor VIII 177 Denervation hypersensitivity 331 Deposition and resorption of bone 286 Descending motor tracts 197 Diabetes mellitus metabolic derangements 211 pathophysiology 287 Dietary fiber 164, 220, 294, 324 intake 260 Diuretics 164 Diurnal variation of ACTH secretion 267 Dog leg appearance 160 Dopa in Parkinson's disease 270 Dopaminergic neurons destroyed in basal ganglia 252 Dumping syndrome 247 Dwarfism 236 Dye dilution method 201 Dyspnea 308 Dyspneic index 276 Dystrophin protein 198

E—C coupling in smooth muscles and skeletal muscle 298 ECG wave recording, negative in AVR 309 Effect of sympathetic and parasympathetic stimulation 201 light on current flow 209 Einthoven’s law 308 Electrical activity 189 Electroencephalogram (EEG) 186 EMG 299 Endocochlear potential 186, 228 Endothelin 247 Endocytosis 274, 291 Enterohepatic circulation 278 bile salts of 175,256 Equilibrium potential of Kt 267 of Na* 284 Erythrocyte sedimentation rate 321

Erythropoiesis 241 Excitation contraction coupling in skeletal muscle and smooth muscle 264 Explicit memory 312 Extrinsic coagulation mechanisms 256 Expiration 258

Facilitated diffusion 293 Fasting glucose levels 329 Fast pain 312 Post prandial plasma glucose level 329 Fat absorption 192 digestion 192 Fetal hemoglobin persistence during adult life 232 Fetoplacental unit 239 FEV 1 230 obstructive and restrictive airway disease in RP-90 Fibrinolytic system 233, 277 Field of vision 213 Filtration fraction 308 Finger nose test 197 Foetoplacental unit 195

G Gap junction 319 Tight junction 319 Gastrectomy total 307 Gastric bypass surgery 191 secretion 232 slow wave 291 Gastroileal reflex 294 Gastrointestinal activity, in cholera 292 Gate control theory of pain 104, 213 General growth curve RP-104 Genesis of REM sleep 316 RMP 332 GFR, when NSAIDs are given to patients with impaired renal circulation RP-92 Gigantism 327 Glomerular filtration rate 174, 204, 292, 322 Glucocorticoid 314 secretion, regulation of RP-100 Glucose 161 reabsorption PCT from RP-91 Galactose 161 Glycosylated hemoglobin 211

Growth hormone GTT curve

287

236

H Hairpin counter current multiplier system 231 Haldane’s effect 164 HCL secretion by parietal cells 242 Heart rate in severe exercise 276 Hemorrhage in left internal capsule 197 Hemorrhagic shock 295 Hemostasis 177 Herring Breuer’s reflexes 233 Heterometric regulation of stroke volume 217 Heteronymous hemianopia 329 Homonymous hemianopia site of lesion 298 Homeostasis 312 Homonymous hemianopia with macular sparing RP-99 Hormone 286 adrenal gland of 184 demonstrating positive feedback mechanism 330 involved in milk production milk ejection 252 which regulate blood calcium 169 Human chorionic gonadotropin 171 Humoral immune responses 256 Hypermetropia 301 Hypocalcemia produces tetany 331 Hypothalamus, functions of 249 in temperature regulation 300 Hypothyroidism 194 infants in RP-102 Hypoxic hypoxia 188, 319 Hypocalcemia 213

Immunity acquired 191, 274 humoral 191 Innate 191, 274 Impedance matching 214, 238 Inner renal medulla 219 ear, in hearing, role of 300 Insulin resistance 270 secretion 211 glucose infusion, effect of 299 secretion, mechanism 196

Intercellular connections RP-102 Intraocular pressure 299 Intrapleural and intrapulmonary pressure changes during normal breathing 306 pressure in upright position 275 Intrinsic pathway of coagulation 230 Inverse stretch reflex 283 Intrinsic coagulation mechanisms 256 Inspiration 258 Inner hair cells 312 Implicit memory 312 lodine requirement per day 314 lron absorption inhibition 307 lron deficiency 204 Irreversible shock 247

J Jugular venous pressure in congestive cardiac failure 204 Juxtaglomerular apparatus 231

K Klinefelter’s syndrome 300 Kluver-bucy animal features 314 Knee jerk reflex in hyperthyroidism 226 Kwashiorkor 290

Lysosomes

309

M Marasmus 290 MCH (mean corpuscular hemoglobin) 276 Mean arterial blood pressure 323 Mean blood pressure 277 Mean corpuscular hemoglobin concentration 258 Megaloblastic anemia 322 Melatonin Action of 330 Membrane potential of smooth muscle, on parasympathetic stimulation 185 Menstrual cycle during post-partum lactation 213 Micturition in complete transaction of the spinal cord 246 Migrating motor complexes (MMC) 205, 322

Milk ejection reflex 283 Mineralocorticoid excess edema not seen 301 Mineralocorticoids escape phenomenon 199 Miniature end plate potential 299 Molecular motors 178, 199 Motor unit 315 Muscarinic receptors 285 Muscle contraction, ECF Ca** content is low RP-/07 Muscle spindle 235 Muscle tone, acute phase of cerebellar disease Myasthenia gravis 171, 185,286, 330

Myocardial contractility 160 Lactation 303 Law of laplace

179 as applied to lung 247 Learning 300 Length tension relationship, in cardiac muscle 181 in skeletal muscle 266 Leptin 249 Lesion of ventromedial nucleus of hypothalamus 300 Lipostatic and glucostatic theory of food intake 282 Local cardiovascular regulatory mechanism 308 Long refractory period in cardiac muscle 205 Loop diuretics 233 Low and high-pressure system 306 Lumbar puncture 268 Lung volumes 174

Myopia 168

Na*— Kt ATP ASE 294 Na*-K*t pump 253 mechanism of operation Natural killer cells 309 Nerve fibers 227 Neural circuits

RP-89

in the olfactory bulb 284 Neuroglial cells RP-703 Neuromuscular excitability in parathyroid hormone deficiency 251 junction in skeletal muscle 328 Neuronal circuits of cerebellum 264 Neurotransmitter 270, 315 Nitrites in cyanide poisoning 294 Non pitting edema in hypothyroidism 286

185

SUBJECT-WISE CUM TOPIC-WISE CONTENT LIST

Gonadotropin level after menopause 251 Goose pimples on exposure to cold 315

REM sleep 330 respiratory function of lungs 295, 261, 322 NSAIDs in patient with compromised renal circulation 177

O Obligatory and facultative water reabsorption 290, 306 Obstructive lung diseases 160 Jaundice 260 Olfactory adaptation 214 Oncotic pressure 179, 260 Oral contraceptive 285 pills 171 Oral glucose tolerance test 182 rehydration therapy, diarrhoea in 192 in infants with diarrhoea 260 Organ of corti 196, 270 Osmolality of urine, in diabetes insipidus 232 Osmotic and water diuresis 274 difference between 217 Outer hair cells 312 Ovulatory cycle 298 Otitis media 330 Oxygen hemoglobin dissociation curve 242,319

SUBJECT-WISE CUM TOPIC-WISE CONTENT LIST

P Pacemaker potential 201 Paraplegia 331 Parasympathetic innervation to salivary gland, interrupted RP-92 Parkinson's disease 171 pCO, in venous blood 291 Peptic ulcer 232, 278 Peripheral chemoreceptors 319 field of vision in complete right optic tract lesion 225 Peristaltic movements 274 Phantom limb 331 phagocytosis 293 Pituitary and thyroid dwarf 298 Placenta functions of 286 Plantar reflex in lesions of upper motor neuron 213 Plasma [K*] concentration after prolonged use of loop diuretics 191 Plasma osmolality 307 proteins 205

Post-synaptic inhibition 264 Presbyopia in myopic 268 Presynaptic inhibition 264 PR interval 291 Pregnancy hormonal changes during RP-100 Protein calorie malnutrition 277 Proton pump inhibitors 260 Prostacyclin 162 Pseudo hermaphroditism 327 Puberty delayed 185 precocious 185 Pulmonary systemic circulation 256 surfactant 258 Pyramidal tract fiber in internal capsule 298

Red glasses 301 Reabsorption in proximal and distal renal tubules 256 Red out on exposure to negative, G force 278 Red muscle fibers 306 Referred pain 171, 181, 198, 235 Reflex cross extensor, pathway for 195 flexor, pathway for 195 REM sleep 213 Renal clearance 174 Renal threshold of glucose 260 Renin-angiotensin system 222 Renin secretion regulation of 292 Renshaw cell inhibition 250 Residual volume 321 Respiratory acidosis 274 alkalosis 274 Restrictive lung diseases 160 Resistance vessels 319 Rh incompatibility 324

S Sarcomere in skeletal muscle 250 Sarcotubular system in skeletal and cardiac muscle 282 Satiety center 314 S. cholesterol 299 Secondary active transport 227, 233 mechanisms in nephron 243

Sclerocorneal angle 185 Semicircular canals 239 Sensorineural conductive deafness 327 Sensory aphasia 268 Sensory motor cortex

315

Sleep pattern 312 Segmental movements 274 Spirogram 256 Skeletal muscle 238 Series elastic component 238 parallel elasticomponents238 Serum albumin globulin ratio 258 calcium level 329 creatinine 276 sodium 307 Sickle cell anemia 276 Sinus arrhythmia 293 Sleep cycle 250 Slow pain 312 Sodium potassium pump 205 Sperm count 299 Spermatogenesis furnace workers in RP-102

Spinal shock 252, 300, 314 Spirogram 174 Starling forces 277, 293 Stomach, mucosal barrier RP-95 Strength duration curve 169, 213 Stress analgesia 314 Stretch and inverse stretch reflex 264, 298 Stroke volume 323 if peripheral resistance is increased RP-93 Subliminal fringe 285 Surfactants

162, 247

T Tables dorsalis 330 Temperature regulatory mechanisms Test tube babies 300 Thermoregulation in neonates 314 Third-degree heart block 219 Threshold for normal hearing 314, 329 Thromboxane A2 162 Thyroid hormone 268 functions of 194 synthesis and release 328 Timed vital capacity 258 Titratable acidity 293 Tonic receptors 330 Transport maximum for glucose (TmG) 321 Transport maximum (Tm) 277 Triple response 220 True hermaphroditism 327 Turner's syndrome 213 Tympanic reflex 199, 252, 301

U Upper motor neuron surfactant 191

331

285

Urinary bladder denervated

WwW

Visual acuity 185 277

is maximum

at fovea centralis

331

Visual

V Venous return 278, 324 Ventilation-perfusion ratio

pathways 298 Visual transduction 271 Vitamin A deficiency 197 Vitamin K,

at the apices of the lungs 233 Vestibulo-ocular reflex 271

in premature babies 309 VO, max 277

Water reabsorption in renal tubules 244 Weber—Fechner law 286 Wenckebach phenomenon White muscle fibers 306

205

Windkessel effect 192, 308, 319

BIOCHEMISTRY

376

D

C

Delayed hypersensitivity 441 Diabetes mellitus 419 diagnosis complications 467 Diagnostic enzymes 336 Dietary fiber in diabetes 337 DNA binding motifs 348, 521 fingerprinting 443 histone interaction 441 repair 458 mechanisms 430 microarrays RP-122 replication 388, 430 proof reading mechanism vaccine 369, 426

Calcitriol 364, 365, 436 action 319 in calcium and phosphorus homeostasis 436 Carnitine role of 412 CD4* cells 403 Cell cycle 444 mediated immunity 457 Chaperones 373 Cholera toxin mechanism of action 347 Cholesterol functions of 505 synthesis 505 Cholestyramine 469 Chylomicrons 438 Ciprofloxacin 388 Citric acid cycle 477 in metabolism 340 Class-switching of immunoglobulins 472 Cloning vectors RP-128 Comparison between, apoptosis and necrosis 497 immunogenicity and antigenicity 462 kwashiorkor and marasmus 462 lab tests of hepatic and posthepatic disease 521 mitochondrial and nuclear DNA

Complement activation 524, 425

472

Electron transport chain 450 Electrophoretic pattern of plasma proteins 510 ELISA 398 Energy metabolism of brain 491 Essential fatty acids, and biomedical importance RP-108 Ethanol, in treatment of methanol poisoning RP-171 Eukaryotic gene expression mechanisms of regulation 351 FF

521

Facilitated diffusion 368 Familial Hypercholesterolemia 505 i

2,3-bisphosphoglycerate Bacterial meningitis 446

Copper biochemical role metabolic defect 505 Creatinine clearance test 457, 478 CTP 426 Cyanide poisoning 357 Cytochrome p450 349 in xenobiotic metabolism 377

SUBJECT-WISE CUM TOPIC-WISE CONTENT LIST

Active methionine 340, 393 biochemical significance 340 Active transport 376 secondary 376 Acute intermittent Porphyria 412 barbiturates administration 378 Acute respiratory distress syndrome 515 in premature babies 339 Adaptive immune response 431 Adaptive immunity 499 Adenylate cyclase 403 AIDS 404 Allopurinol 388 Allosteric enzymes 464 regulation of enzyme activity 490 Allotype 387 Alzheimer’s disease 401 Amino acids inborn errors of metabolism 397 Ammonia detoxification process 380 Anaplerotic pathways 435 reactions 517 Anion gap 442 determination 458 metabolic acidosis, significance in 505 Antibody diversity development 372 Antigen 387 Antioxidant vitamins 518 Apoptosis 354 Aspirin 437 ATP generation chemiosmotic theory 337 ATP sources of 414 synthesis in the electron transport chain 492 Auto-immune diseases 369

Basal metabolic rate 503 in hyperthyroidism 384 Biomolecules 503 derived from phenyl alanine 503 Blood buffers 458 pH 404 regulation RP-123, 347 BMI 516 clinical importance 516


oO

D

=

LU

LL

O

OCO

=" Replicate independently of the bacterial chromosome =" Used for cloning small DNA inserts Bacteriophage: = Low toxicity: should have a minimal effect on the cell physiology

Size of foreign DNA

Cell type specificity

" Used to clone larger DNA inserts 2

Cosmids:

Characteristics of Vectors used in Recombinant DNA

" Hybrids of phages and plasmids

Technology

=

Carry DNA inserts upto 45Kb

O

Capable of replication within the host cell

=

Replicate like plasmids and packaged like phages

O

Have convenient RE sites for inserting the foreign

O

O

©

DNA

=

Have a selectable marker to indicate the uptake of recombinant DNA molecule by host cells Should be small and easy to isolate

translation of DNA insert = Used in eukaryotic genes in bacteria Yeast artificial chromosomes (YACS):

©

Most important vectors are: O

Expression vectors:

=

Plasmids:

=

Small circular DNA molecules separate from the main chromosome

O

Carry host signals to facilitate transcription and

Yeast vectors containing centromere, telomere, origin f replication and selectable marker ;

*_Used for DNA inserts upto 1000KD Bacterial artificial chromosome (BACS): = Bacterial plasmids derived from F plasmid

BD Chaurasia’s Human Anatomy, 7th edition, vol 1, 2, 3 and 4

%

IB Singh’s Textbook of Human Histology, 8th edition IB Singh’s Textbook of Human Neuroanatomy, 10th edition

IB Singh's Human Embryology, 11th edition Langman’s Medical Embryology, 13th edition SD Gangane Human Genetic, 3rd edition

2 =

=e

)

a

Your Roll No. ...............

Name of the Paper

Anatomy Paper-I

Name of the Course

:

MBBS-2018

Semester

>

Annual

Time: 3 Hours

M.M.: 50

INSTRUCTIONS WN

Answers to Parts I, II and III should be written in separate answer sheets provided

&

Write your Roll No. on the top immediately on receipt of this question paper

Attempt parts of a question in sequence

All questions are to be attempted

PART-I 1.

Enumerate the following:

a. c. d. e. 2.

[10]

Muscles supplied by the median nerve in the palm Tributaries of internal jugular vein Layers forming placental barrier at term Types of cartilaginous joints with one example of each

b.

Functional column nuclei of facial nerve

Draw labeled diagram to show the:

a. c. d.

[10]

Microscopic structure of cancellous bone Boundaries and contents of cubital fossa Structure of floor of fourth ventricle

b.

Microscopic structure of mixed salivary gland

e.

Openings in lateral wall of nose

PART-II 3.

4.

Write short notes on:

[9]

a. b.

Lymphatic drainage of mammary gland Extraocular muscles with their action and nerve supply

c.

Fertilization

Describe the thyroid gland under the following subheadings: a.

Extent, surfaces and relations

b. c.

Blood supply Applied anatomy

[6]

PART-III 5.

Write briefly on: a. b.

6.

Pronation and supination of forearm Submandibular Ganglion

Write embryological/anatomical basis of: a. b. c.

[6]

Winging of scapula Spina bifida Vomiting after syringing of ear

[9]

PAPER-I |

2018 PAPER-I

O.

PART-I 1.

Enumerate the following: a.

Muscles supplied by the median nerve in the palm

The site at which exchange take place between the fetal and maternal circulation - placental barrier. Nota true barrier. At 4th month, placental barrier is formed by: Outside to inside = Syncytiotrophoblast = Cytotrophoblast = Extraembryonic connective tissue = Endothelium of the fetal vessels At full term the placental barrier is formed by: Outside to inside = Syncytiotrophoblast = Endothelium of fetal vessels The placenta is hemochorial type in humans as the maternal blood and fetal blood is separated by a

O O

(Ref: BDC Human Anatomy, Vol. 1. 7th ed. pg. 129) ANSWER

O

Muscles Supplied by Median Nerve in Palm

O00 0

O

Flexor pollicis brevis Abductor pollicis brevis Opponens pollicis First and second lumbricals Occasionally: Third lumbrical b.

O.

chorionic tissue.

Functional column nuclei of facial nerve

e.

(Ref: BDC Human Anatomy, Vol. 3. 7th ed. pg. 70) ANSWER

O00

Special General Special General c.

Types of cartilaginous example of each

visceral efferent visceral efferent visceral afferent somatic afferent

Tributaries of internal jugular vein

ANSWER

Growth plate, Soheno-occipital joint, first chondrosternal joint, Costochondral joint

Secondary (Symphyses or fibrocartilaginous joint)

Symphysis pubis, Manubriosternal joint, intervertebral joint, Sacrococcygeal joint, Acromioclavicular joint

2.

Draw labelled diagram to show the:

900000

a.

Inferior petrosal sinus Facial vein Lingual vein Pharyngeal vein Superior thyroid vein Middle thyroid vein Occipital vein-occasionally

Microscopic structure of cancellous bone

(Ref: IB Singh's Textbook of Human Histology, 8th ed. pg. 65)

ANSWER 4 Yea Primitive

Layers forming placental barrier at term (Ref: IB Singh's Human Embryology, 11th ed. pg. 86-87)

osteon Erythropoietic marrow

bin BE Sp.

=

Outer bed

BES -—

B i

a

oe ey

Osteocytes

mssue Spacesin eas

‘ve

ANATOMY

J j | pis,

\wabeculae

ANSWER Adipose tissue

Placental Barrier

The chorion frondosum (fetal portion) desidua_ basalis (maternal portion) constitute the placenta.

:2

Inner cellular) ©

BF SD

95

.

O

one

Primary (Synchondrosis/ hyaline cartilage joint)

Internal Jugular Vein

d.

with

Cartilaginous Joint

(Ref: BDC Human Anatomy, Vol. 3. 7th ed. pg. 158)

O

joints

ANSWER

Functional Column Nuclei of Facial Nerve

O

)

and the together Fig. Microantomy of cancellous bone

PAPER-| b.

Microscopic structure of mixed salivary gland

ANSWER

(Ref: IB Singh's Textbook of Human Histology, 8th ed. pg. 170) 7)

Superior colliculus Inferior colliculus

ANSWER Trochlear nerve}

Se Facial voter Serous

=

\\ BX.

demilunes Blood vessels

Inferior cerebellar peduncle Ss Striae medullares WY

Serous

Excretory

acini

duct

Interlobar

_

connective tissue

.

°

Fig.

Fig. Microanatomy of mixed salivary gland

peduncle

Vy

CS

Vestibular area

Hypoglossal triangle Vagal triangle

_

Posterior median sulcus

Intercalated duct

Interlobular connective tissue

Middle cerebellar

Y/4 AS

Gracile tubercle

Mucous acini

vt

\\

UI

a3)

Si

aul

Floor of fourth ventricle

Superior

c.

Boundaries and contents of cubital fossa

(Ref: BDC Human Anatomy, Vol. 1. 7th ed. pg. 98)

Sulcus

sulcus limitans

SS

\

WG WAT

Cuneate tubercle

.) oP 5

su

SX,

Superior cerebellar peduncl cy

ferruginea

Cerebral

and inferior colliculii

aqueduct

Superior medullary vellum

ANSWER

Musculocutaneous nerve

:

LOM

LAD,

: if

_

ron

Brachialis Biceps brachii

Cerebellum

Cavity of fourth ventricle Pons

Choroid plexus in roof of fourth ventricle

Brachial artery Median nerve

Central canal Medi rture/ edian aperture

Medulla

foramen of magendie

Fig. Roof of fourth ventricle Medial epicondyle and common flexor origin

_f) fy

Brachioradialis

Tendon of biceps brachii Bicipital aponeurosis

Deep branch of

radial nerve Supinator Superficial branch of radial nerve Extensor carpi radialis brevis Extensor carpi radialis longus

e.

Openings in lateral wall of nose (Ref: BDC Human Anatomy, Vol. 3. 7th ed. pg. 252)

ANSWER

Sphenoethmoidal recess Pronator teres

Frontal sinus Secu

Flexor carpi radialis Middle concha 7

+ 7-

Palmaris longus Flexor carpi ulnaris

Frontal sinus opening Fé

Anterior ethmoidal

sinus opening Nasolacrimal

/

a :

|

d.

*\—

5

=e

=

Posterior

Sos

iy ee

concha ethmoidal

promt >

sinus

openings g MaxillaryP

\, sinus opening

Inferior concha

Structure of floor of fourth ventricle (Ref: BDC Human Anatomy, Vol. 4. 7th ed. pg. 115)

Superior

sinus opening Middle ethmoidal

Oca oa Se

duct

Fig. Boundaries and contents of cubital fossa

Sphenoidal air V4, sinus opening

Fig. Openings in the lateral wall of the nose

ANATOMY

Radial nerve

PAPER-I =

Lymphatics from the parenchyma pass laterally towards the axillary tail receiving tributaries on their way: Mostly drain into anterior group and few into posterior group - finally drain into central and apical groups. = Lymphatics from deep parts of breast pierce pectoralis major to drain into axillary and parasternal nodes.

PART-II 3.

Write short notes on:

a.

Lymphatic drainage of mammary gland (Ref: BDC Human Anatomy, Vol. 1. 7th ed. pg. 38)

ANSWER

Applied Anatomy

Lymphatic Drainage of Breast O

O

Dermal network: Drain skin over the breast-except that from nipple and areola-pass radially = Lateral aspect: Drain into axillary lymph nodes = Superior aspect: Supraclavicular nodes, few may reach nodes along cephalic vein in deltopectoral triangle =" Medial aspect: Drain into internal mammary nodes/parasternal nodes =" Inferior aspect: Form a sub-peritoneal plexus and drain into sub-diaphragmatic nodes. =" Cross midline and communicate’ with contralateral lymphatics Lymphatics from parenchyma of breast =" From areola: Subareolar plexus of Sappeycommunicates with breast lymphatics = Axillary lymph nodes drain 75% of breast parenchymal lymphatics.

Predominant pathway of dispersal of carcinoma breast metastasis is by way of lymphatic spread. Axillary lymph nodes: Most common site of metastasis. Blockage of the lymphatic vessels by the carcinoma cells cause lymphedema: Causing elevation of the skin surrounding the pilosebaceous unit which appear as pores giving the appearance of the skin of an orange peel- known as Peau d’ orange sign. Skin dimpling appears when carcinomatous cells invade the glandular tissue leading to fibrosis causing shortening of the suspensory ligaments. Nipple retraction or deviation occurs when the

O O

cancer cells invade the lactiferous ducts and cause

fibrosis and shortening in sub-areolar breast cancer. b.

Extraocular

muscles

with

their

action

and

nerve supply (Ref: BDC Human Anatomy, Vol. 3. 7th ed. pg. 213)

ANSWER Superior rectus

——

Inferior rectus

Superior oblique Inferior

Medial

rectus

rectus

Medial rectus

Lateral

rectus

Figs Table:

Aand B. Extraocular muscles

Extraocular muscles

Muscles

ANATOMY

B |

Insertion

Innervation

Superior rectus

From common tendinous ring

Inserted in front of the equator and 7.7 mm behind the sclerocorneal junction

Oculomotor (SO4, LR6, remaining3- mnemonic)

Inferior rectus

From common tendinous ring

Inserted in front of the equator and 6.5 mm behind the sclerocorneal junction

Oculomotor

Medial rectus

From common tendinous ring

Inserted in front of the equator and 5.5 mm behind the sclerocorneal junction

Oculomotor

Lateral rectus

From common tendinous ring

Inserted into in front of the equator and 6.9 mm behind the sclerocorneal junction

Abducent nerve Contd...

PAPER-| | Origin

Muscles

Superior oblique

| Insertion

| Innervation

Inserted into sclera behind the equator

Trochlear nerve

Oculomotor nerve

maxilla

Inserted into sclera behind the equator in line with superior oblique

From lesser wing of sphenoid

Tarsal plate and to the skin of upper eyelid

Oculomotor

From undersurface of lesser

wing of sphenoid Inferior oblique Levator palpebrae superioris Table:

From orbital surface of

Actions of extraocular muscles

Muscle

Action in primary position

Action in abducted eye

Action in adducted eye

Yolk muscles

Superior oblique

Depression

Only intorsion

Only depression

Inferior rectus of opposite eye

Only extorsion

Only elevation

Superior rectus of opposite eye

Only elevation

Only intorsion

Inferior oblique of opposite eye

Only depression

Only extorsion

Superior oblique of opposite eye

Abduction Intorsion

Inferior oblique

Elevation Abduction Extorsion

Superior rectus

Elevation Adduction Intorsion

Inferior rectus

Depression

Adduction Extorsion Lateral rectus

Only abduction

No action

No action

Medial rectus of opposite eye

Medial rectus

Only adduction

No action

No action

Lateral rectus of opposite eye

Fertilization

Describe the thyroid gland under the following subheadings:

(Ref: IB Singh's Human Embryology, 11th ed. pg. 46) ANSWER Fertilization

Process of fusion of male and female gamete pronuclei. Site - Ampulla of the fallopian tube Occurs within 24 hrs of ovulation Completion of the second meiotic division in the secondary oocyte is signaled by fertilization. The sperms have to get capacitated by modification in the membrane sterols or surface proteins before fertilization. Phases of fertilization: ="

="

=

b.

Blood supply

c.

Applied anatomy (Ref: BDC Human Anatomy, Vol. 3. 7th ed. pg.

144-149)

ANSWER

Thyroid Gland O

Endocrine gland

O

Extent in neck: C5 -T1 Covered by pretracheal layer of deep cervical fascia Parts: 2 lobes joined by a median isthmus

O O

Lobes

Phase 1-Penetration of corona radiata

Phase 2-Penetration of Zona Pellucida

Phase 3-Fusion of the oocyte and sperm membranes Events in fertilization: =" Cortical and Zona reaction = Resumption of the second meiotic division = Metabolic activation of egg Results of Fertilization: = Restoration of the diploid number = Determination of genetic sex = [Initiation of cleavage

Extent, surfaces and relations

cell

O

Conical

O

Dimensions: 5 cm X 3 cm X2 cm

O

Surfaces and borders: Relations

= Apex: Reaching cartilage =

up to oblique

Superficial/Lateral

Sternohyoid,

line of thyroid

surface:

Superior

belly

Sternothyroid,

of

omohyoid

&

Sternocleidomastoid ="

Medial

surface:

Larynx

and

trachea,

Inferior

pharyngeal constrictor, cricothyroid with external

,

OO 0

O

a.

ANATOMY

c.

PAPER-I

"

Recurrent

laryngeal

&

of Superior

*

O

Superior parathyroid glands: Located on the posterior border of the middle third of the gland beneath the false musculofascial capsule. Inferior parathyroid glands: Located on the posterior border of the lower pole of the gland beneath the false musculofascial capsule

O O

Isthmus O

Dimension: 1.25 cm x 1.25cm

O

Opposite 2nd and 3rd tracheal rings

O

Relations:

=

Sternothyroid, Sternohyoid, and Anterior jugular veins. = Superior thyroid arteries anastomose along the upper border. O

=" Inferior thyroid vein: Lower border Levator glandulae: Fibromuscular band extending between hyoid and isthmus of thyroid gland/ pyramidal lobe.

Vascular Supply-Arterial Supply

O

O

O

Pretracheal and paratracheal nodes Laterally: Deep cervical nodes Occasionally - drain into brachiocephalic nodes or directly into thoracic duct

Applied Anatomy

Posterior border:

¢

O O

Posterolateral surface: Carotid sheath

= Anterior border: Anterior branch thyroid artery ="

nerve

O

laryngeal nerve, esophagus

Superior thyroid artery = Branch of external carotid artery = Related to external laryngeal branch of Superior laryngeal nerve (branch of Vagus nerve) = Divides into anterior and posterior branches Inferior thyroid artery =" Branch of thyrocervical trunk of first part subclavian artery

of

=" Related to Recurrent laryngeal branch of Vagus nerve = Divides into ascending and inferior laryngeal branches Arteria thyroidea ima: Branch of either brachiocephalic trunk or arch of aorta

Ectopic thyroid gland: Along the path taken by the thyroglossal duct-most common site-base of tongue Goitre: Thyroid enlargement-pressure symptoms on trachea or recurrent laryngeal nerves or veins Thyroidectomy: Surgical considerations = External laryngeal nerve deviate away from the Superior thyroid artery near the superior pole of the gland: Near the upper pole of the gland best site to ligate the superior thyroid artery " Recurrent laryngeal nerve approaches the Inferior thyroid artery near the lower pole of the gland: Best site to ligate the Inferior thyroid artery is away from the gland. =" Rich vascular plexus exists just deep to the true capsule of the thyroid gland: Removal of the thyroid gland is by excising the avascular musculofascial false capsule and removing the encapsulated gland (with true capsule intact). =" Pararthyroids are at a danger of damage because they lie in the same plane as of the thyroidectomy surgery (beneath the false capsule): Protected by exposing and securing before removal of the gland. =" Because of the complexity of the lymphatic drainage, detailed regional lymph node removal in metastatic papillary carcinoma is advocated (instead of standard radical neck dissection). =" Arteria thyroidea ima passes in the midline ventral to trachea to reach the isthmus of thyroid gland: May cause hemorrhage while performing tracheostomy surgery.

PART-III 5.

Write briefly on: a.

Pronation and supination of forearm (Ref: BDC Human Anatomy, Vol. 1. 7th ed. pg. 107)

Venous Drainage O

ANATOMY

O O

Superior thyroid vein: Drains into Internal jugular vein Middle thyroid vein: Drains into Internal jugular vein Inferior thyroid vein: Drains via pretracheal plexusinto right and left brachiocephalic veins respectively

ANSWER Supination and Pronation O

Supination and pronation occurs at Humeroradial, proximal and distal radioulnar joints.

O

Humero-radial is a part of the complex hinge joint of elbow. Proximal and distal radioulnar joints are pivot variety of synovial joint.

Lymphatic Drainage O

Via tracheal plexus into Prelaryngeal nodes above the isthmus)

(just

O

PAPER-| O

O

O

O

O

Inanatomical position: The palm facing anteriorly, and forearm is supinated with radius being lateral and parallel to ulna and interosseous membrane non- spirallized.

*

Posterior filament between ganglion lingual nerve. Pathway for secretomotor fibers: Superior Salivatory nucleus(Pons)

*

1

Pronation: Radius turns anteromedially and obliquely across the ulna. Upper end lying lateral and its lower end medially to ulna with the interosseous membrane spiralized.

Facial nerve

L Chorda Tymapni nerve

Supination: Radius turns back to its lateral position. Range: With elbow extension 140°-150°. Increased upto 360° with scapular movements and humeral rotation.

L Lingual nerve

L

Axis:

= A line passing through the centre of the head of radius proximally to the ulnar attachment of articular disc (of distal radio-ulnar joint) distally: Axis of movement of radius relative to

Submandibular ganglion (Synapse)

L Branches to submandibular gland join lingual nerve

L

ulna, and it is not fixed.

Sublingual gland

=" Head of radius rotates (on the capitulum of humerus) in fibro-osseous ring and distal end of radius swings around the head of ulna.

=" Sympathetic root: * Pathway for vasomotor fibers Superior cervical sympathetic ganglion (Synapse)

=" From full pronation to supination the distal end of ulna also moves along a curved course which is posterolateral in pronation and anteromedial in supination: Due to which axis of supination and pronation does not remain stationary.

1 Plexus around facial artery

L Submandibular ganglion (Without synapse)

1 Submandibular, sublingual and anterior lingual glands

Muscles Causing Movements

O

=" Sensory root:

Supination:

*

= Supinator: Slow supination in extension (without resistance) = O

Biceps: Fast supination flexion (with resistance)

in fast movement

in

Pronation:

= =

and

6.

From lingual nerve

Write embryological/anatomical basis of: a.

Winging of scapula (Ref: BDC Human Anatomy, Vol. 1. 7th ed. pg. 12)

Pronator quadratus for rapid movement. Pronator teres for pronation against resistance.

= Assisted by gravity

ANSWER

b.

Winging of scapula is a clinical sign which can be elicited when the patient is asked to push his hands against a wall or in abduction-resulting in prominence of the medial border and inferior angle of the scapula.

Submandibular Ganglion (Ref: BDC Human Anatomy, Vol. 3. 7th ed. pg. 139)

ANSWER

Anatomical Basis

O

Small, fusiform structure

O

Location: Topographically related to the lingual nerve, but functionally related to the chorda tympani branch of facial nerve.

O

Situated superficial to the hyoglossus muscle, just above the deep part of the submandibular gland.

O

Roots:

= Motor/parasympathetic root:

The medial border of the scapula is kept pressed onto the thoracic wall by the serratus anterior muscle. Paralysis of serratus anterior either in injury to the Long Thoracic nerve or injuries to the thoracic wall. Over activity of rhomboids and levator scapulae make the medial border of the scapula prominent. b.

ANATOMY

Submandibular Ganglion

Spina bifida (Ref: IB Singh's Human Embryology, 11th ed. pg. 308)

9

PAPER-I c.

ANSWER

Vomiting after syringing of ear (Ref: BDC Human Anatomy, Vol. 3. 7th ed. pg. 286)

Spina bifida comprises of a constellation of defects involving the neural tube esp. of the spinal region. ANSWER

Embryological Basis Normally the closure of ventral neuropore and dorsal neuropore happens by 25 and 27 days respectively. Abnormal

of

neural

folds

lead

to

these

abnormalities.

Anatomical Basis

Neural tube defects are divides into two categories:

External acoustic meatus is supplied by:

O

O

Anterosuperior wall-auriculotemporal nerve

O Defects involving secondary neurulation Various categories include:

O

Posteroinferior wall-auricular branch of nerve (Alderman’s nerve/Arnold’s nerve)

O

If either very hot or cold water is used, it irritates the

OO 0

ANATOMY

closure

Syringing is a procedure used by otolaryngologists to clean the external acoustic meatus with a gush of warm saline water.

Defects involving primary neurulation

Spina bifida occulta Meningocele Meningomyelocele Rachischisis

Vagus

vagus nerve, which may lead to stimulation of reflex gastric peristalsis (reverse) lead to vomiting and coughing.

Your Roll No. ...............

Name of the Paper

:

Anatomy Paper-Il

Name of the Course

:

MBBS-2018

Semester

:

Annual

Time: 3 Hours

M.M.: 50 INSTRUCTIONS

1.

Write your Roll No. on the top immediately on receipt of this question paper

2.

All questions are to be attempted

3.

Answers to Parts I, II and III should be written in separate answer sheets provided

4.

Attempt parts of a question in sequence

PART-I 1.

Enumerate the following:

a. b. c. d. e. 2.

[10]

Derivatives of paramesonephric duct in female Layers of adrenal cortex with their secretions Branches of right coronary artery Lateral rotators of hip joint with their nerve supply Contents of spermatic cord

Draw labeled diagram to show the: a. b. c. d. e.

[10]

Microanatomy of ovary Microanatomy of spleen Anterior relations of left kidney Transverse section of thorax at upper border of T3 vertebral level Cutaneous innervation of dorsum of foot

PART-II 3.

Write short notes on: a. b. c.

4.

[9]

Great saphenous vein Autosomal dominant disorders Rotation of midgut

Describe the urinary bladder under the following headings: a.

Borders, surfaces and relations

c.

Applied anatomy

[6] b.

Innervation

PART-III 5.

Write briefly on: a. b.

6.

[6]

Blood supply of stomach Bronchopulmonary segment

Write embryological/anatomical basis of: a.

Femoral hernia

b.

Patent ductus arteriosus

c.

Referred pain in myocardial infarction

[9]

PAPER-II |

2018 PAPER-II

)

PART-I 1.

Enumerate the following a.

Derivatives of paramesonephric duct in female (Ref: IB Singh's Human Embryology, 11th ed. pg. 284)

ANSWER Derivatives of Paramesonephric Duct

Paramesonephric ducts/Mullerian ducts are a pair of O

Males

= Appendix of testis O

=" Utricular cord Females

=

Fallopian tubes

=

Uterus

=

Cervix

= Upper part of vagina b.

Layers of adrenal cortex with their secretions (Ref: BDC Human Anatomy, Vol. 2. 7th ed. pg. 358)

ANSWER

Layers of Adrenal Cortex O O O

Zona glomerulosa: Mineralocorticoids-eg. Aldosterone Zona fasciculata: Glucocorticoids - eg. Cortisol Zona reticulata: Gonadocorticoids-eg. Adrenal androgens, Dehydroepiandrosterone, Androstenedione

Adrenal Cortex Layer and Products Surface of adrenal gland

Layer: * Glomerulosa

en

— Adrenal gland

- Baibeage s

Connective tissue

| /- Capsule oa “1+ Zona glomerulosa

* Fasiculata * Reticulata

— Zona fasciculata

Respective Products: Adrenal * Mineralocorticoids * Glucocorticoids

cortex

J

{>

Adrenal cortex—

/

7

1 + Zona reticularis

* Sex hormone

2

Adrenal medulla

ANATOMY

(a) Mnemonics:

c.

:

Adrenal medulla] | (b)

“Go Find Road, Make Good Start”

Branches of right coronary artery (Ref: BDC Human Anatomy, Vol. 1. 7th ed. pg. 278)

O

ANSWER

Lymphatics from testis and epididymis

O

PAPER-II Nerves

Branches

=

Genital branch of genitofemoral nerve

O

=

Sympathetic plexus of nerves around testicular artery

First segment - between origin and right cardiac margin = Right conus artery (anastomoses with similar branch of left coronary to form the Anulus of Vieussens). = Anterior Atrial branches - Artery of Sinuatrial node ="

O

Lateral Atrial branches

= Anterior ventricular branches - right marginal artery Second segment - between the right border and the crux of the heart = Posterior Atrial branches = Posterior (Inferior) ventricular branches = Posterior Interventricular branch and _ septal branches ="

d.

O

Loose areolar tissue

O

Remnants of processus Vaginalis

2.

Draw labelled diagram to show the: a.

Microanatomy of ovary

(Ref: IB Singh's Textbook of Human Histology, 8th ed. pg. 233) ANSWER

aN

Secondary

Germinal epithelium

follicle

Cortical stroma

Ramus cristae terminalis

Primordial follicle

Lateral rotators of hip joint with their nerve supply (Ref: BDC Human Anatomy, Vol. 1. 7th ed. pg. 150)

Mf

Theca externa Theca interna

Medullary ——\ ge \

vessels

.

-

\\

Granulosa cells

Cumulus oophorus Antrum Ovum in mature follicle

ANSWER

00000 0

Chief muscles of Lateral Rotation

Obturator internus: Nerve to obturator Superior gemellus: Nerve to obturator Inferior gemellus: Nerve to quadratus Quadratus femoris: Nerve to quadratus Piriformis: Ventral ramiS, , Gluteus maximus: L5S. ,

Fig. Microanatomy of ovary

Internus internus femoris femoris

L_S, L.S, LS, L_S,

b.

Microanatomy of spleen

(Ref: IB Singh's Textbook of Human Histology, 8th ed. pg. 98) ANSWER

Assisting Muscles

Sartorius: Femoral nerve L,,

O

Obturator externus: Obturator nerve L, ,

e.

Contents of spermatic cord (Ref: BDC Human Anatomy, Vol. 2. 7th ed. pg. 233)

ANSWER

Spermatic cord contents: O O O

Vas deferens Pampiniform plexus of veins Arteries = Testicular =" Cremasteric = Artery to vas deferens

. **

>

=

+ 4

et

F205)

Eccentrically

placed central artery

Po

.

~

»

PC °

our

ae awd

react shes FS :-

Se

A BS" Se ao}

are pie,

ring bodies Pituicytes

e

oonenes

ae Sr

OTRAS

Acidophils Adenohypophysis

tract of neuro-

SSS

,

Colloid

Pars

Ba ‘ ‘

e

Capillaries . » «

Se >

5

a4

. Se



;

o

i~

Ms

intermedia

” Ja

es

Connective tissue septa

Fig. Microanatomy of pituitary gland

c.

TS of pons at the level of facial colliculus

(Ref: BDC Human Anatomy, Vol. 4. 7th ed. pg. 97)

ANSWER Abducent nerve nucleus

Facial motor nucleus —

— Vestibular nucleus

Superior salivatory nucleus

Inferior cerebellar peduncle Dorsal and ventral cochlear nuclei

Medial longitudinal fasciculus bas

(a

NS

»)

ZY

Spinal leminiscus

“hy 4 eX

Spinal nucleus and tract of

trigeminal nerve

Middle cerebellar peduncle Tectospinal tract

Trigeminal leminiscus

Trapezoid body Corticospinal and corticobulbar tracts

Rubrospinal tract

Pontine nuclei

#

Cortico-ponto-cerebellar fibers

Fig. TS of pons at the level of facial colliculus

ANATOMY

Medial leminiscus

PAPER-I d.

Structures passing through superior orbital fissure

(Ref: BDC Human Anatomy, Vol. 1. 7th ed. pg. 215)

ANSWER Lateral rectus muscle

Common tendinous ring

Superior orbital fissure

Superior rectus muscle Levator palpebrae superioris muscle

Recurrent meningeal branch of lacrimal artery

:

Vif

Superior oblique muscle

—*4 WV ~

Optic nerve with its meningeal coverings

iim, il! | |), -

4

‘7

“-

\ w/s. :

Trochlear nerve Lacrimal nerve .

~

.

— Frontal nerve

Superior ophthalmic vein

Medial rectus muscle Optic canal

Superior division of occulomotor nerve Ophthalmic artery Nasociliary nerve

Inferior rectus muscle

Abducent nerve

Inferior ophthalmic vein Inferior division of occulomotor nerve

Infraorbital nerve

Zygomatic nerve

Infraorbital artery

Inferior orbital fissure

Fig. Structures passing through superior orbital fissure e.

Sensory supply of face

(Ref: BDC Human Anatomy, Vol. 3. 7th ed. pg. 70)

ANSWER

Supraorbital nerve Vy, Supratrochlear nerve

Zygomaticotemporal nerve Lacrimal nerve

Auriculotemporal nerve

Infratrochlear nerve Zygomaticofacial nerve

AG ;

— Greater occipital nerve (C2 dorsal ramus)

Infraorbital nerve

External nasal nerve

-(7_/

C3 dorsal ramus Lesser occipital nerve

Buccal nerve Mental

nerve

[ Great auricular nerve C4 dorsal ramus

\ Transverse cervical nerve

Fig. Sensory supply of scalp, face & Neck

ANATOMY

PART-II 3.

Write short notes on: a.

Development of interatrial septum

(Ref: BI Singh's Textbook of Human Embryology, 11th ed. pg. 233)

ANSWER Development of Interatrial Septum

O

Interatrial septum divides the chamber into right and left atria

O

At 4th week of gestation a sickle shaped partition appears in the common atrial chamber known as the septum primum

common.

atrial

PAPER-|

O

O O

O

O

It grows towards the developing atrioventricular endocardial cushions but is left short by a narrow gap known as ostium primum With further growth and approximation of septum primum with endocardial cushions to obliterate the ostium primum-small defects appear in the upper part of septum primum which coalesce to form ostium secundum. Another crescentric fold appears in the right atrial cavity known as Septum Secundum Septum secundum is an incomplete fold directing the flow of blood from the inferior vena cava into the left atrial chamber-this new direction to the flow of blood forms the oblique cleft known as Foramen Ovale. At birth: The pulmonary circulation begins and foramen ovale obliterates by increased pressure in the left atrial chamber Interatrial septum formed by fusion between septum secundum and septum primum

O

= Accompanied by lateral rotation and vice versadue to shape of bones and attachment of the palmar ligament. = Muscles *¢

O

Adduction =" Muscle: Adductor pollicis Opposition

=" Muscles: Opponens pollicis and flexor pollicis Brevis together cause flexion and medial rotation of an abducted thumb to cause opposition. Force of Digital grasp is increased by Adductor Pollicis and Flexor Pollicis Longus.

the O

b.

we

i

Flexion

Extension

nag

c.

in this

spductin en

Circle of willis and its clinical importance (Ref: BDC Human Anatomy, Vol. 4. 7th ed. pg. 171)

ANSWER

Movements at lst carpometacarpal joint (Ref: BDC Human Anatomy, Vol. 1. 7th ed. pg. 161)

Circle of Willis O

Anastomotic arterial network in the interpeduncular fossa within the subarachnoid space.

O

Formed by the internal carotid and vertebral arteries of both sides.

ANSWER First Carpometacarpal Joint-between base of Ist Metacarpal and Trapezium

Saddle variety of synovial joint Movements

O

Circumduction

= A combination of following movements order or reverse cause circumduction

Atrial septal defects =" Ostium secundum is the most common type: Due to excessive cell apoptosis in the upper part of Septum Primum or by insufficient formation of Septum secundum. =" Ostium primum defect: Septum primum fails to fuse with endocardial cushions. =" Cor triloculare biventriculare: complete deficiency of atrialseptum. = Premature closure of foramen ovale: Leads to hypertrophied right side of heart and hypoplastic left side.

Abductor pollicis longus and extensor pollicis brevis * Accessory muscle: Extensor pollicis longus Abduction = Muscles: Abductor pollicis longus and brevis

Applied Anatomy

O

Extension

Flexion

= Accompanied by medial rotation and vice versadue to shape of bones and oblique attachment of the dorsal ligament. = Muscles ¢ Flexor pollicis brevis assisted by opponens pollicis (Causes flexion with medial rotation) ¢

Accessory muscle: Flexor pollicis longus

Formation: From behind forwards

=" Posteriorly: Terminal branches of basilar artery Right and left Posterior Cerebral arteries =" Posterolaterally: arteries both sides

Posterior

communicating

= Laterally: Terminal segment of internal carotid arteries with branches-middle and anterior cerebral arteries = Anterolaterally: both sides

Anterior

cerebral

arteries

=" Anteriorly: Anterior communicating artery.

of

ANATOMY

O

PAPER-I 4.

Branches O

Central/Perforating following groups: ="

arteries:

Subdivided

into

Anteromedial branches:

From Anterior communicating artery and anterior cerebral arteries-pierce anterior perforated substance. * Supply: Opticchiasma, preoptic andsupraoptic areas of hypothalamus, septum pellucidum, anterior columns of fornix, cingulate gyrus, rostrum of corpus callosum, anterior part of putamen and head of caudate nucleus. =" Anterolateral branches

Describe parotid gland under the following headings: a.

Relations and nerve supply

b.

Contents

c.

Applied anatomy

*

¢

From proximal part of the middle cerebral arteries-also known as striate/lateral striate/ lenticulostriate arteries. 0 Pierce anterior perforated substance. 6 Supply posterior striatum, part of globus palllidus,

*

anterior

limb,

genu,

and

posterior limb of internal capsule. Medial striate artery: From middle or anterior cerebral arteries-supplies rostral part of caudate

nucleus

and

putamen,

anterior

limb and genu of internal capsule. =" Posterolateral branches: * From distal part of posterior cerebral arteries * Supply: Cerebral peduncles, colliculi, pineal gland,

and

posterior

thalamus

and

(Ref: BDC Human Anatomy, Vol. 3. 7th ed. pg. 109-113) ANSWER Parotid Gland O O O

Largest salivary gland Predominantly serous gland Parotid duct: Opens into vestibule opposite 2nd maxillary molar tooth

Relations O

Surfaces: Superficial, Superior (base), Anteromedial and Posteromedial surfaces = Apex-blunt-lies on the posterior belly of digastric muscle and carotid triangle. =" Superior surface-related to: * Cartilaginous part of external acoustic meatus, and posterior part of temporomandibular joint. ¢

geniculate body (bythalamogeniculate branches) =" Posteromedial branches: ¢ From proximal part of posterior cerebral

*

Clinical Importance O

O

ANATOMY

O

Most important collateral arterial network to maintain cerebral blood flow. Obstruction in any one of the arterial systems (Internal carotid or Vertebral), the flow of blood from the unobstructed side replenishes for the deficient supply. Berry aneurysm-rupture cause Subarachnoid hemorrhage. Anterior choroidal artery-from internal carotid artery-artery of cerebral thrombosis. Charcot’s artery-one of the lateral striate arteries is more susceptible to rupture-artery of cerebral hemorrhage.

nerve: Curves around embedded in capsule of

=" Superficial surface:

medial

arteries and posterior communicating arteries - pierce posterior perforated substance. Supply: Parts of thalamus, hypothalamus, pituitary gland, mammillary _ bodies, subthalamus, globus pallidus.

Auriculotemporal neck of mandible, parotid gland.

=

*

Skin

*

Superficial fascia with branches of great auricular nerve, superficial parotid lymph nodes, & platysma

Anteromedial surface-related to:

*

Posterior border of ramus of mandible grooves the surface

* * *

Masseter Temporomandibular joint Facial nerve branches emerge at the anterior border =" Posteromedial surface:

* * * *

Mastoid process Sternocleidomastoid Posterior belly of digastric Styloid process and its associated apparatusseparating carotid sheath contents from the gland. *¢ External carotid artery grooves the surface and then enters the gland = Anteromedial and posteromedial surfaces meet at the medial margin-related to medial pterygoid and lateral wall of the pharynx.

PAPER-| Nerve Supply

Applied Anatomy

Parasympathetic Secretomotor Pathway

O

Inferior salivatory nucleus

:

O

O

Glossopharyngeal nerve (9th nerve)

+

Tympanic branch of glossopharyngeal nerve

Tympanic plexus

O

Inflammation of parotid gland: Acute Sialadenitisvery painful-due to stretching of the parotid capsule supplied by great auricular nerve. Mumps: Viral infection of the parotid gland Parotidectomy: = For parotid tumours. = The plane of the facial nerve is used as a guide for resection of gland. Frey’s syndrome: Sweating, redness and warmth of the angle of the mouth at the site/smell of food. = Occurs after parotidectomy surgery = Due to regrowth of the severed ends of parasympathetic fibers of auriculotemporal nerves into the skin and sweat gland innervation of the angle of mouth by the Great Auricular nerve.

4

=

Lesser petrosal nerve

Otic ganglion (SYNAPSE)

:

Auriculotemporal nerve

Sympathetic Supply Post ganglionic fibers from superior cervical ganglion

PART-III

:

Sympathetic plexus around external carotid artery

5.

Sensory Supply

Write briefly on: a.

by great auricular

and

auricu-

(Ref: BDC Human Anatomy, Vol. 3. 7th ed. pg. 70)

Contents

ANSWER

From deep to superficial-the following structures with branches:

Lacrimal apparatus comprises of the lacrimal gland and the lacrimal drainage pathway.

O

Lacrimal Gland

O

Retromandibular vein

=" Formation: Union of maxillary and superficial temporal veins. = At apex of gland divides into anterior and posterior branch Facial nerve with its branches:

= Facial nerve enters posteromedial surface. =" Branches:

O

v

Superior/temporofacial trunk

Inferior/cervicofacial trunk

O O

y ¢ ¢ ¢ ¢

Temporal branch Zygomatic branch Buccal branch Marginal mandibular branch

Winds

around

the

lateral border

of LPS

to lie

below the muscle

v

:

and Immunoglobulin A. Orbital and palpebral parts. Orbital part: = Almond shaped lodged in the lacrimal fossa on the medial aspect of the zygomatic process of the frontal bone. = Lies superior to levator palpebral superioris (LPS) Palpebral part: ="

Facial nerve

Parotid plexus/pes anserinus

Produces the aqueous component of tear film with antimicrobial substances like Lysozyme, Lactoferrin,

O

O

External carotid artery = Maxillary artery emerges from anteromedial surface = Superficial temporal artery (Superior surface) with its transverse facial branch. = Posterior auricular artery: Posteromedial surface

O

O

O

Main ducts open up into the conjunctival sac of superior lateral fornix. Ducts from the orbital part penetrate the LPS to join the ducts of palpebral part. Accessory lacrimal glands: Glands of Krause and Wolfring-occur in or near the fornix.

ANATOMY

Sensory innervation lotemporal nerves.

Lacrimal apparatus

PAPER-I Lacrimal Drainage Pathways

Sympathetic Supply

Tears collect at medial canthus of eye

Superior cervical ganglion

Punta of upper and lower eyelids

Sympathetic plexus around internal carotid artery

Lacrimal canaliculus—Superior and inferior

Deep petrosal nerve

|

:

i Lacrimal sac

. Joins greater petrosal to form

'

Sensory Supply

|

Lacrimal nerve-branch trigeminal nerve

Nasolacrimal duct

nerve of pterygoid canal

of

ophthalmic

division

of

Inferior meatus of nose

b.

Internal capsule (Ref: BDC Human Anatomy, Vol. 4. 7th ed. pg. 146)

Blood Supply Arterial Supply

O

Lacrimal

ANSWER

branch

of

ophthalmic

artery

infraorbital artery branch of maxillary artery

Venous Drainage O

Into superior ophthalmic vein

and

Internal Capsule ©

Type

O

Inatransverse section of the brain-shape-V shaped

O

Parts: = Anterior limb = Genu

Nerve Supply

. Parasympathetic Supply

="

Lacrimatory nucleus |

of projection fibers

er Posterior limb Retrolenticular

=" Sublenticular O

Constituent fibers

Nervus intermedius (sensory branch of facial nerve)

|

Anteriorlimb

© Frontopontine e Anterior thalamic radiations

Facial nerve trunk

|

Genu

e Corticobulbar (for contralateral motor nuclei

of cranial nerves) Geniculate ganglion (no relay)

e Anterior fibers of superior thalamic radiations

| Greater petrosal nerve

Posterior limb

© Corticospinal/Pyramidal tract (Anteroposterior—Upper limb, Trunk, Lower limbs)

| Nerve of pterygoid canal | Pterygopalatine ganglion

e Frontopontine e Corticorubral e Superior thalamic radiations

(SYNAPSE) Retrolenticular

¢ Parietopontine

|

e Occipitopontine

Maxillary nerve

e Posterior thalamic radiations

ANATOMY

| Zygomatic branch

e Optic radiations Sublenticular

| © Temporopontine e Few parietopontine

| Zygomaticotemporal branch

|

e Auditory radiations

Blood Supply O

Lacrimal nerve

Central

arteries from

the circle of Willis: Lateral

and medial striate arteries.

PAPER-| O

Lateral striate arteries supply: Anterior limb, genu, most of posterior limb of internal capsule

O

Medial striate artery supply: Anterior limb, and genu of Internal capsule and the basal ganglia. Anterior choroidal artery: Ventral part of posterior

O

=" Deficient growth of these processes leads to non fusion between the maxillary process or medial nasal prominence-leading to a cleft in the upper lip O

limb and retrolenticular part of Internal capsule.

6.

=" Incomplete fusion prominences.

Wrist drop (Ref: Langman’s Medical Embryology, 13th ed. pg. 292; BDC Human Anatomy, Vol. 1. 7th ed. pg. 102)

c.

O

Paralysis ofthe extensors ofthe wristand handleading to overactivity of the flexors of forearm and handthus the hand assuming its characteristic position of flexion at the wrist and metacarpophalangeal joints. Weak extension of interphalangeal joints may be due to action of lumbricals and interossei that insert into dorsal digital expansion, thus aiding extension. b.

nasal

Meningocele

Meningocele O

Fluid filled outpouchings through defects in vertebral arch or skull is known as meningocele. Type of Neural tube defect.

O

May occur in the lumbosacral or occipital region

Embryological Basis Oo

Abnormalities in closure of the neural folds in 3" or 4'" week of development lead to a constellation of disorders known as Neural tube defects.

O

Vertebral arch development is induced underlying developing neural tube.

O

Defective

Cleft Lip (Ref: IB Singh's Human Embryology, 11th ed. pg. 157)

medial

ANSWER

Anatomical Basis

O

two

(Ref: Langman’s Medical embryology, 13th edition, pg. 315)

Wrist Drop

The radial nerve injury leads to wrist drop. Wristdrop patientpresents with inability ofextension of wrist and fingers at metacarpophalangeal joints, with weak extension at proximal and distal interphalangeal joints

the

Mutations in IRF6 (Van der Woude syndrome) and MSX1; teratogens like anticonvulsant drugs like Valproic acid and cigarette smoking are associated with orofacial clefts.

ANSWER

O. O

of

= Associated with impaired cognition, midline defects of brain (holoprosencephaly) etc.

Write embryological/anatomical basis of: a.

For median cleft lip

neural

folding

therefore

by

the

causes

deficient vertebral arch formation with or without

ANSWER

In the Occipital region the defect involves primary neurulation process with the skull overlying is deficient.

O

Forthe lumbosacral region the defect is in secondary neurulation process involving the caudal part of the neural tube-with lumbosacral vertebra deficient.

Types: Unilateral or bilateral; median cleft or lateral cleft

Embryological Basis O

O

For lateral cleft which may be U/L or B/L = The upper lip is formed by the union of the medial nasal prominences (part of Frontonasal process) with the maxillary processes of both the sides.

i

O

Defects in facial development: Leading to gap in upper lip.

Subsequent herniation of meninges and CSF through the defect leads to cyst like sac covered by the skin.

ANATOMY

O

O

N

Cleft Lip

involvement of neural tissue it.

Your Roll No. ...............

Name of the Paper

:

Anatomy Paper-Il

Name of the Course

:

MBBS-2017

Semester

:

Annual

Time: 3 Hours

M.M.: 50 INSTRUCTIONS

1.

Write your Roll No. on the top immediately on receipt of this question paper

2.

All questions are to be attempted

3.

Answer to Parts I, If and III should be written in separate answer sheets provided

4.

Attempt parts of a question in sequence

PART-I 1.

Enumerate the following:

a. c. e. 2.

[10]

Ventral branches of abdominal aorta b. Derivatives of midgut loop Components of extra-hepatic biliary system d. Evertors of foot with nerve supply Developmental components of interventricular septum of heart

Draw labeled diagram to show the:

a. c. d. e.

[10]

Microscopic structure of hyaline cartilage Posterior relations of left kidney Boundaries and contents of poplieteal fossa Structure in the inter condylar area of tibia

b.

Microscopic structure of ureter

PART-II 3.

Write short notes on: a. b. c.

4.

[9]

Down’s syndrome Pelvic diaphragm Medial longitudinal arch of the foot

Describe the stomach under the following headings (very imp):

a. b. c.

[6]

Parts and surfaces Stomach bed Applied anatomy

PART-III 5.

Write briefly on: a. b.

6.

Femoral hernia Openings in the diaphragm

Write embryological/anatomical basis of: a. b. c.

[6]

Tracheo-esophageal fistual Descent of testis Foot drop

[9]

PAPER-II |

2017 PAPER-II

O

PART-I 1.

Enumerate the following: a.

Ventral branches of abdominal aorta

(Ref: BDC Human Anatomy, Vol. 2. 7th ed. pg. 368)

Parts of extrahepatic biliary apparatus = Right and left hepatic ducts =

Common hepatic ducts

="

Gall bladder

=

Cystic duct

"="

Common bile duct

d.

ANSWER

)

Evertors of foot with nerve supply (Ref: BDC Human Anatomy, Vol. 2. 7th ed. pg. 108)

Ventral Branches of Abdominal Aorta O

Abdominal

aorta

branches

are

ventral,

lateral,

ANSWER

dorsal, and terminal.

O

Ventral branches are: =" Coeliac trunk-T,, = Superior mesenteric artery-L, = Inferior mesenteric artery-L, b.

Evertors of Foot O O

O

Derivatives of Midgut Loop

O

at

the

Subtalar

and

Peroneus/fibularis longus: Superficial peroneal nerve (L5 S1) = Peroneus/fibularis brevis: Superficial peroneal nerve (L5 S1) Accessory muscle =

Peroneus/fibularis tertius: Deep peroneal nerve (L4, 5 S1, 2)

e.

Developmental components of interventricular septum of heart

ANSWER

c.

O

(Ref: BDC Human Anatomy, Vol. 2. 7th ed. pg. 315)

occu3rs

Chief muscles

Midgut: Gut loop that extends from the level of Anterior intestinal portal to the Posterior intestinal portal Derivatives of midgut loop: Epithelium of: = Duodenum (Distal to the opening of major duodenal papillae) = Jejunum = Tleum =" Caecum appendix = Ascending colon =" Right 2/3rd of transverse colon Components of extra-hepatic biliary system

foot

=

(Ref: Langman’s Medical Embryology, 13th ed. pg. 239)

O

of

Intertarsal Joints

Derivatives of midgut loop

ANSWER

Eversion

(Ref: Langman’s Medical embryology, 13th ed. pg. 195)

Developmental Components

Interventricular septum divides the ventricular cavity into right and left chambers Components of interventricular septum

O

Muscular part: Ingrowth from the medial walls of the expanding ventricles Membranous part: Two conotruncal swellings (right dorsal and left ventral) fuse with the inferior endocardial cushions

2.

Draw labeled diagram to show the:

ANSWER

O O

Biliary apparatus collects bile from liver and transports it after further modification to the duodenum Divided into intrahepatic and extrahepatic parts.

a.

Microscopic structure of hyaline cartilage

(Ref: IB Singh's Textbook of Human Histology, 8th ed. pg. 57)

ANATOMY

Components of Extra-hepatic Biliary System

PAPER-II

Outer | fibrous

Perichondrium

ANSWER

Capsular matrix oe

Territorial matrix -—————

lsogenous

Interterritorial

matrix

cell nests

Fig. Microanatomy of hyaline cartilage

b.

Microscopic structure of ureter

(Ref: IB Singh's Textbook of Human Histology, 8th ed. pg. 214)

ANSWER

Transitional epithelium Lamina propria

Inner longitudinal muscle layer Outer circular muscle layer Serosa

Fig. Microanatomy of ureter

c.

Posterior relations of left kidney

(Ref: BDC Human Anatomy, Vol. 2. 7th ed. pg. 342)

ANSWER

R 5 |

mae

ais

11th RIB

=

12th RIB Lateral arcuate ligament

Right crus of diaphragm lliohypogastric nerve llioinguinal nerve

Medial arcuate ligament

Psoas major

Quadratus lumborum

ANATOMY

Transversus abdominis

Subcostal vessels and nerve

Fig. Posterior relations of right and left kidneys

PAPER-II Boundaries and contents of popliteal fossa (Ref: BDC Human Anatomy, Vol. 2. 7th ed. pg. 85)

ANSWER

Down’s Syndrome O

ANSWER Sartorius

Vastus lateralis

Gracilis

O

¢

f

| ps f femoris Bice

i mbranosus

Autosomal Recessive

Medial and lateral meniscus are semilunar fibrocartilage partition in knee joint. Anatomical Basis

Defective Planar cell polarity (regulated by primary cilia)

O

v Defective primary ciliary mechanisms

¥ * Increased fluid secretion

* Abnormally increased cyst lining epithelium cell proliferation

2 Cysts

O

knee joint capsule and Tibial collateral ligament attach and fix the medial margin of the medial meniscus - making it relatively immobile. Relative mobility of lateral meniscus is due to attachments of meniscofemoral ligaments and popliteus muscle.

The aforementioned factors lead to high shearing stress on the medial meniscus during flexion and extension of knee joint-making it more prone to injury.

i

2.

ANATOMY

O O

Autosomal Dominant

ANSWER

io

O

Mutations causing defects in the primary cilia (sensory cilia) and cilia associated proteins Polycystinl encoded by PKD1 Se

Your Roll No. ...............

Name of the Paper

:

Anatomy Paper-I

Name ofthe Course

:

MBBS-2013

Semester

:

Annual

Time: 3 Hours

M.M.: 50 INSTRUCTIONS

1.

Write your Roll No. on the top immediately on receipt of this question paper

2.

All questions are to be attempted

3.

Answers to Parts I, II and III should be written in separate answer sheets provided

4.

Attempt parts of a question in sequence

PART-I 1.

Enumerate the following:

a. c. 2.

[10]

Derivative of neural crest Nuclei of special visceral efferent column

b. d.

Tributaries of cavernous sinus of brain Elevators of eye ball with their nerve supply

Draw labelled diagrams to illustrate:

a.

Microscope structure of hypophysi

b. c. d. e.

Microscope Transverse Superficial Boundaries

[10]

structure of compact bone section of medulla oblongata at the level of sensory decussation relations of hyoglossus muscle and contents of mixed salivary gland

PART-II 3.

Write short notes on:

a. c. 4.

[9]

Internal capsule of Brain Interossei of hand

b.

Describe Palatine tonsil under the following heading:

a. b. c. d.

Bell’s palsy [6]

Relations Blood supply Development Applied anatomy

PART-III 5.

Write briefly on: a. b.

6.

[6]

Lateral spinothalamic tract Deltoid muscle

Give the anatomical/embryological basis of: a.

Claw hand

b. c.

Poliomyelitis Hyderocephalus

[9]

PAPER-| |

2013 PAPER-I

PART-I

)

Parts of

subclavian artery

1.

Enumerate the following.

a.

Derivative of neural crest.

Second part

Costocervical trunk

-

Third part

Dorsal scapular

Dorsal scapular

(Ref: Langman’s Medical Embryology, 13th ed. pg. 77)

2.

a.

Draw labelled diagrams to illustrate:

a.

Microscope structure of hypophysis

ANSWER

For answer, refer paper-1 2015, Q. 1 (a), Pg. 48 b.

(Ref: IB Singh's Textbook of Human Histology, 8th ed. pg. 249)

Tributaries of cavernous sinus of brain

(Ref: BDC Human Anatomy, Vol. 3. 7th ed. pg. 199)

ANSWER

For answer, refer 2017 paper-1, Q. 2 (b), Pg. 21

b.

For answer, refer paper-1 2014, Q. 3 (b), Pg. 67 c.

ANSWER

Nuclei of special visceral efferent column

Microscope structure of compact bone

(Ref: IB Singh's Textbook of Human Histology, 8th ed. pg. 66)

(Ref: BDC Human Anatomy, Vol. 4. 7th ed. pg. 49)

ANSWER ANSWER an

Nuclei of Special Visceral/Branchial Efferent Column

O O

Motor nucleus of Trigeminal nerve Nucleus of Facial nerve

O

Nucleus Ambiguus: [Xth, Xth, & XIth



o

ce

Internal (—"*, > circumferen- |

“.

eye

>

-. S*« r * 3

we QA "ee Seee

tial lamellae > © -

Cr ‘

OR

~, a

-——-——

.

Haversian

BZ Sek «8\ canal pee me ~ © *>—Canaliculi ies PA Osteon



Concentric

d.

Elevators of eye ball with their nerve supply (Ref: BDC Human Anatomy, Vol. 3. 7th ed. pg. 215)

lamellae += Lacunae

Volkmann's

containing

canal

Interstitial \Y lamellae jw External circum- |"

ANSWER

osteocytes

TN

as

TR ON

SSF te

ferential lamellae

For answer, refer 2016, paper-1, Q. 1 (d), Pg. 35

Branches of subclavian artery (Ref: BDC Human Anatomy, Vol. 3. 7th ed. pg. 154)

c.

Transverse section of medulla oblongata at the level of sensory decussation (Ref: BDC Human Anatomy, Vol. 4. 7th ed. pg. 94)

ANSWER

Subclavian Artery Branches Parts of subclavian artery

Right

First part

Internal thoracic vertebral artery thyrocervical trunk

ANSWER Left

costocervical trunk

Contd...

d.

Superficial relations of hyoglossus muscle (Ref: BDC Human Anatomy, Vol. 3. 7th ed. pg. 137)

i

Internal thoracic vertebral artery thyrocervical trunk

ANATOMY

For answer, refer 2015 paper-1, Q. 2 (e), Pg. 50

N

e.

Fig. Microanatomy of compact bone

PAPER-I ANSWER Styloid process Stylohyoid muscle Glossopharygeal nerve

Submandibular ganglion Submandibular gland duct

Styloglossus muscle

Mucosa over dorsum of tongue

Stylopharyngeus muscle

Terminal branches of lingual nerve

meu’ AiO

Lingual nerve

Sublingual gland

=

Hypoglossal nerve

Submandibular gland duct (Deep to sublingual gland)

Deep part of submandibular gland

Mandible

Middle constrictor muscle

Genioglossus muscle Lingual artery Geniohyoid muscle

HY)

Annual

Time: 3 Hours

M.M.: 50 INSTRUCTIONS

1.

Write your Roll No. on the top immediately on receipt of this question paper

2.

All questions are to be attempted

3.

Answers to Parts I, II and III should be written in separate answer sheets provided

4.

Attempt parts of a question in sequence

PART-I 1.

Enumerate the following:

a. c. e. 2.

[10]

Contents of spermatic cord b. Structures in the lesser omentum Lateral rotators of the hip joint d. Tributaries of coronary sinus Major openings of thoraco-abdominal diaphragm with their vertebral levels

Draw labelled diagrams to show: a. c. e.

[10]

Microantomy of duodenum Relations of medial surface of right lung Cutaneous nerve supply of the dorsum of foot

b. d.

Microscopic anatomy of ovary Arterial supply of stomach

PART-II 3.

Write short notes on:

a. b. c. 4.

[9]

Development of inter-atrial septum Poplitieus Lymphatic drainage of uterus

Describe porto-Caval anastomisis under following headings: a. b. c.

[6]

Sites Veins participating at each site Clinical manifestation at each site

PART-II 5.

Write briefly on: a. b.

6.

Down’s syndrome Prostatic urethra

Give anatomical/embryological basis of: a. b. c.

[6]

Meckel’s diverticulum Avascular necrosis of head of femur Referred pain over the right shoulder in case of gall stones

[9]

PAPER-II |

2011 PAPER-II

PART-I 1.

Enumerate the following: a.

ANSWER For answer, refer 2018 paper-2, Q. 1 (d), Pg. 13

d.

Contents of spermatic cord (Ref: BDC Human Anatomy Vol 2, 7th ed. pg. 233)

For answer, refer 2018 paper-2, Q. 1 (e), Pg. 13

ANSWER

O

Coronary sinus drains into the right atrium

O

Tributaries of coronary sinus are: =" Great cardiac vein =" Middle cardiac vein

Structures in the lesser omentum

(Ref: BDC Human Anatomy Vol 2, 7th ed. pg. 258)

=" Small cardiac vein

ANSWER Structures in the Lesser Omentum

O

O

Tributaries of coronary sinus

Tributaries of Coronary Sinus

ANSWER

b.

)

=

Posterior vein of left ventricle

=

Oblique vein of left atrium (of marshall)

All have valves at their orifices except oblique vein of left atrium.

Double layered fold of peritoneum that extends from the stomach (Gastrohepatic ligament) and first half of first part of duodenum (Hepatoduodenal ligament) to the porta hepatis and fissure for ligamentum venosum.

ANSWER

Structures in the lesser omentum

For answer, refer 2017 paper-2, Q. 5 (b), Pg. 33

=

Left gastric vessels

2.

=

Right gastric vessels

= Anterior vagus nerve and branches =

Posterior vagus nerve and branches

=

Gastric lymph nodes and lymphatics

=

Right free margin contents:

c.

*

Portal vein

* * *

Hepatic Artery Proper Bile duct Lymphatics

*

Autonomic

*

Plexus of nerves

Lateral rotators of the hip joint (Ref: BDC Human Anatomy Vol 2, 7th ed. pg. 150)

e.

Major openings of thoraco-abdominal diaphragm with their vertebral levels

Draw labelled diagrams to show: a.

Microanatomy of duodenum

(Ref: IB Singh’s Textbook of Human Histology 8th ed. pg. 187)

ANSWER For answer, refer 2014 paper-2, Q. 2 (b), Pg. 71 b.

Microscopic anatomy of ovary

(Ref: IB Singh’s Textbook of Human Histology 8th ed. pg. 233) ANSWER For answer, refer 2018 paper 2, Q. 2 (a), Pg. 13

c.

Relations of medial surface of right lung

>

= O

bo

a at

Whit [ 3

LA Mir

ms

ceniie —_ Posterior

atlanto-

occipital

|

membrane

Vertebral artery

= Frontal nerve = Lacrimal nerve =" Trochlear nerve

Dorsal ramus C1 spinal nerve Rectus capitis posterior major Inferior oblique

=

Capitis

e.

Recurrent meningeal branch of ophthalmic artery Features of medial wall of tympanic cavity

Fig. Suboccipital triangle and its contents

PAPER-| d.

Transverse section through lower part of pons (Ref: BDC Human Anatomy Vol 4, 7th ed. pg. 97)

Roots

Parasympathetic Lacrimatory nucleus

,

ANSWER For answer, refer 2017 paper-1, Q. 2 (c), Pg. 21

e.

Nervus intermedius

Sympathetic plexus

i

around internal

carotid artery

Lateral derivatives of pharyngeal pouches

Facial nerve

'

ANSWER

Geniculate ganglion (no synapse)

.

Lateral lingual swellings Tuberculum impar

Eustachian

tube

Greater petrosal nerve it 4

Tympanic

Tympanic cavity

membrane External

¥ Deep petrosal nerve

Nerve of pterygoid canal/vidian nerve

acoustic meatus

Palatine

Foramen caecum

tonsil

Inferior parathyroid

Pterygopalatine ganglion (Synapse)

Ectoderm Copula/

:

Thymus

Endoderm

parathyroid gland Ultimobranchial body- c cells

-

Zygomatic branch

Cervical sinus

Epiglottal swelling

‘ Zygomaticotemporal branch i

Laryngeal orifice

* Palatine glands * Pharyngeal glands ¢ Nasal glands (Cell of origin separate from lacrimatory nucleus)

Lacrimal gland

Fig. Derivatives of pharyngeal pouches

PART-II

——>) and nasal branches

:

eminence

Superior

Palatine, pharyngeal

Maxillary nerve

hypobranchial

gland

'

Sympathetic Superior cervical sympathetic ganglion

3.

.

Write short notes on:

a.

Sympathetic plexus around internal carotid artery

Supination and pronation of forearm

:

Deep petrosal nerve

ANSWER



For answer, refer 2018 paper-1, Q. 5 (a), Pg. 9 b.

Nerve to pterygoid canal

.

Pterygo- palatine ganglion

Pterygopalatine ganglion (No synapse)

.

ANSWER

O

O

Flat, reddish peripheral parasympathetic ganglion (Largest). Location: Within the pterygopalatine fossa, near sphenopalatine foramen, in front of pterygoid canal and foramen rotundum. Topographically connected to the maxillary nerve but functionally related to facial nerve.

Sensory

Sensory fibers distributed by following maxillary nerve branches pass through the ganglion without relay: O

Orbital

O

Nasopalatine

ANATOMY

O

Supply vasculature and orbitalis muscle via branches of maxillary nerve

PAPER-I

2008 O

Superior alveolar

O

Palatine

O

Pharyngeal c.

ANSWER

Pharyngeal Constrictors O

Arachnoid granulations

Attachments:

ANSWER Arachnoid Granulations O

Focal outpouchings of the arachnoid mater into the dural venous sinus.

O

Present in superior sagittal, Petrosal and straight sinuses.

transverse,

Superior

O

OO

Granulations are different from arachnoid villi

Villi can only be seen under microscope in the fetus and new born infant.

O

Granulations can be seen with naked eye from 18 months ofage - become more lobulated and complex with advancing age with patches of calcification (known as Pacchionian bodies).

Attachments: = Anteriorly: *

Thyropharyngeus: Arises from oblique line of thyroid cartilage lamina and inferior cornu * Cricopharygeus: Arises from side of the cricoid cartilage * Cricopharygeus consists of two parts: Upper oblique (pars oblique) and lower deep parts (pars fundiformis). =" Posteriorly: * Thyropharyngeus: Median raphe * Cricopharygeus: 6 Upper oblique (pars oblique)-Median raphe 6 Lower deep parts (pars fundiformis)forms a continuous circular band without intervening raphe.

Each granulation consists of a globular structure with a narrow neck.

= The neck penetrates through the dural lining of the venous sinus. = The arachnoid mater that projects into the core of the granulation through the neck is modified to form trabeculae and interwoven channels. = The arachnoid core is surmounted by a cap of arachnoid cells. The region of the cap is covered directly by the endothelium of the sinus. = The remaining part of the arachnoid granulation has a covering of duramater intervening between the endothelium of sinus and arachnoid mater known as dural cupola.

Longitudinal Muscles of Pharynx

Function-not completely understood.

O

ANATOMY

a.

Origin

b.

Insertion

c.

Nerve supply

d.

Action

of

pharynx

=" Posteriorly: Pass backwards to get inserted into the median pharyngeal raphe Inferior constrictor = ‘Thickest =" Two parts: Thyropharyngeus & Cricopharyngeus

Structure of Arachnoid granulation:

Describe the muscle following headings:

= Anteriorly: Pterygoid hamulus, posterior border of pterygomandibular raphe, and posterior end of mylohyoid line =" Posteriorly: Curve posteriorly to get inserted into the median pharyngeal raphe (attached above to the pharyngeal tubercle) Middle constrictor Attachments: = Anteriorly: Lesser cornu of hyoid bone, lower part of stylohyoid ligament and to entire upper border of greater cornu of hyoid bone

With advancing age their size and number increase - present as filling defects in the venous sinus on angiograms and CECT scans.

=

Superior constrictor

under

the

(Ref: BDC Human Anatomy Vol 3, 7th edition pg 241)

Palatopharyngeus: = Lies below palatopharyngeal arch. = Attachments: * Two fasciculi separated by levator veli palatini attach to the superior surface of the palatine aponeurosis. * Unite posterolaterally and descend to get inserted into the posterior border of thyroid cartilage. = Passavant’s muscle/ Palatopharyngeal sphincterComprised of upper fibers of palatopharyngeus (which encircle the lumen) and superior constrictor.

PAPER-| ANSWER Ulnar Nerve in Hand O

= At wrist-ulnar nerve passes below the superficial part of Flexor retinaculum (in Guyon’s Canal) along with ulnar artery. = Itthen divides into Superficial and Deep Terminal branches. O

O O

Pharyngeal plexus: All constrictors and longitudinal muscles of pharynx except Stylopharyngeus, which is supplied by Glossopharyngeal nerve. Cricopharyngeus additionally supplied by recurrent laryngeal and external laryngeal nerves.

Superior, middle and thyropharyngeus constrict upper, middle and lower parts of pharynx during deglutition. Cricopharyngeus-pharyngo-oesophageal sphincter. Longitudinal muscles acting together elevate the larynx and pharynx. Palatopharyngeus approximates the corresponding arches. Salpingopharyngeus helps to open the cartilaginous part of eustachian tube in conjunction with Tensor veli palatini.

Applied Anatomy O

Killian’s triangle: = Old concept: Between the thyropharyngeus and cricopharyngeus = Recent observations: Between the upper oblique and lower deep parts of cricopharyngeus. Laimer’s triangle: Beneath cricopharyngeus between the longitudinal fibers of oesophagus (laterally attach to side of cricoid cartilage) Both Killian’s Triangle and Laimer’s triangle-are sites of pharyngeal pulsion diverticula.

PART-III 5.

Dorsal branch of ulnar nerve: *

Actions O

Branches: "

Nerve Supply O

Course:

O

Ulnar nerve in the hand (Ref: BDC Human Anatomy Vol 1, 7th ed. pg. 127)

5

cm

proximal

to

wrist,

passes

dorsally and distally deep to Flexor carpi ulnaris and then descends along the medial aspect of the dorsum of wrist and hand. * It gives 2 or 3 dorsal digital nerves for the ring and little fingers. * Dorsal digital nerve for little finger supply till base of distal phalanx and for ring finger supply till base of middle phalanx. * The remaining distal portions the dorsal branches from the proper palmar digital branch of ulnar nerve. =" Superficial terminal branch: * Supplies Palmaris brevis and medial palmar skin. * Divides into 2 palmar digital nerves. * One palmar digital nerve supplies medial side of little finger and other is a common digital nerve supplies adjoining aspects of little and ring fingers. =" Deep terminal branch: * Runs along with deep branch of ulnar artery. * Supplies: 5 Deep head of flexor pollicis brevis 5 Adductor pollicis 5 Occasionally a branch to opponens pollicis 56 Three hypothenar muscles-abductor digiti minimi, flexor digiti minimi, and opponens digiti minimi. 6 All palmar and dorsal interossei 0 Third and fourth lumbricals Applied anatomy: =" Ulnar tunnel syndrome: Entrapmentneuropathy of ulnar nerve in the guyon’s canal. =" Ulnar nerve division at the wrist: *¢ All above mentioned muscles are paralysed. * It leads to clawing of the hand due to the action of long extensors, and flexors, causing,

Write briefly on:

a.

Originates

*

extension of metacarpophalangeal joint and flexion at interphalangeal joints. Clawing less intense in the index and middle fingers as the first two lumbricals supplied by median nerve.

ANATOMY

=" Contraction of Passavant’s muscle leads to elevation of a Passavant’s ridge which approximates the nasopharyngeal isthmus. Salpingopharyngeus: = Attachments: * Inferior part of cartilaginous portion of Eustachian tube. * Descends to blend with palatopharyngeus. Stylopharyngeus Attachments: = From medial aspect of base of styloid process =" Descends along the pharyngeal wall to enter the interval between the superior and middle constrictors. = Few fibers merge with the constrictors and rest join palatopharyngeus to attach to the posterior border of thyroid cartilage.

137

PAPER-| ANSWER Ulnar Nerve in Hand O

= At wrist-ulnar nerve passes below the superficial part of Flexor retinaculum (in Guyon’s Canal) along with ulnar artery. = Itthen divides into Superficial and Deep Terminal branches. O

O O

Pharyngeal plexus: All constrictors and longitudinal muscles of pharynx except Stylopharyngeus, which is supplied by Glossopharyngeal nerve. Cricopharyngeus additionally supplied by recurrent laryngeal and external laryngeal nerves.

Superior, middle and thyropharyngeus constrict upper, middle and lower parts of pharynx during deglutition. Cricopharyngeus-pharyngo-oesophageal sphincter. Longitudinal muscles acting together elevate the larynx and pharynx. Palatopharyngeus approximates the corresponding arches. Salpingopharyngeus helps to open the cartilaginous part of eustachian tube in conjunction with Tensor veli palatini.

Applied Anatomy O

Killian’s triangle: = Old concept: Between the thyropharyngeus and cricopharyngeus = Recent observations: Between the upper oblique and lower deep parts of cricopharyngeus. Laimer’s triangle: Beneath cricopharyngeus between the longitudinal fibers of oesophagus (laterally attach to side of cricoid cartilage) Both Killian’s Triangle and Laimer’s triangle-are sites of pharyngeal pulsion diverticula.

PART-III 5.

Dorsal branch of ulnar nerve: *

Actions O

Branches: "

Nerve Supply O

Course:

O

Ulnar nerve in the hand (Ref: BDC Human Anatomy Vol 1, 7th ed. pg. 127)

5

cm

proximal

to

wrist,

passes

dorsally and distally deep to Flexor carpi ulnaris and then descends along the medial aspect of the dorsum of wrist and hand. * It gives 2 or 3 dorsal digital nerves for the ring and little fingers. * Dorsal digital nerve for little finger supply till base of distal phalanx and for ring finger supply till base of middle phalanx. * The remaining distal portions the dorsal branches from the proper palmar digital branch of ulnar nerve. =" Superficial terminal branch: * Supplies Palmaris brevis and medial palmar skin. * Divides into 2 palmar digital nerves. * One palmar digital nerve supplies medial side of little finger and other is a common digital nerve supplies adjoining aspects of little and ring fingers. =" Deep terminal branch: * Runs along with deep branch of ulnar artery. * Supplies: 5 Deep head of flexor pollicis brevis 5 Adductor pollicis 5 Occasionally a branch to opponens pollicis 56 Three hypothenar muscles-abductor digiti minimi, flexor digiti minimi, and opponens digiti minimi. 6 All palmar and dorsal interossei 0 Third and fourth lumbricals Applied anatomy: =" Ulnar tunnel syndrome: Entrapmentneuropathy of ulnar nerve in the guyon’s canal. =" Ulnar nerve division at the wrist: *¢ All above mentioned muscles are paralysed. * It leads to clawing of the hand due to the action of long extensors, and flexors, causing,

Write briefly on:

a.

Originates

*

extension of metacarpophalangeal joint and flexion at interphalangeal joints. Clawing less intense in the index and middle fingers as the first two lumbricals supplied by median nerve.

ANATOMY

=" Contraction of Passavant’s muscle leads to elevation of a Passavant’s ridge which approximates the nasopharyngeal isthmus. Salpingopharyngeus: = Attachments: * Inferior part of cartilaginous portion of Eustachian tube. * Descends to blend with palatopharyngeus. Stylopharyngeus Attachments: = From medial aspect of base of styloid process =" Descends along the pharyngeal wall to enter the interval between the superior and middle constrictors. = Few fibers merge with the constrictors and rest join palatopharyngeus to attach to the posterior border of thyroid cartilage.

137

PAPER-I

2008 b.

6.

Scalenus anterior muscle

(Ref: BDC Human Anatomy, Vol. 2, 7th ed. pg. 172)

Give anatomical/embryological basis of a.

Dangerous area of face (Ref: BDC Human Anatomy, Vol. 3, 7th ed. pg. 72)

ANSWER ANSWER

Scalenus Anterior Muscle O

Location: Posteromedial to Sternocleidomastoid in the side of the neck.

O

Attachments:

O

=" Origin: Tendinous fascicles from the Anterior tubercles of the transverse processes of the C 3-6. = Insertion: Converges and descends vertically to get attached to the Scalene tubercle on the medial border of the first rib (anterior to groove for Subclavian artery). Relations: ="

Anterior: Clavicle, Subclavius, Sternocleidomas-

toid,

Omohyoid,

lateral part of carotid

sheath,

transverse cervical, Suprascapular and ascending cervical

O O

O

Subclavian

vein,

Prevertebral

fascia and Phrenic nerve. =" Posterior: Suprapleural membrane, Pleural, roots of brachial plexus and Subclavian artery. =" Medial border: Below Cé attachment separated from Longus colli by a space which contains Vertebral vessels, sympathetic trunk, and its cervicothoracic ganglion. On the left Thoracic duct crosses this space. = Lateral: Brachial plexus Nerve Supply: Ventral branches of C 4-6 spinal nerves. Actions: Contracting bilaterally causes flexion of the cervical part of the vertebral column forwards and contracting unilaterally rotates the vertebral column to the opposite side. Elevates the first rib acting as accessory muscle in respiration. Applied anatomy = Scalenus anterior syndrome*

ANATOMY

arteries,

Compression of the thoracic outlet structures by muscular abnormalities in scalenus anterior muscle.

Dangerous area includes-medial canthus of eye, nose, and upper lip. These areas drained by Facial vein. Anatomical Basis

O

O

Facial vein has following connections: =" With Cavernous sinus: Via Superior Ophthalmic vein. =" With Pterygoid venous plexus: Via Inferior ophthalmic vein and Deep Facial vein. (Which in turn is also connected to cavernous sinus). Due to the above valve: Less connections, any infection in these regions may enter cavernous veins leading to Facial thrombophlebitis and subsequently cavernous thrombophlebitis. b.

Horner’s syndrome

ANSWER

For answer, refer 2015 paper-1, Q. 6 (c), Pg. 55 c.

Crutch palsy

ANSWER

Paralysis caused by compression from improper or illfitting crutches is known as crutch paralysis. Anatomical Basis

Leads to compression of radial nerve (most commonly affected) leading to wrist drop and sensory loss in the region of the nerve. Loss of extension at elbow depends upon which place the compression occurs (Axilla or radial groove).

Your Roll No. ...............

Name of the Paper

:

Anatomy Paper-Il

Name ofthe Course

:

MBBS-2008

Semester

:

Annual

Time: 3 Hours

M.M.: 50 INSTRUCTIONS

1.

Write your Roll No. on the top immediately on receipt of this question paper

2.

All questions are to be attempted

3.

Answers to Parts I, II and III should be written in separate answer sheets provided

4.

Attempt parts of a question in sequence

PART-I 1.

Enumerate the following:

a. b. c. d. e. 2.

[10]

Tributaries of azygos vein Branches of splenic artery Posterior relations of the head of pancreas Muscles producing inversion and eversion of foot Structures passing through the greater sciatic foramen

Draw labelled diagrams to show:

a. c. e.

[10]

Microanatomy of the large artery Major positions of vermiform appendix Cutaneous innervation of foot

b. d.

Openings in the diaphragm Microanatomy of epididymis

PART-II 3.

Write short notes on: a. b. c.

4.

[9]

Prostatic urethra Root of the mesentery Ihiotibial tract

Describe uterine tube under the following headings: a. b. c.

[6]

Gross anatomy Development Applied anatomy

PART-III 5.

Write briefly on:

a. b. 6.

[6]

Turner’s syndrome Menisci of the knee joint

Give anatomical/embryological basis of:

a.

Caput medusa

b.

Coarctation of dorsal aorta

c.

Liver biopsy

[9]

PAPER-II

2008 |

2008 PAPER-II

PART-I 1.

Enumeratethefollowing a.

O 9000000 0

Origintributaries: = Lumbar Azygos vein = Union of right ascending lumbar and subcostal veins Right superior intercostal vein - from 2-3 right posterior intercostal veins 5-11 right posterior intercostal veins Mediastinal veins Oesophageal veins Pericardial veins Right bronchial vein Hemiazygos vein Accessory hemiazygos vein Branches of splenic artery

ANSWER

Splenic Artery Branch of Coeliac Trunk from Abdominal Aorta Branches of Splenic Artery

O

O

O

O

Pancreatic branches:

= Dorsal pancreatic artery =" Greater Pancreatic artery = Arteries to the tail of pancreas Short gastric arteries Left gastroepiploic artery Posterior Gastric artery (40%) c.

d.

Muscles producing inversion and eversion of foot

ANSWER

Tributaries of Azygos Vein

b.

Renal veins Termination of right gonadal vein

Tributaries of azygos vein

ANSWER

O

O O

Posterior relations of the head of pancreas (Ref: BDC Human Anatomy Vol 2, 7th ed. pg. 327)

For answer, refer 2015 paper-2, Q. 3 (a), Pg. 60 e.

Structures passing through the greater sciatic foramen

ANSWER

Structures Passing Through Greater Sciatic Foramen O

Piriformis

Above Piriformis

O

Superior gluteal nerve

O

Superior gluteal vessels

Below Piriformis

O

Inferior gluteal nerve Inferior gluteal vessels

O

Sciatic nerve

O O

Posterior cutaneous nerve of thigh Nerve to quadratus femoris

O

Nerve to obturator internus

O

Internal pudendal vessels

O

Pudendal nerve

2.

Draw labelled diagrams to show:

a.

Microanatomy of the large artery

ANSWER Simple squamous/ Tunica

endothelium

intima

. Tunica >

= O

bh

© Crossing over

L

Chiasma

oe)

O > tT QO

Contd...

PAPER-II Alignment and disjunction Centromeres do not split

Secondary spermatocyte (23 duplicated chromosomes, 2N)

Meiosis II

Alignment and disjunction Centromeres split

A

Cell division

Cell division

Spermatids (23 single chromosomes, 1N)

Spermiogenesis




(

>

Normal human eye

ANSWER O

Near sightedness or inability to see far objects Image is focused before retina

O O

Far point less then 1 meter in serene myopia Near point becomes nearer

O O O

) -------

0 e----..Q

fej ~nneancnaan ©

.

Rangeofaccommodation decreases Presbyopia gets corrected in old age Error can be corrected by concave lens

sss

L

(

>

a

@------- 6 - oe

a

ey >» oe

nn nnn ——oe Correction for myopic eye

>

>\

Myopia

Anterior chamber

©

4 Myopic eye

Normal vision

>

>

Sclera Chorold

Cornea Pupli

Focal point

Light

Central \ |

Anterior chamber

|

Sclera Chorold

S

Cornea Pupli

Focal point

Light

Central

retinal artery

retinal artery

O oe)

> tT a

lris Zonules

9

Optic nerve

lris Zonules

a

Central Ciliary

body

Retina

retinal vein

Central Ciliary

body

Retina

retinal vein

Optic nerve

PAPER-II c.

2018

Regulation of cortisol secretion (Ref: AK Jain, Vol. 2, Chap. 74, pg. 725-726)

O

Chronaxie:Itis the duration required bythe stimulus double the rheobase to produce contraction.

PART-II

ANSWER Stress, low blood glucose \ CRH

3.

\

Enumerate the hormones which regulate blood calcium. Discuss the regulation of blood calcium and add a note on tetany:

i aiaaaaiatalla

(Ref: AK Jain, Vol. 2, 7th ed. pg. 715-717) ANSWER

Hypothalamo-

hypophysial p ortal y system

O

PuSLOT PHO Rey

Parathyroid hormone, calcitonin . = The hormone responsible for

.

increase

.

in the

plasma concentration of calcium is Parathyroid Hormone =" Increase calcium uptake from intestine and reabsorption from kidney occurs from Vitamin D assisted by PTH " Decrease in the plasma calcium levels occurs by

Cortisol

Adrenal cortex (zona fasciculata)

calcitonin

Regulation of Blood Calcium Stimulatory}!

Calcitonin

—> Inhibitory || * Increases blood glucose

O

¢ Anti-inflammatory effects

Strength duration curve

d.

(Ref: AK Jain, 7th ed. Vol. 2, pg. 145) ANSWER

-

90000 0

—+

Target tissues: - Increases fat and protein breakdown

Lowers plasma calcium No deficiency disease

Bone calcium storage

Inhibits osteoclasts

Teeth and skeletal development Decreases gastric motility; increases gastric secretion Inhibits renal 1 alfa hydroxylase enzyme Stimulates osteoblasts

Calcium Homeostasis

= 100 -

90 -

TT =|

80

=

ro

5 ”

6050-

PTH promotes and calcitonin inhibits Ca* release in the

Ca™ removed from blood by

F =

40 o>

blood by osteoclasts

osteoblast

~

20

=

!



7

' | 2x rheobase

a

! |

200

Ca” reabsorption P from the urine 400

300

current

required

to

OO

))

cause

Unabsorbed Ca” Ingested

500

Ca”

lost in the feces

e

Strength duration curve was given by louis Lapicque Curve that is obtained by plotting stimulus strength on y axis and stimulus duration on x axis Rheobase: Measure of membrane potential i.e. minimum

Blood

PTH promotes

Pulse duration (usec)

contraction

of

muscle.

O

ey

=

Chronaxie

O

Bone

-

0

):

As strength of the stimulus increase the duration for the application decreases

ney

>

\

©

= O

oe)

Ca”

lost inl

PTH

a

promotes

active vitamin

the urinel formation

D

> tT

6

Kamin

promotes Ca

absorption

4

. Small

QO

intestine ee

.. 1 ele! ¢ ~)

PAPER-II Parathyroid Hormone O

O O.

O

Increase movement of calcium from bone fluid into

Increase intestinal absorption of calcium through

extra cellular fluid through functional membrane of

TRPV 6 receptors

bone.

PTH stimulated calbindin (calcium binding protein) production in intestinal cells Increase calcium reabsorption from kidney by TRPV

Tetany

9 receptors

Oo O.

Increase in serum calcium Increase activity of osteoclasts

4.

What will happen and why? a.

O

Decreased calcium levels decrease the threshold for

O. O

membrane excitability Increase in the irritability of nerves Increased neuronal excitability causes features of tetany

Tovision in right optic tract lesion

(Ref: AK Jain, Vol. 2, pg. 1094)

ANSWER Visual Field Deffects “wi

Se

hah

Ph

Lesion before optic chiasm 1. Monocular visual loss = lesion at

Left eye

Right eye

Ops TISEKS

\

2. Bitemporal hemianopia = lesion at optic chiasm

Bitemporal hemianopia

Monocular

. \|



Upper 5 homonymous

q>

Lesion after optic chiasm

Homonymous

3. Homonymous hemianopia = lesion

hemianopia

at right optic tract

4. Lower homonymous quadrantanopia = lesion at left arietal radiation - Meyer's loo

homonymous quadrantanopia

oak a 5. Upper homonymous

CP

ae

,

P

quadrantanopia = lesion at left temporal radation

6

6. Homonymous hemianopia with central sparing = lesion at right occipital pole

q)

Homonymous hemianopia with central sparing

b.

Ifintake of iodide exceeds 2 mg/day

c.

(RefL AK Jain, Vol. 2, pg. 694)

To hearing if an individual excessively loud sound

is exposed

to

(Ref: AK Jain, Vol. 2, Chap. 109, pg. 1070, 1076 >

©

L O

ANSWER

O O

oe)

> tT OW

O

Normal requirement of iodine is 100-200 pg/day Increase iodine intake inhibits NIS pump (sodium iodine symporter)

ANSWER

O

Acoustic reflex or Tympanic Reflex

O

Sudden loud sound causes contraction of the tensor

O

tympani muscle of tympanic membrane Prevents injury to the internal ear by loud noise.

Wolff-Chaikoff Effect

=

Inhibitory effect of iodine lasts for 1-2 weeks, then it wears off.

PAPER-II d.

Iftestis fails to descend

O

(Ref: AK Jain Vol. 2 pg. 797-798)

Increase catecholamines

c.

Myasthenia gravis (Ref: AK Jain, Vol. 2, 7th ed. pg. 160)

Cryptorchidism

ANSWER

Testes in inguinal canal or abdomen Spermatogenesis absent Infantile seminiferous tubules Associated malignant testicular tumors

PART-III 5.

90000 0

O00 0

O

ANSWER

Write short notes on the following: a.

Fatigue

Disorder of neuromuscular junction Ptosis, diplopia Slurred speech Antibodies against Ach receptors on post synaptic membrane

Parkinson’s disease (Ref: AK Jain, Vol. 2, 7th ed. pg. 999-1002)

6.

Give physiological/clinical following: a.

ANSWER

of the

Oral contraceptive pills

Flexion attitude Rigidity of proximal muscles, cog wheel rigidity Mask like face

ANSWER O

Festinant gait

Pill rolling movement of fingers Coarse tremors Akinesia/bradykinesia b.

significance

(Ref: A K Jain, Vol. 2, 7th ed. pg. 821)

Role of hypothalamus in regulation of body temperature

000 0

9000dD0Wd6LDDdLD

Weakness of skeletal muscle

(Ref: AK Jain, Vol. 1, pg. 591)

Inhibits LH

Prevents ovulation-anovulatory cycle Withdrawal bleed Thickened endometrium and cervical mucous

Types: Combined, sequential, estrogen, minipills, morning after pills and progesterone antagonists b.

Human chorionic gonadotropin (Ref: AK Jain, Vol. 2, 7th ed. pg. 826-827)

ANSWER

O OO O

Cutaneous vasodilation Sweating

ANSWER

Increase in respiration Decrease in BMR

Earliest hormone that appears after fertilization Helps in confirmation of pregnancy/pregnancy tests Referred pain (Ref: AK Jain, Vol. 2, 7th ed. pg. 904-906)

Decrease sympathetic activity

Muscular apathy

ANSWER

Decrease TSH secretion

Shivering Hunger Increase voluntary activity Increase TSH

OO 0 O

OO O

Luteinizing by function

Anorexia

Posterior hypothalamus is stimulated by cold and causes increase in body heat by the following thermoregulatory responses O

Secreted by syncytiotrophoblast

c.

O

9000980860 OQ

Anterior hypothalamus is stimulated by heat and causes decrease in body temperature by following thermoregulatory mechanisms:

Injury or pathology at one site, pain felt on other site

>

Cardiac pain felt in left arm Gallbladder pain felt on tip of right shoulder

©

Dermatomal rule: Structures developing from same embryonic segment Convergence theory: Nerves that develop from the same embryonic segment, converge at same spinothalamic neurons (law of projection)

O

L

oe)

> tT QO

PAPER-II Facilitation theory: Stimulation of afferent neurons from viscera causes subliminal fringe in somatic neurons carrying sensation in the same spinothalamic nerve fibers. Central pain inhibition: Segmental suppression: Gate control theory, stimulation of A beta fibers causes presynaptic inhibition of A delta fibers and C fibers. E.g. TENS, ice pack or hot fomentation, counterirritants.

Supraspinal suppression:

Raphe magnus nucleus (RMN) in lower pons and medulla has serotonergic neurons that release

>

©

L O oe)

> tT OW

5HT and substance P that descend down

and

cause post synaptic inhibition of dorsal neurons. Periaqueductal

grey

area

in

midbrain

has

opioid receptors which in response to opioids and

natural

endorphins,

enkephalins

produce

analgesia. PAG also stimulate RMN Antinociception

induced

by adenosine

as pre

and post synaptic inhibition Stimulation of noradrenergic inhibitory system Analgesic effect of cholinergic drug epibatidine via nicotinic.

Your Roll No................

Name ofthe Paper

:

Physiology Paper-I

Name ofthe Course

:

MBBS-2017

Semester

Annual

Time: 3 Hours

M.M.: 50

INSTRUCTIONS Write your Roll No. on the top immediately on receipt of this question paper All questions are to be attempted

Attempt parts of a question in sequence

PART-I Define renal clearance. How renal clearance can be used to determine glomerular filtration rate? Discuss the determinants of glomerular filtration rate. [3]

ao

SF

Draw labeled diagram/flow chart of the following:

[8]

Spirogram showing various lung volumes and capacities Enterohepatic circulation of bile salts Left ventricular pressure and volume changes during a normal cardiac cycle Mechanism of glucose absorption across intestinal epithelium

PART-II [8]

What will happen and why?

[8]

fF PS

Define blood pressure. Describe short term regulation of blood pressure. Add a note on Bainbridge reflex.

ao

a

Answers to Parts I, II and III should be written in separate answer sheets provided

If NSAIDs are given to a patient having compromised renal circulation To A-V PO, difference in stagnant hypoxia When there is deficiency of clotting factor VIII To intestinal motility when sympathetic discharge increases

PART-III Write short notes on the following:

a. b. c.

[9]

Hemostasis Factors affecting vital capacity Molecular motors

Give physiological/clinical significance of the following: a.

Cross-matching

b. c.

Law of Laplace Oncotic pressure

[9]

al

2017 PAPER Define renal clearance. How renal clearance can be used to determine glomerular filtration rate? Discuss the determinants of glomerular filtration rate.

(Ref: Textbook of Physiology, AK Jain, Vol. 1, 7th ed. pg. 541-542)

[

=" Thus in inulin volume of plasma completely cleared equals volume of plasma filtered per unit time.

PART-I

O

Clearance of inulin or GFR = U (urine concentration

of Inulin) x V (urine concentration of Inulin) O

flow

rate)/P

(plasma

Clearance of creatinine can also be used to measure GER.

Determinants of GFR

ANSWER

O00 0

1.

PAPER-I

O

Filtration Coefficient Kf: product of glomerular capillary membrane permeability and its filtration area. Hydrostatic pressure in Bowman’s space fluid Hydrostatic pressure in glomerular capillary Osmotic pressure in glomerular capillary Sympathetic stimulation

2.

Drawlabeled diagram/flow chart of the following:

Clearance

It is ability

of kidney

to

clear

plasma

of various

substances. Thus, “The renal clearance can be defined

as the volume of plasma that is cleared of a substance in one minute by excretion of the substance in urine.” Unit: mL/min C = UV/P, where

C = Renal clearance U = Urine concentration of the substance

a.

V = Rate of flow of urine P = Plasma concentration of substance.

Glomerular markers: Clearance = GFR (those substances which neither reabsorbed nor secreted)

O

Renal clearance < GFR (substances that are partially reabsorbed)

O

Lowest

clearance

e.g.

Glucose,

5

Na,

OO O >

—| Tidal

amino

L O oe)

> tT a

Capacity

(VC)

{\,

capacity

(te)

§ LExpiratory reserve

S|

volume _

(ERV) ‘

Functional

Residual volume (RV)

residual capacity (FRC)

wy

Measures GFR

Measures capacity of tubular secretion

,

Y

Time (S)

Measures renal plasma flow and renal blood flow Measures

osmotic

clearance

and

free

Fig. Spirogram

water

Static Lung Volume

Measurement of GFR

O

t

\/\ S| volume YY) —

Highest Renal Clearance (filtered and secreted, also represent range of blood flow) e.g. PAH, Diotrast.

clearance

©

Inspirat Vita]

Functions of Renal Clearance

O

volumes

A

Inspiratory __ reserve volume

acids, Clion and HCO,

O

lung

ANSWER

e.g. Inulin, urea, creatinine

Renal

various

(Ref: Textbook of Physiology, AK Jain, Vol. 1, 7th ed. pg. 415)

Points To Remember

O

Spirogram showing and capacities

O

Tidal volume: The amount of air that either moves into or out of lung in quiet respiration, 500-750 mL

O

Inspiratory reserve volume (IRV): The maximum air that can be expired over tidal volume with maximal inspiratory effort. It is approximately 3L (2-3.2L)

Inulin clearance test: Inulin is freely filtered, inert, non-toxic,

not stored or metabolized

and

can be

easily estimated. ="

Give iv bolus dose, then continuous iv infusion so

the rate of infusion equals loss in urine.

PAPER-| O

Expiratory reserve volume (ERV): It is volume of air that can be expired maximally after passive respiration. Approx. 1.2L

Residual volume (RV) approx. 1.3L: Itisthe amount of air left in the lungs after maximal expiratory effort.

O

Once in the liver, they are taken up from the blood using Na* taurocholate co-transporting polypeptide (NTCP) and organic anion transport proteins (OATPs).

O

At the canaliculus, bile acids are secreted through bile salt excretory pump (BSEP) and multidrug resistance protein 2 (MRP2).

It is approx. 1.3 L

Closing volume: It is the volume of air at which lowermost alveolar airways begins to close during expiration.

Red Blood Cells

7

Hemoglobin

Static Lung Capacity O

y Biliverdin

Inspiratory capacity (IC): IRV + TV. It is the maximum volume of the air that can be inspired after normal tidal volume. Normal volume approx.

Bilirubin

3.5L

Bilirubin-albumin

Functional residual capacity: It is the volume of air present inside the lung after normal expiration. ERV + RV. Approx. 2.5L Vital lung capacity: It is the maximum amount of air expired after maximal inspiratory effort. Approx. 4.7 L, IRV+TV +ERV Total lung capacity: is the total amount of air present inside the lung, i.e. ERV+TV+IRV+RV, approx. 5.9L Closing capacity: RV+ CV. It is the volume of air that remains in the lung when lowermost alveoli are closed.

Reticuloendothelial system

Bilirubin UDP glucuronyl

4 EXCITONS

—————-_

manne —

Enterohepatic circulation

«--——

transferase Conjugated

Bloodstream

_

Liver

bilirubin

Conjugated bilirubin

Bile

Conjugated bilirubin

Small intestine

1 Urobilinogen

Terminal ileum

Point to Remember

Lung volumes spirometry

and

capacities

that

cannot

be

measured

Urobilin stercobilin

by

| Excreted in feces

e Residual volume (RV) e Functional residual capacity (FRC) e Total lung capacity (TLC)

Fig: Bilirubin metabolism. UDP = uridine diphosphate

c.

They are usually measured by: e Helium dilution technique e Nitrogen washout technique e Body plethysmography

b.

Enterohepatic circulation of bile salts.

Colon

Left ventricular pressure and volume changes during a normal cardiac cycle

(Ref: Textbook of Physiology, AK Jain, Vol. 1, 7th ed. pg. 289)

ANSWER

(Ref: Textbook of Physiology, AK Jain, Vol. 1, 7th ed. pg. 244) Ventricular Pressure-Volume Loops ANSWER

O

Are constructed by combining systolic and diastolic pressure curves.

Enterohepatic Circulation

O

The diastolic pressure curve is the relationship between diastolic pressure and diastolic volume in the ventricle.

O

The systolic pressure curve is the corresponding relationship between systolic pressure and systolic volume in the ventricle.

O

Most of the bile acids are absorbed from the intestine and travel through the portal blood to the liver, where they are taken up by the hepatocyte and re-secreted-“Enterohepatic circulation” In the intestine, they are absorbed actively in ileum

using Na* - dependent transport proteins. This ileal transport is highly efficient delivering more than 90% of the bile acids to portal blood.

O

A single left ventricular cycle of contraction, ejection, relaxation, and refilling can be visualized by combining the two curves into a pressure-volume loop.

>

©

L O

oe)

> tT QO

PAPER-I

Left ventricular pressure

ANSWER 150 _=

Absorption of Carbohydrates 3

a

I =

— 75-

intestine

2

|

i

Na’

Glucose or

A

0

T

T

0

4L

T

50

T

>—

T

100



1

T

150

Secondary A active

*%_ Facilitated ~

O

1 > 2 (isovolumetric contraction). The cycle begins at the end of diastole at point 1. The left ventricle is filled with blood from the left atrium and its volume is about 140 mL (end-diastolic volume). Ventricular pressure is low because the ventricular muscle is ventricular pressure increases. The mitral valve closes when left ventricular pressure is greater than left atrial pressure. Because all valves are closed, no blood can be ejected from the ventricle

=

Poisoning the Na‘t-K* pump inhibits glucose and galactose absorption by dissipating the Na* gradient. Fructose

(isovolumetric). 2 — 3 (ventricular ejection). The aortic valve opens at point 2 when pressure in the left ventricle exceeds pressure in the aorta. Blood is ejected into the aorta, and ventricular volume decreases. The volume that is ejected in this phase is the stroke volume. Thus, stroke volume can be measured graphically by the width of the pressure-volume loop. The volume remaining in the left ventricle at point 3 is endsystolic volume.

O oe)

> tT OW

Glucose and galactose = Are transported from the intestinal lumen into the cells by a Na*-dependent co-transport (SGLT 1) in the luminal membrane. The sugar is transported “uphill” and Na+ is transported “downhill.” = Are then transported from cell to blood by facilitated diffusion (GLUT 2). = The Na*-K* pump in the basolateral membrane keeps the intracellular [Na*] low, thus maintaining the Na* gradient across the luminal membrane.

relaxed. On excitation, the ventricle contracts and

L

diffusion

Fig. Absorption of carbohydrates

Steps in the Cycle

©

Glucose or alactose

Na’

Fig. Left ventricular pressure-volume loop

>

K"

galactose

Left ventricular volume (mL)

O

Blood

Epithelial cell of small intestine

Lumen of

=

It

is

transported

exclusively

by

facilitated

diffusion; therefore, it cannot be absorbed against

a concentration gradient.

PART-II

3 — 4 (isovolumetric relaxation). At point 3, the ventricle relaxes. When ventricular pressure

Define blood pressure. Describe short term regulation of blood pressure. Add a note on Bainbridge reflex.

decreases to less than aortic pressure, the aortic valve

(Ref: Textbook of Physiology, AK Jain, Vol. 1, 7th ed. pg. 349-354)

closes. Because all of the valves are closed again, ventricular volume is constant (isovolumetric) during this phase. 4 — 1 (ventricular filling). Once left ventricular pressure decreases to less than left atrial pressure, the mitral valve opens and filling of the ventricle begins. During this phase, ventricular volume increases to about 140 mL (the end-diastolic volume). d.

Mechanism of glucose intestinal epithelium.

absorption

across

(Ref: Textbook of Physiology, AK Jain, Vol. 1, 7th ed. pg. 20. 265)

ANSWER Blood Pressure

It is the lateral pressure exerted on the wall of the blood vessel. It is measured in mm Hg. Regulation of Blood Pressure O

Rapid blood pressure control mechanism = Baroreceptorreflex: Highpressure(carotidsinus, arch of aorta), Low pressure ( cardiopulmonary

PAPER-| receptorscauses

bradycardia,

decrease

in

increased

CO,,

excitation of VMC,

=

Renin Angiotensin vasoconstrictor mechanism

=

Stress relaxation mechanism

reverse

stress

(Ref: Textbook of Physiology, AK Jain, Vol. 1, 7th ed. pg. 463) ANSWER

Decreased blood flow to tissue No oxygenation in tissue

Arterial pO, normal In Venous blood of tissue pO, decreases markedly A-V difference increases markedly E.g.

Shock

failure,

relaxation

due

to

Congestive

hemorrhage heart

and

failure

circulatory

due

to

which

decrease blood flow to tissues, localized in cases of atherosclerosis, thrombosis, embolism.

=" Capillary fluid shift O

ToA-V PO, difference in stagnant hypoxia.

strong

sympathetic discharge, causing vasoconstriction, increase in blood pressure. = Chemoreceptor reflexes: Carotid bodies and aortic bodies respond to lack of oxygen, increased CO, and decreased pH. Intermediate control mechanism: (After several minutes) and

Inhypertensive patients causes edema

b.

cardiac output and BP. CNS ischemic response: Low BP, less supply to CNS,

O

vasodilation,

O

90000 0

=

atrial receptors), increase discharge

c.

Long term control mechanism: (Several hour) " Renal body fluid feedback mechanism

When there is deficiency of clotting factor VIII

(Ref: Textbook of Physiology, AK Jain, Vol. 1 7th ed. pg. 105)

(direct): Decrease BP, effect on renal glomerular

return,

increased

= Aldosterone system =" Renin Angiotensin System. Bain Bridge Reflex Sudden rise in atrial pressure, increased stretch in atrial wall,

stimulation

of atrial stretch receptors,

O

Hemophilia A/Classic hemophilia deficiency of factor VIII

O

X- linked recessive characteristics

O O

Absence of clotting VIII increases the clotting time Bleeding time remains normal

O

Bleeding tendencies-soft tissue, muscle, joint,GIT,

O

retention, increased venous cardiac output, increased BP

ANSWER

respiratory and urinary tract bleed Severely damaged joints

O

capillaries, reduced excretion of salt and water, increased ECF volume due to sodium water

Recurrent

afferents

from vagus to medulla, efferent through vagus with stimulation of sympathetic nerves to cause tachycardia and increased contractility and prevents damming of blood in veins, atria and pulmonary circulation.

blood

transfusions,

in

congenital

treated with frozen

plasma, crystallized factor VIII. d.

To intestinal motility discharge increases

when

sympathetic

(Ref: Textbook of Physiology, AK Jain, Vol. 1 7th ed. pg. 252-253)

4.

What will happen and why?

a.

If NSAIDs are given to a patient compromised renal circulation

having

(Ref: Textbook of Physiology, AK Jain, Vol. 1, 7th ed. pg. 523)

ANSWER

O.

ANSWER

O

NSAIDs inhibit PGI2 that regulates renal vascular tone and PGE2 regulates sodium and chloride transport in Henle’s loop, moderating water transport and medullary

O

Increased in the sympathetic activity causes release of adrenal medullary hormones epinephrine and nor-epinephrine Decrease in intestinal motility Increase in constipation

the

O

Decreases GFR

O

Decreased urinary sodium excretion

O

Acute renal failure, severe impairment of renal functions

activity

causes

PART-III

blood flow.

Thus when given to renal compromised patients

sympathetic

>

© 5.

Write short notes on the following: a.

Hemostasis

(Ref: Textbook of Physiology, AK Jain, Vol. 1, 7th ed. pg. 98-99)

L O

oe)

> tT QO

PAPER-I b.

ANSWER

Factors affecting vital capacity

(Ref: Textbook of Physiology, AK Jain, Vol. 1, 7th ed. pg. 416) Hemostasis

Arrest of bleeding is hemostasis

ANSWER

Primary Hemostasis

O O

O

Vasoconstriction: Initially spasm _ of _ vessel, damage to vessel and platelets release 5HT and vasoconstrictors.

O

Injury to blood vessel

Formation of temporary hemostatic plug: Platelet adhesion place hyphen instead of colon stick to injured vessel wall, Platelet Activation-ADP and Thromboxane A2, Platelet Aggregation-Platelet aggregating factor (PAF)

O O

Formation of definitive temporary plug

hemostatic

clot

0.5 Ss

—Vf a a “

Molecular motors

ANSWER

fo

Platelet , Fi recruitment

c.

(Ref: Ganong review of medical physiology, 25th ed. pg. 39-40)

Thrombin on Fibrin J47 Fibrinogen vati Acti ting of clot

O

obstruction, pneumothorax etc reduces VC.

over

nitive Defift Temporary SS See plug SS SS i— Y

O

Size of thoracic cavity: Males>females Age: Elasticity decreases in old age thus decreases with age Strength of muscles: Directly proportional Position: More in standing then sitting or lying (because of gravity increased size of thoracic cavity, reduced blood flow) Pregnancy reduces VC Diseases: Ascites/Pulmonary disease like fibrosis,

O

Molecular motors move the cargo (cell protein, organelle and other substances) inside the cell.

O.

They are 100-500 kDa ATPases.

O

At head end they bind actin or microtubule, at tail end bind cargo and utilize ATP for their movement and transportation across cytoskeleton.

Collagen thromboplastin

Contraction

Secondary Hemostasis Intrinsic System HMW kininogen kallikrein

X|l ———>

|xlla [ra

4nm

Conventional kinesin

80 nm

kininogen

X| ——~> Xla

Extrinsic system Myosin V

| —_—

~~

IX IXa Vill —— | ca* Villa

VIIA — a PL TPL

PL

Xx

——>

V—

Xa

|

rede

C

2+

PL

PHYSIOLOGY 178

VII

Super Families O

Kinesin:

O

microtubule. Associated with mitosis and miosis Dynein: Double headed with neck piece imbedded in protein, moves to-ve end on microtubule.

O

Myosin:

Ca” Va

Prothrombin

Fig. Molecular motors

——*

6.

Blood clot retraction

of

18

moves

types,

bind

to

+end

to

actin

on

for

Give physiological/clinical following:

significance

of the

go

XII| ——> Xilla

©

L O

oe)

> tT QO

Your Roll No................

Name of the Paper

:

Physiology Paper-II

Name of the Course

:

MBBS-2017

Semester

Annual

Time: 3 Hours

M.M.:

50

INSTRUCTIONS Write your Roll No. on the top immediately on receipt of this question paper All questions are to be attempted

Attempt parts of a question in sequence

PART-I What is referred pain? How is the sensation of pain transmitted to the brain? Describe the pain control system of the brain and the spinal cord. [3]

eo

SF

Draw labeled diagram/flow chart of the following:

[3]

Length tension relationship in a cardiac muscle Feedback control of testicular activity Oral glucose tolerance test in normal and diabetic persons Audiogram in sensorineural deafness of left ear

PART-II List the hormones secreted from the adrenal glands. Discuss briefly the role of glucocorticoids on inflammation and immune system. Add a note on adrenal insufficiency. [8] What will happen and why?

[3]

FS

If there is narrowing of sclerocorneal angle

ao

a

Answers to Parts I, II and III should be written in separate answer sheets provided

To muscle tone during the acute phase of cerebellar disease in humans To membrane potential of smooth muscle on parasympathetic stimulation To feeding behavior in lateral hypothalamic lesion

PART-III Write short notes on the following:

a. b. c.

Myasthenia gravis Delayed and precocious puberty Visual acuity

Give physiological/clinical significance of the following:

a. b. c.

[9]

Electroencephalogram (EEG) Circadian rhythm Endocochlear potential

[9]

PAPER-II

2017

|

2017 PAPER-II

PART-I

1.

O

pain transmitted to the brain? Describe the pain

(Dynorphin, Enkephalin and Morphine)

control system of the brain and the spinal cord.

©

ANSWER

O

Due to same dermatome or structure from same embryonic segment. Convergence projection theory = Convergent fibers from visceral and somatic area to dorsal horn (2nd order)

= Dorsal horn to thalamus

PAG acts on nucleus raphe magnus (serotonergic), Rostral ventromedial medulla (cholinergic)

O

RMN and RVM inhibits dorsal horn

O

Locus Coeruleus (norepinephrine) inhibits dorsal horn

O

Stress

Pain in visceral organ felt at different somatic site.

e.g. cardiac pain felt at left arm. O

Decrease transmission of nociceptor afferents due to activation of both opioid receptors.

What is referred pain? How is the sensation of _ pain Control by Periaqueductal Grey and Brainstem

(Ref: Textbook of Physiology, AK Jain, Vol. 2, 7th ed. pg. 904)

O

|

.

induced

oy

analgesia

by

neurons in Amygdala

tgene

catecholaminergic

O

Cannabinoids induced with CB1 receptors in brain, CB2 on microglia

2.

Drawlabeled diagram/flow chart of the following:

= Thalamus to Somatosensory cortex a.

Cortical

influence v

Periaqueductal gray matter

Length muscle

tension

relationship

in

a cardiac

(Ref: Ganogn's Review of Medical Physiology 25th ed. pg. 115)

ai Midbrain

Ventrolateral pain pathway

XS

if

ANSWER

v

Frank-Starling Relationship

O

Medula

Nucleus raphe

magnus

——.

Catecholaminergic

Rostral ventromedial— medulla

pathway

O

Serotonergic pathway

It describes the increase in stroke volume and cardiac output that occurs in response to an increase in venous return or end-diastolic volume. It is based on the length-tension relationship in the

. Spinal cord ee

ventricle.

Increases

in

end-diastolic

volume

Cc ause a n incr crease in ventricular entricular fiber fiber len length, which Cc produces an increase in developed tension.

| Afferent”

pain fibers

270 -

ee

chevenkiiculer

240 210 >

Somatic

pressure

structures

Visceral

2

structures (

D

aE

180 7 450 7

= Fig. Convergence projection theory

®

Developed

tension 420 -

a

Pain Control System in Spinal Cord

O

;

;

2

,

Gate Control mechanism: stimulation of cutaneous

mechanoreceptors modulates pain. E.g. TENS O

Pre synaptic opioid receptor stimulation causing decrease Ca influx, decrease glutamate and Substance P.

>-

60

Diastolic

©

2

Mosca

O

Post synaptic opioid receptors stimulation causing

hyperpolarization of dorsal horn interneurons O

oO

90 7

©

ea

OH Sh

a0

Bo

Bow

Diastolic volume (mL) Fig. Tension relationship in cardiac muscle

> ae

PAPER-II O

O

Itis the mechanism that matches cardiac output to venous return. The greater the venous return, the greater the cardiac output. Changes in contractility shift the Frank-Starling curve upward (increased contractility) or downward (decreased contractility). —_——_

=

—**

Pa

intratesticular paracrine mechanism to reinforce the spermatogenic effects of FSH in the Sertoli cells. Hypothalamus (arcuate nucleus)

*

Positive inotropic effect

eh

| l | | l l l

So

®

®

+

v

I I I I I I

Negative inotropic effect

+— 4

Stroke volume or cardiac output

Control

Right atrial pressure

Inhibin

Steps in Photoreception in the Rods

_)

O

4 Oo

/]\\

pituitary

tissues

Effects oflight on current flow in visual receptors 1110-1111)

ANSWER

Hypothalamus

Target

c.

hormone

LWA Liver

Adipocyte

Bone

Ghrelin

The photosensitive element is rhodopsin, which is composed of opsin (a protein) belonging to the superfamily of G-protein-coupled receptors and retinal (an aldehyde of vitamin A). Light on the retina converts 11-cis retinal to all-

Stomach

trans retinal, a process called photoisomerization. A series of intermediates is then formed, one of which

Muscle

IGF-1

y

Secondary target tissues

Fig. Feedback control of growth hormone secretion

is metarhodopsin II. = Vitamin A is necessary for the regeneration of 11-cis rhodopsin. Deficiency of vitamin A causes night blindness. Metarhodopsin II activates a G protein called transducin (Gt), which in tum activates a phosphodiesterase. Phosphodiesterase catalyzes the conversion of cyclic guanosine monophosphate (cGMP) to 5'-GMP, and cGMP levels decrease.

>

©

= O

oe)

> tT QO

PAPER-II 11-cis retinal

| Light All-trans retinal

4

Metarhodopsin II

|

Activation of G protein (transducin)

. Activation of phosphodiesterase

|

4cGMP

|

Closure of Na’* channels

|

Hyperpolarization

v Decreased glutamate release

Fig. Steps in photoreception in rods. cGMP = cyclic guanosine Decreased glutamate release monophosphate.

O

Decreased

O

hyperpolarization of the photoreceptor cell membrane. Increasing light intensity increases the degree of hyperpolarization. When the photoreceptor is hyperpolarized, there is decreased release of glutamate, an excitatory neurotransmitter. There are two types of glutamate receptors on bipolar and horizontal cells, which determine whether the cell is excited or inhibited. d.

levels of cGMP

cause

closure

of Na* channels,

decreased

inward

Na*

current,

and,

as a result,

Electrical and mechanical responses of a mammalian skeletal muscle fiber to a single maximal stimulus (Ref: Textbook of Physiology, AK Jain, Vol. 2, 7th ed. pg. 168

ANSWER Axon terminal of somatic motor neuron

ACh

Muscle fiber

4)

Somatic motor neuron releases ACh at neuro-

muscular junction.

von potential rT

Action potential»

= a ee

e|

2) T-tubule

:

a

iP Motor end

|

FE

OOTTTI '

'

_

~DHP

receptor

°

P

ereill

©

= O

wv

> tL QW

PAPER-II OCa?*

Calcium channel

Sulphonylurea receptor Insulin secretion

ATP-sensitive K* channel

Glucose transporter

(GLUT-2)

Membrane depolarisation ay

Metabolism

Fig. Insulin secretion

Pathophysiology of Diabetes Mellitus Insufficient insulin

Reduced tissue uptake of glucose

-——_ Extracellular hyperglycaemia

Intracellular

hypoglycemia

f Glucogenesis

——_——

and gluconeogenesis

——_—— Breakdown of fats

High levels of ketones

Diabetic ketoacidosis

Diabetes control card

HbA, test score

* Decreased protein

synthesis * Cachexia ¢ Lethargy * Polyphagia * Decreased gamma

globulins * Susceptibility to infections ¢ Impaired wound healing

MEAN BLOOD GLUCOSE mmol/L mg/dL

PHYSIOLOGY

21.1

Hyperosmotic

Blood glucose

Peeme

renal threshold

f Dehydration

of

cells

f Hyperglycaemic coma

>

' Glucosuria: urine has a highSG

! Osmotic diuresis -

Polyuria polydipsia Hypokalaemia Hyponatraemia

What period of time do the various clinical tests for diabetes reflect? Blood glucose level at the time of blood sampling

Blood glucose

Mean glycemia over the previous 3 weeks

Glycated albumin

Mean glycemia over the previous 2 months

Glycated hemoglobin

PAPER-II ANSWER

These differences in the period of time over which mean blood glucose level is reflected are related to the life spans (half-life) of cells and proteins.

OO O

O

Several hours

glucose

Glaucoma Blocked canal of Schlemm Increased in intra ocular pressure above 80 mm Hg Retinal degeneration due to increased pressure causes blindness

PART-III

Glycated | About 2 weeks albumin

About 4 weeks

About 1 month

Glycated hemoglobin

About 2 months

Life span

5.

Write short notes on the following a.



Strength duration curve

(Ref: Textbook of Physiology, AK Jain, Vol. 1, 7th ed. pg. 145) Fig. Effect of consistent increased glucose levels above normal

What will happen and why?

a.

ANSWER

To the excitability of nerve hypocalcemia

and muscle

in

O O

(Ref: Textbook of Physiology, AK Jain, Vol. 2, 7th ed. pg. 717-718)

O ANSWER

O

Excitability of Nerve and Muscle in Hypocalcemia O

Decrease calcium increase permeability to sodium

O

Increased membrane excitability of excitable tissues

O

Resulting

convulsions,

laryngospasm,

O

bronchos-

b.

pasm, abdominal cramps, obstetrician’s hand b.

ANSWER O

(Ref: Textbook of Physiology, AK Jain, Vol. 2, 7th ed. pg. 919

Dorsiflexion of big toe

O

Fanning of toes

O

Flexion of the limb c.

To menstrual lactation

O9O00 90

ANSWER

cycle

during

Anovulatory cycle Amenorrhea Prolactin inhibits FSH and LH

Increased threshold for arousal Desynchronized EEG Turner’s syndrome

ANSWER

OO 0

0900 0

O

Follows non REM sleep Paradoxical sleep

(Ref: Textbook of Physiology, AK Jain, Vol. 2, 7th ed. pg. 779)

O

ANSWER

Rapid eye movement

c.

post-partum

(Ref: Textbook of Physiology, AK Jain, Vol. 2, 7th ed. pg. 809)

REM sleep

(Ref: Textbook of Physiology, AK Jain, Vol. 2, 7th ed. pg. 989

To plantar reflex in lesions of upper motor neuron

O

Given by Louis Lapicque Curve that is obtained by plotting stimulus strength on y axis and stimulus duration on x axis Rheobase: Measure of membrane potential i.e. minimum current required to cause contraction of muscle. As strength of the stimulus increase, the duration for the application decreases Chronaxie: It’s the duration required by the stimulus double the rheobase to produce contraction.

XO 45 chromosomes

Phenotype: Female Webbed neck, dwarf, delayed sexual development, amenorrhea

6 weeks in non lactating female 25-30 weeks in lactating female d.

To the field of vision

if there

is increase

in

intraocular pressure (Ref: Textbook of Physiology, AK Jain, Vol. 2, 7th ed. pg. 1090)

6.

Write physiological/clinical significance of a.

PHYSIOLOGY

4.

Gate control theory of pain

(Ref: Textbook of Physiology, AK Jain, Vol. 2, 7th ed. pg. 904-905)

213

O00 90

PAPER-II By Ronald Melzack and Patrick Wall in 1965

O

Duration: Seconds to minutes

Stimulation of A afferent fibers and dorsal column Excitation of inhibitory interneurons

O

Calcium

c.

tract

in

threshold

of the

Impedance matching

(Ref: Textbook of Physiology, AK Jain, Vol. 2, 7th ed. pg. 1076

Olfactory adaptation

(Ref: Textbook of Physiology, AK Jain, Vol. 2, 7th ed. pg.

1060-1061)

ANSWER

ANSWER

0

O

Olfactory fatigue

OO

O

PHYSIOLOGY

increase

receptor for the specific odor

Inhibition transmission cells oflateral spinothalamic

b.

mediated

O

Odor perceived at first gradually disappears after some time

Itrefers to increase pressure in middle ear 22 times increase in pressure Sound magnification 1.2-1.3 times by ear ossicles 17 times reduction in the surface area of oval window i.e. 3 mm square (tympanic membrane 50 mm square)

Your Roll No................

Name ofthe Paper

:

Physiology Paper-I

Name ofthe Course

:

MBBS-2014 Annual

Semester Time: 3 Hours

M.M.: 50

INSTRUCTIONS Write your Roll No. on the top immediately on receipt of this question paper All questions are to be attempted

Attempt parts of a question in sequence

PART-I What are the causes and characteristic features of hypoxic hypoxia? What are the physiological compensatory response to chronic hypoxia? [8]

a ©

9

f

Draw labeled diagram/flow chart of the following:

[8]

CO, - ventilatory response curve Stages of erythropoiesis Mechanism of HCI secretion in the stomach Heterometric regulation of stroke volume

PART-II Define counter-current mechanism.

Describe the mechanism

of concentration and dilution of urine. Differentiate

[3]

What will happen and why?

[8]

8

2

between osmotic and water diuresis.

a ©

a

Answers to Parts I, II and III should be written in separate answer sheets provided

If If If If

a sea diver ascends very rapidly from deep waters to the surface third-degree heart block sets in suddenly blood flow to inner renal medulla increases bile duct is completely blocked

PART-III Write short notes on:

a. b. c.

Regulation of salivary secretion Hypoxic hypoxia Triple response

Give physiological/clinical significance of: a. b. c.

[9]

Negative intra-pleural pressure Dietary fiber Long duration of the action potential in the ventricular muscle

[9]

PAPER-I |

2014 PAPER-I

O

Cyanosis

O

Dizziness

What are the causes and characteristic features

©

Headache, nausea vomiting

are the physiological

O

Numbness or tingling sensation in extremities

O O

Increase in respiratory rate and depth Impaired judgement

PART-I 1.

of hypoxic hypoxia? What

compensatory response to chronic hypoxia?

Altered behavior, talkativeness

ANSWER

Effects of Chronic Hypoxia/Acclimatization

Causes o O O O O

|

Decrease OXYSEN SUPPIY nee of gas exchange:

. . Respiratory obstructive

©

CASES Venous to arterial shunts: Cyanotic congenital heart disease disease, lun Interstitial lung patholo Lung

O O

5

O

. ; Reduced partial pressure of oxygen: High Altitude, Drowning

O

5y: 5 P fibrosis, under ventilated alveoli



5

Features

O

Ventilation perfusion imbalance

2.

Draw labeled diagram/flow chart of the following: a.

Hypoxia induced factors cause proteins like Erythropoietin Polycythemia

O

4

transcription

of

Increased vascularity, formation of new blood vessels ; ; ; ; Increased mitochondria, myoglobin and _ tissue ; cytochrome oxidase Increased ventilatory response a

dioxide

Carbon alkalosis

;

oe

washout

causing

respiratory

;

O

Oxygen dissociation curve shifts to left

O.

Increase in 2,3 DPG in RBC decreases affinity for Hb

(Ref: Textbook of Physiology, AK Jain, Vol. 1, 7th ed. pg. 439)

CO, - ventilatory response curve

ANSWER Arterial PCO, is the most important stimulus to ventilation under most conditions

sia ba

e This graph depicts the effects of increased arterial blood PCO, and decreased arterial pH

PHYSIOLOGY

e

Note that if the arterial PCO, increases between 35 and

75 mm Hg, there is marked increase in ventilation e On the other hand, the increase in ventilation produced by changes in pH between 7.3 and 7.5 is only one-tenth as great produced by PCO, changes e This graph signifies that the most important regulator of respiration is always arterial PCO, levels

PCO,

Normal

ri

Alveolar ventilation (basal rate = 1)

10-

1-1 )

20 30 40 50 60 70 80 90 100 PCO,(mm Hg)

7675747372717069

>

pH

b.

Stages of erythropoiesis

(Ref: Textbook of Physiology, AK Jain, Vol. 1, 7th ed. pg. 70)

PAPER-|

For answer refer 2016 paper-1, Q. 2(a) Pg. 188

cr

HCI

K

Colony forming

HCO;

:

(“alkaline tide”)

—-H,CO oe

Ne

K

CO, +H,0

+

Proerythyroblast

Xe

(Pronormoblast) y

Hb ‘Ist appears

Fig. Simplified mechanism of H* secretion by gastric parietal cells. CA = carbonic anhydrase

Normoblast

v

Oo

Early Normoblast (Basophilic)

t

O

Intermediate normoblast

(Polychromatophilic)

t

Late Normoblast

(Orthochromatic)

In the parietal cells, CO, and H,O are converted to

H* and HCO, , catalyzed by carbonic Ht is secreted into the lumen of the H*-K* pump (H*, K*-ATPase). Cl is with H*; thus, the secretion product cells is HCl. d.

¢

anhydrase. stomach by the secreted along of the parietal

Heterometric regulation of stroke volume

(Ref: Textbook of Physiology, AK Jain, Vol. 1, 7th ed. pg. 343) ——»>

Nucleus removed

}

Normal RBC

c.

+ HCO,

|

Unit-Erythroids (CFU-E)

Fig.

cr

H* a

is due to NaCl.

of

namely UT-A1 to UT-A4

Fig. Countercurrent multiplier

©

50%

O

O

= g °

y

contributed ;

only

It is for this reason, high protein diet increases the concentrating ability of kidney and low protein diet decreases kidneys’ ability to concentrate the urine

8

£ 3} ts -

that

O

aA

600

;

metabolism - urea

300

600

Remember

hyperosmolarity

Materials absorbed into the capillaries diffuse directly from veins into arterioles

300

;

Role of Urea in Concentration of Urine

° For exchange of heat and

ie

Vasa recta serves as countercurrent exchangerthey minimize washout of solutes and maintain the osmolarity gradient developed by loop of Henle ;

veine

Intestine

Vasa recta are straight capillaries that run parallel to loop of Henle

to the cold venous blood coming from the extremities e Between spermatic arteries and

Testis

>

Thick ascending loop of Henle - “the diluting segment” " Thick ascending loop of Henle is permeable to solutes and impermeable to water = Sodium, chloride and potassium moves out of the thick ascending limb through Na-K-2Cl cotransporter = Because of selective movement of solutes leaving water inside - fluid inside thick ascending limb becomes hypotonic to plasma

through

achieved due to which no further reabsorption of the solutes occur e Diuresis of urine having osmolality equals to plasma.

4.

water) e¢ Hypotonic plasma ® Inhibition of ADH secretion e Diuresis of diluted urine

What will happen and why?

a.

Ifasea diver ascends very rapidly from deep

waters to the surface (Ref: Textbook of Physiology, AK Jain, Vol. 1, 7th ed. pg. 477)

PAPER-| PART- Ill

ANSWER

O

O00 9

O

O

Caisson’s disease/decompression palsy/bends/dysbarism

sickness/diver’s 5.

Write short notes on:

Diving in deep sea causes diffusion of nitrogen in plasma, nitrogen narcosis: Euphoria, loss of mental functions like reasoning etc.

a.

(Ref: Textbook of Physiology, AK Jain, Vol. 1, 7th ed. pg. 207)

Rapid ascend/sudden decompression causes dissolved nitrogen to be released as Bubbles.

ANSWER

Bubbles in fat and myelin sheath causing paralysis

Regulation of Saliva Production

Lung capillaries causing dyspnea

O

Saliva production is_ controlled by _ the parasympathetic and sympathetic nervous systems (not by GI hormones). Saliva production is unique in that it is increased by both parasympathetic and sympathetic activity. Parasympathetic activity is more important, however.

Heart causing Myocardial infarction In joints sever pain known as Bends. b.

O

Ifthird-degree heart block sets in suddenly

(Ref: Textbook of Physiology, AK Jain, Vol. 1, 7th ed. pg. 301)

Parasympathetic Stimulation (Cranial Nerves VII and IX)

OO O

O

ANSWER

O

No conduction from atria to ventricle

Increases saliva production by increasing transport processes in the acinar and ductal cells and by causing vasodilation. Cholinergic receptors on acinar and ductal cells are muscarinic.

Ventricles have idio-ventricular rhythm (45 bpm) Stokes Adam Syndrome

O

Dizziness, loss of consciousness, convulsion due to

decreased blood supply to brain c.

Ifblood flow to inner renal medulla increases

(Ref: Textbook of Physiology, AK Jain, Vol. 1, 7th ed. pg. 551-552) ANSWER

Thesecond messenger is inositol 1, 4, 5-triphosphate

O

(IP3) and increased intracellular [Ca”*]. Anticholinergic drugs (e.g., atropine) inhibit the production of saliva and cause dry mouth.

Conditioning Food Nausea Smell

{

Dehydration Fear Sleep Anticholinergic drugs

;

Parasympathetic

Sympathetic

Increase in blood flow will decrease the osmolality resulting in inefficient counter current mechanism

ACh

NE

O

Low blood flow in renal medulla is important for production of hyperosmolality in medullary interstitium

O

Loss of concentration mechanism of urine

=

O

O

Regulation of salivary secretion

Hence diluted urine

O

d.

Atropine

1

Muscarinic receptor

Ifbile duct is completely blocked

|

Acinar and ductal cells

IP,, Ca”

68 Receptor |

cAMP

ANSWER Saliva

Due to biliary stone

O

Increased pressure on hepatic duct

O

Cholestatic

Jaundice

and

its features

like icterus,

O

Absent digestion of fats

Sympathetic Stimulation

O

loss of appetite, itching etc.

Fig. Regulation of salivary secretion. ACh = acetylcholine; cAMP = cyclic adenosine monophosphate; IP3 = inositol 1, 4, 5-triphosphate; NE = norepinephrine.

Steatorrhea,

O

absent absorption of Vitamin A, D, E

and kK

O

Impaired liver function tests

Increases the production of saliva and the growth of salivary glands, although the effects are smaller than those of parasympathetic stimulation.

PHYSIOLOGY

O

PAPER-I

is

cells

cyclic

are

beta-

adenosine

Saliva Production

It is increased (via activation of the parasympathetic nervous system) by food in the mouth, smells, conditioned reflexes, and nausea.

It is decreased (via inhibition of the parasympathetic nervous system) by sleep, dehydration, fear, and anticholinergic drugs. Hypoxic hypoxia

(Ref: Textbook of Physiology, AK Jain, Vol. 1, 7th ed. pg. 461-470) ANSWER For answer refer 2016, paper-1, Q. 1, Pg. 188

c.

Triple response

(Ref: Textbook of Physiology, AK Jain, Vol. 1, 7th ed. pg. 379-380) ANSWER O O

O

6.

PHYSIOLOGY

Decrease in elasticity, chest expand barrel shaped, e.g emphysema Traumatic Pneumothorax: pressure, lung collapse

positive

Intrapleural

Dietary fiber

O

Undigested and not absorbed

O

Increases bulk, stimulates peristalsis

O

Decreases absorption by getting into way of nutrient and absorption surface Stimulates bile salt synthesis from liver by binding of bile salts and there excretion through feces, thus decreases plasma cholesterol Reduces cancer by binding and increased excretion of carcinogen subjecting to increased GIT motility. c.

Long duration of the action potential in the ventricular muscle

(Ref: Textbook of Physiology, AK Jain, Vol. 1, 7th ed. pg. 180) ANSWER

Increased duration of absolute refractory period

Give physiological/clinical significance of:

(Ref: Textbook of Physiology, AK Jain, Vol. 1, 7th ed. pg. 414)

Less negative at base

Cellulose, hemicellulose etc,

Axon reflex: axon of cutaneous receptors on stimulation activates interneurons stimulating arterioles locally causing dilatation due to Ach

Negative intra-pleural pressure

More negative in apex of lung

O

A wheal is local edema that results from the local release of histamine, which increases capillary filtration.

a.

On expiration it decrease

ANSWER

Trauma produces the “triple response” in skin—a red line, a red flare (axon reflex), and a wheal.

release, hence typical flare is formed.

More the negative pressure more will be expansion

(Ref: Textbook of Physiology, AK Jain, Vol. 1, 7th ed. pg. 260-636)

O

b.

Transmural pressure or recoil pressure

b.

OO O

O

ANSWER 090000 0

ductal

O

Receptors on acinar and adrenergic. The second messenger monophosphate (cAMP).

Allows filling of Atria Adequate emptying of ventricles Can never be tetanized

Your Roll No................

Name ofthe Paper

:

Physiology Paper-Il

Name ofthe Course

:

MBBS-2014

Semester

Annual

Time: 3 Hours

M.M.: 50

INSTRUCTIONS Write your Roll No. on the top immediately on receipt of this question paper 2.

All questions are to be attempted

3.

Answers to Parts I, II and III should be written in separate answer sheets provided

4.

Attempt parts of a question in sequence

PART-I 1.

Name the functional division of the cerebellum. Briefly describe the afferent connections of the cerebellum. Add a note on the functions of the cerebellum. [3]

2.

Draw labeled diagram/flow chart of the following: a. b. c. d.

[3]

Renin-angiotensin system The action potential and the contractile response of the ventricular muscle on the same time axis Structure of organ of corti Graph showing the levels of human hcG with respect to fertilization and the period of pregnancy

PART-II 3.

Enumerate the various hormones secreted by adrenal gland. Write five functions of glucocorticoid hormones. Add a note Cushing’s syndrome. [3]

4.

What will happen and why? a. b. c. d.

[3]

If bilateral lesion in the ventromedial nuclei of the hypothalamus occurs To peripheral field of vision in complete right optic tract lesion To the knee jerk reflex in hyperthyroidism To spermatogenesis in furnace workers.

PART-III 5.

6.

Write short notes on:

a. b.

Accommodation reflex Contraceptive pills

c.

Classification of nerve fibers

Give physiological/clinical significance of:

a. b. c.

Secondary active transport Blood testis barrier Endocohlear potential

[9]

[9]

PAPER-II |

2014 PAPER-II

PART-I 1.

Name

ANSWER

the functional division of the cerebellum.

Renin-Angiotensin-Aldosterone System

Briefly describe the afferent connections of the

It is

cerebellum. Add

It is used in long-term blood pressure regulation by adjustment of blood volume.

a note on the functions of the

(Ref: Textbook of Physiology, AK Jain, Vol. 2, 7th ed. pg. 902-905)

ANSWER Functional Division of the Cerebellum

O O

Vestibulocerebellum: Flocculonodular lobe, control of body posture Spinocerebellum: Anterior lobe except lingual, pyramids, uvula and paraflocculous Corticocerebellum: Posterior lobe except uvla and pyramids

O00 0

cerebellum.

O

O O

O

O O

range,

extent

= Angiotensin receptor (AT 1) antagonists (e.g., losartan) block the action of angiotensin II at its receptor and decrease blood pressure. Angiotensin II has four effects: = It stimulates the synthesis and secretion of aldosterone by the adrenal cortex. * Aldosterone increases Na* reabsorption by the

and

Renin-angiotensin system

(Ref. Ganong’s Review of Medical Physiology, 25th ed, pg. 699)

cells of the afferent arteriole to

therefore, decrease blood pressure.

=

renal

distal

tubule,

thereby

increasing

extracellular fluid (ECF) volume, blood volume, and arterial pressure. * This action of aldosterone is slow because it requires new protein synthesis. Jt increases Na*-H* exchange in the proximal convoluted tubule.

*

Drawlabeled diagram/flow chart of the following: a.

A decrease in renal perfusion pressure causes the

conversion of angiotensin I to angiotensin II and,

timing, integration and co ordination of movements (comparator function). = Prevents overshooting of movement—Damping effect =" Controls ballistic actions (rapid alternate movements) e.g. playing piano, dancing =" Servomechanism: Correct execution of motor movements 2.

Angiotensin II is degraded by angiotensinase. One of the peptide fragments, angiotensin III, has some of the biologic activity of angiotensin II.

Angiotensin-converting enzyme (ACE) catalyzes the conversion of angiotensin I to angiotensin II, primarily in the lungs. =" ACE inhibitors (e.g., captopril) block the

OO

PHYSIOLOGY

their

Angiotensin II is physiologically active.

Renin is an enzyme that catalyzes the conversion of angiotensinogen to angiotensin I in plasma.

Vestibulocerebellum involved in control of posture and equilibrium of the body. Spinocerebellum involved in control of tone of muscles and stretch reflex. Corticocerebellum is involved in control of movements,

Angiotensin | is inactive.

secrete renin.

Inferior cerebellar peduncle = Dorsospinocerebellar tract =" Reticulocerebellar tract = Olivocerebellar tract = External arcuate fibers = Vestibulocerebellar tract Middle cerebellar peduncle: Cerebropontine cerebellar fibers Superior cerebellar peduncle: Ventral spinocerebellar tract and tectocerebellar tract

voluntary

Renin is an enzyme.

juxtaglomerular

Functions of Cerebellum

O

aslow, hormonal mechanism.

Steps in the Renin-Angiotensin-Aldosterone System

Afferent Connections

O

|

This action of angiotensin II directly increases Na* reabsorption, complementing the indirect stimulation of Na* reabsorption via aldosterone. * This action of angiotensin II leads to contraction alkalosis. It increases thirst and therefore water intake. It causes vasoconstriction of the arterioles, thereby increasing TPR and arterial pressure.

PAPER-II ions (favoured by both chemical and electrical gradients) out of the cell and in part because of a decrease in Na+ conductance.

Role of the Renin-Angiotensin-Aldosterone System in the Cardiovascular Response to Hemorrhage Acute hemorrhage

c)

¢ YP,

Phase 2 It is the plateau of the action potential.

¢

It is caused

by

Renal perfusion pressure

conductance,

which

¢

¢

*Conversion of angiotensinogen to angiotensin | Angiotensin-converting

in

Ca**

results in an inward

Ca**

enzyme (ACE)

d)

During phase 2, outward and inward currents are approximately equal, so the membrane potential is stable at the plateau level. Phase 3 It is repolarization.

? Conversion of angiotensin | to angiotensin II

¢

During and

tAngiotensin II

a a

K*

phase

3,

Ca**

conductance

conductance increases

decreases,

and

therefore

predominates.

TAldosterone

TNa*—H* exchange

tThirst

TNa’*

TNa’

Water

reabsorption

reabsorption

intake

The high K* conductance results in a large outward K* current (IK), which hyperpolarizes the membrane back toward the K* equilibrium potential.

. | | Vasoconstriction

TTPR

e)

¢

Phase 4 It is the resting membrane potential. It is a period during which inward and outward currents (IK1) are equal and the membrane potential approaches the K+ _ equilibrium potential.

TP, toward normal *Pa = mean arterial pressure

*TPR = total peripheral resistance

b.

increase

current, and by an increase in K+ conductance.

TRenin

v

a transient

Theaction potential andthe contractile response

-

Action potential of myocardial cells

of the ventricular muscle on the same time axis.

(Ref. Ganong’s Review of Medical Physiology, 25th ed, pg. 524)

O

Occurs in Ventricles, atria, and the Purkinje system.

O

Have stable resting membrane potentials of about -~90 millivolts (mV). This value approaches the K* equilibrium potential. Action potentials are of long duration, especially in Purkinje fibers, where they last 300 milliseconds (msec). Phase 0 It is the upstroke of the action potential. It is caused by a transient increase in Nat conductance. This increase results in an inward Na* current that depolarizes the membrane. At the peak of the action potential, the membrane potential approaches the Na* equilibrium potential.

b)

Phase 1

It is a brief period of initial repolarization. Initial repolarization is caused by an outward current, in part because of the movement of K*



I

100 msec

|

Fig. Action potential of myocardial cells Cc.

Structure of organ of corti

(Ref: Textbook of Physiology, AK Jain, Vol. 2, 7th ed. pg. 1071) ANSWER For answer refer 2016, paper-2, Q. 2 (c), Pg. 196

d.

Graph showing the levels of human hcG with respect to fertilization and the period of pregnancy

(Ref: Textbook of Physiology, AK Jain, Vol. 1, 7th ed. pg. 826)

PHYSIOLOGY

Action Potential of Myocardial Cells

Millivolts

ANSWER

223

PAPER-II PART-II

ANSWER Pregnancy O

3.

It is characterized by steadily increasing levels of estrogen and progesterone, which maintain the endometrium for the fetus, suppress ovarian follicular function (by inhibiting FSH and LH secretion), and stimulate development of the breasts.

(Ref: Textbook of Physiology, AK Jain, Vol. 2, 7th ed. pg. 722-729) ANSWER

Fertilization O

If fertilization occurs, the corpus luteum is rescued

For answer refer 2017, paper-2, Q. 3, Pg. 184

from regression by human chorionic gonadotropin (HCG), which is produced by the placenta.

Cushing's Syndrome (Demonstrated by Harvey Cushing) Cause: Excessive glucocorticoids (Increased ACTHACTH dependent, Low ACTH-ACTH independent as in cortical tumor of adrenal glands) = Increased protein catabolism = Hyperglycemia due to glycogenolysis, gluconeogenesis causing insulin resistant DM =" Lipolysis with differential redistribution of fat causing buffalo hump and central obesity =" Osteoporosis = Increase activity of collagenase enzyme- purple stria = Increase in sodium water retention causing hypertension and diuresis =" Hirsutism = Moon face = Restlessness and psychosis = Increase susceptibility to peptic ulcers = Frequent infections due to decreased immunity = Amenorrhea and loss of libido

O First Trimester

The corpus luteum (stimulated by HCG) is responsible for the production of estradiol and progesterone. Peak levels of HCG occur at gestational week 9 and then decline.

Hormone level

O

Enumerate the various hormones secreted by adrenal gland. Write five functions of glucocorticoid hormones. Add a note Cushing’s syndrome.

Personality changes

a)

Hyperglycemia

@

CNS Irritability Fat round “Moon” Face

Red face

Increased susceptibility

Weeks of pregnancy

Fluid retention

to infection

(Edema)

Gynecomastia Corpus luteum

Placenta

Thin Extremities

-

°

(in males)

Gl! Distress-Acid

Fat deposition

Fig. Hormone levels during pregnancy. HCG = human chorionic gonadotropin; HPL = human placental lactogen

on abdomen and back of neck

7

("Buffalo Hump’)

|

J;

Purple striae “)

!

Amenorrhea, Hirsutism

(in females)

Second and Third Trimesters O

PHYSIOLOGY

O

by

Progesterone is produced by the placenta. Estrogens are produced by the interplay of the fetal adrenal gland and the placenta. The fetal adrenal gland synthesizes dehydroepiandrosteronesulfate (DHEA-S), which is then hydroxylated in the fetal liver. These intermediates are transferred to

the

placenta,

where

enzymes

remove

Osteoporosis (Increased risk of fractures)

—_—— Thin skin Bruises & Petechiae

1 ¢

Fig. Cushing's syndrome

sulfate

and aromatize to estrogens. The major placental estrogen is estriol. Human placental lactogen is produced throughout pregnancy. Its actions are similar to those of growth hormone and prolactin.

4.

What will happen and why? a.

If bilateral lesion in the ventromedial nuclei

of the hypothalamus occurs (Ref: Textbook of Physiology, AK Jain, Vol. 2, 7th ed. pg. 1010)

PAPER-II ANSWER

O O

Hypothalamic

The ventromedial hypothalamus is a satiety center and regulates food intake. Lesions of the ventromedial hypothalamus result in obesity.

nuclei

Anterior region

Temperature regulation; lesion > hyperthermia Stimulates the parasympathetic nervous system

Posterior region

Temperature regulation; lesion > poikilothermia (inability to thermoregulate) Stimulates sympathetic nervous system

Preoptic area

Regulates release of gonotrophic hormones; contains sexually dimorphic nucleus Lesion before puberty > arrested sexual development; lesion after puberty > amenorrhea or impotence

Dorsomedial

Stimulation > savage behavior

Hypothalamic Nuclei’s Functions and their Lesions Hypothalamic

__| Functions and lesions

nuclei

Lateral

Feeding center; lesion —> starvation

hypothalamic Satiety center; lesion savage behavior

Ventromedial

— hyperphagia, obesity,

b.

Suprachiasmatic

Regulates circadian rhythms, receives direct retinal input

Supraoptic and paraventricular

Synthesizes ADH and oxytocin; regulates water balance Lesion — diabetes insipidus, characterized by polydipsia and polyuria

| Functions and lesions

To peripheral field of vision in complete right optic tract lesion

(Ref: Textbook of Physiology, AK Jain, Vol. 2, 7th ed. pg. 1094) ANSWER

O

Mamillary body

Input from hippocampus; damaged in Wernicke encephalopathy

Arcuate

Produces hypothalamic releasing and inhibiting factors and gives rise to tuberohypophysial tract Has neurons that produce dopamine (prolactininhibiting factor)

Lesions of the optic tract result in a loss of visual input from the contralateral visual field. For example, a lesion of the right optic tract results in a loss of input from the left visual field. This is called a homonymous hemianopia;

in this example, a left

homonymous hemianopia.

Contd... rae

aed

i

Pedy

Lesion before optic chiasm 1. Monocular visual loss = lesion at

Left eye

Right eye

pane GE 2. Bitemporal hemianopia = lesion at optic chiasm

Bitemporal

Monocular

we

Optic ~

3 =

Lesion after optic chiasm

Chiasma

Upper

5

homonymous

quadrantanopia

\ |

Homonymous

—)

hemianopia

3. Homonymous hemianopia = lesion at right optic tract

4. Lower homonymous quadrantanopia = lesion at left arietal radiation - Meyer's loo

P

homonymous quadrantanopia

y

P

5. Upper homonymous

CP

quadrantanopia = lesion at left temporal radation >

6



Homonymous hemianopia

with central sparing

Fig. Visual pathways

6. Homonymous hemianopia with central sparing = lesion at right occipital pole

©

= O

oe)

> tT QO

PAPER-II c.

Tothe knee jerk reflex in hyperthyroidism

d.

(Ref: Textbook of Physiology, AK Jain, Vol. 2, 7th ed. pg. 697)

(Ref: Textbook of Physiology, AK Jain, Vol. 2, 7th ed. pg. 790-791) ANSWER

ANSWER O

Increased

nerve

conduction

due

to

O O O

decreased

sodium leakiness

O

Tospermatogenesis in furnace workers

Decrease sperm count due to excessive heat exposure Infertility Lower temperature than normal body temperature is required for spermatogenesis.

Brisk reflex

PART-III 5.

Write short notes on:

a.

Accommodation reflex (Ref: Textbook of Physiology, AK Jain, Vol. 2, 7th ed. pg. 1102)

ANSWER O

Accommodation refers to the reflex that increases the curvature of the lens needed for near vision.

O

Preganglionic parasympathetic fibers arise in the Edinger-Westphal nucleus and pass via the Oculomotor nerve to the ciliary ganglion. Pretectal area

Afferent Limb: CN Il

Light stimulates geanglion retinal cells > impulses travel up CNII which projects bilaterally to the pretectal nuclei (midbrain) The pretectal nucleus projects bilaterally > Edinger-Westphal nuclei (CN II)

Edinger-

Efferent Limb: CN III

Westphal nucleus

Edinger-Westphal nucleus (pre-ganglionic parasympathetic) —> ciliary ganglion (postganglionic parasymphathetic) > pupillary sphincter muscle — miosis

Ciliary Ganglion

Pupil

Because cells in the pretectal area supply the Edinger-Westphal nuclei bilaterally, shining light in one eye—constriction in the ipsilateral pupil (direct light reflex) and the contralateral pupil (consensual light reflex) Because this reflex does not involve the visual cortex, a person who is cortically blind can still have this reflex

Fig. Pupillary light reflex pathway

O >

(5

O aa O

Ts > a

O

Postganglionic parasympathetic fibers from the ciliary ganglion supply the ciliary muscle. Contraction of this muscle relaxes the suspensory ligaments and allows P ry 5 the lens to increase its convexity (become more round).

ANSWER

This increases the refractive index of the lens, . permitting the image of a nearby object to focus on the reti e retina.

OQ

_

b.

O

estrogen inhibits ovulation by feedback inhibition of

;

ov

,

,

;

LH, thickening of cervical mucous and endometrium el . ; , Minipills/progesterone only pills: Thickens cervical ; vy: mucous and endometrium, decrease tubal motility

O

Sequential pills: 15 days estrogen followed by 5 days progesterone and estrogen

O

Estrogen only pills causing anovulatory cycles

Contraceptive pills

(Ref: Textbook of Physiology, AK Jain, Vol. 2, 7th ed. pg. 822)

Combined pills/classical pills: Progesterone with

PAPER-II c.

Classification of nerve fibers

(Ref: Ganong’s Review of Medical Physiology, ed, pg. 94-95)

Extracellular fluid

Glucose

>

Na’ -glucose

ANSWER Table:

symport transporter

releasing glucose to the cytoplasm

Erlanger and Gasser classification

Type of fiber

Physiological role

Diameter (uum)

|Conduction velocity

|Most susceptible to

a) Aa

Proprioception, Somatic motor

12-20

70-120

B

Touch, pressure

5-12

30-70

Y

Motor to muscle 3-6 spindles

15-30

Pain, Cold,

12-30

6

2-5

PRESSURE

(4) Sodium-

Preganglionic autonomic

C

Dorsal Pain root Temperature

Table 13:

©

Tight junctions

Fig. Blood-testis barrier

= O

oe)

> tT QW

PAPER-II Formed by = Capillary endothelium, = Capillary basement membrane,

PHYSIOLOGY

O00

="

Connective tissue,

= Basement membrane of germinal epithelium, = Sertoli cells tight junction. Permeable to testosterone and other steroids Poorly permeable to proteins Protects germ cells from autoimmune reaction on exposure to blood

O

Maintenance of seminal fluid

c.

Endocohlear potential

(Ref: Textbook of Physiology, AK Jain, Vol. 2, 7th ed. pg. 1078)

ANSWER For answer refer 2017 paper-2, Q. 6(c), Pg. 188

Your Roll No. ...............

Name ofthe Paper

:

Physiology Paper-I

Name ofthe Course

:

MBBS-2013

Semester

Annual

Time: 3 Hours

M.M.: 50

INSTRUCTIONS Write your Roll No. on the top immediately on receipt of this question paper 2.

All questions are to be attempted

3.

Answers to Parts I, II and III should be written in separate answer sheets provided

4.

Attempt parts of a question in sequence

PART-I 1.

Define Cardiac Output. Enumerate various factors affecting it. Describe one method of measuring it.

[3]

2.

Draw a well labelled diagram/flow chart/graph:

[3]

a. b. c. d.

Intrinsic pathway of coagulation FEV1 in normal subject and in obstructive and restrictive lung diseases Hairpin counter current multiplier system in kidney Juxtaglomerular apparatus

PART-II 3.

4.

Name the different phases of gastric secretion. Explain the regulatory mechanisms of each phase. Add a note on

Peptic ulcer.

[3]

What happens and why?

[3]

a. b. c. d.

If fetal hemoglobin persists during adult life If pulmonary surfactant is absent in newborn To osmolality of urine in a patient of diabetes insipidus If parasympathetic innervations to the salivary glands are interrupted

PART-III 5.

6.

Write short notes on:

a.

Secondary active transport

b. c.

Fibrinolytic system Hering-Breuer reflexes

Write the physiological/clinical significance of the following:

a. b. c.

Coronary blood flow in sub-endothelial portion of left ventricle Ventilation-perfusion ratio at the apices of the lungs Loop diuretics

[9]

[9]

PAPER-I |

2013 PAPER-I

PART-I 1.

Define Cardiac Output. Enumerate various factors affecting it. Describe one method of measuring it.

O

The upward curve after decline shows recirculation thus graph is extrapolated from decline showing the first passage of dye

O

Cardiac Output=total amount of dye injected (I) / (mean dye conc. -c ) (t- duration of first pass)

2.

Drawa

(Ref: Textbook of Physiology, AK Jain, Vol. 1, 7th ed. pg. 342) ANSWER

|

well labelled diagram/flow chart/graph:

Cardiac Output

a.

Cardiac output is the amount of blood pumped from each ventricle per minute

(Ref: Textbook of Physiology, AK Jain, Vol. 1, 7th ed. pg. 98) ANSWER

90000%0d~d0D80dL6O

Factors Affecting Venous Return Respiratory pump

For answer refer 2015 paper-1, Q. 2(b), Pg. 202

Muscle pump Cardiac pump Capacitance of veins Body posture Fluid volume Ventricular distensibility " Homometric regulation: When there is increase or decrease in myocardial contractility independent of preload conditions.

b.

OOO

Pathology

O

Sympathetic stimulation/catecholamines: Increase Glucagon, digitalis, xanthine: Increases Parasympathetic/vagal discharge: Decreases

hyperkalemia and hypercapnia decreases quinidine Anesthetics, barbiturates,

MI,

heart

failure,

hypoxia,

FEV1 in normal subject and in obstructive and restrictive lung diseases.

(Ref: Textbook of Physiology, AK Jain, Vol. 1, 7th ed. pg. 416)

ANSWER

Forced Vital Capacity Measurement O

In this test, subject inspires maximally and then exhales as hard and as completely as he or she can into the spirometer for 6 seconds

O

The record obtained is called flow volume which is shown below

Factors Affecting Contractility

O

Intrinsic pathway of coagulation.

4 Flow (L/s) PEF

acidosis,

and

procainamide: Decreases

Dye Dilution Method of Measuring Cardiac Output (Evan’s Blue/Radioactive Isotope) O O

PHYSIOLOGY

O O

After withdrawal of venous blood addition of the dye to the blood and injection of the same into the vein Immediate withdrawal of arterial sample in series of tubes with 0.5 -2 sec interval Analysis of the concentration of dye in successive samples After reaching a peak it declines when plotted on graph

TLC

Inspiration

Fig.

Flow Volume loop

loop

PAPER-I Obstructive Lung Diseases (Emphysema, Bronchial Asthma) In obstructive lung diseases,

4.

The predicted curve

e The height of the curve (PEF) is much less than predicted e Descending part of expiration (effort

low PEF

%,

independent part) is predominantly affected. It is concave due to airflow limitation in small

a

2

=5

Fi FEV1

Co-transport Gath

Glucose

©

Na

O

Glucose

Na’

= oe)

—70 mV

> tT QO

Amino acids

PAPER-I Secondary active transport

Another example of a secondary active transport is Na-K-2Cl co-ransport In thick ascending limb of Henle Recall from pharmacology! “loop” diuretics as furosemide, inhibits the action of this transporter

The direct source of the energy is that liberated by

| simultaneous facilitated diffusion

The of another transported substance Down

its own

Thick ascending limb

electrochemical

gradient

Lumen +7 mV

The electrochemical gradient for Na is maintained by Na/K pump

Cell of the thick ascending limp Cs

Blood 0 mV

Na’

xk

O

PART-II 3.

O O

Describe the types and mechanism of water reabsorption in renal tubules. What are the factors affecting these mechanisms?

Factors Affecting

(Ref: Textbook of Physiology, AK Jain, Vol. 1, 7th ed. pg. 535-537)

ANSWER

O

PCT: 65-75% by osmosis, isotonic to plasma

tubular

Thin descending loop of Henle by osmosis due to hypertonic interstitium Early DCT: 5-8% by osmosis Late DCT and collecting tubule under effect of antidiuretic hormone through aquaporin channels

fluid remains

O

Osmolality of plasma

O O

Hypertonicity of medullary interstitium Anti-diuretic hormone

Distal convoluted tubules

4

“a

™ Reabsorption of ions, water and all organic

Renal capsule

Distal convoluted

Secretion of ions, drugs and toxins. Variable reabsorption of water,

production of

tubules

sodium ions and calcium ions

filtrates

(under hormonal control)

Hy

—— r

LAS

it

S

=>

Collecting ducts

Proximal convoluted

limb

Thin descending

—>

aa> Actively

Thick — ascending

tubules

—>

=

~

WW

—>

==>

limb

PHYSIOLOGY

-_ Loop of Henle Further reabsorption of

in —

mam> Passively

ascending limb

water (descending limb) and both sodium

\

Collecting ducts Variable reabsorption of

sf

and chloride ions

(ascending limb)

>

|

e.

ie

water and reabsorption

Variable ====,>-

or secretion of sodium. potassium, hydrogen and bicarbonate ions

Water =>

Loop of henle

Fig. Mechanism of tubular absorption

Solutes —>

PAPER-| Renal physiology Glomerulus Does not allow proteins through (due to podocytes), Mesh of collagen and connective tissue, Filters ECF

DCT Active transport of Na” and CI under control of Aldosterone Ca” absorption under the control of PTH Proximal part ! Distal part i K

!

Urea

Creatinine, Antibiotics,

Diuretics, Uric acid

PCT

!

Reabsorb all nutrients

by active transport k H,0 HCO, oe Secrete Urea and toxins NaC! Absorb water driven by sodium

NaCl

aA

Cortex

! H,O __—|NaCl

Ca “PTH”

ae

| “Aldosterone”

ae

enone ge eee

H.O

NaCl

Cl HCO, H K Na

es

Piney

H,0

Medulla

basins

Collecting duct and tubules ADH from CNS thirst centres due to angiotensin Il causes H,O absorption through

increasing aquaporin ADH

Mg

Ca

Loop of Henle Thin descending limb only reabsorbs water, thick ascending limb co-trasports Na’ and Cl'along with passive absorption of Mg and Ca

Fig. Events occuring in renal tubules

4.

What happens and why? a.

Torespiration following bilateral vagotomy

(Ref: Textbook of Physiology, AK Jain, Vol. 1, 7th ed. pg. 446)

'

NPBM

\

\

KF

\ \

O

°

°

.

Bilateral pneumotaxic centre (PC): Ventilated with phrenic triggered pump, suppression of lung

NTS

Bilateral

vagotomy:

Unilateral

Vagotomy

/

\

\

\ \

N.p.Amb

La 7»



lesion

with

I

/ /

\

1,

,

/ \

YP \/ v,

/

f

/

A S

/

/

\ S

/

/

/

/

/

/

f \

-.

/

X

VPA -—{ — — infl.

MALL 4 PC coag.

Fy

:

ce

consequent apneusis.

PC Lesions DARKO 7~— T

\ \

inflammation produced apneusis. O

/

/

\

\

ANSWER ee

\ \

\

'

AKA 7 — = _ 9S sec

Fig. Schematic representation of bed nuclei from which , , . inspiratory drive originates. X= Vagus nerve, NPBM= Nucleus Para brachialis Medialis, NTS= Nucleus of Solitary tract, N. p.

Amb= Nucleus Periambigualis 40sec >

©

= O

oe)

> tT QO

PAPER-I b.

To systemic blood pressure when both renal arteries are clamped

(Ref: Textbook of Physiology, AK Jain, Vol. 1, 7th ed. pg. 354-355)

sympathetic input contracts the internal sphincter via a-1 receptors. O

As a result, the bladder can fill with urine.

Voiding Phase ANSWER O

O

distension of the bladder.

Condition mimics renal disease O

O00 9

Decrease RBF

Stimulation of renin angiotensin system Renal hypertension

in the sufficiently mature individual, it sends input

To micturition in complete transaction of the spinal cord

to the medulla and cortex signaling that voiding is needed.

(Ref: Textbook of Physiology, AK Jain, Vol. 2, 7th ed. pg. 935-938)

PNS activation causes contraction of the detrusor muscle (M3). This initiates voiding. However, the external sphincter is controlled voluntarily (nicotinic receptor). If voiding is inappropriate at that moment, voluntary contraction of this sphincter stops the voiding process.

ANSWER Micturition Reflex

O

Micturition is a reflex regulated by the peripheral nervous system. Theautonomiccomponentexists at birth, continuing throughout life, but the motor component requires sufficient maturation of the nervous system (occurs around age 2).

O

O

If the

,

Pelvic nerve

ear

However, continued filling of the bladder increases pressure and _ re-initiates the sensory input attempting to start the voiding process. Typically,

Urethra

— = Relaxation

Internal sphincter




©

L O

oe)

> tT QO

Your Roll No................

Name of the Paper

:

Physiology Paper-II

Name of the Course

:

MBBS-2012

Semester

Annual

Time: 3 Hours

M.M.:

50

INSTRUCTIONS Write your Roll No. on the top immediately on receipt of this question paper 2.

All questions are to be attempted

3.

Answers to Parts I, II and III should be written in separate answer sheets provided

4.

Attempt parts of a question in sequence

PART-I 1.

Enumerate the functions of hypothalamus. Describe any one of them in detail. Add a note on leptin.

[8]

2.

Draw labelled diagram/Flow charts of the following:

[8]

a. b. c. d.

Sleep cycle in an adult Renshaw cell inhibition Visual pathway with lesion at optic chiasma Sarcomere in skeletal muscle

PART-II 3.

Enumerate glucocorticoid hormones. Discuss their role as anti-inflammatory and antistress hormone. Add a note on cushing syndrome. [8]

4.

What happen and why? a. b. c. d.

[8]

To gonadotropin level after menopause To neuromuscular excitability in parathyroid hormone deficiency When pain fibers in the stump of an amputated that get stimulated When dopamenergic neurones are destroyed in basal ganglia

PART-III 5.

Write short notes on the followings:

a. b. c. 6.

[9]

Hormones involved in milk production and milk ejection Dark adaptation in eye Spinal shock

Give physiological/clinical significance of the followings: a.

Tympanic reflex

b. c.

Regulation of intraocular pressure Na —K’* pump

[9]

PAPER-II

2012

|

2012 PAPER-II

O

PART-I 1.

O

Enumerate the functions of hypothalamus. Describe any one of them in detail. Add a note on leptin.

O

(Ref: Textbook of Physiology, AK Jain, Vol. 2, 7th ed. pg. 990)

O ANSWER

Functions of Hypothalamus

O

O

O O

Endocrinal function: secretion of releasing hormones (GnRH, PRH, GHRH, TRH, CRH ) and inhibiting hormones (prolactin inhibiting hormone and growth hormone inhibiting hormone) Autonomic function: Anterior hypothalamus has parasympathetic function and_ posterior hypothalamus has sympathetic function Sleep: Sleep faciliatory centers are present in anterior hypothalamus, waking centers in posterior hypothalamus. Circadian rhythm maintenance through retinohypothalamic fibers and lateral geniculate nuclei Feeding regulation is done by feeding center in lateral hypothalamic nucleus and satiety is regulated by ventromedial nucleus.

Reproductive function, maintenance of female estrous cycle and ovulation through tuberal region Instinctual and emotional behavior is regulated hypothalamus along with limbic system. Reward center along medial forebrain bundlein lateral and ventromedial nucleus and punishment center in periventricular zone of medial hypothalamus Body temperature regulation: Anterior hypothalamus involved in heat loss and posterior hypothalamus involved in heat gain. Thirst stimulated by lateral nucleus of hypothalamus and osmoreceptors in supraoptic nucleus secrete ADH.

Leptin O O

O

Hormone produced by adipose tissue Energy expenditure hormone, helps in energy homeostasis by hunger, use of food energy and physical activity. JAK stat receptors on arcuate nucleus in hypothalamus, and adipose tissues

O

Adipose tissue mass regulation

O

Facilitates immunity

O O O

Facilitatory function in beta cells of islets. Permissive action in puberty Facilitates action of growth hormone

PFA Ca labolic; Pathways ag” Response to satiety signals

POMC

Fat mass

Adiposity

signals ~

Anabolic pathways

NTS

:

Gl tract

Satiety .

~~

“ie

-

ar

\

4

g

a

.

\ A

Mechanical

a

agus nerve

Superior cervical li

Cervical spine SNS afferents

ov

Chemical ——__— >

Energy Va

O

O

)

metabolism

4 CCK release

Fig. Central nervous system involvement in feeding behavior via leptin

>

©

= O

oe)

> tT QW

PAPER-II 2.

Draw labelled following: a.

diagram/Flow

charts

of

the

ANSWER O

Renshaw cells are inhibitory cells in the ventral horn of the spinal cord.

O

They receive input from collateral motoneurons and, when stimulated, feedback (inhibit) on the motoneuron.

Sleep cycle in an adult

(Ref: Textbook of Physiology, AK Jain, Vol. ??, 7th ed. pg. ??)

ANSWER

axons of negatively

Descending pathways

Sleep Cycle O

Sleep consists of 2 distinct states: NREM and REM.

O

Non-rapid Eye Movement (NREM) alternates with REM sleep throughout the sleep period. It is divided into 3 stages on the basis of EEG criteria: = Slowing of the EEG rhythms

la inhibitory interneuron

Motor neurons

= Higher muscle tone = Absence of eye movements = Absence of “thought-like” mental activity Renshaw cell (Interneuron)

=" NREM is an idling brain in a movable body.

O

Rapid eye movement (REM) is an awake brain in a paralyzed body: = Aroused EEG pattern = Sexual arousal = Saccadic eye movements

al

Extensor

Flexor

muscle

muscle

=" Dreaming Sleep

REM

NREM

2

:

.

Stages: 1

2

3

Fig. Renshaw cell inhibition

c.

(Ref: Textbook of Physiology, AK Jain, Vol. 2, 7th ed. pg. 1094)

Delta sleep

Fig. Stages of sleep

Visual pathway with lesion at optic chiasma

ANSWER

Sleep Facts Stage 2

Longest stage of sleep

Stage 3

e Deepest stage of sleep; delta sleep is restorative e Tends to decrease in the elderly

PHYSIOLOGY

d.

Sarcomere in skeletal muscle

(Ref: Textbook of Physiology, AK Jain, Vol. 1, 7th ed. pg. 164)

About 5-15 min from time one goes to bed and falls asleep

ANSWER

REM latency

About 90 min from time one falls asleep to first REM period

O

REM

e First REM period of night is 5-15 min and last one is 20—40 min REM increases as night goes on Greater amounts in second half of night Easiest to arouse Memories are consolidated by hippocampus

Amuscle is made up of individual cells called muscle fibers. Longitudinally within the muscle fibers, there are bundles of myofibrils.

O

A myofibril can be subdivided into individual sarcomeres. A sarcomere is demarked by Z lines.

O

Sarcomeres are composed of filaments creating bands. Contraction causes no change in the length of the A band, a shortening of the I band, anda shortening in the H zone (band).

Sleep latency

250

For answer refer 2014 paper-2, Q. 4 (b), Pg. 225

Greater amounts in first half of night

NREM

b.

Renshaw cell inhibition

(Ref: Textbook of Physiology, AK Jain, Vol. 2, 7th ed. pg. 862-865)

O

PAPER-II O

Actin anchors myosin and is an _ important component of striated muscle’s elasticity.

Sarcomere canna

M creatine kinase

ees ' '

!

Vascular reactivity: Sensitization of arterioles to NE for maintained of arteriolar tone. Absence increase the permeability and loss of fluid from capillaries to ECF,

Actin

H-Zone

Permissive action: Adjunction with other hormones to cause calorigenic effect (glucagon and catecholamines), NE and E for lipolysis and bronchodilation.

1

;



!

——

or

B08

Mild mineralocorticoid action: Sodium and water

> aA

retention,

ra >

Nh

a

>

@

Myosin

N

M-Line

Titi

5

C protein

>>

Desmin

A-Band

excretion,

inhibits

ADH _ secretion

Cushing’s Syndrome (Demonstrated by Harvey Cushing) O

Fig. Sarcomere

PART-II 3.

K

causing diuresis.

*

i“

Enumerate glucocorticoid hormones. Discuss their role as anti-inflammatory and anti -stress hormone. Add a note on Cushing syndrome.

Cause: Excessive glucocorticoids (Increased ACTHACTH dependent, Low ACTH-ACTH independent as in cortical tumor of adrenal glands) = =

Increased protein catabolism Hyperglycemia due to glycogenolysis, gluconeogenesis causing insulin resistant DM

=

Lipolysis with differential redistribution causing buffalo hump and central obesity

of fat

=" Osteoporosis

(Ref: Textbook of Physiology, AK Jain, Vol. 2, 7th ed. pg. 722-726)

=

Increase activity of collagenase enzyme-purple stria

=" Increase in sodium water hypertension and diuresis

ANSWER

retention

causing

=" Hirsutism

Glucocorticoid Hormones

= Moon face

Adrenal Cortex O

Glomerulosa: Aldosterone

O

Fasciculata: Cortisol

O

Reticularis: Dehydroepiandrosterone

=

Restlessness and psychosis

=

Increase susceptibility to peptic ulcers

=

Frequent infections due to decreased immunity

= Amenorrhea and loss of libido

Adrenal Medulla

Epinephrine 20% Norepinephrine 80%

What happen and why.

a.

Functions

O

Role of Glucocorticoids inhibiting =" Phospholipase A2 =

O

in

Inflammation

Synthesis of Leukotrienes

by

To gonadotropin level after menopause

(Ref: Textbook of Physiology, AK Jain, Vol. 2, 7th ed. pg. 808-809) ANSWER

= Thromboxane

O

Ovaries no longer synthesis 17 beta oestradiol No negative feedback

= Prostaglandins

O

Increase amount of FSH and LH secretion

O

= Prostacyclin Role in Immunity

b.

= Lymphopenia

To neuromuscular excitability in parathyroid hormone deficiency

(Ref: Textbook of Physiology, AK Jain, Vol. 2, 7th ed. pg. 718)

= Neutrophilia =" Basopenia

= Decrease size of lymph nodes = Inhibits NF-kB = Inhibits cytokines and interleukins

ANSWER O

Parathyroid levels

deficiency decreases

plasma

calcium

PHYSIOLOGY

O O

PAPER-II O O

Decrease in the threshold potential of nerve Increase in the neuromuscular excitability c.

When pain fibers in the stump amputated that get stimulated

of

an

O

Slowresponse: Over 25 min, adaption of peripheral portion of retina, mydriasis, rods take over

O

Rhodopsin is synthesized

O O O

Phantom limb Lawof projection The sensory neural pathway when stimulated at any point, the pain will be perceived from the site of sensory receptor. d.

When dopaminergic neurons are destroyed in basal ganglia

+#————_The

cones are more sensitive here

millilamberts

ANSWER

Log intensity of minimal effective stimulus (visual threshold stimulus)

(Ref: Textbook of Physiology, AK Jain, Vol. 2, 7th ed. pg. 902-905)

Cones Rod Cone break

The rods are more sensitive here

———

(Ref: Textbook of Physiology, AK Jain, Vol. 2, 7th ed. pg. 999-1002)

Time in dark (min)

O

ANSWER

c.

Spinal shock

(Ref: Textbook of Physiology, AK Jain, Vol. 2, 7th ed. pg. 935-937)

Parkinsonism

O00

Festinate gait

Pill rolling movement

ANSWER

Coarse tremors

O

PART-III O Write short notes on the followings: a.

Hormones involved in milk production and milk ejection

(Ref: Textbook of Physiology, AK Jain, Vol. 2, 7th ed. pg. 845-846)

O00 0

5.

First stage, flaccid paralysis occurs in any kind of spinal trauma Body below injury experience motor as well as sensory loss Decreased venous return Blue extremities Dry scaly skin Bedsores

ANSWER

PHYSIOLOGY

O

Prolactin: Lactogenesis =" Secreted from anterior pituitary = Stimulates galactosyltransferase to produce lactose Oxytocin: Milk letdown or milk ejection reflex =" Baby suckles = Stimulation of receptors present on nipples = Stimulation of posterior pituitary = Release of oxytocin = Contraction of lactiferous ducts and sinuses =" Release of milk and baby suckles more. b.

ANSWER

O

Fast response: (4-5 min)

Paradoxical

adaptation

of cones

significance

of the

Tympanic reflex (Ref: Textbook of Physiology, AK Jain, Vol. 2, 7th ed.

pg. 1075-1076)

ANSWER O

Also known as attenuation reflex

O

Sudden loud sound

O

Reflex contraction muscle

Dark adaptation in eye

(Ref: Textbook of Physiology, AK Jain, Vol. ??, 7th ed. pg. ??)

Give physiological/clinical followings: a.

Decrease

OO

O

6.

of tensor

tympani,

stapedius

sound transmission

Protection from loud noise to prevent damage auditory receptors b.

of

Regulation of intra-ocular pressure

(Ref: Textbook of Physiology, AK Jain, Vol. 2, 7th ed. pg. 1090)

PAPER-II ANSWER

O00

O

Production of aqueous humor from ciliary processes From posterior chamber to anterior chamber Canal of schlemm into intrascleral venous plexus From venous plexus into anterior ciliary veins c.

Na*t-K* pump (Ref: Textbook of Physiology, AK Jain, Vol. 1, 7th ed. pg. 18)

ANSWER 6. K+ is released and Na’ sites are

ready to bind Na* again; the cycle repeats

Extracellular

1. Binding of cytoplasmic Na’ to

fluid

the pump protein stimulates phosphorylation by ATP

Cytoplasm

(=

@-/—e @) @_

2. Phosphorylation causes the

protein to change its shape

Concentration gradients

of K’ and Na’

3. The shape change expels

5. Loss of phosphate restores the original conformation of the pump protein

Na’ to the outside, and extracellular K* binds

4. K’ binding triggers release of the phosphate group

Fig. Sodium-Potassium pump

>

©

= O

oe)

> tT QW

Your Roll No

Name of the Paper

:

Physiology Paper-!

Name of the Course

:

MBBS-2011

Semester

:

Annual

Time: 3 Hours

M.M.: 50

INSTRUCTIONS 1.

Write your Roll No. on the top immediately on receipt of this question paper

2.

All questions are to be attempted

3.

Answers to Parts I, II and III should be written in separate answer sheets provided

4.

Attempt parts of a question in sequence

PART-I 1.

2.

Compare and contrast:

[3]

a.

Extrinsic and intrinsic coagulation mechanisms

b. c. d.

Reabsorption in proximal and distal renal tubules Pulmonary and systemic circulation Humoral and cell mediated immune responses.

Draw labelled diagrams/flowcharts to illustrate:

a. b. c. d.

[3]

Enterohepatic circulation of bile salts Normal spirogram Left atrial left ventricular and aortic pressure changes and heart sound during a normal cardiac cycle Juxtaglomerular apparatus and its role in physiology.

PART-II 3.

a.

Five normal value/range and its applied/clinical significance. 1. 11. 11. iv.

b.

FS

[2]

Heart rate during inspiration and expiration? Rh negative woman if she is transfused with Rh positive blood?

Write short notes on:

ao

[2]

Pulmonary surfactant HCl secretion.

What happens and why to? i. 11.

4.

albumin globulin ratio corpuscular hemoglobin concentration vital capacity pressure.

Enumerate two points each:

1. 11. c.

Serum Mean Timed Blood

[4]

Neural regulation of respiration Cardiac output and its regulation Stage of erythropoiesis Counter current mechanisms in kidney.

[10]

PART-III 5.

a.

Define the following: 1. 11. 111. iv.

b.

Renal threshold of glucose Obstructive jaundice Caisson’s disease Oncotic pressure.

Explain why? (3-4 sentences) 1. 11. 111. iv.

Oral rehydration therapy is advised in infants with diarrhoea Dietary fiber intake is recommended Proton pump inhibitors are given for peptic ulcer. Cardiac muscle cannot be tetanized.

oF SP

Discuss briefly:

ao

6.

[4]

Tubuloglomerular feedback Changes in blood at high altitude. Aganglionic megacolon Non-respiratory functions of lung.

[8]

PAPER-I |

2011 PAPER-I

PART-I

d.

)

Humoralandcellmediatedimmuneresponses

(Ref: Textbook of Physiology, AK Jain, Vol. 1, 7th ed. pg. 124-130)

1.

Compare and contrast: a.

Extrinsic

and

intrinsic

ANSWER

coagulation

mechanisms (Ref: Textbook of Physiology, AK Jain, Vol. 1, 7th ed. pg. 101) ANSWER

Humoral Immune response

Cell mediated immune response

e Formation of IgG, IgA,IgE antibodies

e Formation of cytotoxic CD-8 cells

e

e T-cell mediated

B-cell mediated

e IL-4, IL-5 Intrinsic Mechanism

Extrinsic Mechanism

e Activated due to exposure to damaged endothelium or collagen

e Activated due to injury of tissue or vessel wall

e

e Release of factor Ill (tissue thromboplastin)

Factor XII converted to

activated factor XII e Factor 12, 11 and 9 involved

b.

Reabsorption tubules

e

2.

and

distal renal

(Ref: Textbook of Physiology, AK Jain, Vol. 1, 7th ed. pg. 527-537)

Draw labelled diagrams/flow charts to illustrate: (Ref: Textbook of Physiology, AK Jain, Vol. 1, 7th ed. pg. 244)

a.

Factor 3 and 7 involved

in proximal

For answer refer, 2017 paper-I, Q. 2 (b), Pg. 175

ANSWER

Normal spirogram

(Ref: Textbook of Physiology, AK Jain, Vol. 1, 7th ed. pg. 415) PCT

DCT

Na

70-85 percent

5-20 percent

Glucose

Below T max 100% reabsorbed

-

K

90%

Remaining 10 percent goes in DCT and resorbed there

HCO3-

90% (secondary active transport)

10-15%

H ions

85%

10%

Water

65-75% passive reabsorption

ADH -aquaporin dependent facultative reabsorption, 10-12%.

Pulmonary and systemic circulation

ANSWER

For answer refer, 2017 paper-I, Q. 2 (a), Pg. 174 c.

ANSWER Phases—Left Ventricle

O

O ANSWER

e Pulmonary trunk, arterioles have deoxygenated blood

e Pulmonary venules and veins have oxygenated blood

e Low resistance, low pressure

e High resistance, increase blood pressure

O

Systemic Circulation

O

O Pulmonary Circulation

Left atrial left ventricular and aortic pressure changes and heart sound during a normal cardiac cycle

(Ref: Textbook of Physiology, AK Jain, Vol. 1, 7th ed. pg. 289)

(Ref: Textbook of Physiology, AK Jain, Vol. 1, 7th ed. pg. ??)

PHYSIOLOGY

Enterohepatic circulation of bile salts

ANSWER

b.

c.

e IL-2, gamma interferon

Isovolumetric contraction—period between mitral valve closing and aortic valve opening; period of highest O, consumption Systolic ejection—period between aortic valve opening and closing Isovolumetric relaxation—period between aortic valve closing and mitral valve opening Rapid filling—period just after mitral valve opening Reduced filling—period just before mitral valve closing

PAPER-| Heart Sounds

O O O O

$§1—mitral and tricuspid valve closure. Loudest at mitral area. §2—aortic and pulmonary valve closure. Loudest at left upper sternal border. §3—in early diastole during rapid ventricular filling phase. Associated with increasing filling pressures (e.g., mitral regurgitation, HF) and more common in dilated ventricles (but can be normal in children and young adults). §4—in late diastole (“atrial kick”). Best heard at apex with patient in left lateral decubitus position. High atrial pressure. Associated with ventricular noncompliance (e.g., hypertrophy). Left atrium must push against stiff LV wall. Consider abnormal, regardless of patient age. Systole

© “ao

= E

4+ 80 =

60 -

=

=

% o

©

o

5

eg @§

§ 8

2 @c

= =

5

oES.

365 28

85 =

&

| §2 roe:

|

Aortic valve

opens a ee

3 2s

38 | 88 Yo 2o

82 SE

&§ “8

>i \¢ >i |¢ —— >| It



100 -

rs

0

>| 14-91 |—> || \+——

120 =

0 53 Of

£2 te

Diastole

355 r=

o

>

y Aoevene closes -

/

ee

|

Aortic

~~-..___ pressure remy

Dicrotic notch

- Left ventricular pressure

40-

| Mitral valve closes 20 4 | S| Ze

Left atrial pressure

ee

eee

|

ee eee a

0-

valve opens

ae

i i

ae

ee

alia =

S2 Heart

ala INN.

sounds

Ventricular

~

Ae S

(NV

[am

~ NO a NT

a

c

Jugular

Vv

pulse

ECG

ed

!

R =,

_—

Je.

P

Q S 0

|

t

0.1

|

0.2

|

0.3

0.4 Time (sec)

|

0.5

|

0.6

t

0.7

0.8

Fig. The cardiac cycle

d.

Juxtaglomerular apparatus and its role in physiology ae

an

(5

an

>

(Ref: Textbook of Physiology, AK Jain, Vol. 1, 7th ed. pg. 511)

ANSWER For answer refer, 2013 paper-I, Q. 2 (d), Pg. 231

O

O

a=

aw

PAPER-I PART-II 3.

a.

ANSWER

Give normal value/range clinical significance.

i.

and

its applied/

For answer refer, 2016 paper-I, Q. 4 (a), Pg. 191 ii.

HCl Secretion

(Ref: Textbook of Physiology, AK Jain, Vol. 1, 7th ed. pg. 222)

Serum albumin globulin ratio

(Ref: Textbook of Physiology, AK Jain, Vol. 1, 7th ed. pg. 51) ANSWER ANSWER

For answer refer, 2014 paper-I, Q. 2 (c), Pg. 217

Albumin: 3-5 mg/dL Globulin: 2-3 mg/dL

c.

What happens and why to: i.

Ratio is 1.7: 1 ="

Decrease

serum

Heart’

during

rate

inspiration

and

expiration?

in serum

albumin

oncotic pressure

causes decrease in

predisposing

to tissue

(Ref: Textbook of Physiology, AK Jain, Vol. 1, 7th ed. pg. 111, 339)

edema

ii.

Mean corpuscular hemoglobin concentration

(Ref: Textbook of Physiology, AK Jain, Vol. 1, 7th ed. pg. 74) ANSWER

OO O

Normal FEV] is 80% of FVC FEV2 is 95% of FVC FEV3 is 98-100% of FVC Differentiation

of

obstructive

from _

restrictive

Blood pressure

(Ref: Textbook of Physiology, AK Jain, Vol. 1, 7th ed. pg. 349-351) ANSWER

OO

PHYSIOLOGY

It is the lateral pressure exerted on the walls of the vessels Arterial blood pressure Systolic 120 mm Hg Diastolic 80 mm Hg b.

Enumerate two points each i.

Expiration HTH

HeH

@

_ Inspiration @

Hit

Se

@

Rh negative woman if she is transfused with Rh positive blood?

(Ref: Textbook of Physiology, AK Jain, Vol. 1, 7th ed. pg. 110-112)

ANSWER

About 85% of people are Rh+. If a pregnant woman is Rh- and the father is Rh+, there is a chance that

pulmonary disorder

O

ii.

O

ANSWER

O

“Respiratory sinus arrhythmia = ——=*~*~S=«S mms

Timed vital capacity

(Ref: Textbook of Physiology, AK Jain, Vol. 1, 7th ed. pg. 416-418)

iv.

Tachycardia during inspiration Bradycardia during expiration

OPI SROs er O

Determination of type of anemia along with other parameters like MCH, MCV.

O

O O

Inspiration

Approx. 35% Decreases in iron deficiency anemia

iii.

ANSWER

Pulmonary Surfactant

(Ref: Textbook of Physiology, AK Jain, Vol. 1, 7th ed. pg. 419)

the fetus will also be Rh+. This situation will pose no problem in the first pregnancy, as the mother’s immune system will not usually encounter fetal blood cell antigens until placental separation at the time of birth. At that time, however,

Rh+ fetal

red blood cells will enter the maternal circulation and

stimulate

a T-dependent

immune

response,

eventually resulting in the generation of memory B cells capable of producing IgG antibody against RhD. In a subsequent pregnancy with another Rh+ fetus, this maternal IgG can be transported across the placenta,

react with

fetal Rh+

red

cells,

and

activate complement, producing hemolytic disease. Hemolytic disease of the newborn can be prevented by treating the Rh- mother with RhoGAM, a preparation of human anti-RhD antibody, at 28 weeks of gestation and again within 72 hours after birth. This antibody effectively eliminates the fetal Rh+ cells before they can generate RhD-specific

PAPER-| =" Hypoventilation (breath-holding) is limited by

memory B cells in the mother. Anti-RhD antibody should be given to any Rh- individual following any termination of pregnancy.

4.

the resulting increase in Pco, and decrease in Po,,. A previous period of hyperventilation extends the period of breath-holding.

Write short notes on:

Chemoreceptors for CO,, H*, and O,

a.

Comparison of Central and Peripheral Chemoreceptors

Neural regulation of respiration (Ref: Textbook of Physiology, AK Jain, Vol. 1, 7th ed. 444)

Control of Breathing O

Location

Stimuli that Increase Breathing Rate

Central

Medulla

J pH 4 Pco2

Peripheral

Carotidand aortic bodies

J Po2 (if tT QW

PAPER-II ii.

Lumbar puncture

ANSWER O O

Used to determine constituents of cerebrospinal fluid Used to study pathological changes e.g. Tubercular meningitis c.

O

Naand water retention

O O

Metabolic Alkalosis Kand H ion excretion

O

Pressure diuresis, aldosterone escape b.

Sensory aphasia (Ref: Textbook of Physiology, AK Jain, Vol. 2, 7th ed. pg. 1037-1039)

What will happen and why if: i.

Presbyopia occurs in a myopic person

ANSWER Left

(Ref: Textbook of Physiology, AK Jain, Vol. 2, 7th ed.

Right

pg. 1105-1106) Broca area

ANSWER

Myopia image is formed before retina Presbyopia due to aging of lens image is formed behind retina Due to senile changes in myopic the presbyopia gets corrected because near point would decrease ii.

a~

Facial area of motor cortex (area 4)

;

(area 22)

occurs

3 =

Urinary bladder is completely denervated

(Ref: Textbook of Physiology, AK Jain, Vol. 1, 7th ed. pg. 583-584)

Denervated bladder/isolated or decentralized bladder means no afferent or efferent supply.

O

Cauda equina injury

O

Voluntary control is lost Early stages involve incomplete micturition followed by dribbling

4.

by

chest

5.

I. Define the following: a.

Mean blood pressure

(Ref: Textbook of Physiology, AK Jain, Vol. 1, 7th ed. pg. 349-350)

various

Secondary active transport

(Ref: Textbook of Physiology, AK Jain, Vol. 1, 7th ed. pg. 20-21)

ANSWER

ANSWER

O

Average cycle

O

It determines pressure head

O

All cardiovascular changes in MBP 95-100 mm Hg

MBP = DBP + 1/3 pulse pressure

For answer refer 2014 paper-2, Q. 6 (a), Pg. 227

b.

measles,

PART-III

Write short notes on:

a.

(like-

OO

O

O

Later involves automatic emptying neuromuscular mechanisms.

diseases

=" Use of overdiluted milk, premature termination of pregnancy, etc.

ANSWER

O

Protein calorie malnutrition (2010 paper I) Most common form of malnutrition Commonly seen in infants and young children Common etiological factors are: = Inadequate diet (quantity and quality)

b.

blood

pressure

in the complete

reflexes

are

cardiac

dependent

on

Transport maximum (Tm)

(Ref: Textbook of Physiology, AK Jain, Vol. 1, 7th ed. pg. 529)

Starling’s forces

(Ref: Textbook of Physiology, AK Jain, Vol. 1, 7th ed. pg. 57) ANSWER ANSWER

For answer refer 2016 paper-1, Q. 2 (d), Pg. 190

It is the maximum physiological limit substance to pass through the membrane

O

E.g. Renal T max of glucose is 300 mg/dL Maximum reabsorption of glucose will only occur till 300 mg/dL beyond which it will be excreted in

Fibrinolytic system O

Cc.

O

(Ref: Textbook of Physiology, AK Jain, Vol. 1, 7th ed. pg. 103) ANSWER

O

Helps dissolution of clot

of

the >

©

=

urine.

O

c.

> tT

VO, Max

(Ref: Textbook of Physiology, AK Jain, Vol. 1, 7th ed. pg. 487)

oe)

QO

PAPER-I ANSWER

OO0 0

O

ANSWER

Maximum volume of oxygen consumed Maximum aerobic work capacity HR=220-age in years

O

RQ>1.15

O

O

Increase in plasma lactic acid d.

d.

Enterohepatic circulation

Ca** deficiency does not produce coagulation defects

(Ref: Textbook of Physiology, AK Jain, Vol. 1, 7th ed. pg. 707)

(Ref: Textbook of Physiology, AK Jain, Vol. 1, 7th ed. pg. 244)

ANSWER

ANSWER

O

For answer refer 2017 paper-1, Q. 2(b), Pg.??

5.

In apical region alveolar pressure is higher then pulmonary arterial pressure Ventilation is more perfusion is less Results in higher oxygen levels

II.

Explain why (in 3-4 sentences)

a.

Heart rate increase with increase temperature

O

Calcium levels of only 2 mg/dL is required for activation of coagulation factors Calcium levels below 5 mg/dL causes Tetany (convulsions, laryngospasm and coma) Thusa state for coagulation defects never reaches

6.

Discuss briefly:

a.

Peptic ulcer

O in body

(Ref: Textbook of Physiology, AK Jain, Vol. 1, 7th ed. pg. 339, 366)

ANSWER

O

Increase in body temperature increases coronary blood flow Increase in body metabolism causes increase in CBF

O

b.

Red out occurs on exposure to negative ‘G’ force

ANSWER

|

D

| aia

=

D

O O

O oe)

> tT OW

Downward negative G drives blood to the head (opposite of venous pooling of blood on standing) -2 to-3 g causes red out were vision is reddened Increased pressure of blood in head may cause bursting of blood vessels of brain or eyes causing red out

c.

CO, transport in blood

(Ref: Textbook of Physiology, AK Jain, Vol. 1, 7th ed. pg. 438-439) &>

L

Caused due to break in continuity of gastric mucosa Disruption of mucosal barrier (Helicobacter pylori inhibits HCO3-secretion, NSAIDs, Ethyl alcohol) Increase acid insult to the gastric mucosa (increased acid production by gastrinomas/Zollinger Ellison Syndrome, hyper responsive parietal cells and elevated pepsinogen I levels ) b.

ZS

©

O O

Equal & opposite to acceleration

Me

>

ANSWER

O

G Force

O

(Ref: Textbook of Physiology, AK Jain, Vol. 1, 7th ed. pg. 228-229)

Apical alveoli have alveoli

higher

PO,

than basal

(Ref: Textbook of Physiology, AK Jain, Vol. 1, 7th ed. pg. 427-429)

ANSWER

O O O

First dissolves in plasma (60%) Second binds to RBC (40%) Transport occurs in 3 forms dissolved, carbamino compounds and bicarbonates

O

In plasma: Dissolved as carbonic acid (H,CO,) only 0.2 ml/dl, combines with carbamino protein only 0.1 ml/dl and as bicarbonate of Na which is 2.1 ml/dl In RBC: Dissolved as carbonic acid due to carbonic anhydrase enzyme 0.1 ml/dl, forms carb amino hemoglobin 0.6 ml/dl and bicarbonates of K* 0.9 ml/dl

O

c.

Factors affecting venous return

(Ref: Textbook of Physiology, AK Jain, Vol. 1, 7th ed. pg. 344)

PAPER-I ANSWER

ANSWER

O O

O O O O O

O O O

Respiratory/thoracic pump Cardiac pump: Forward moving force from behind and suction force from front Muscle contraction Blood volume Venous system capacity

O

Body Posture

O

Compliance of ventricles d.

Renin-Angiotensin aldosterone system Regulation of Acid-base balance Renin secretion Erythropoietin release in response to hypoxia Synthesis of active form of Vitamin D by 1 alfa hydroxylase enzyme present in kidney that converts 25 HOCC to 1, 25 DHCC

O

Gluconeogenesis

Nonexcretory functions of kidney (Ref: Textbook of Physiology, AK Jain, Vol. 1, 7th ed. pg. 512-513, 563, 55 7)

Arteriolar constriction

0-

=

>

1p |

Adernal

;

.

Sieeme

.

\ .

_

-

(ACE)

a

Angiotensinogen ——» Angiotensin-|———» Angiotensin-II

4

2

8

A

>

Renalperfusion (via i GA)

{|

|

—-» Active transport _. » Passive , transport

eee ee ee eee eee eee eee eee eee eee

Inhibition

GFR

Efferent arteriolar constriction

[DCT] Na’ -}i>

5)

bs ECFV |

K*

@}i-----

'

cr

>

!

rr ee

6)

So

—» Restore

.

e :

——+» Aldosterone

H,07)> .

sabre

ICT

-

|

|

op

!

Posterior

!

pituitary

!

TTT Trt nnn rns seers ss sccssssss

Fig. Renin-angiotensin- aldosterone system

>

©

= O

oe)

> tT QW

Your Roll No................

Name of the Paper

:

Physiology Paper-II

Name of the Course

:

MBBS-2010

Semester

>:

Annual

Time: 3 Hours

M.M.: 50 INSTRUCTIONS

1.

Write your Roll No. on the top immediately on receipt of this question paper

2.

All questions are to be attempted

3.

Answers to Parts I, II and III should be written in separate answer sheets provided

4.

Attempt parts of a question in sequence

PART-I 1.

2.

Compare and contrast:

[3]

a. b. c.

Decorticate and decerebrate rigidity Endo cochlear potential and cochlear microphonics Sarcotubular system in skeletal and cardiac muscle

d.

Lipostatic and glucostatic theory of food intake.

Draw labelled diagrams/flow charts/graphs to illustrate:

a. b. c. d.

[3]

Accommodation reflex Milk ejection reflex Inverse stretch reflex Neural circuits in the olfactory bulb

PART-II 3.

I.

Give normal value/range with clinical significance: a. b. c. d.

II.

Ill.

2 FS

[2]

To CSF pressure in communicating hydrocephalus If there is bilateral destruction of primary visual cortex.

Write short notes on:

eo

[2]

Features of brown sequard syndrome on the same side below the lesion Muscarinic receptors.

What will happen and why: a. b.

4.

Equilibrium potential of Na* Normal gestation period Absolute refractory period in cardiac muscle Near point of vision at the age of 60.

Enumerate two points each: a. b.

[4]

Oral contraceptive Temperature regulatory mechanisms Theories of referred pain Mechanism of release of neurotransmitter at neuromuscular junction.

[10]

PART-III 5.

I.

Define the following: a. b. c. d.

Il.

fF

[4]

Muscle weakness occurs in myasthenia Gravis Non pitting oedema occurs in hypothyroidism Rise in basal body temperature is an indicator of ovulation Administration of adrenaline causes adrenaline apnoea.

Discuss briefly:

ao

6.

Subliminal fringe Weber—fechner law Hormone Memory.

Explain why? (3-4 sentences) a. b. c. d.

[4]

Deposition and reabsorption of bone Functions of placenta Growth hormone Pathophysiology of Diabetes mellitus.

[8]

PAPER-II |

2010 PAPER-II

PART-I 1.

Compare and contrast: a.

|

Skeletal muscles

Cardiac muscles

e T-system at A-I junction

e T-system at Z lines

e Non-synctial

e Synctial

Decorticate and decerebrate rigidity

d.

(Ref: Textbook of Physiology, AK Jain, Vol. 2, 7th ed. pg. 954-958

Lipostatic intake

and

glucostatic

theory

of food

(Ref: Textbook of Physiology, AK Jain, Vol. 2, 7th ed. pg. 1009) ANSWER Decorticate rigidity

Decerebrate rigidity

e

. Transaction between

Removal of cerebral cortex

superior colliculi and inferior colliculi

with intact brainstem and

basal ganglia e Moderate rigidity

e Increased muscle tone, release from extrapyramidal inhibitory discharge, gamma rigidity

e Extensor hyperactivity in lower limb, flexed wrist and fingers with pronated forearm and semi flexed elbow across the chest

b.

Endo cochlear microphonic

e Hyperactive reflexes due to faciliatory discharge from descending faciliatory reticular fibers

potential

and _ cochlear

e Glucostatic theory

e Response to fatty acids

e Response to glucose

e Leptin receptor (leptin nediated)

e Glucose receptors involved (insulin mediated action)

e Leptin inhibits neuropeptide Ye inhibit feeding center

Glucose stimulate satiety center inhibit feeding center

Draw labelled illustrate:

a.

ANSWER

diagrams/flow

charts/graphs

Endo cochlear potential

Cochlear microphonic

e Endolymphatic potentials

e Cochlear potentials e Potential difference between part of chochlea to the entire body

Sarcotubular system in skeletal and cardiac muscle (Ref: Textbook of Physiology, AK Jain, Vol. 1,

7th ed. pg. 166, 179,197)

ANSWER Skeletal muscles

Cardiac muscles

e Well developed terminal cisterns

e Terminal cisterns

poorly developed

to

Accommodation reflex

(Ref: Textbook of Physiology, AK Jain, Vol. 2, 7th ed. pg. 1102) ANSWER

e Between endolymph of scala media and perilymph of scala vestibuli, aaprox. + 80 mV

PHYSIOLOGY

e Lipostatic theory

2.

(Ref: AK Jain, vol. 2, 7th ed. pg. 1078-1080)

c.

ANSWER

Retina

From the para striate cortex

y

Y

Via optic nerve, chaisma optic tract

Via

v Lateral geniculate body

occipitomesen cephalic Tract and Pontine centre

EW

nucleus

y

y

Striate cortex

Via Ill n. to sphincter pupillae

Fig. Pathway accommodation reflex

PAPER-II — Sphincter pupillae

.-Optic nerve

Optic chiasm

Mid-brain

Pretectal

+” nucleus Edingerwestphal nucleus

Fig. Pathway of accomodation reflex

b.

Milk ejection reflex (Ref: Textbook of Physiology, AK Jain, Vol. 2, 7th ed. pg. 682)

ANSWER

For answer refer 2012 paper-2, Q. 5 (a), Pg. 252 c.

Inverse stretch reflex

(Ref: Textbook of Physiology, AK Jain, Vol. 2, 7th ed. pg. 886-887)

ANSWER 1. Tension on tendon} activates sensory neuron

Spinal cord

\

[

\.| 2. Sensory neuron |. Xx Y stimulates interneuron

Gwe

>

©

3. Interneuron inhibits

=

motoneuron

O

Alpha motoneuron

inhibited

Fig. Pathway of inverse stretch reflex

oe)

4. Tension on tendon is reduced

> tT QW

PAPER-II d.

Neural circuits in the olfactory bulb. (Ref: Textbook of Physiology, AK Jain, Vol. 2, 7th ed. pg. 1058)

ANSWER Olfactory bulb

Olfactory

*

tract || Olfactory

bulb

/

Interneurons|

Mitral cell

(Secondary neuron) Ly

-— Tufted cell (secondary neuron) =— Glomeruli

== —

Cribriform plate of ethmoid bone

0

= Olfactory receptor

0

‘|

0 0

A

0\0

neurons Columnar epithelium

Cilia

Nasal cavity

Orbitofrontal

X

To olfactory

Olfact

cortex and related

Olfactory\tubercle

temporal lobe

bulb

Olfactory tract Olfactory receptor cell

PART-II

Cc.

candies”

Absolute refractory period in cardiac muscle

(Ref: Textbook of Physiology, AK Jain, Vol. 2, 7th ed. pg. 186)

3.

i.

a.

Give normal value/range with significance: (Kindly provide the significance

clinical clinical

ANSWER O

Equilibrium potential of Na*

250-300

msec,

heart muscle

cannot

be tetanized

due to its long absolute refractory period.

(Ref: Textbook of Physiology, AK Jain, Vol. 2, 7th ed. pg. 37)

d.

Near point of vision at the age of 60

(Ref: Textbook of Physiology, AK Jain, Vol. 2, 7th ed. pg. 1104) ANSWER

O

+60 mV, generation of resting membrane potential

ANSWER

b.

O

85 cm, thus in old age myopic never suffer from presbyopia

3.

II.

Enumerate two points each:

a.

Features of brown Sequard syndrome on the same side below the lesion.

Normal gestation period

PHYSIOLOGY

(Ref: Textbook of Physiology, AK Jain, Vol. 2, 7th ed. pg. 830) ANSWER

O

40 weeks/280 days, important development of the fetus

for

growth

and

( Ref: Textbook of Physiology, AK Jain, Vol. 2, 7th ed. pg. 938

PAPER-II O

Sensory:

ANSWER Fine

touch,

two

point

discrimination,

tactile localization are lost and temperature, crude touch and pain remains O

Motor: Upper motor neuron type paralysis and cutting of descending VMC fibers causes vasomotor loss. b.

O

Neuro effector junction of all parasympathetic endings Cholinergic endings in sweat gland, skeletal muscle and blood vessels Sweating, vasodilation PLC and K* channel opening Antagonist-Atropine

sequential,

estrogen,

minipills,

Temperature regulatory mechanisms

Thermoregulation in cold O

Production of heat: Catecholamines and TSH secretions, increase voluntary activity and shivering, hunger

O

Conservation ofheat: Piloerection, vasoconstriction

Thermoregulation in Heat

III.

What will happen and why:

a.

To CSF pressure _ hydrocephalus

in

O communicating

Heat dissipation: Vasodilation, sweating and increase in the respiratory rate Production of heat decreases: Decreased hunger, apathy decreased TSH c.

(Ref: Textbook of Physiology, AK Jain, Vol. 2, 7th ed. pg. 375)

Theories of referred pain

(Ref: Textbook of Physiology, AK Jain, Vol. 2, 7th ed. pg. 904)

ANSWER

ANSWER

O

Increase in pressure

O

O

Production is more then absorption

O

Meningeal thrombosis b.

Combined,

(Ref: Textbook of Physiology, AK Jain, Vol. 2, 7th ed. pg. 591-593)

O 3.

Types:

ANSWER

O O

Prevents ovulation-anovulatory cycle Withdrawal bleed Thickened endometrium and cervical mucous

b.

ANSWER

O

Inhibits LH

morning after pills and progesterone antagonists

Muscarinic receptors.

(Ref: Textbook of Physiology, AK Jain, Vol. 2, 7th ed. pg. 928

O

O

OO0 0

ANSWER

Inflammation

and

Facilitation theory: Subliminal fringe lowers the threshold of nerve fibers from same somatic area

venous _ sinuses

O

If there is bilateral destruction of primary visual cortex (Ref: Textbook of Physiology, AK Jain, Vol. 2, 7th ed. pg. 1093-1098)

Convergence theory: Afferents from the visceral organs converge at common spinothalamic neuron d.

Mechanism

of release of neurotransmitter at

neuromuscular junction (Ref: Textbook of Physiology, AK Jain, Vol. 2, 7th ed. pg. 157-159) ANSWER

Macular sparing with loss of peripheral vision as macular representation on occipital lobe is more extensive then of retina.

0

O

O

OO

ANSWER

Action potential reaches presynaptic ending Calcium channels open Fusion of Ach filled vesicles Release of Ach

4.

Write short notes on:

a.

Oral contraceptive.

(Ref: Textbook of Physiology, AK Jain, Vol. 2, 7th ed. pg. 821)

5.

I. Define the following a.

Subliminal fringe

(Ref: Textbook of Physiology, AK Jain, Vol. 2, 7th ed. pg. 866)

PHYSIOLOGY

PART-III

PAPER-II C.

ANSWER O

Excitation of a nerve causes subliminal effect on the

Risein basal body temperature is an indicator of ovulation

(Ref: Textbook of Physiology, AK Jain, Vol. 2, 7th ed. pg. 809)

non-discharging neuron decreasing its threshold for excitation.

b.

ANSWER

Weber—Fechner law

(Ref: Textbook of Physiology, AK Jain, Vol. 2, 7th ed. pg. 874)

O O

ANSWER

O

It is the relationship between the intensity of the stimulus felt to the magnitude of the stimulus. = Perception=constant factor (k) X logarithm of stimulus + constant of integration c.

Hormone

(Ref: Textbook of Physiology, AK Jain, Vol. 2, 7th ed. pg. 657) ANSWER O

O

d.

d.

O O O

High quantity of adrenaline when injected Sudden increase in systemic blood pressure Inhibition of respiratory center

6.

Discuss briefly:

II.

Explain why—(3-4 sentences)

a.

Muscle Gravis

weakness

occurs

O

ANSWER

Itis defined as an ability to recollect past events.

O O O

in

Myasthenia

O

O O

Antibodies against Ach nicotinic receptors Difficult depolarization of muscle Fatigue and weakness

Osteocytes responsible for bone formation Osteoclasts responsible for bone reabsorption Both processes are continuous Bone synthesis requires synthesis of collagen and its mineralization (endochondral) Membranous bone form without cartilage phase Endosteal bone is remodeled bone Hormones involved are growth hormone, thyroid hormone, Insulin like growth factors, parathyroid hormone, Vitamin D and Calcitonin

O

OO

Autoimmune disease

b.

Deposition and resorption of bone

(Ref: Textbook of Physiology, AK Jain, Vol. 2, 7th ed. pg. 710)

ANSWER O

causes

ANSWER

a.

(Ref: Textbook of Physiology, AK Jain, Vol. 2, 7th ed. pg. 160)

0

adrenaline

(Ref: Textbook of Physiology, AK Jain, Vol. 2, 7th ed. pg. 448)

Memory

ANSWER

OO

of

adrenaline apnea

(Ref: Textbook of Physiology, AK Jain, Vol. 2, 7th ed. pg. 1041)

5

Administration

Hormonesare substances (protein, steroid or amine)

released from the gland goes into the circulation and produce their effect on the target organ.

O.

Increase in body temperature from 0.3-0.5 degree centigrade Due to increase in progesterone levels Secondary to LH surge

Non pitting edema occurs in hypothyroidism

Mineral deposited in the form salts of Ca2+ and PO4+

Growth occurs at epiphyseal plate Osteoblasts differentiates into both osteocytes and osteoclasts b.

(Ref: Textbook of Physiology, AK Jain, Vol. 2, 7th ed. pg. 698)

of hydroxyapatite

Functions of placenta

PHYSIOLOGY

(Ref: Textbook of Physiology, AK Jain, Vol. 2, 7th ed. pg. 835-837)

ANSWER

O

Accumulation of chondroitin sulfate and hyaluronic acid in subcutaneous layer of skin which forms a gel like substance with tissue fluid causing edema which is non-pitting

ANSWER

O

O

Itbecomes lungs, kidney and intestine for developing fetus Oxygen transport and other gases

000 0 0

PAPER-II Transportation of fluids

d.

Production of hormones necessary for gestation

(Ref: Textbook of Physiology, AK Jain, Vol. 2, 7th ed. pg. 757)

Provides nutrition Formation of amniotic fluid

ANSWER

Prevention of passage of IgM antibodies c.

Decreased or absent production of insulin Insulin absence causes no uptake of glucose from GLUT 4 transporters and insulin dependent glucose transporters inside the cells

Growth hormone

O

Secreted from anterior pituitary

O

Increase protein synthesis

O

Increase uptake of amino acids in cells, transcription and translation

OO

Increase in the collagen and bone synthesis

Effect through Insulin like growth factor or somatomedin C Gluconeogenesis, glycogenolysis, increase insulin resistance, diabetogenic

Lipolysis, increase FFA, beta-oxidation, ketogenic Hypersecretion: Gigantism and acromegaly Hyposecretion: Dwarfism

OO

(Ref: Textbook of Physiology, AK Jain, Vol. 2, 7th ed. pg. 669-678)

OO O

Pathophysiology of Diabetes mellitus

Shift of metabolism from glucose to energy from gluconeogenesis and lipolysis Protein degradation and ketoacidosis Absence of glucose uptake in satiety center of Hypothalamus causes polyphagia

Hyperglycemia but inhibited uptake of due to insulin absence causes increased osmolarity Hyperosmolar plasma causes stimulation centers-Polydipsia Increased glucose concentration causes diuresis-Polyuria

glucose plasma of thirst osmotic

>

©

L O

oe)

> tT QO

Your Roll No. ...............

Name of the Paper

:

Physiology Paper-!

Name of the Course

:

MBBS-2009

Semester

:

Annual

Time: 3 Hours

M.M.: 50 INSTRUCTIONS

1.

Write your Roll No. on the top immediately on receipt of this question paper

2.

All questions are to be attempted

3.

Answers to Parts I, II and III should be written in separate answer sheets provided

4.

Attempt parts of a question in sequence

PART-I 1.

Compare and contrast: a. b. c. d.

2.

[3]

Kwashiorkor and Marasmus Innate and acquired immunity Central and peripheral chemoreceptors Obligatory and facultative water reabsorption.

Draw labeled diagram/flow charts/tables to illustrate:

a. b. c. d.

[3]

Mechanism of HCI secretion in stomach Mechanism for glucose transport across renal epithelium Normal pacemaker potential and effects of sympathetic and parasympathetic stimulation on it. Sequence of events in endocytosis.

PART-II 3.

I.

Give normal values/range of: a. b. c. d.

II.

Ill.

[2]

To gastrointestinal activity if person suffers from cholera To central venous pressure in cardiac failure

SF

Chemical regulation of respiration Regulation of renin secretion

ao

Write short notes on:

Starling forces Dysfunctions of phagocytosis

®

[2]

C—reactive proteins Factors affecting GFR

What happens and why? a. b.

4.

PR interval pCO, in venous blood Conduction velocity in Purkinje’s fiber Gastric slow wave

Enumerate (two points each)

a. b.

[4]

[10]

PART-III I.

[4]

Explain why (not exceeding 3—4 sentences)

[4]

Nitrites are used to treat cyanide poisoning Dietary fibers are essential for health Apical alveoli have higher pO2 than basal alveoli

d.

Apremenopausal Rh negative female should not be given Rh positive blood

9

2

Il.

Define the following: a. Titrable acidity b. Sinus arrhythmia c. Facilitated diffusion d. Gastroileal reflex

o

5.

fF PS

Discuss briefly:

ao

6.

Na* — K* ATP ase Application of renal clearances Non respiratory function of lungs Hemorrhagic shock

[8]

PAPER-I |

2009 PAPER-I

PART-I 1.

ANSWER

Compare and contrast:

a.

Kwashiorkor and Marasmus

(Ref: Textbook of Physiology, AK Jain, Vol. 1, 7th ed. pg. 642-643) ANSWER Kwashiorkor

Marasmus

e Adequate energy intake with protein deficiency

e Deficiency of carbohydrates, fat, proteins and nutrients

e Edema present due to protein deficiency

e¢ Edema absent

e Rust colored skin and hair, fatigue, diarrhea, decreased muscle mass, stunted growth,

e Emaciated look, loss of

subcutaneous fat with low immunity, stunted growth and chronic diarrhea

low immunity e Treated with good diet high in protein content

b.

e Treated by giving all the nutrients in diet

Innate and acquired immunity

(Ref: Textbook of Physiology, AK Jain, Vol. 1, 7th ed. pg. 123)

Obligatory reabsorption

Facultative reabsorption

e PCT

e DCT, CT

e Reabsorption along with active reabsorption of solute like Na+

e¢ Need based reabsorption

e Passive reabsorption

e Active reabsorption

e Passive diffusion

e Aquaporin channels inserted in cell membrane

2.

Draw labeled illustrate: a.

under the effect of Anti Diuretic

Hormone (ADH)

diagram/flow

For answer refer 2014 paper-1, Q. 2(c), Pg. 217

ANSWER b.

Mechanism for renal epithelium

glucose

Glucose Absorption and Handling

ANSWER Peripheral chemoreceptors

Central chemoreceptors

e Carotid bodies: Bifurcation of common carotid artery, glossopharyngeal nerve

e

e Aortic bodies: Arch of Aorta,

e Connected to respiratory

vagus nerve

© Stimulated by increase in H+ ions

e Stimulation causes hypertension and increase HR along with increase in depth and rate of respiration

e No effect on CVS

e Early response to increase in

e Late response to increase

arterial pCO2

c.

Apical membrane

Na-Glucose co transporter rely upon Na

Basolateral membrane

Glucose transporters (GLUTs) Does not rely on Na

near respiratory centers

e Stimulated by decrease oxygen, increase CO2 and H ions

Basolateral membrane

SGLT 1/2

Na’

\

Q

Gitizose

Brush border membrane

Na’

° K

o,

in arterial pCO,,

Obligatory and facultative water reabsorption

(Ref: Textbook of Physiology, AK Jain, Vol. 2, 7th ed. pg. 535-537)

across

Glucose absorption also relies upon the Na* gradient. It is absorbed by Na-glucose co-transport mostly at the proximal tubule.

Present on ventral medulla

center

transport

(Ref: Textbook of Physiology, AK Jain, Vol. 1, 7th ed. pg. 529-530)

For answer refer 2016 paper-1, Q. 3, Pg. 191

(Ref: Textbook of Physiology, AK Jain, Vol. 1, 7th ed. pg. 335)

to

(Ref: Textbook of Physiology, AK Jain, Vol. 2, 7th ed. pg. 221-223)

ANSWER

Central and peripheral chemoreceptors

charts/tables

Mechanism of HCI secretion in stomach.

ANSWER

c.

PHYSIOLOGY

|

Tight junction

Na’/K* 4 Lateral intracellular space

pump

PAPER-| c

Tubular Transport Maximum for Glucose

Normal pacemaker potential and effects of sympathetic and parasympathetic stimulation on it

Essentially (100%) all glucose is reabsorbed Tubular maximal transport

Men: 375 mg/min

(Ref: Textbook of Physiology, AK Jain, Vol. ??, 7th ed. pg. ??)

Women: 300 mg/min 180 mg/dl

Threshold

ANSWER

Handling of glucose is limited by saturation of the transport mechanism i.e. carriers

d.

For answer refer 2015 paper-1, Q. 2(a), Pg. 201

Sequence of events in endocytosis (Ref: Textbook of Physiology, AK Jain, Vol. ??, 7th ed. pg. ??)

ANSWER Mammalian endocytosis

GD \ Stage 1 Coat module recruited tothe bud deformsthe membrane.

==

Membrane

Stage 2 Theresulting curvature rapidly recruits dynamin tothe neck first. Dynamin further deforms membrane.

=

PIP2

=

Stage 4 Dynamin protection of the neck PIP,createsaPIP,phase segregation. The resulting interfacial force squeezes the neck, creating even higher curvature that induces more PIP, hydrolysis.

Stage 3 Actin begins to polymerize and dynamin further accumulates at the neck, resulting an ever higher membrane curvature that recruits phosphatase to the endocytic site.

Coat

Actin

Stage5 Rapid vesicle scission and disassembly of endocytic apparatus.

BDP (yeast) Dynamin (mammalian)

VA

© Phosphatase

Fig. Mammalian endocytiosis

d.

PART-II

Gastric slow wave

(Ref: Textbook of Physiology, AK Jain, Vol. 1, 7th ed. pg. 226)

3.

I.

Give normal values/range of:

a.

PRinterval

(Ref: Textbook of Physiology, AK Jain, Vol. 2, 7th ed. pg..296-297)

ANSWER

O

BER: < 1 cm/sec beginning from interstitial cells of Cajal on greater curvature near fundus and increases to 4cm/ sec till it reaches antrum.

3.

II.

Enumerate (two points each)

a.

C—reactive proteins

ANSWER

O

0.12 to 0.16 sec when heart rate is 72 per minute.

b.

pCO, in venous blood

(Ref: Textbook of Physiology, AK Jain, Vol. 1, 7th ed. pg. 439)

(Ref: Textbook of Physiology, AK Jain, Vol. 1, 7th ed. pg. 124)

9.

46mm Hg

ANSWER

c.

©

Non specific or innate immunity

©

Present since birth

O

Antigens activate/increase complement system.

O

Insertion of membrane attack complex (porins) in the invading microorganisms causing their destruction

Conduction velocity in Purkinje’s fiber

(Ref: Textbook of Physiology, AK Jain, Vol. 1, 7th ed. pg. 182)

ANSWER O

4m/sec

C-reactive

activating

PHYSIOLOGY

ANSWER

PAPER-I b.

Factors affecting GFR

ANSWER

(Ref: Textbook of Physiology, AK Jain, Vol. 1, 7th ed. pg. 522-523)

ANSWER O

Age: Decrease with age.

O

Renal blood flow: Directly proportional.

O

Glomerular’ capillary Directly proportional.

hydrostatic

O

Increase in systemic venous pressure

O O

Right heart failure Congestion of venous system

4.

Write short notes on:

pressure: a.

O

Bowman’s capsule hydrostatic pressure: Inversely proportional.

O

Glomerular membrane permeability.

O

Mesangial cell’s regulation of capillary size.

Chemical regulation of respiration

(Ref: Textbook of Physiology, AK Jain, Vol. 2, 7th ed. pg. 449-451) ANSWER O

Peripheral bodies,

3.

To gastrointestinal activity if person suffers from cholera

(Ref: Textbook of Physiology, AK Jain, Vol. 1, 7th ed. pg. 251)

OO O

O

ANSWER

Sever watery diarrhea/rice water stools, fishy smell Dehydration Cholera toxin binds GM1 ganglioside receptor and endocytosis occurs Activates adenyl cyclase/cAMP system in intestinal basolateral membrane

b.

(Ref: Textbook of Physiology, AK Jain, Vol. 2, 7th ed. pg. 394-395)

PHYSIOLOGY

b.

fibrosis

Tocentral venous pressure in cardiac failure

by increase

and

in pCO,,

carotid decrease

Myocardial chemoreceptors: Known for Bezold Jarisch reflex stimulated by drugs like veratridine/ nicotine cause bradycardia, hypotension and rapid shallow breathing.

Vomiting

Increase cAMP activates CFTR (cystic transmembrane conductance regulator Efflux of ions and water from enterocytes

stimulated

Aortic

pO, causes hypertension and tachycardia along with increase in depth and rate of respiration. Central or medullary chemoreceptors: Ventral medullary centers near respiratory centers, respond to H+ ion concentration and cause increase in rate and depth of respiration on stimulation Pulmonarychemoreceptors: Known for Pulmonary chemoreflex, stimulated by drugs like veratridine/ nicotine cause bradycardia, hypotension and rapid shallow breathing.

III. What happens and why?

a.

chemoreceptors:

Regulation of renin secretion

(Ref: Textbook of Physiology, AK Jain, Vol. 1, 7th ed. pg. 565) ANSWER O

Increase arteriolar pressure causes increase stretch causing inhibition of secretion from JG cells

PAPER-| O

Renin secretion increases due to sympathetic stimulation via beta 1 adrenergic receptor, decrease arteriolar pressure Arteriolar constriction

0-

=

>

1eP |

Adernal

-

Q@—

: e

*

(A

e

——+» Aldosterone

Q©-

Angiotensinogen ——» Angiotensin-I———» Angiotensin-ll *

X

—» Restore

.

CE)

GFR

Efferent arteriolar

A

constriction

O

alt

—-» Active transport

+ Renal perfusion (via eA )

_.

ai

Ky

«ll...

Cr

-yj>

bo

ECFV

07>

’ » Passive transport

ot

6 ]

ss Sympathetic activity

abe

oP

Posterior

pituitary

c

Starling forces

PART-III

(Ref: Textbook of Physiology, AK Jain, Vol. 1, 7th ed. pg. 57)

5.

ANSWER For answer refer 2016 paper-1, Q. 2(d), Pg. 190

d

Congenital absence of myeloperoxidase enzyme:

O

deficiency Chronic granulomatous disease: No NADPH oxidase activity, hereditary, defective phagocytosis, formation of granulomas. Neutrophil hypomotility: Problem with actin that

O

do not polymerize normally Congenital neutropenia: Autosomal gene mutation HAXI Bene

O

.

.

.

Mmol/L

amount

of alkali other

then

bicarbonate

that must be added to urine to return its pH to 7.4. b.

Sinus arrhythmia

(Ref: Textbook of Physiology, AK Jain, Vol. 1, 7th ed. pg. 303) ANSWER

O

Normal phenomenon

O

Variation in sinus rhythm due to variation in P-P interval causing irregular ventricular rate

O

Variation of heartrate due variation in vagal tone

05

O

Inspiration causes decrease decrease in heart rate.

O O yY 7

dominant,

Agranulocytosis: Sever leukopenia, eosinopenia, Basopenia and neutropenia, caused by drugs like antiepileptics, anti thyroids, ACE inhibitors, anti psychotics and anti depressants Myeloid leukemia

Itis H*ion that reacts with anion of buffer other than

bicarbonate anion is titratable acidity

Autosomal resistive, normal NADPH oxidase activity, defective killing of microorganism, immune

O

Titratable acidity

ANSWER

ANSWER

©

a.

Dysfunctions of phagocytosis O

O

Define the following:

(Ref: Textbook of Physiology, AK Jain, Vol. 1, 7th ed. pg. 560)

(Ref: Textbook of Physiology, AK Jain, Vol. 1, 7th ed. pg. 89)

O

I.

c.

in vagal tone hence

Facilitated diffusion

(Ref: Textbook of Physiology, AK Jain, Vol. 1, 7th ed. pg. 15-17)

PAPER-I c.

ANSWER

O

higher

PO, than basal

(Ref: Textbook of Physiology, AK Jain, Vol. 1, 7th ed. pg. 428)

AN SWER

Gastroileal reflex

(Ref: Textbook of Physiology, AK Jain, Vol. 1, 7th ed. pg. 253) ANSWER

O

alveoli have

alveoli

It is carrier mediated diffusion of large molecules across the cell membrane, e.g. glucose transporters, gated ion channels. d.

Apical

O

Intrapleural pressure-7 mm Hg at apex and-2 mm Hg at base Reduced perfusion at apex leads to increased pO2

O

More susceptible to tuberculosis

O

Relaxation of caecum along with opening ofileocecal valve occurs once the food passes from stomach.

d.

A premenopausal Rh-negative female should not be given Rh positive blood

(Ref: Textbook of Physiology, AK Jain, Vol. 2, 7th ed. pg. 110-112) 5.

II.

Explain why (not exceeding 3-4 sentences)

a.

Nitrites are used to treat cyanide poisoning

AN SWER

(Ref: Textbook of Physiology, AK Jain, Vol. 2, 7th ed. pg. 464)

ANSWER O O

Cytochrome oxidase is inhibited by cyanide Treated with im. hydroxocobalamin, i.v. sodium thiosulfate and sodium nitrate Nitrites turn hemoglobin ferrous into ferric forming meth Hb thatbind cyanide into cvanmethemoglobin, releasing cytochrome c oxidase

O

Exposure of Rh positive blood to Rh negative female cause formation of anti Rh IgG antibodies The possibility of conception during reproductive years may cause Rh incompatibility in the fetus if the fetus is Rh positive.

6.

Discuss briefly:

O

a.

Na‘*-K* ATP ase

(Ref: Textbook of Physiology, AK Jain, Vol. 1, 7th ed. pg. 18-19)

Dietary fibers are essential for health

(Ref: Textbook of Physiology, AK Jain, Vol. 1, 7th ed. pg. 260, 636)

ANSWER O

ANSWER O

Forms bulk, helps in digestion

O

Absorbs water

O

Forms barrier digesting food

between

intestinal

mucosa

and

O00 90

b.

ATP dependent pump 3 alfa subunits and 3 beta subunits ATP converted into ADP

Moves 2 potassium inside Moves 3 sodium outside

Dilutes carcinogens

Binding site for 2k

4

ECF

PHYSIOLOGY

>» Cellmembrane Bs




Annual

Time: 3 Hours

M.M.: 50 INSTRUCTIONS

1.

Write your Roll No. on the top immediately on receipt of this question paper

2.

All questions are to be attempted

3.

Answers to Parts I, II and III should be written in separate answer sheets provided

4.

Attempt parts of a question in sequence

PART-| 1.

Compare and contrast:

a. b. c. d.

Ovulatory and anovulatory cycle Pituitary and thyroid dwarf E—Ccoupling in smooth muscles and skeletal muscle Stretch and inverse stretch reflex

Draw labeled diagram/flow charts/tables to illustrate:

a Oo

F

2.

[3]

[3]

Visual pathways and mention the site of lesion resulting in homonymous hemianopia. Arrangement pyramidal tract fiber in internal capsule Effect of glucose infusion on insulin secretion EMG

PART-II

a

ES

Give normal values/range of:

Il.

Ill.

If blood supply from thalamogeniculate artery is interrupted In lesion of ventromedial nucleus of hypothalamus

FS

Write short notes on:

a Oo

[2]

Suprasegmental centers affecting gamma efferent discharge. Two features of klinefelter’s syndrome

What happens and why? a. b.

4.

S. cholesterol Miniature end plate potential Sperm count Infraocular pressure

Enumerate (two points each)

a. b.

[4]

Spinal shock Test tube babies Role of hypothalamus in regulation of temperature Role of inner ear in hearing

[2]

PART-III 5.

I.

Define the following: a. b. c. d.

d.

Ionic basis of excitation and conduction in a nerve

fF

[4]

It is necessary to evaluate the thyroid status in newborn Glucocorticoids are given in organ transplant Radiologist and aircraft pilot wear red glasses in bright light before going to dim light Edema is not observed in mineralocorticoid excess

Discuss briefly

ao

6.

Learning Tympanic reflex Conditioned reflex Hypermetropia

Explain why (not exceeding 3—4 sentences) Oo fF

Il.

[4]

Regulation of serum Ca‘ Sequence of events during transmission at neuromuscular junction Physiology of lactation.

| 8]

PAPER-II |

2009 PAPER-II

PART-I 1.

|

S.no

Compare and contrast:

a.

Skeletal muscle

Smooth muscle

Ca binds troponin C, binding of myosin head with Actin

Phosphorylation and activation of myosin head ATPase, Latch bridge mechanism

Ca-Mg ATPase actively pumps calcium in

Calcium moves slowly out of smooth muscle

Ovulatory and anovulatory cycle

(Ref: Textbook of Physiology, AK Jain, Vol. 2, 7th ed. pg. 809)

Relaxation

terminal cistern

ANSWER

Detachment from actin

Dissociation from

Ovulatory cycle

Anovulatory cycle

when myosin head binds

Calcium calmodulin

e Ovulation occurs, release of oocyte

© Ovulation absent

ATP

cause relaxation

e Increase in BBT, cervicalmucous changes

e° Nochanges

e Occurs in reproductive age group

e Early menarche and close to menopause

e Sufficient progesterone

e Insufficient progesterone

b.

b.

Pituitary and thyroid dwarf

(Ref: Textbook of Physiology, AK Jain, Vol. 2, 7th ed. pg. 675) ANSWER Pituitary Dwarf

Thyroid Dwarf

e Decreased level of Growth hormone

e Decreased level of thyroid hormone

e Fatty with body structure proportional to chronological age

e Stalky body, protruding tongue

e Mental age corresponding to chronological age

e Mental retardation

e Immature face

e Retarded physical development

c.

( Ref: Textbook of Physiology, AK Jain, Vol. 2, 7th ed. pg. 881-887) ANSWER Stretch reflex

Inverse stretch reflex

e Monosynaptic

e Bisynaptic

e Muscle spindle involved

e Golgi tendon organ involved

e Involved in length

e Protective reflex to regulate

servomechanism

E-C coupling in smooth muscles and skeletal muscle

(Ref: Textbook of Physiology, AK Jain, Vol. 2, 7th ed. pg.

Stretch and inverse stretch reflex

168-70, 192)

tension in muscle

e Knee jerk, ankle jerk, biceps jerk etc e la fiber afferent from muscle’ spindle e Efferent through motor neuron

2.

Draw labeled illustrate:

a. ANSWER

° Relaxation of contracted muscle e

Ib fiber afferent from golgi tendon organ e Efferent through inhibitory interneurons

diagram/flow

charts/tables

to

Visual pathways and mention the site oflesion resulting in homonymous hemianopia.

(Ref: Textbook of Physiology, AK Jain, Vol. 2, 7th ed. pg. 1093) S.no

Skeletal muscle

Smooth

Contraction

Nicotinic receptors binds Ach

Muscarinic receptor

PHYSIOLOGY

Membrane depolarization due to Na influx at neuromuscular junction Spread of action potential through T -tubules release Ca from terminal cisterns

muscle

binds ACh = Ca influx that bind Ca = calmodulin

ANSWER

For answer refer 2014 paper-2, Q. 4(b), Pg. 125 b.

Activation of calmodulin dependent light chain kinase

Arrangement pyramidal tract fiber in internal capsule

(Ref: Textbook of Physiology, AK Jain, Vol. 2, 7th ed. pg. 996)

PAPER-II A

ANSWER

NSWER

Internal capsule

i

1 1 1 \ " 1 " 1

"

oO

Optic radiation

=e

A

Auditory radiation

7)

>

Thalamocortical fiber

ee

&

Corticothalamic fiber

|+Phase|

ie'

BN

Corticospinal tract

ga

Corticobulbar tract

So

1.

Amplitude

Muscle



14. Corticorubral fiber

Duration —

Fig. Electomyogram

PART-II Fig. Internal capsule

c.

3.

Effect of glucose infusion on insulin secretion

(Ref: Textbook of Physiology, AK Jain, Vol. 2, 7th ed. pg. 756)

ANSWER

Give normal values/range of:

a.

S.cholesterol

(Ref: Textbook of Physiology, AK Jain, Vol. 2, 7th ed. pg. 621) ANSWER

* Glucose

(\I() J |\ J cut 2

(—

I.

v Glucose | Css Glucose-6-phosphate

* Insulin

O

120-200 mg/dL b.

+

Miniature end plate potential

(Ref: Textbook of Physiology, AK Jain, Vol. 2, 7th ed. pg. 159)

o

rs)

ANSWER

©

= O

oe)

> tT QO

PAPER-II 3

II.

Enumerate (two points each)

a.

Suprasegmental centers efferent discharge.

4.

affecting

Write short notes on:

a.

gamma

Spinal shock

(Ref: Textbook of Physiology, AK Jain, Vol. 2, 7th ed. pg. 935-937)

( Ref: Textbook of Physiology, AK Jain, Vol. 2, 7th ed. pg. 952) ANSWER ANSWER

O O

For answer refer 2012 paper-2, Q. 5(c), Pg.??

Facilitatory reticular formation Inhibitory reticular formation b

b.

(Ref: Textbook of Physiology, AK Jain, Vol. 2, 7th ed. pg. 827)

Two features of Klinefelter’s syndrome

O O00 0

ANSWER

O

ANSWER

O00

( Ref: Textbook of Physiology, AK Jain, Vol. 2, 7th ed. pg. 779-780)

Feminine stigmata

Somniferous tubules dysgenesis XXY Mental retardation

Hypogonadism genital organs

with under development

Test tube babies

Fallopian tube obstruction In vitro isolation of ovum In vitro fertilization with sperm At blastocyst stage inserted in female uterus for implantation c.

of male

Role of hypothalamus temperature

in

regulation

of

(Ref: Textbook of Physiology, AK Jain, Vol. 2, 7th ed. pg. 592-593)

3.

III. What happens and why? a

If blood supply from artery is interrupted

ANSWER

thalamogeniculate

O

Anterior causes

( Ref: Textbook of Physiology, AK Jain, Vol. 2, 7th ed. pg. 983)

O

ANSWER

Thalamic syndrome due to ischemia of postero ventral and posterolateral thalamic nuclei. Loss

of

stereognosis,

tactile

discrimination

d_

and

Kinesthetic sensations are lost (inability to locate a part of the body with closed eyes)

Posterolateral lesion

Ataxia, muscle weakness, intention tremors

O

Hyperextended fingers with flexion at wrist

O

Choreoathetosis In

lesion

of

ventromedial

nucleus

O of

>

(Ref: Textbook of Physiology, AK Jain, Vol. 2,

© oe)

ANSWER

OW

O O O

> tT

Destruction of satiety center Polyphagia Obesity

increased

hunger,

body

Temperature cutaneous

sympathetic

activity

Basilar membrane vibration in organ of corti causes depolarization of hair cells Upward movement of basilar membrane causes depolarization Downward movement causes hyperpolarization Apex of cochlea stimulated by low frequency sounds Base of cochlea stimulated by high frequency sounds

7th ed. pg. 1006-1009)

L O

O O

hypothalamus

shivering,

increase

ANSWER

O

b.

vasodilation,

(Ref: Textbook of Physiology, AK Jain, Vol. 2, 7th ed. pg. 1076-1078)

O

O

causes.

lowering

Role of inner ear in hearing.

localization

O

cutaneous

vasoconstriction, increased release of catecholamines

Postero ventral lesion

O

sweating,

Temperature

respiratory rate in response to temperature Posterior hypothalamus: raising

O

hypothalamus:

PART-III 5.

I.

Define the following:

a.

Learning

(Ref: Textbook of Physiology, AK Jain, Vol. 2, 7th ed. pg. 1039)

PAPER-II b.

ANSWER

Learning is alteration in behavior as a result of an experience. b

Tympanic reflex.

(Ref: Textbook of Physiology, AK Jain, Vol. 2, 7th ed. pg. 1076) ANSWER

O

Aprotective reflex in which there occurs contraction of tensor tympani muscle in response to loud sound c.

(Ref: Textbook of Physiology, AK Jain, Vol. 2, 7th ed. pg. 726-729) ANSWER

O O00 0

O

Glucocorticoids are given in organ transplant.

Conditioned reflex

Decrease allergic reaction Decrease inflammatory reaction Lowers immunity Decrease differentiation of lymphocytes Decrease production of cytokines, interleukin and prostaglandins involved in inflammatory reaction c.

(Ref: Textbook of Physiology, AK Jain, Vol. 2, 7th ed. pg. 1039)

Radiologist and aircraft pilot wear red glasses in bright light before going to dim light

(Ref: Textbook of Physiology, AK Jain, Vol. 2, 7th ed. pg. 1110)

ANSWER

O

Acquired response that is achieved on repeated pairing of the stimulus with an other stimulus. d.

ANSWER

O

Scattering of red light is minimum due wavelength that stimulates the rod and function is not inhibited Dark adaption does not take 20 minutes

Hypermetropia

(Ref: Textbook of Physiology, AK Jain, Vol. 2, 7th ed. pg. 1105)

O

d.

ANSWER

O

Edema

to its cones

is not observed in mineralocorticoid

excess

Long sightedness, person is able to see distant objects clearly but unable to see near objects

(Ref: Textbook of Physiology, AK Jain, Vol. 2, 7th ed. pg. 731)

ANSWER 5.

II.

Explain why (not exceeding 3-4 sentences)

a.

It is necessary to evaluate the thyroid status in newborn.

(Ref: Textbook of Physiology, AK Jain, Vol. 2, 7th ed. pg. 717)

O O

Aldosterone escape occurs Excess mineralocorticoid causing increase in blood pressure more then 25 mm Hg

O

Increase in BP causes increase GFR

O

Pressure diuresis occurs

6.

Discuss briefly

ANSWER

Thyroid hormone is responsible for the growth, branching and myelination of neurons

O

Maturation of CNS, risk for mental retardation that

O

can be prevented Overall growth and development of body

a.

Ionic basis of excitation and conduction in a nerve

(Ref: Textbook of Physiology, AK Jain, Vol. 1, 7th ed. pg. 39-41)

PHYSIOLOGY

O

PAPER-II ANSWER Na’* gates closes...|....00.0J..00.f.. cepeeee|eceeceeeehee.

|

S

\

ay

LS

Action potential

¢ K+ gates open

©

(}--

I= £ Q

oF 5 1S

eee

=

©

g

Saar eees bene



Permeability to Ne

Daten rene lame

|.

Threshol d

Permeability to K*

excitation

e | IO

MWA See ee ee ® Y.

SLAVS

IO. No oeeeeeefeceereeee|eoeeM..

55

e

TT

nel

ECE

na

Z

5

4 5 Time (m sec)

6

3Na '

+

+

+

+

+—

+

7

+

Na’ - K” pump pum

+

ICF_| Na* 10; K* 140 Orit

ee

1

Na * 142; 42: K’ K* 5

+

y +

+

+

Passively Resting state

Tremendous’ Na’ influx

_'m' gate \,

Decrease in Na’ influx

Na* channel (voltage-gated)

Rapid K’*

; ul 3H +

Na* +

+

+

'h' gate +

+

4__k* channel

'n' gate

OO

(voltage gated)

Repolarization

Depolarization

y

Resting state: K* permeability more then Na* Depolarization: Na* channel activation occurs

Bone ——~

above 7mV, causing Na* influx

Repolarization:

Opening

of

voltage

gated

K*

OO

channels, K* efflux

After depolarization: Delayed efflux of K* After hyperpolarization: Prolonged efflux of K’, activation of Na-K ATPase pump. b.

Regulation of serum Ca*

(Ref: Textbook of Physiology, AK Jain, Vol. 2, 7th ed. pg. 712)

PTH promotes and calcitonin inhibits Ca” release in the blood by osteoclasts (4

Ca™ removed from blood by osteoblast eee,

ee

- 2.

Tn

»)

PTH promotes

Ca” reabsorption from the urine

7 ;

Unabsorbed Ca

Ingested Ca”

lost in the feces

e

‘C

ANSWER Calcium Homeostasis

> (5 O

5

O

O

The hormone responsible for increase in the plasma , Lo, ; concentration of calcium is Parathyroid Hormone

Increase

.

calcium

uptake

from

.

;

intestine

and

a

reabsorption from kidney occurs from Vitamin D

=

assisted by PTH

On

O

Decrease in the plasma calcium levels occurs by calcitonin

Ca” | PTHHM promotes promot lost in} active vitamin D the urine] formation

Vitamin D 2 promotes Ca absorption

Fig. Calcium homeostasis

_ Small intestine

——"

PAPER-II c.

2009

Sequence of events during transmission neuromuscular junction

at

(Ref: Textbook of Physiology, AK Jain, Vol. 1, 7th ed. pg. 158) ANSWER

= = =" = ="

Fusion of Ach containing secretory vesicles Release of Ach Ach binds Ach receptor Influx of Na* in post synaptic membrane Membrane depolarization of postsynaptic membrane

Action potential at presynaptic ending of nerve

O

Depolarization causes opening of calcium channels "

Calcium influx

Motor neuron action potential

Cholinergic synaptic vesicle

b Propagated action potential in muscle plasma membrane

@ Ca** entry voltage-gated channels

Voltage-gated

Na* channels _——

—_——

} Acetylcholine

©

Muscle fiber action potential initiation

Acetylcholine degradation

T Nicotinic\& receptor

Acetylcholinesterase

d.

Local current between depolarized end plate and adjacent muscle plasma membrane

Physiology of lactation (Ref: Textbook of Physiology, AK Jain, Vol. 2, 7th ed. pg. 845, 679

ANSWER Lactogenesis

For answer refer 2013 paper-2, Q. 5(b), Pg. 238

>

©

= O

oe)

> tT QW

Your Roll No................

Name of the Paper

:

Physiology Paper-!

Name of the Course

:

MBBS-2008

Semester

>

Annual

Time: 3 Hours

M.M.:

50

INSTRUCTIONS 1.

Write your Roll No. on the top immediately on receipt of this question paper

2.

All questions are to be attempted

3.

Answers to Parts I, II and III should be written in separate answer sheets provided

4.

Attempt parts of a question in sequence

PART-I 1.

Compare and contrast:

a. b. c. d. 2.

[3]

Osmotic and water diuresis Low and high pressure system Obligatory and facultative water reabsorption White and red muscle fibers

Draw labelled diagram/flow chart/table to illustrate:

a.

b. c. d.

[3]

Activity of a muscle spindle afferent during steady stretch, muscle contraction and simultaneous a, y neuron stimulation Mechanism for glucose transport across intestinal epithelium Normal pacemaker potential and effect of sympathetic c and parasympathetic stimulation on it Intrapleural and intrapulmonary pressure changes during normal breathing [8]

PART-II 3.

I.

Give normal value/range of:

a. b. c. d. Il.

Ill.

4.

Central venous pressure Life span of platelets Plasma osmolality Serum sodium

Enumerate (two points each):

a.

Two conditions causing cheyne stroke respiratiory

b.

Two factor when present in diet inhibit iron absorption

What happens and why (not exceeding 3—4 sentences) a. b.

[2]

[2]

If total gastrectomy is done To bleeding and clotting time in hepatoecellular diseases

Write short notes on: a. b. c. d.

[4]

Tubuloglomerular feedback Anion gap Windkessel effect Local cardivascular regulatory mechanism

[10]

PART-III 5.

I.

Il.

Define the following: a. Dyspnoea b. Filtration fraction c. Linthoven’s law d. Limiting pH of urine. Explain why (not exceeding 3—4) sentences) a. b. c. d.

Cyanosis is not observed in sever anemia Phototherapy is given in cases of physiological jaundice of newborn Most of the ECG waves are recorded negative in AVR Vit. K is given in premature babies.

fF PS

Discuss briefly:

ao

6.

[4]

Natural killer cells Stages of erythropoiesis Fibrinolytic system Lysosomes.

[8]

PAPER-I

2008 |

2008 PAPER-I

PART-I 1.

Compare and contrast:

|

White fibers

Red fibers

e Increase glycogen, increase ATPase activity, decreased

e Moderate glycogen, low ATPase, increase myoglobin

myoglobin

a.

Osmotic and water diuresis

(Ref: Textbook of Physiology, AK Jain, Vol. 1, 7th ed. pg. 533)

e Slow conducting, fatigue resistant

e Power full contraction, low

e Highly vascular, increased

vascularity with decreased level of mitochondria

ANSWER Osmotic Diuresis

Water Diuresis

e Increase in osmotically active substances in plasma, glucose mannitol

e Increase in plasma volume (drinking increased amount of water) e Hypotonic plasma e Inhibition of ADH secretion e Diuresis of diluted urine

e Tmax of filtered substances is

achieved due to which no further reabsorption of the solutes occurs e Diuresis of urine having osmolality equals to plasma.

b.

e Fast conducting, easily fatigued

2.

Draw labelled illustrate a.

level of mitochondria

diagram/flow

chart/table

to

Activity of a muscle spindle afferent during steady stretch, muscle contraction and simultaneous a, y neuron stimulation

(Ref: Textbook of Physiology, AK Jain, Vol. 1, 7th ed. pg. 881)

Lowand high-pressure system

(Ref: Textbook of Physiology, AK Jain, Vol. 1, 7th ed. pg. 313-314) ANSWER

Activity of a muscle spindle afferent during steady stretch, muscle contraction and simultaneous a, y neuron stimulation

ANSWER High Pressure

Low pressure

e >25mmHg

e +2 a

-5 ve

5

2 +1 aig

6

-3End

Relaxation volume

5

=o Sn

4

CI.

expiratory

on A - 2 position

o>)

©

©ée

5

—1

pono nao nn nncncnnnsac bon Ryo kat

|

©LH LZ

Airways pressure measuring device

Spirometer

—2 -120

|

-80

yl -40

0

Vv

! 80

40

|

|

Maximum 9 expiratory +4:

120 160

200 Position

intrapulmonary (or Airway) pressure (mmHg)

Fig. Graph showing changes in intrapulmonary pressure while breathing

PART-II 3.

I. a.

ANSWER

O

Physiological: hyperventilation

O

Pathological:

Give normal value/range of: Central venous pressure

b. ANSWER

voluntary

Left ventricular failure, uremia

and

Two factor when present in diet inhibit iron absorption.

(Ref: Textbook of Physiology, AK Jain, Vol. 1, 7th ed. pg. 271-272)

4-6mm Hg

b.

altitude,

damage to medulla

(Ref: Textbook of Physiology, AK Jain, Vol. 1, 7th ed. pg. 317)

O

High

Life span of platelets

ANSWER

(Ref: Textbook of Physiology, AK Jain, Vol. 2, 7th ed. pg. 95)

O ANSWER

8-12 days c.

O

decreased HCL secretion in stomach

Plasma osmolality

(Ref: Textbook of Physiology, AK Jain, Vol. 2, 7th ed. pg. 17-18) ANSWER

III.

What happens and why (not exceeding 3-4

sentences)

d.

a.

Serum sodium

ANSWER

153 mEq/L

Iftotal gastrectomy is done.

(Ref: Textbook of Physiology, AK Jain, Vol. 2, 7th ed. pg. 228-229)

ANSWER

of water

O

causing

Cheyne _ stroke

(Ref: Textbook of Physiology, AK Jain, Vol. 2, 7th ed. pg. 456-457)

Syndrome:

Loss

of reservoir function,

Megaloblastic anemia, absence of castle’s intrinsic

II. Enumerate (two points each): a. Two conditions respiration.

Dumping

after meal hyperglycemia followed by increased insulin resulting in rapid hypoglycemia Dizziness, weakness and sweating Iron deficiency anemia due absent HCL

O

3.

3

290 mosm/L

(Ref: Textbook of Physiology, AK Jain, Vol. 2, 7th ed. pg. 29-30)

O

Phytates and oxalates bind iron

O

O

O

O

O

Partial gastrectomy causes decreased HCL that converts dietary Fe3+ (ferric ) iron into absorbable Fe2+ ferrous state Achlorhydria causes iron deficiency due _ to

factor

>

©

= O

oe)

> tT QO

PAPER-I

2008 b.

Tobleedingandclottingtimeinhepatocellular diseases

ANSWER

O

(Ref: Textbook of Physiology, AK Jain, Vol. 2, 7th ed. pg. 99-104)

O

Increase clotting time as clotting factors II, VII, IX

O

O

and X are synthesized in liver Bleeding time remain same as it depends function of platelets and condition of capillary

O

4.

on

Write short notes on:

a.

It is the ability of tissue to autoregulate their own blood flow Due to basal myogenic tone, of pace maker cells in vessel walls Local

vasodilators:

acid, adenosine temperature O

Local

and

Metabolites

decreased

vasoconstrictors:

like

pH,

K’‘,

lactic

increase

Serotonin,

in

decrease

temperature

Tubuloglomerular feed back

PART-III

(Ref: Textbook of Physiology, AK Jain, Vol. 1, 7th ed. pg. 516)

5.

I.

Define the following:

a.

Dyspnea

ANSWER

O

O

O

Increase in glomerular capillary pressure due to increased afferent arteriolar pressure increases GFR Increase in the filtration and reabsorption function of PCT and DCT causes increase sodium and chloride entry in macula densa cells Increase activity of Na K ATPase pump causes increase adenosine formation Increase adenosine in macula densa cells causes increase calcium uptake causing afferent arteriolar constriction. b.

(Ref: Textbook of Physiology, AK Jain, Vol. 1, 7th ed. pg. 455) ANSWER

0000 0

O

Anion gap

Difficult breathing MVV is less Decreased vital capacity Increase pulmonary ventilation Physiological cause: Sever exercise Pathological causes: Asthma,

Emphysema,

Metabolic acidosis, cardiac failure

(Ref: Textbook of Physiology, AK Jain, Vol. 2, 7th ed. pg. 570-571)

b.

Filtration fraction

(Ref: Textbook of Physiology, AK Jain, Vol. 1, 7th ed. pg. 524) ANSWER

O

O O

O O

Difference between the concentration of cations and anions Plasma anion gap is 12 mEq/L Clinical acid -base status is assessed by measuring cations other then Na* and anions other then Cl and HCO3Anions like albumin, sulphates and phosphates Cations like Calcium, potassium and magnesium

ANSWER

c.

In electrocardiogram the sum of any complex in lead II equal sum of the potential in lead III and lead I

Windkessel effect

(Ref: Textbook of Physiology, AK Jain, Vol. 1, 7th ed. pg. 314-315)

It is the ratio of glomerular filtration rate to the renal plasma flow c.

(Ref: Textbook of Physiology, AK Jain, Vol. 1, 7th ed. pg. 299) ANSWER

d.

PHYSIOLOGY

ANSWER

308:

O. O

O.

Large elastic vessels or windkessels vessels Onventricular systole on increased pressure of blood these vessels distend and on decreased pressure during diastole these return to original state. This property of elasticity and elastic recoil helps maintain steady outflow of blood d.

Local cardiovascular regulatory mechanism

(Ref: Textbook of Physiology, AK Jain, Vol. 1, 7th ed. pg. 325-327)

Einthoven’s law

Limiting pH of urine

(Ref: Textbook of Physiology, AK Jain, Vol. 1, 7th ed. pg. 557-558)

ANSWER

O O

Lowest pH attained in urine is pH4.4, which is the limiting pH of urine The maximum H ions that can be secreted in the urine against the concentration gradient

PAPER-| 5.

II.

Explain why (not exceeding 3-4) sentences)

a.

Cyanosis is not observed in severe anemia

6.

a.

(Ref: Textbook of Physiology, AK Jain, Vol. 1, 7th ed. pg. 467) ANSWER

O O

ANSWER

In sever anemia Hb is 5 mg/dL Reduced hemoglobin up to 5 mg/dL is required for cyanosis to develop Thus due to decreased Hb levels, reduced Hb never

reaches up to 5 mg/dLthus cyanosis does not develop. b.

Phototherapyisgivenin cases of physiological jaundice of newborn.

(Ref: Textbook of Physiology, AK Jain, Vol. 1, 7th ed. pg. 82)

Innate cellular immunity Large granular lymphocytes CD 56

Scavenges tumor cells and viral infected cells Cytotoxic Contains perforins, granzymes and proteases IL-2,IL-12, IL- 15, IL-18 and CCL5 for their activation

b. ANSWER

Jaundice is excess of bilirubin

O

Bilirubin converted to lumirubin in white light Lumirubin is water soluble excreted through urine c.

are responsible

Stages of erythropoiesis

(Ref: Textbook of Physiology, AK Jain, Vol. 1, 7th ed. pg. 70)

O

O

Natural killer cells

(Ref: Textbook of Physiology, AK Jain, Vol. 2, 7th ed. pg. 124)

9O0O0006=~dOdLO

O

Discuss briefly:

For answer refer 2016 paper-1, Q. 2(a), Pg. 188 c.

(Ref: Textbook of Physiology, AK Jain, Vol. 1, 7th ed. pg. 103)

Most of the ECG waves are recorded negative in AVR

(Ref: Textbook of Physiology, AK Jain, Vol. 1, 7th ed. pg. 294-298)

Fibrinolytic system

ANSWER For answer refer 2013 paper-1, Q. 5(b), Pg. 233

ANSWER

Represents electrical activity of ventricular cavity recorded on VR right arm due to negative deflection Vit. Kis given in premature babies.

(Ref: Textbook of Physiology, AK Jain, Vol. 1, 7th ed. pg.

ANSWER 105-106)

ANSWER Factor II, VII, [X and X are synthesized in liver

O

Vitamin K is responsible for post translational modification of these factors by causing gamma carboxylation of these factor and forming calcium binding sites Premature babies lack vitamin K Thus injecting Vitamin K_ prevents bleeding tendency and post translational modification of clotting factors synthesized in liver.

OO

O

O O

250-750 nm, irregular structures having granules (5-8 nm in diameter) present in cytoplasm Contains various proteolytic enzymes: Ribonuclease, deoxyribonuclease, phosphatase, glycosidase, arylsulphatases, collagenase and cathepsins Acidic pH of 5.0 is maintained by proton pump Digests bacteria or macromolecules Autolysis of cell

PHYSIOLOGY

d.

Lysosomes

(Ref: Textbook of Physiology, AK Jain, Vol. 1, 7th ed. pg. 9)

O00

O

d.

Your Roll No. ...............

Name of the Paper

:

Physiology Paper-II

Name of the Course

:

MBBS-2008

Semester

>

Annual

Time: 3 Hours

M.M.: 50 INSTRUCTIONS

1.

Write your Roll No. on the top immediately on receipt of this question paper

2.

All questions are to be attempted

3.

Answers to Parts I, II and III should be written in separate answer sheets provided

4.

Attempt parts of a question in sequence

PART-I 1.

Compare and contrast:

a. b. c. d. 2.

[3]

Central and peripheral retina Explicit and implicit memory Slow and fast pain Outer and inner hair cells

Draw labelled diagrams/flow charts tables to illustrate:

a. b. c. d.

[3]

Normal sleep pattern in young adult Inverse stretch reflex Hormonal interaction in the mediation of calctum homeostasis Milk ejection

PART-II 3.

I.

II.

Give a. b. c. d.

Enumerate (two points each):

a. b. Ill.

PS F

[2]

[2]

Ifsatiety centre is destroyed. To blood cells and lymphatic organs during gluco-cortricoid excess

Write short notes on:

ao

[4]

Two functions of Aqueous humor Two features of kluver-bucy animal

What happens and why (not exceeding 3—4 sentences) a. b.

4.

normal values range of: Power of reduced eye Iodine requirement per day in a normal healthy adult Duration of spinal shock in human being Threshold for normal hearing

Thermoregulation in neonates Stress analgesia Regulation of body fluid osmolality Sensory motor cortex

[10]

PART-III 5.

I.

Il.

Define the following: a.

Motor unit

b.

Adequate stimulus

c.

Neurotransmitter

d.

Conditioned reflex.

Explain why (not exceeding 3-4 sentences)

[4]

[4]

ofF

Hyperthyroid patients lose weight despite excessive eating

d.

oF PS

Describe briefly:

ao

6.

Calcium stabilizes the cell membrane Colour and depth of object cannot be appreciated at night ‘Goose pimples” occur when exposed to cold.

Muscle spindle Genesis of REM sleep Physiological basis of bradykinesia in Parkinson’s disease Mechanism of stimulation of taste receptors.

[8]

PAPER-II

2008 |

2008 PAPER-II

PART-I 1.

Compare and contrast: a.

Central and peripheral retina

|

Fast pain

Slow pain

Few in number

More in number

Stimulated by only noxious stimuli

Stimulated also by thermal and mechanical stimuli

Pressure sensitive

Chemical sensitive / anesthetic

agents

(Ref: Textbook of Physiology, AK Jain, Vol. 2, 7th ed. pg. 1096)

d. ANSWER

Outer and inner hair cells

(Ref: Textbook of Physiology, AK Jain, Vol. 2, 7th ed. pg.

1076)

Central retina

Peripheral retina

® 6mm around fovea centralis

e Extends beyond central retina to ora serrata

e Predominantly cones

e Predominantly rodes

Outer hair cells

Inner hair cells

e Thicker, densely packed photoreceptors

e Thinner

e 3-4 layers

e Single layer

e 3500

e 20,000

e Inner nuclear layer thicker

e INL thinner

e Responsible for highest visual acuity sharpness of vision

© Responsible dark adaptation

e Inner phalangeal cells support them

e Outer phalangeal cell supports them

e Sensory, fine auditory discrimination

e Sound detection, amplitude and clarity

b.

Explicit and implicit memory

ANSWER

(Ref: Textbook of Physiology, AK Jain, Vol. 2, 7th ed. pg. 1041-1043)

2. ANSWER

labelled

diagrams/flow

charts

tables

to

illustrate:

a.

Explicit memory

Implicit memory

e Declarative

e Reflexive, non-declarative

e Working, short term and long term

e Procedural or priming memory for skills, words and object recognition

e Hippocampus involved in processing

e Not involved

e Consciousness involved

e Not involved

c.

Draw

Normal sleep pattern in young adult

(Ref: Textbook of Physiology, AK Jain, Vol. 2, 7th ed. pg. 881)

ANSWER

For answer refer 2012 paper-2, Q. 2(a), Pg. 250 b.

Slow and fast pain

Inverse stretch reflex

(Ref: Textbook of Physiology, AK Jain, Vol. 2, 7th ed. pg. 881)

(Ref: Textbook of Physiology, AK Jain, Vol. 2, 7th ed. pg. 902

ANSWER

ANSWER Fast pain

Slow pain

A delta fibers

Group C nerve fibers

12-30 m/sec

0.5-2 m/sec

Neurotransmitter: Glutamic acid

Substance P

For answer refer 2011 paper-2, Q. 1(d), Pg. 264 c.

interaction

in

the

mediation

of

Calcium homeostasis Contd...

PHYSIOLOGY

Hormonal

(Ref: Textbook of Physiology, AK Jain, Vol. 2, 7th ed. pg. 707)

PAPER-II ANSWER Bone

PTH promotes and cacitonin

inhibits Ca** release

Ca™ removed from

into the blood by

blood by osteoblasts

osteoclasts

PTH promotes

Ca” reabsorption

Unabnsorbed Ca”

from the urine

lost in the feces

Ingested Ca”

VitaminD

PTH Ca” lost in the urine | Promotes

Small

promotes Ca”

intestine

absorption

active vitamin D

V formation

Fig. Calcium homeostasis

Osteoclast

Ca” reabsorption

activity is inhibited

in the kidneys decreases

Thyroid gland

ace

releases calcitonin

%

|

< gor

~/

(10 mg/dL) Normal calcium level

Ca” level in blood ey,

+10mg/db

\

Homeostasis

Normal calcium level (10 mg/dL)

|

Ca” level in blood

@sed C2

Parathyroid glands

decreses

increases

release PTH

Osteoclasts release Ca” from bone

Calcium is Calcium reabsorbed fromjabsorption in the urine by the small intestine kidneys increases via vitamin D synthesis

a



>

©

= O

oe) a

Fig. Calcium Homeostasis

> tT QO

PAPER-II d.

Milk ejection

(Ref: Textbook of Physiology, AK Jain, Vol. 2, 7th ed. pg. 682) ANSWER

000 0

2008

III.

For answer refer 2013 paper-2, Q. 5(b), Pg. 238

Loss of fear, memory and anger response Oral exploration of objects by mouthing Inappropriate sexual behavior Omniphagia and hyperphagia What happens and why (not exceeding sentences) a.

PART-II

3-4

Ifsatiety center is destroyed.

(Ref: Textbook of Physiology, AK Jain, Vol. 2, 7th ed. pg. 1006)

I. a.

Give normal values range of: ANSWER

Power of reduced eye

(Ref: Textbook of Physiology, AK Jain, Vol. 2, 7th ed. pg. 1105) ANSWER O

Iodine requirement healthy adult

per

day

in a normal

(Ref: Textbook of Physiology, AK Jain, Vol. 2, 7th ed. pg. 689) ANSWER 100-200 microgram

c.

Present in ventromedial nucleus of hypothalamus Destruction causes uninhibited feeding center Hyperphagia and obesity occurs b.

Approx. 59 Diopters

b.

O

O O O

Duration of spinal shock in human being

(Ref: Textbook of Physiology, AK Jain, Vol. 2, 7th ed. pg. 935)

To blood cells and lymphatic organs during glucocorticoid excess

(Ref: Textbook of Physiology, AK Jain, Vol. 2, 7th ed. pg. 726) ANSWER 90000 0

3.

Neutrophilia Eosinopenia Basopenia Lymphopenia Thrombocytosis Polycythemia

4.

Write short notes on:

ANSWER

O

Approx. 3 weeks d.

Threshold for normal hearing

a.

(Ref: Textbook of Physiology, AK Jain, Vol. 2, 7th ed. pg. 1075)

(Ref: Textbook of Physiology, AK Jain, Vol. 2, 7th ed. pg. 588)

ANSWER

ANSWER

O II.

O O O

Zero decibel is auditory threshold Enumerate (two points each): a.

Two function of aqueous humor

(Ref: Textbook of Physiology, AK Jain, Vol. 2, 7th ed. pg. 1090)

Nutrition to cornea

O

Nutrition to lens

O

Intra ocular pressure maintenance

L

b.

O

Two features of kluver-bucy animal

(Ref: Textbook of Physiology, AK Jain, Vol. 2, 7th ed. pg. 1025)

oe)

> tT OW

ANSWER

O

Destruction of bilateral amygdala/bilateral temporal lobe

Stress analgesia

(Ref: Textbook of Physiology, AK Jain, Vol. 2, 7th ed. pg. 904) ANSWER

O O00 0

©

O

First 48 hours: T4 After 48 hours: Brown adipose tissue/brown fat Brown fat: Dense sympathetic nerve supply, rich vascularity, increased BMR due to increased oxidative phosphorylation b.

ANSWER

>

Thermoregulation in neonates

Stress like fear, anxiety, injury, were increased sympathetic activity Intrinsic pain inhibitory mechanisms

there

is

Endogenous opioids, endorphins, enkephalins Neuronal stimulation of A delta fibers and C fibers Stimulation of A afferent fibers and dorsal column-

Gate control theory.

PAPER-II c.

2008

Regulation of body fluid osmolality

ANSWER

(Ref: Textbook of Physiology, AK Jain, Vol. 2, 7th ed. pg. 679, 377)

O

ANSWER

O O

d.

Circumventricular organ in hypothalamus has osmoreceptors 1% change in osmolarity causes stimulation of ADH secretion

O

Increase in osmolarity causes ADH secretion

O

Osmoreceptors are plasma Na‘* receptors

ADH/aquaporin _ insertion/water collecting ducts and tubules Osmolarity maintained d.

Conditioned reflex

(Ref: Textbook of Physiology, AK Jain, Vol. 2, 7th ed. pg. 1039) ANSWER

O

O

O

Chemical transmitters released from nerve endings that inhibit or stimulate post synaptic membrane.

reabsorption/

Sensory motor cortex

Acquired response that is achieved on repeated pairing of the stimulus with another stimulus. ii.

Explain why (not exceeding 3-4 sentences)

a.

Hyperthyroid patients excessive eating

lose

weight

despite

(Ref: Textbook of Physiology, AK Jain, Vol. 2, 7th ed. pg. 699)

(Ref: Textbook of Physiology, AK Jain, Vol. 2, 7th ed. pg. 1015) ANSWER

Sensorimotor cortex: The area of the brain that comprises the precentral and post-central gyni and covers the primary sensory and motor areas of the brain. O

O

The area sulcus is the brain The area

between central sulcus and precentral called precentral gyrus; the motor area of and comprises broad mann'"s area 4. between central sulcus and post-central

sulcus is called post-central gyrus; the sensory area

of the brain and comprises areas 2, 3 and 4.

I.

Define the following:

a.

Motor unit

Increase oxidative phosphorylation, increase BMR Increase requirement of nutrition Increase in metabolic functions Protein catabolism

b.

Calcium stabilizes the cell membrane.

(Ref: Textbook of Physiology, AK Jain, Vol. 2, 7th ed. pg. 145, 717) ANSWER O

PART-III 5.

O00

O

ANSWER

Plasma

calcium

levels

are

involved

in _ the

stabilization of resting membrane potential c.

Color and depth appreciated at night

of

object

cannot

be

(Ref: Textbook of Physiology, AK Jain, Vol. 2, 7th ed. pg. 1110)

(Ref: Textbook of Physiology, AK Jain, Vol. 2, 7th ed. pg. 173) ANSWER ANSWER

It the motor fibers innervated by single neurons axon terminals. b.

Adequate stimulus

d.

(Ref: Textbook of Physiology, AK Jain, Vol. 2, 7th ed. pg. 173) ANSWER

c.

“Goose pimples” occur when exposed to cold.

(Ref: Textbook of Physiology, AK Jain, Vol. 2, 7th ed. pg. 591) ANSWER

required

to produce

OO

of stimulus

0

O

Threshold stimulus

Minimum amount action potential

Function of cones Atnightrod cell responsible for night vision Cones are inhibited

Neurotransmitter

(Ref: Textbook of Physiology, AK Jain, Vol. 2, 7th ed. pg. 1047-

1054)

Horripilation Increase in column of trapped of trapped air Prevents heat loss Contraction of piloerector muscle

PHYSIOLOGY

O

O O O

2008 6.

PAPER-II Describe briefly:

a.

Cortex

Muscle spindle.

|

(Ref: Textbook of Physiology, AK Jain, Vol. 2, 7th ed. pg. 881-884) Neostriatum (Indirect

ANSWER

‘Do 1 Dopamine

pathway)

‘ receptors

b.

Genesis of REM sleep.

Inhibition of Substantia nigra-

pallidus-

pars compacta

~ | \ext. segment

(Ref: Textbook of Physiology, AK Jain, Vol. 2, 7th ed. pg. 988)

3

O

Stage 1 - light sleep (high voltage, low frequency slee

900000 0

Excitatory output

spinal cord

d.

bradykinesia

‘55

:

v GP-Internal

52 rs 3 TUR: ---: 6

Segnent ¥

©

= O

wv

> tT QW

PAPER-I ANSWER Severe haemorrhage (Blood loss >40%)

+++!

/Blood Pressure

+++Decreased

(10 min, then

Sphincters and venules spasm

Coronary blood flow decreased

77

CNS ischemia

|

|

Decrease Heart rate and myocardial contraction

VMC activity decreased | Venous pooling

+ve

of blood

feedback

GIT

Systemic

v

circulation



ow Collection of =“ Release of

vasodilators

Blood -

GIT mucosa anes

Blood

Entry of gram

coagulation intravascularly

negative bacteria Into the circulation via toxins

Cardiac depression

Venous pooling v

Cardiac output «——_ { Cardiac failure

+ve feedback

Venous return + Vasomotor failure

Venous return

———®

+ve feedback t Peripheral circulatory failure (PCF)

Fig. Feedback mechanism for irreversible shock

Left ventricular and aortic pressure changes during a normal cardiac cycle in correlation with ECG.

(Ref: Textbook of Physiology, AK Jain, Vol. 1, 7th ed. pg. 287-290)

ANSWER

For answer refer 2011 paper-1, Q. 2(c), Pg. 256 d.

Stages of leucopoiesis.

PHYSIOLOGY

(Ref: Textbook of Physiology, AK Jain, Vol. 2, 7th ed. pg. 89-91)

ANSWER 900000:090

c.

Stem cells Myeloblasts Pre-myelocyte Myelocyte proper Metamyelocyte Mature WBC

Cardiac output i Septic or endotoxic shock

PAPER-I IL-1 IL-6 IL-3 GM-CSF



v

tbe

e

&

(ILs)

EYMprON ast

(GM-CSF)

e

Promonocyte

y

Prolymphocyte

ee. CSF] o.. Hemophilc

Neutrophilic

Basophilic

pre-myelocyte

pre-myelocyte

pre-myelocyte

¢

. y

Myelocyte proper

v

M-CSF & Vv

Ce

Meta myelocyte

Eosinophilic

Neutrophillic

minantl

Neutrophil

es

é

Granular leucocytes

Large lymphocyte

@)

—_ hilic

v

S

Besophh

v

Monocyte

Lymphocytes Peripheral blood

Fig. Leucopoiesis

PART-II

c.

Transport maximum for glucose (TmG)

(Ref: Textbook of Physiology, AK Jain, Vol. 2, 7th ed. pg. 529)

3.

I.

Give normal value/range.

a.

Erythrocyte sedimentation rate

ANSWER

ANSWER For answer refer 2010 paper-1, Q. 5(b), Pg. 277

d.

Normal Values (by Wintrobe Method)

Residual volume

(Ref: Textbook of Physiology, AK Jain, Vol. 2, 7th ed. pg. 415)

Males = 0 to 10 mm in 1 hour, Females = 0 to 20 mm in 1

b

Compliance of lungs alone

(Ref: Textbook of Physiology, AK Jain, Vol. 2, 7th ed. pg. 421-423) ANSWER O

ANSWER

O

Amount of air left inside the lung after maximal forceful expiratory effort

O

1200mL

PHYSIOLOGY

hour, Infants = 0 to5 mm in 1 hour

0.22 L/cm of water

321

PAPER-I 3

II. Enumerate (two point each) a

4.

Choleretics

Write short notes on:

a.

Non-respiratory functions of lungs

(Ref: Textbook of Physiology, AK Jain, Vol. 1, 7th ed. pg. 245)

(A K jain, Vol 1, pg407-408 )

ANSWER

ANSWER

O

O

Substances that increase secretion of bile acids and bile salts

O

Ursodeoxycholic acid b.

O O

Causes of megaloblastic anemia.

(Ref: Textbook of Physiology, AK Jain, Vol. 1, 7th ed. pg. 73)

Ig A immunoglobulins in respiratory mucosa protects against infections Humidification of inspired air Cilia of bronchial mucosa helps in removing particles above 2 micrometer in diameter

ANSWER

O O O

O O O

Synthesis of surfactant Synthesis and release of PGE2 and PGF2 alfa Angiotensin converting enzyme _ secreted by endothelial cells converts angiotensin ! into angiotensin II

O

Reservoir action for left ventricle

Defective DNA synthesis Deficiency of folic acid and B12 deficiency Pernicious anemia is said to occur when there is deficiency of intrinsic factor

b.

Factors affecting GFR

(Ref: Textbook of Physiology, AK Jain, Vol. 1, 7th ed. pg. 522-523)

3

III. What will happen and why (not exceeding 3-4 sentences)

ANSWER

a.

When a deep sea diver rapidly rises to the surface?

O

Age: Decreases with age

O-

Renal blood flow: Increases with increase in RBF

(Ref: Textbook of Physiology, AK Jain, Vol. 1, 7th ed. pg. 477-478)

O

Glomerular’ capillary hydrostatic pressure: Increases with its increase Bowman’s’~ capsule _ hydrostatic _— pressure:

O

ANSWER

OOO

O

Decreases with its increase

Rapid ascent Formation of nitrogen bubbles Plugging of small capillaries Stroke, joint pains, dyspnea, chokes, MI b.

O O

Plasma proteins: Decrease causes increase in GFR as decrease oncotic pressure Permeability of the glomerular membrane

c.

To blood pressure during isometric exercise?

Migrating motor Complexes (MMC) in GIT

(Ref: Textbook of Physiology, AK Jain, Vol. 1, 7th ed. pg. 227-228)

(Ref: Textbook of Physiology, AK Jain, Vol. 1, 7th ed. pg. 170-171,

pg485)

ANSWER Occurs in fasting, 90-120 min post meal

Tension increased, length remains same

O

Contraction of contractile component compensated by series elastic components Antigravity muscles, static exercises

O

Work

done

zero

as

no

displacement,

is

0000 90

O

O

ANSWER

increase

OO

PHYSIOLOGY

O

sympathetic activity, increase in peripheral resistance.

Increase in systolic and diastolic BP Increase in MABP

Rhythmic smooth muscle contraction 5 cm/min Motilin Stomach to distal ilium Phase 1: Spike potential Phase 2: Period ofirregular electrical and mechanical activity Phase 3: Period of regular electrical and mechanical activity

PAPER-| Phase |

Phase Il

— No electrical spike potentials or mechanical activity (no contractions)

Phases of MMC

(a) Antrum

|

Duodenum

.

|

Proximal jejunum |— tT

OW

PAPER-II 2.

Draw labelled diagram/flow chart/table to illustrate a.

Neuromuscular junction in skeletal muscle (Ref: Textbook of Physiology, AK Jain, Vol. 2, 7th ed. pg. 157) Action potential propagation in motor neuron

Axon of motor neuron

Myelin sheath

Axon terminal

Terminal button Vesicle of acetylcholine

——

Voltage-gated calcium channel

Acetylcholine receptor site Acetylcholinesterase Plasma membrane

Action potential propagation in muscle fiber



of muscle fiber }



©

;

7

>,

“An

,

:

[

)

i

)

D

—O (A242 Pi 2) /

\

y WY

(

,

(

OO]



12.

)

\_-

;



~ O Og f

-

©

;

) .

Ȣ



.

\

.

\

'

{ nn

)

ry ,

:

/;

) )

:

/ J)

(2 )

;

-

a

" eZ

>

oP }

\

Nee

ly

: :

.) '

‘ .

; (

:

“al Mi, A



4

:‘

'

et :

'

.



2) .

: Ay

:

/

.

,

Voltage-gated Na* channel

Q

O50"

©

Nat

chemically gat

Motor end plate

cation channel

Q

HA

PA

AA ) AA AA

Contractile elements within muscle fiber

1. An action potential in a motor neuron is propagated to the terminal button. 2. The presence of an action potential in the terminal button triggers the opening of voltage-gated Ca” channels and the subsequent entry of Ca” into the terminal button. 3. Ca” triggers the release of acetylcholine by exocytosis from portion of the vesicles. 4. Acetylcholine diffuses across the space separating the nerve and muscle cells and binds with receptor sites specific for it on the motor and plate of the muscle cell membrane. 5. This binding brings about the opening of cation channels. leading to a relatively large movement of Na’ into the muscle cell compared to a smaller movement of K* outward.

b.

6. The result is an end-plate potential. Local current flow occurs between the depolarized end plate and adjacent memebrane. 7. This local current flow opens voltage-gated Na™ channels in the adjacent membrane. 8. The resultant Na” entry reduces the potential to threshol. initiating an action potential, which is propagated throughout the muscle fiber

9. Acetylcholine is subsequently destroyed by acetylcholinesterase, and enzyme located on the motor end-plate membrane, terminating the muscle cell’s response.

Thyroid hormone synthesis and release

PHYSIOLOGY

(Ref: Textbook of Physiology, AK Jain, Vol. 2, 7th ed. pg. 691)

PAPER-II ANSWER Bio-synthesis and secretion of thyroid hormone

Na*

Deiodination

Thyroglobulin prec ursor (T,)

'

—> crib —>_ 6) ER

an

Golgi

MIT, DIT

Pinocytosis

LS

drop let

Fig. Synthesis and secretion of thyroid hormone

PART-II

Presynaptic inhibition in spinal cord

Cc.

(Ref: Textbook of Physiology, AK Jain, Vol. 2, 7th ed. pg. 864)

3. ANSWER

I. Give normal value/range of: a.

Presynaptic inhibition at an axoaxonic synapse

Fasting levels

and

post prandial

plasma

glucose

(Ref: Textbook of Physiology, AK Jain, Vol. 2, 7th ed. pg. 758) Presynaptic

Nerve

neuron

ANSWER

Nerve

impulse

impulse

ee

Sa

——

a

O O

Fasting glucose: 70-99 mg /dL Post Prandial (2 hours after): less then 140 mg /dl b.

Threshold for normal hearing

(Ref: Textbook of Physiology, AK Jain, Vol. 2, 7th ed. pg. 1078) Inhibitory

Nerve

impulse Postynaptic membrane

ANSWER

y/,

O

c.

Fig. Presynaptic inhibition

d.

Visual pathways lesion and heteronymous hemianopia

Zero decible

(Ref: Textbook of Physiology, AK Jain, Vol. 2, 7th ed. pg. 149)

depicting

(Ref: Textbook of Physiology, AK Jain, Vol. 2, 7th ed. pg. 1093)

ANSWER O

ANSWER

For answer refer 2014 paper-2, Q. 4(b), Pg. 225

Conduction velocity of A o fibers

70-120 m/sec

d.

Serum calcium level.

(Ref: Textbook of Physiology, AK Jain, Vol. 2, 7th ed. pg. 713)

PHYSIOLOGY

neuron

PAPER-II ANSWER

O

10meg/dL

3.

II. a.

Enumerate (two points each): Two hormones’ demonstrating feedback mechanism

positive

(Ref: Textbook of Physiology, AK Jain, Vol. 2, 7th ed. pg. 681)

09000 0 0

ANSWER

ANSWER

Disorder of neuromuscular junction Ptosis, diplopia Slurred speech Antibodies against Ach receptors on post synaptic membrane

Luteinizing hormone

(Ref: Textbook of Physiology, AK Jain, Vol. 2, 7th ed. pg. 955)

Two actions of melatonin

(Ref: Textbook of Physiology, AK Jain, Vol. 2, 7th ed. pg. 765)

ANSWER

O

Transaction colliculi

O

Interruption from the corticospinal, corticobulbar and rubrospinal tract

O

Reticulospinal tract inhibitory and_ excitatory impulses remain intact to extensor muscles

O

Decerebrate rigidity

O

O

Decerebrate rigidity

Extensor muscle hyperactivity

O

Oxytocin

Opisthotonus

ANSWER

O

Circadian rhythm

O

Color of the skin

3.

III. What happens and why (not exceeding 3-4 sentence) a.

Fatigue

b.

O

b.

Weakness of skeletal muscle

Tosensations in tables dorsalis?

c.

(Ref: Textbook of Physiology, AK Jain, Vol. 2, 7th ed. pg. 940, 583)

between

the

superior

and _ inferior

Non REM sleep

(Ref: Textbook of Physiology, AK Jain, Vol. 2, 7th ed. pg. 988)

ANSWER

Dorsal nerve root degeneration results in loss of sensations Loss of tactile localization and discrimination Loss of stereognosis Loss of fine touch Loss of kinesthetic sensation Inability and loss of position sense Charcot’s joints Ulcers on pressure points Lightening pain b.

PHYSIOLOGY

High voltage Slow frequency Decreased muscle tone Decreased BP, HR and RR

Release of Growth hormone

d.

To hearing in otitis media?

ANSWER

O

Occurs before and after REM sleep

Referred pain

(Ref: Textbook of Physiology, AK Jain, Vol. 2, 7th ed. pg. 904)

(Ref: Textbook of Physiology, AK Jain, Vol. 2, 7th ed. pg. 1075)

O

ANSWER 090000 0

99000000

O

ANSWER

O

Injury or disease present at one site and pain felt on other site

ossicles and eustachian tube

O

Cardiac pain felt in left arm

Conduction type of hearing loss

O

Convergence theory: Visceral supplied by same nerve root

5.

Define the following:

Injury

or

disease

to

tympanic

membrane,

ear

and

somatic

area

PART-III 4.

Write short notes on:

a.

Myasthenia gravis

(Ref: Textbook of Physiology, AK Jain, Vol. 2, 7th ed. pg. 160)

a.

Tonic receptors

(Ref: Textbook of Physiology, AK Jain, Vol. 2, 7th ed. pg. 873)

PAPER-II ANSWER

ANSWER

O

Oo O

These adaptors discharge tonically for many hours as they adapt incompletely very poorly and slowly b.

Law of projection Afferent neural pathway when stimulated at any point alongits course, the sensation will be perceived , as coming from sensory organ.

Arterial blood pressure

(Ref: Textbook of Physiology, AK Jain, Vol. 2, 7th ed. pg. 349)

d.

Visual acuity is maximum at fovea centralis (Ref: Textbook of Physiology, AK Jain, Vol. 2, 7th ed.

ANSWER O

pg. 1089-1090)

Itis lateral pressure exerted on the walls of the artery

by blood flowing through it. c.

ANSWER

Denervation Hypersensitivity

(Ref: Textbook of Physiology, AK Jain, Vol. 2, 7th ed. pg. 175, 194) ANSWER

O

Increased sensitivity to chemical stimuli of post synaptic membrane after it is deprived of nerve

O

Itsrod free

© O O

Cones densely packed No blood vessels Sharpest vision

6.

Describe briefly:

supply. d.

a.

Paraplegia

Aldosterone escape

(Ref: Textbook of Physiology, AK Jain, Vol. 2, 7th ed. pg. 732)

(Ref: Textbook of Physiology, AK Jain, Vol. 2, 7th ed. pg. 916)

ANSWER ANSWER O

Paralysis of both lower limbs

5.

II.

Increase in mineralocorticoid levels

O O

Sodium and water retention Increase in the systemic arterial > 25 mm Hg

neuron

lesion

produces

(Ref: Textbook of Physiology, AK Jain, Vol. 2, 7th ed. pg. 919)

O

Upper motor plasticity

Increase in GFR

O

Explain why (not exceeding 3-4 sentences)

Pressure diuresis occurs

O

a.

O

Further no increase in arterial BP

b.

blood

pressure

Brown squared syndrome

(Ref: Textbook of Physiology, AK Jain, Vol. 2, 7th ed. pg. 938)

ANSWER

O

Increased gamma motor brain and spinal cord

O

Brisk reflexes, spastic paralysis

©

Hemi transaction of spinal cord

©O

Abnormal Babinski sign

O

Changes above the injury: Hyperesthesia both side

b.

neuron

discharge

from

Hypocalcemia produces tetany

(Ref: Textbook of Physiology, AK Jain, Vol. 2, 7th ed. pg. 717)

ANSWER

Changes at the level O

Complete sensory, vasomotor and motor loss on the same side.

O

Nomotor loss but some loss of pain sensation on the opposite side.

ANSWER

O

© O

Decreased calcium levels decrease the threshold for

Changes below the level

membrane excitability

O

Upper motor neuron type

5

Increase in the irritability of nerves Increased neuronal excitability causes features of

rigidity, vasomotor loss. Sensory-fine touch, tactile localization and discrimination, stereognosis lost

oO GH

tetany c. Pain in phantom limb

but pain, crude touch and temperature remains. On other side: Motor-no loss. Sensory-loss of pain, temperature and crude touch but intact fine touch,

O-

(Ref: Textbook of Physiology, AK Jain, Vol. 2, 7th ed. pg. 876)

O

On same

side: motor:

>

tactile localization and discrimination, stereognosis.

331

PAPER-II c.

= Anopia

Color blindness

(Ref: Textbook of Physiology, AK Jain, Vol. 2, 7th ed. pg.

color

blindness,

anomaly

represents weakness

1114-111)

d.

Genesis of RMP

ANSWER

(Ref: Textbook of Physiology, AK Jain, Vol. 2, 7th ed. pg. 36)

Inability of a person to perceive different colors is color blindness

It is generated from potential difference generated from the difference in the intracellular and extracellular concentration gradient of sodium,

Types

potassium, calcium and chloride ions across the cell membrane.

Monochromatic color blindness: Prot (red)/green (deuter)/trit (blue) anopia, single color anopia

O

Dichromatats: Color blindness of 2 colors

O

Trichromat: Color blindness of all 3 color types =" X-linked inheritance =

Incidence in males is more then female

Neurons-70 mV, muscle-90 mV OO

O

PHYSIOLOGY

means

Negative value means that the cell has got more negative ions/charge inside from outside Maintained by Na +, K + and Cl-channels and Na-K ATPase pump.

Biochemistry

Biochemistry by Satyanarayana, 5th edition

Your Roll No. ..

Name ofthe Paper

:

Biochemistry Paper-I

Name ofthe Course

:

MBBS-2018

Semester

Annual M.M.: 50

Time: 3 Hours

INSTRUCTIONS Write your Roll No. on the top immediately on receipt of this question paper All questions are to be attempted

Attempt parts of a question in sequence

PART-I

[10]

Write briefly on:

a b. c. d

Diagnostic enzymes Phenylketonuria Chemiosmotic theory of ATP generation Functions of glutathione

[10]

Explain why: aoSF PS

a

Answers to Parts I, II and III should be written in separate answer sheets provided

Dietary fiber is recommended in patients with diabetes Myoglobin is not a suitable carrier of oxygen Chronic alcohol intake leads to fatty liver Premature babies often suffer from acute respiratory distress syndrome

PART-II a.

What do you understand by Mucosal block theory in iron absorption? Discuss iron-deficiency disorder

b.

Discuss the pivotal role of citric acid cycle in metabolism

What is active methionine? Discuss its biochemical significance?

[10] [10]

PART-III Explain different types of lipoproteins. Discuss their role in cholesterol transport:

[S|

OR Discuss the synthesis and break down of glycogen. Write a note on its regulation: Write short notes on:

a.

Biochemical tests for differentiating various types of jaundice

b.

Glycemic index of food

[S|

PAPER-I |

2018 PAPER-I

PART-I 1.

"

Write briefly on: O. a.

Diagnostic enzymes

Introduction

Enzymes are divided into two groups =" Plasma_ functional/plasma _ specific—normally present in plasma =" Plasma — nonfunctional/nonplasma present in low or absent in plasma

O

Serum enzyme level indicates the balance between the synthesis and release of enzyme. Important for diagnosis and prognosis of certain diseases Raised enzyme levels are due to = Cellular damage = Increased rate of cell turnover

BIOCHEMISTRY

Bone disorders 0 6Rickets 5 Hyperparathyroidism 0 Osteocarcinoma * Liver disorders 5 Obstructive jaundice * Ref values - 50-100 U/L Acid phosphatase

and

prognosis

of prostate

it helps

Lactate dehydrogenase * * * * = Ref

O

in detection

Myocardial infarction Infective hepatitis Leukemia Muscular dystrophy values - 200-450 U/1

Creatine kinase "

Increased in

* Early myocardial infarction * Muscular dystrophy =" Ref values - 50-200 U/l (Males); (females)

of

b.

30-150

U/1

Phenylketonuria (Ref: Biochemistry by Satyanarayana 5th ed. pg. 352)

Increased in acute pancreatitis

= Ref values - 30-150 U/1 =" Peaks at 3th or 4th day after onset of disease = Serum Amylase is also used in diagnosis of * Chronic pancreatitis * Acute parotitis * Pancreatic duct obstruction Alanine transaminase

= Elevated in * * * = Ref O

Alkaline phosphatase

Amylase =

O

Ref values - 10-50 U/1

=" Used in the diagnosis of

Examples of Diagnostic Enzymes O

Liver diseases

=" Used in diagnosis gland cancer O

Mainly nonplasma specific enzymes

O

*

= Ref values - 0.5-5.0 U/I

O

= Proliferation of cells = Increased synthesis Used as biomarker—as cellular damage

O

specific—

Diagnostic Enzymes

O

Myocardial infarction

*

ANSWER

O

*

=" Used in diagnosis of

(Ref: Biochemistry by Satyanarayana Sth ed. pg. 110)

O

|

Viral hepatitis Jaundice Liver cirrhosis values - 10-45 U/l

Aspartate transaminase

=" Used in diagnosis of

ANSWER Introduction O

Most common

amino acid metabolic disorder

O

Occurs due to deficiency of enzyme - phenylalanine hydroxylase

O

Autosomal recessive disorder

O

Phenylalanine is not converted to tyrosine, leading to accumulation of phenylalanine in tissues

Phenylalanine Metabolism in Phenylketonuria O

Increased excretion in urine

O

Excessive production of phenylacetate and phenyllactate

phenylpyruvate,

PAPER-| O

These metabolites are excreted in urine

O

Phenylacetate imparts mousey odour.

urine

of PKU

O patients

a

Clinical/Biochemical Manifestations of PKU

O

Effect on central nervous system = Characterized by

=

*

Mental retardation

*

Seizures

* Failure to walk and talk This is due to

O

MoreHtions onthe outer side ofinner mitochondrial

O.

membrane (low pH) than on its inner side. This proton motive force (PMF) is sufficient to result in the ATP synthesis from ATP synthase complex.

O

Protons

*

O

Accumulation of phenylalanine in brain impairing the metabolism and transport of other amino acids. * Deficiency of serotonin * Defective myelin formation Hypopigmentation - defective melanin synthesis

O.

Increased plasma phenylalanine levels in newborns to about 20-65 mg/dL

O

Normal values Guthrie test phenylalanine Ferric chloride

O

O

O.

O

O

- 1-2 mg/dL - bioassay for the detection of in newborns test - detection of phenylpyruvate in

Chemiosmotic theory of ATP generation (Ref: Biochemistry by Satyanarayan 5th ed. pg. 228)

ANSWER

Introduction Theories proposed for oxidative phosphorylation are O

Chemiosmotic theory

O

Boyer’s binding change mechanism

O

Chemical coupling hypothesis

Functions of Glutathione

ANSWER Functions of Glutathione

Biological function is due to reversible reduction capability of glutathione O O O O

O

O

O

Most widely accepted theory of electron transport chain Proposed by Peter Mitchell Explains the mechanism of ATP production by the transport of electrons in the respiratory chain According to this theory, ATP synthesis in the respiratory chain is linked to the H* electrochemical gradient

Proton Gradient

O

intermembrane

(Ref: Biochemistry by Satyanarayana 5th ed. pg. 68)

Inner mitochondrial membrane is impermeable to protons

O

O

O

the

ComplexV - has the ATP synthase - uses the proton gradient for the ATP generation Also known as ATPase as it hydrolyzes ATP to ADP and iP ATP synthase = Complex enzyme = Consists of 2 functional subunits - Fl and FO = Structure is similar to ‘lollipops’ d.

Chemiosmotic Theory

O

in

Prenatal diagnosis - using cultured amniotic cells c.

O

accumulate

Enzyme System for ATP Synthesis

urine

O

that

space re-enter the mitochondrial matrix leading to ATP synthesis.

O

Diagnosis

Electron transport through the respiratory chain is coupled with the translocation of protons across the inner mitochondrial membrane to the intermembrane space. This results in the generation of a proton gradient across the membrane.

oxidation

-

Serves as a coenzyme for certain enzymes such as prostaglandin synthase, glyoxalase Prevents oxidation of sulfhydryl groups It has a role in formation of disulfide bonds in proteins Reduced glutathione plays a role in maintenance of structure and integrity of RBC membrane Protects hemoglobin from oxidation Involved in the transport of amino acids via the y glutamyl cycle or Meister cycle Plays a role in detoxification Glutathione peroxidase - acts as free radical scavenger.

> oO

D 2.

=

Explain why:

a.

Dietary fiber is recommended with diabetes

Li

LL

in patients

(Ref: Biochemistry by Satyanarayana 5the ed. pg. 507)

O

OCO

PAPER-I 100 -

ANSWER

Myoglobin pO, in lungs

Non-digestible by human enzymes

O

2types

=

Soluble: * Present in legumes and fruits * Water soluble * Results in gel formation * E.g. Pectins = Insoluble: * Found in vegetables and grains * Adsorb water * E.g. Cellulose and hemicellulose O Intestinal bacterial can digest certain fibers e.g. Pectins Beneficial effects of fiber in patients with diabetes O O

O

Monomeric

O

Resembles _ individual structurally

BIOCHEMISTRY

O O

100 change

at

=

Has strong affinity for oxygen

=

Structure prevents superoxide from leaving heme group

= Thus myoglobin functions as a more favorable oxygen storage protein than oxygen transport protein

hemoglobin

Chronic alcohol intake leads to fatty liver

moieties ANSWER

O

Heart

Functions

Oxygen reservoir/storage oxygen to rapidly respiring muscle

When oxygen diffuses through interstitial fluids, the amount of oxygen dissolved in the fluid is called the Partial Pressure of Oxygen and is represented as PO.,. Oxygen diffuses from areas with high levels of PO, to areas with low levels of PO.,. Represents the oxygen binding capacity of hemoglobin and myoglobin Shows that myoglobin tightly binds oxygen as compared to Hb.

“Fatty liver” (hepatic steatosis) occurs in conditions in which there is an imbalance between hepatic triacylglycerol synthesis and the secretion of VLDL. Such

conditions

include

obesity,

uncontrolled

diabetes mellitus, and chronic ethanol ingestion.

O

Chronic alcohol consumption can also result in alcoholic fatty liver due to increased synthesis of triacylglycerols. This occurs as a result of decreased fatty acid oxidation due to a fall in the NAD+/NADH ratio, and increased lipogenesis due to the increased availability of fatty acids (decreased catabolism) and of glyceraldehyde 3-phosphate (the dehydrogenase is inhibited by the low NAD+/NADH ratio).

O

Alcohol-induced hepatitis results from acetaldehyde and _ free-radical generation from ethanol metabolism in the liver (via the MEOS oxidation pathway).

Oxygen Dissociation Curve

O

- No

|

(Ref: Biochemistry by Satyanarayana 5th ed. pg. 324)

=" Skeletal muscle

O

|

Myoglobin not suitable as oxygen carrier because

c.

Foundin

= Transports cells

|

=" When oxygen concentration is low, oxygen released from oxyhemoglobin is taken up by myoglobin which supplies it to the tissues.

Oxygen binding hemoprotein

=

50 pO, (mm Hg)

|

O

Myoglobin is not a suitable carrier of oxygen

O

O

|

Myoglobin Dissociation Curve normal physiologic chemistries.

Myoglobin

"

|

O

ANSWER

O

pO, in tissues

|

Fiber improves glucose tolerance by the body Reduces the rate of glucose absorption from the intestine. b.

Hemoglobin |

O

uo

Complex carbohydrates

©

O

Saturation with O2

Introduction

PAPER-| CH,CH,OH

Symptoms

O O O O

Ethanol

NADPH

NADP+ +2H,O

PART-II “a

O

3.

CHCp SH Acetaldehyde

a.

What do you understand by Mucosal block theory in iron absorption? Discuss iron deficiency disorder: (Ref: Biochemistry by Satyanarayan Sth ed. pg. 417)

Fig. The reaction catalyzed by the microsomal ethanol oxidizing system (MEOS) in the endoplasmic reticulum (ER).

O

Cirrhosis occurs as an accumulation of damage to the hepatocytes, leading to fibrosis and loss of liver function. Acute

effects

of ethanol

arise from

the increased

Alterations in fatty acid metabolism occur as fatty acid oxidation is inhibited by the high levels of NADH. Fatty acids accumulate in the liver, produce

Unique to Iron Homeostasis

O

triacylglycerols, and increase the production of VLDL. The export of VLDL is diminished in chronic alcoholics, leading to a fatty liver, due to impairment in protein synthesis due to chronic liver dysfunction.

Regulation of iron absorption - to maintain homeostasis of iron. Iron stores are depleted - increased absorption Adequate stores - decreased absorption Only 10% of dietary iron is absorbed Increased in high body demands like anemia Usually present as ferric state, binds to proteins or organic acids. Iron is released as Ferric in the presence of acid in stomach Ferric form is converted to ferrous state by reducing substances such as vitamin C and cysteine Ferrous form is soluble and readily absorbed.

ANSWER

O

O

Mucosal Block Theory O

NADH/NAD‘ ratio due to ethanol metabolism.

O

ANSWER

0000 0

O

In mucosal cells, ferrous is converted back to ferric

Infant Respiratory Distress Syndrome

O

O O

O

d.

Premature babies often suffer respiratory distress syndrome

from

O

acute O

(Ref: Biochemistry by Satyanarayan 5th ed. pg. 38)

O O

Deficiency of surfactant Characterized by =" Collapsed air-spaces alternating expanded areas = Vascular congestion = Hyaline membranes. Increasing the workload of breathing Resultant hypoxia

with

hyper-

lron Deficiency Disorders O O

Also Known as

O O O

Newborn respiratory distress syndrome Hyaline membrane disease Surfactant deficiency lung disease

O

Etiology Normal babies start to produce surfactant by the age of 24-28 weeks of pregnancy. If the baby is born prematurely, amount of surfactant in lungs may not be sufficient, causing IRDS

form by enzyme ferroxidase. Ferric combines with protein apoferritin to form ferritin (temporary storage form) When required, iron enters the blood stream from mucosal cells.

O.

Most prevalent nutritional disorder Causes =" Inadequate intake of dietary iron =" Inadequate absorption of iron =" Chronic blood loss = Repeated pregnancies =" Hookworm infections More common in strict vegetarians as = Relatively low content of iron in diet = Presence of iron absorption in vegetarian diet Iron deficiency occurs in = Young children = Adolescent girls

= Pregnant and lactating women

BIOCHEMISTRY

nome

+H'* +O,

Rapid shallow breathing Blue coloured lips, fingers and toes Grunting sound while breathing Flared nostrils

PAPER-I O

O

Characteristics = Reduced hemoglobin levels (>12 g/dL) = Microcytic hypochromic anemia = Sluggish metabolic activities = Retarded growth = Loss of appetite = Apathy Treatment = Iron supplementation along with folic acid and vitamin C

ANSWER

b.

O

Discuss the pivotal role of citric acid cycle in metabolism (Ref: Biochemistry by Satyanarayan 5th ed. pg. 256)

Introduction Involves

the

oxidation

of acetyl

CoA

to carbon

O

Active methionine - donates during transmethylation

group

the

methyl

Synthesis of Active Methionine

O

O O

Occurs by transfer of adenosyl group from ATP to sulfur atom of methionine Enzyme involved - methionine S-adenosyltransferase During activation, sulfur gets converted to sulfonium atom Active methionine - SAM -isasulfonium compound 3ATPs are consumed

Utilizes about 2/3rds of the oxygen consumed by the body Also known as

O

Presence of positive charge - makes it highly reactive

O

Enzymes involved - methyl transferases

O

Active methionine transfers methyl group to an acceptor and gets converted to S-adenosyl homocysteine

O

Irreversible reaction

Methionine is regenerated back in the process. Active methionine is useful in the synthesis of = Polyamines - spermidine, spermine

O

Amphibolic nature

O

Provides intermediates for the synthesis of various compounds

0000 0

(SAM)

Biochemical Functions of Active Methionine

Role of Citric Acid in Metabolism

=" Creatine

Both catabolic and anabolic in nature

=

Involved in

= N-Methylnicotinamide

Gluconeogenesis

=

Transamination

= Methylated tRNA bases

Deamination

=

Synthetic (Anabolic) Reactions

O O

Active methionine - S-adenosyl methionine

dioxide and water

=" Tricarboxylic acid cycle =" Krebs cycle

O

O

O

O

Sulfur containing essential amino acids Needs to be activated for transmethylation

O

O

O O

O

ANSWER

O

Introduction

Oxaloacetate and a - ketoglutarate - serve as precursor for synthesis of aspartate and glutamate, used in the synthesis of non-essential amino acids, purine and pyrimidines.

Epinephrine Choline Protein methylated amino bases

O

Active methionine is required for the activation of certain inactive compounds

O

Controls protein turnover - methylation prevents the protein degradation

O

Precursor of plant hormone, involved in ripening of fruits.

Succinyl CoA - synthesis of porphyrins and heme Acetyl CoA - biosynthesis of fatty acids, sterols

ethylene in plants -

PART-III

BIOCHEMISTRY

Metabolic (Catabolic) Reactions

5.

O O

Central metabolic hub of the cell. Aerobic metabolism of carbohydrates

O

Provides ATP and NADH, FADH2 - provides energy for various metabolic pathways.

Explain different types of lipoproteins. Discuss their role in cholesterol transport: (Ref: Biochemistry by Satyanarayana 5th ed. pg. 316)

ANSWER 4.

What

is_

active

methionine?

Discuss

its

biochemical significance: (Ref: Biochemistry by Satyanarayan 5th ed. pg. 361)

Introduction O

Lipoproteins - lipids + proteins

O

Transport lipids in plasma

PAPER-| 5 major types - differentiated based on specific proteins attached to the phospholipid outer layer, called the apolipoprotein = Chylomicrons - transport dietary triacylglycerol = Very low-density lipoproteins (VLDL) - transport of endogenous triacylglycerol Low density lipoproteins (LDL) cholesterol from liver to other tissues

O

O

transport

* Consists of high concentration of cholesterol * Considered bad cholesterol =" High-density lipoproteins (HDL) - involved in reverse cholesterol transport. * Considered good cholesterol

Chylos - juice; micron - small

= Transports exogenous/dietary triacylglycerols to various tissues. = Consist of * Triglycerides - 85-92% * Phospholipids - 6-12% * Cholesterol - 1-3% * Proteins - 1-2% (mainly apoprotein B,,) = Least in density in lipoproteins = Largest size. = =

Stabilizes the lipid droplets Increases the solubility of lipids.

O

Dependent on enzyme lecithincholesterol acyltransferase (LCAT) and high-density lipoproteins

O

LCAT = Plasma enzyme = Synthesized by liver

O

Consists of protein, fats and cholesterol synthesized in the liver.

O

O O

Associated with 5 different apoproteins =

B-100

= = =

C-I C-Il C-Il

= E. Converted to IDL and LDL by removal of the apoproteins Second only to chylomicrons in the percentage triglyceride content.

LDL O O

= Substrate - HDL-cholesterol = Transfers the fatty acid on second position of lecithin (phosphatidyl choline) to hydroxyl group of cholesterol = Reversible reaction = Associated with apo-A, protein of HDL Forward Transport of Cholesterol O O

VLDL

O

Bad-cholesterol. Transports cholesterol from liver to peripheral tissues. LDLcontains only one lipoprotein Apo B-100.

O O

LDL particles are derived from VLDL. Small part is directly released from liver.

HDL is a good cholesterol. Transports cholesterol from peripheral tissues to liver. Synthesized in liver. Major apoproteins in HDL are Apo Al, with some Apo A2, Apo C and Apo E. HDLis a plasma reservoir of Apo C and Apo E, which can be transferred to VLDL and chylomicrons.

Transport of Cholesterol

Chylomicrons =

Binding of LDL to receptor is by apo-B-100. Half-life is 2 days.

HDL O

to tissues

=

O

75% of plasma cholesterol is incorporated into LDL particles. LDL transports cholesterol from liver to peripheral tissues. The transported cholesterol has following fates: = For synthesis of steroids. = May be incorporated into membranes. = May be esterified to MUFA and stored. Extracellular domain of LDL receptors is responsible for binding of apo-B-100 and apo-E. Intracellular domain is responsible for the clustering of LDL receptors into regions of plasma membrane termed coated pits. Apo-B-100 binds to apo-B-100 receptor, receptor LDL complex is internalized by endocytosis. The endosome vesicles fuses with lysosomes. The receptor is recycled and returned to the cell surface. LDL particles, apoproteins and cholesterol esters are hydrolyzed by hydrolases, forming free amino acids and free cholesterol. 70% of LDL is degraded in the liver and remaining is in extra-hepatic tissues. plasma Free cholesterol incorporated into membrane

or stored in cells.

BIOCHEMISTRY

O

PAPER-I Forward and reverse transport of cholesterol

Liver

-—>|

VLDL

/+—>

LDL “Bad” cholesterol

deposit

“Good” cholesterol

(Excretion)

HDL

Reverse Cholesterol Transport

O

Mostly stored in liver and muscles

O O

O

Synthesis of glycogen - glycogenesis

O

Breakdown of glycogen - glycogenolysis

O

Multi-step process Results in movement of cholesterol from peripheral tissues back to the liver. Process

=

Cholesterol is transferred to HDL from nonhepatic tissues by: * ABCAI1 - ATP-binding cassette transporter acts as a Carrier * Apoprotein Al - protein of HDL - acts as acceptor * Phospholipid component of HDL - acts as sink for mobilized cholesterol. = Cholesterol is esterified to form cholesteryl esters by LCAT = ‘These are transferred by cholesteryl ester transfer protein (CETP) in exchange for triglycerides =

Cholesterol esters form a part of HDL.

=" Uptake of HDL by liver is through lipoprotein lipase (hepatic lipase). = Cholesterol is metabolized and degraded for excretion in liver. O

Hepatic lipase activity = Increased by androgens =" Decreased by estrogens. Or Discuss the synthesis and break down of glycogen. Write a note on its regulation

> oO

D

(Ref: Biochemistry by Satyanarayana 5th ed. pg. 259)

Glycogenesis O.

Site - cytosol

O

Requires glucose, UTP and ATP

O

Steps of Glycogenesis =

Synthesis of UDP- glucose

*

Enzymes required - hexokinase (in muscle) or Glucokinase (in liver) 5 Phosphoglucomutase * Glucose is converted to glucose 6 phosphate, which is further catalyzed to glucose 1 phosphate * Glucose 1 phosphate gets converted to UDP glucose in the presence of UTP and UDP glucose phosphorylase = Initiation of glycogenesis with the help of primer * Primer - accepts glucose from UDP- glucose * Small fragment of pre - existing glycogen * In absence of glycogen, glycogenin acts as primer

=" Glycogen synthesis *

Enzyme glycogen synthase transfers glucose from UDP-glucose to the non reducing end of glycogen to form alpha 1,4 - linkages =" Formation of branches *

=

LU

LL

ANSWER

O

OCO

Branching enzyme - glucosyl alpha 4,6 transferase - transfers a small fragment of 5 - 8 glucose residues to another glycogen residues by alpha 1, 6 bonds.

Introduction

Glycogen O

Storage form of glucose in animals

*

Glycogen is further elongated and branched, respectively, by the enzymes glycogen synthase and glucosyl 4-6 transferase

PAPER-| = Formation of glucose 6 phosphate and glucose * Enzymes’ glycogen’ phosphorylase and debranching enzyme produce glucose 1 phosphate and glucose in the ratio of 8:1. * Phosphoglucomutase converts glucose 1 phosphate to glucose 6 phosphate. * Fate of glucose 6 phosphate is further tissue dependent.

Glucose

ATP Glucokinase ADP

Glucose 6-phosphate | Phosphoglucomutase

Glucose 1-phophate UTD

Glycogen PiC-D

UDP-glucose pyrophosphorylase

Glycogen phosphorylase

PP;

Glu-1 phosphate

UDP - glucose Limit dextrin

OH

Debranching enzyme (transferase activity)

Glycogen initiator synthase

UDP

Debranching enzyme (a- 1,6 glucosidase activity)

Glycogen primer Glucosyl (X-4-6) transferase

Phosphorylase

Elongation by glycogen synthase

Glucose 1-phosphate

Branching enzyme

|

Glycolysis
oO

Produced by the electron transport chain and at other sites. Cannot diffuse far from the site of origin. Generates another ROS.

D

Not a free radical, but can generate free radicals by reaction with a transition metal (e.g., Fe2*). Can diffuse into and through cell membranes.

LL

=

Li

O

OCO

PAPER-I Reactive species

OHe, hydroxy!

| Properties

The most reactive species in attacking biologic molecules. Produced from H,O, in the Fenton reaction in the presence of Fe** or Cut

radical

ANSWER Introduction O

Coffee contains caffeine

O

Harmful Effects of Free Radicals

Caffeine plays a role in Increases both selective and sustained attention * Selective attention: Focus on relevant stimulus * Sustained attention: Focus over an extended period of time Consumed shortly after waking to increase alertness and fight sleep inertia Improves working memory Beneficial effects on performance Caffeine modulates neural activity in a network involving the parietal and prefrontal cortexsustained attention. Beneficial in multi tasking and mental arithmetic Caffeine acts as adenosine A,, receptor antagonist Thus inhibits all the actions of adenosine As a result caffeine Increases neural activity Decreases sleep Increases alertness Enhances dopamine signally by increases D,/D, receptors.

O

b.

ROe, Re, R-S, organic radicals

Organic free radicals (R denotes the remainder

RCOOs, peroxyl

An organic peroxyl radical, such as that occurs during lipid degradation (also denoted LOOe)

radical

of the compound). Produced from ROH, RH (e.g., at the carbon of a double bond in a fatty acid), or RSH OHe attack

hypochlorous acid 0,7, singlet oxygen

Oxygen with antiparallel spins. Produced at high oxygen tensions from absorption of UV light Decays so fast that it is probably not a significant in vivo source of toxicity

NO, nitric oxide

RNOS. A free-radical produced endogenously by nitric oxide synthase. Binds to metal ions. Combines with O, or other oxygen-containing radicals to produce additional RNOS.

ONOO-, peroxynitrite

RNOS. A strong oxidizing agent that is not a

O

O

Produced in neutrophils during the respiratory burst to destroy invading organisms. Toxicity is through halogenation and oxidation reactions. Attacking species is OCI"

HOCI,

free radical. It can generate NO, (nitrogen dioxide), which is a radical

Damage the biomolecules Causes oxidation of sulfhydryl groups of proteins, causes fragmentation and crosslinking of amino acids: Loss of protein function Causes lipid peroxidation: |. PUFA-most susceptible to free radical damage Cause DNA breakage, fragmentation of bases.

fructose

leads

to

(Ref: Biochemistry by Satyanarayana 5/e, pg. 280)

ANSWER Mechanism of Fructose Metabolism

Disease association Fructose

Cardiovascular disease: Oxidized LDL promote atherosclerosis Cancer: Due to effect on nucleic acidCauses mutagenicity and cytotoxicity-role in carcinogenesis Inflammatory diseases: Rheumatoid arthritis Diabetes mellitus: Glycation ofproteins increases its susceptibility to free radical damage-basis of diabetic microangiopathy, diabetic nephropathy. Other disease Male infertility Aging Cataract Parkinson’s Alzheimer’s

ae|

Fructokinase

Fructose - 1 - phosphate

-—_—_ Glyceraldehyde Dihydroxyacetone | hosphate)

NADH+H’

ee

(DHAP

Alcohol

Dehydrogenase NAD*

@ -?er-t & &

BIOCHEMISTRY

Excessive ingestion of hypertriglyceridemia

Vv Glycerol ATP

Glycerol

kinase

v Glycerol 3-phosphate

Liver cirrhosis, etc. Role of Fructose

2.

Explain why: a.

Intake of coffee causes alertness

O

More readily absorbed/metabolized glucose

in liver than

PAPER-| O O

Preferred for energy needs in sperms Increased dietary intake - increased Acetyl CoA = Increased lipogenesis = Increased fatty acids, triacylglycerol and VLDLs =" Leads to hypertriglyceridemia and increased

ANSWER

Procollagen Suicide

cholesterol in blood

. . = Increases risk of atherosclerosis c.

d.

O

Procollagen suicide - selective destruction of aberrantly folded trimeric molecules containing mutant pro- alphal chains

O

Mutations cause procollagen suicide. J, ;

suicide takes place in human

O O

(Ref: Biochemistry by Satyanarayana 5/e, pg. 487)

O

Procollagen body

; Mutation in genes coding for procollagen synthesis Glycine is replaced by another amino acid leading to defective procollagen structure and brittleness Seen in Osteogenesis imperfecta (Brittle bone disease)

Premature cataract is observed in children with galactosemia (Ref: Biochemistry by Satyanarayana 5/e, pg. 278)

ANSWER

O

Important enzymes for Galactose Metabolism are galactokinase and galactose 1-phosphateuridyltransferase. Genetic deficiencies of these enzymes produce galactosemia. Cataracts, a characteristic finding in patients with galactosemia, result from conversion of the excess galactose in peripheral blood to galactitol in the lens of the eye, which has aldose reductase. Accumulation of galactitol in the lens causes osmotic damage and cataracts. The same mechanism accounts for the cataracts in diabetics because aldose reductase also converts glucose to sorbitol, which causes osmotic damage. Deficiency of galactose 1-phosphate uridyltransferase produces a more severe disease because, in addition to galactosemia, galactose 1-P accumulates in the liver, brain, and other tissues. Intestine

Lactose

Symptoms of lactase deficiency after ingestion

Lactase

of lactose (milk products) include: ¢ Diarrhea * Bloating

|

t

Glucose

Galactose

Blood

Vv Galactose

* Cramps Lens

}/---------- -» Galactose- PUGOSS TOGUCTASS +> Galactitol lf galactose accumulates}

Liver, Brain and Other tissues

ATP)

Galactitol trapped in the lens causes swelling and cataracts

Galactokinase deficiency: ¢ Cataracts early in life

Galactokinase

App«| Vv

Gal 1-P uridyltransferase deficiency: ¢ Cataracts early in life * Vomiting, diarrhea following lactose ingestion * Lethargy * Liver damage, hyperbilirubinemia

Galactose 1-P

Gal 1-P uridyl

epimerase

»

transferase

UDP-Gal

* Mental retardation bs

Glucose 1-P In the well-fed state, galactose can enter glycolysis or contribute to glycogen storage

is

7;

Glycogen

Glucose 6-P v

y

Glycolysis

Glucose

Administration of galactose during hypoglycemia induces an increase in blood glucose

Fig. Galactose metabolism

BIOCHEMISTRY

OO

O O O

PAPER-I PART-II 3.

a.

Discuss briefly how iron is absorbed in the body (Ref: Biochemistry by Satyanarayana 5/e, pg. 416)

O

Ferric combines with protein apoferritin to form ferritin (temporary storage form)

O

When required, iron enters the blood stream from mucosal cells.

Factors Affecting Fe Absorption O.

O

Site of absorption - stomach, duodenum and upper jejunum.

Factors promoting iron absorption =" Acidity, =" Vitamin C =" Presence of Alcohol = Fructose

O O

Only 10% of dietary iron is absorbed Increased in high body demands like anemia

= Cysteine = Iron deficiency anemia

O

Usually present as ferric state, binds to proteins or

= Diet rich in small peptides and amino acids

organic acids.

=

ANSWER lron Absorption

0 O O O

Iron is released as Ferric in the presence of acid in

©

Diet deficient in phosphates

Factors interfering with iron absorption

stomach

=

Ferric form is converted to ferrous state by reducing substances such as vitamin C and cysteine Ferrous form is soluble and readily absorbed. In mucosal cells, ferrous is converted back to ferric form by enzyme ferroxidase.

= Oxalate (found in leafy vegetables) = Phosphate rich diet = Malabsorption syndrome such as steatorrhea = In patients with partial or total surgical removal of stomach and/or intestine

Lumen of GIT Food Fe

Mucosal cells of GIT Apoferritin

Ferntin

HCl

(Fe*)

organic acids

Fe™

Transferrin (Fe")

~|

Ferro-

Ascorbic acid

Fe”

Plasma

> Fe”

Ferritin

Ceruloplasmin

v

or ferroxidase I

Fe”

‘.

Liver

Hemosiderin

Fe”

reductase

[Feros

Tissues

Transferrin

Fe”

| cysteine

Phytate (found in cereals)

>Fe”

BONS MEW Muscle (Mb)

(0)

Other tissues

(Cyts & NH)

Fig. Absorption of iron

b.

Explain the role of Vitamin D in mineral metabolism. Add a note on the deficiency manifestations (Ref: Biochemistry by Satyanarayana 5/e, pg. 127)

ANSWER

O O

Biosynthesized from cholesterol sunlight - sunshine vitamin Antirachitic vitamin

O

to

Calcitriol (1,25-DHCC) - biologically active form. = Regulates the plasma calcium and phosphate

Introduction > Vitamin D: Eat soluble vitamin

levels. = Acts at intestine, ney

= C)

O O

low P asm meee . . . Action of calcitriol on the intestine:

Oco

exposure

Role of Vitamin D in Mineral Metabolism

a2 2

Functions like hormone Derived from ergocalciferol and cholecalciferol

on

O

and bone in response to

PAPER-| = Increases the intestinal absorption calcium and phosphate.

O

Symptoms "

Rickets in the children

*

Bone deformities Due to incomplete mineralization Delayed tooth eruption Bow legs * Elevated alkaine phosphatase activity =" Osteomalacia in adults. * Demineralization of bones * Increased susceptibility to fracture = Diagnosis of vitamin D deficiency * Plasma levels

O

Detoxification refers to the process of conversion of toxic compounds (xenobiotic) in the body to nontoxic or less toxic products that can be easily excreted.

D =

Li

LL

O

OCO

PAPER-I = Aldehydes undergo oxidation to form acids * Benzaldehyde to benzoic acid * Chloral to chloroacetic acid

OO

Liver is the main site for detoxification.

O

Xenobiotic refers to the chemical/foreign substance that is toxic to the body. Mechanism of Detoxification of xenobiotics =" The mechanism of detoxification in liver occurs in two phases: *

Phase

I

includes

reactions

such

= Amines

*

Aliphatic amines are oxidized to corresponding acids and urea * Aromatic amines undergo oxidation to form phenols = Aromatic hydrocarbons - benzene are converted to phenols = Sulfur is oxidized to sulfuric acid = Drugs such as meprobamate are processed by oxidation

as

oxidation, reduction and hydrolysis: Liver is a storehouse of various enzymes catalyzing the oxidation-reduction reactions such as oxidases, cytochrome P 450. Oxidation

O

BIOCHEMISTRY

O.

Detoxification of substances is by their oxidation to form nontoxic products These include: = Aromatic and aliphatic alcohols * Methanol to formic acid *

Ethanol to acetic acid

*

Benzyl alcohol to benzoic acid

Reduction

O

For example = Picric acid is reduced to picramic acid, chloral to trichloroethanol.

Your Roll No. ...............

Name ofthe Paper

:

Biochemistry Paper-II

Name ofthe Course

:

MBBS-2017

Semester

Annual

Time: 3 Hours

M.M.: 50

INSTRUCTIONS 1.

Write your Roll No. on the top immediately on receipt of this question paper

2.

All questions are to be attempted

3.

Answers to Parts I, II and III should be written in separate answer sheets provided

4.

Attempt parts of a question in sequence

PART-I Write briefly:

ao

SF

1.

2.

[10]

Mechanism of action of tetracycline and puromycin Myasthenia Gravis Facilitated diffusion During protein translation AUG serves both as initiation codon and codon for internal methionine residues in polypeptide. Explain how translation machinery is able to discriminate the initiation codon from internal AUG codons?

Explain why: a. b. c. d.

[10]

DNA vaccine provide a better approach for protection Lesch-Nyhan syndrome Auto-immune diseases with one example Tumour markers with two examples

PART-II 3.

Write a note on the significance of human genome project in health and disease

[5]

Or Explain the role of post-translations modifications 4.

in ensuring strength of collagen

Discuss the purine salvage pathway. Add a note on the related disorders

[10]

Or Explain the cooperation between B-cell and T-cell in generation of immune response

PART-III 5.

Explain the mechanisms involved in the development of antibody diversity

[5]

OR Explain the application of PCR in biomedical research and diagnosis 6.

Write short notes on:

a. b.

Southern blotting Chaperones

[10]

PAPER-II |

2017 PAPER-II

PART-I 1.

ANSWER Facilitated Diffusion

Write briefly:

a.

Mechanism puromycin

of action

of tetracycline

O

and

O O

OO

(Ref: Biochemistry by Satyanarayana 5/e, pg. 556) ANSWER

O

Compounds that inhibit protein synthesis are also utilized as antibiotics. Streptomycin, tetracycline, chloramphenicol, and erythromycin inhibit protein synthesis on prokaryotic (70S) ribosomes, and are used to treat a variety of infections. Because mitochondria contain 70S-type ribosomes that function similarly to those in prokaryotic cells, these compounds also inhibit mitochondrial protein synthesis. Tetracycline binds to the 30S ribosomal subunit of prokaryotes and inhibits the binding of aminoacy]ltRNA to the A site. Puromycin binds at the A site, forms a peptide bond with the growing peptide chain, and prematurely terminates synthesis. It acts in both prokaryotes and eukaryotes.

Diffusion is facilitated by carrier/transport proteins Occurs along the concentration gradient Solutes moves from higher concentration to lower concentration No energy is used Carrier or transport proteins used in facilitated diffusion are = Glucose =" Galactose =

Leucine

=

Phenylalanine etc

Mechanism of Facilitated Diffusion—PING PONG MODEL O O O

O

O

|

Transport protein occurs in two forms: Ping and

pong In pong conformation - it is exposed higher solute concentration. The solute binds to the specific sites Upon binding with the solute, the undergoes conformational changes

to the side with at the carrier carrier protein (ping form)

The solute is thus released on the other side of the membrane

b.

Myasthenia gravis (Ref: Biochemistry by Satyanarayana 5/e, pg. 701)

ANSWER O

O

0

BIOCHEMISTRY

O

c.

Facilitated diffusion (Ref: Biochemistry by Satyanarayana 5/e, pg. 625)

099

eE1 00

Pong

Myasthenia gravis is a neuromuscular disorder due to an autoimmune condition that produces antibodies against the acetylcholine receptor. The receptor cannot be activated by acetylcholine, and the muscle cell cannot respond to the

The overlying symptom of this disorder is muscle fatigue. The treatment consists of acetylcholinesterase inhibitors (such that acetylcholine is present for an extended period at the neuromuscular junction) and immunosuppressants to reduce the production of the autoantibodies targeted against the receptor.

Oo

C=1—-

i

Ping

Diagrammatic representation of ‘Ping-Pong’ model of facilitated diffusion

d.

During protein translation AUG serves both as initiation codon

and

codon

for internal

methionine residues’ in _ polypeptide. Explain how translation machinery is able

neurotransmitter, and contraction will not occur. O

Po

to discriminate the initiation internal AUG codons?

codon

from

(Ref: Biochemistry by Satyanarayana 5/e, pg. 551) ANSWER Introduction O

Initiation codons - start codon - first codon of mRNA

in the initiation of protein translation.

PAPER-II O.

Initiation codon - AUG - also codes for methionine

O

O

Stop codons - termination codons - last codon of mRNA during protein translation Stop codons - UAA, UAG, UGA

After its introduction in the target tissue, it is expressed using both transcriptional and translation mechanism.

O

Behave as viruses.

O

O

O

AUG codon serves both as initiation codon and for methionine. 2sets of AUG codon -

=" Methionine-bearing initiator tRNA = Methionine-specific tRNAs—for internal AUG codons. Translation machinery is able to discriminate between the initiation codon from internal AUG due to the presence of additional sequences present before the initiation codon. Shine-Dalgarno sequence =" Additional recognition sequence contained within the mRNA before the intended initiation codon AUG,

=

O

Helps the ribosome to recognize where it should start translating. = Present in prokaryotes. = AUG sequences on the 5’ side of the initiation codon is disregarded. Kozak consensus sequences = Seen in eukaryotic cells =" Used to recognize the initiating AUG codon. ="

O

internal

AUG

after initiation are recognized methionine-bearing tRNA.

2.

codons

ANSWER

Lesch-Nyhan Syndrome

present

by a non-initiator

O

DNA vaccine provide a better approach for protection (Ref: Biochemistry by Satyanarayana 5/e, pg. 588)

ANSWER

OOO

O

O.

Novel approach Introduced in 1990 Third generation vaccine Involves direct introduction of plasmid containing the DNA sequence encoding the antigen into the body. Itis a genetically engineered bacterial plasmid

Neurological symptoms = Mental retardation =" Aggressive behavior = Learning disability = Self-mutilation

Biochemical Basis

O

DNA Vaccine

Due to deficiency in HGPRT enzyme Inherited disorder Sex-linked metabolic disorder Affects only males Associated with primary gout Also known as juvenile gout Leads to: = Increased synthesis and accumulation of purines =" Decreased utilization of guanine and hypoxanthie by salvage pathway =" Reduced levels of GMP and IMP = Increased production of uric acid

Manifestations

Explain why: a.

Lesch-Nyhan syndrome (Ref: Biochemistry by Satyanarayana 5/e, pg. 397)

= Aids in attachment of ribosome to MRNA During protein translation in both prokaryotes eukaryotes,

Easier to insert into the host body Low cost of production b.

Present at 5’ end of mRNA,

and

Stimulates either or both B and T cells Absence of any infective potential Improved vaccine stability Highly specific

O

Brain depends on salvage pathway purine nucleotide synthesis: Basis of symptoms in Lesch-Nyhan syndrome Uric toxicity is not seen in brain allopurinol does not affect the symptoms of syndrome

for de novo neurological tissues: So neurological

Treatment

O

Allopurinol - drug of choice c.

Auto-immune diseases with one example (Ref: Biochemistry by Satyanarayana 5/e, pg. 701)

BIOCHEMISTRY

O

Advantages of DNA Vaccine-Better Approach for Protection 000000

Recognition of Initiation Codon

9O0O0006=~dOdLO

O

PAPER-II +—h6€*M

ANSWER

O

Self-reactive immunity-auto immunity. Diseases produced due to autoimmunity are autoimmune diseases Mechanism/pathophysiology = Consequence of infection by foreign antigen-

¢-

O O

¢

Auto-Immunity and Autoimmune Disease

Lung carcinoma Pancreatic carcinoma Colon cancer

PART-II 3.

Write anote on the significance ofhuman genome project in health and disease

bacterial, viral or fungal

(Ref: Biochemistry by Satyanarayana 5/e, pg. 598)

=

O

O

Some antigens are homologous to host protein epitopes = Leading to cross reaction between these epitopes and antibodies. Characteristics = Destruction of cells and tissues = Altered growth = Altered function = Usually chronic debilitating disease Examples =" Type I diabetes * Insulin dependent diabetes * Auto-reactive T cells damage Beta cell ofpancreas = Rheumatoid arthritis * Antibodies act against the joint proteins * Causes inflammation and stiffness of all joints

Cancer of germ cells of testis

d.

ANSWER Introduction

O

Started in 1990

O

Primary objective = Mapping of human genome = To elucidate the entire nucleotide sequence of human DNA

Significance of Human Genome Project O

O

Tumour markers with two examples (Ref: Biochemistry by Satyanarayana 5/e, pg. 662)

ANSWER Introduction

O O O

Biochemical markers that detect cancer Produced by tumor cells Canbe = Surface antigens = Cytoplasmic proteins = Enzymes or =" Hormones

Examples

O

Carcinoembryonic antigen (CEA) =" Colon cancer =" Colorectal cancer

>

="

oO

=" Stomach cancer

D

=

LU

LL

O

OCO

O

Pancreatic cancer

="

Lung cancer

="

Breast cancer

= Applications Alpha-fetoprotein (AFP) = Elevated serum levels are seen in * Hepatic cancer

Impact on biology and technology =" Understanding complex biological system ="

Decodingthe human DNA, genes and their function

= =

Development of pharmacogenomics Development of proteomics

Impacton health and disease = To understand the complex polygenic diseases e.g. cancer, hypertension, diabetes = Better understanding of human genetic variations = Identifying cancer causing gene mutations = Developments in gene therapy = Early diagnosis, monitoring disease progression and response to treatment Or Explain the role of post-translations modifications in ensuring strength of collagen (Ref: Biochemistry by Satyanarayana 5/e, pg. 488)

ANSWER Post-translational Modifications

O O

About 8 post-translational modifications occur in procollagen. Intracellular post-translational modifications =" Hydroxylation * Ofselective proline residues to form hydroxyproline * Ofselective lysine residues to form hydroxylysine * Requires vitamin C =" Glycosylation * Ofselective lysine and hydroxylysine residues * Addition of glucose or galactose

PAPER-II = Formation of triple helix O

=

Purine salvage pathway

* Covalent intermolecular cross linking Extracellularly post-translational modification

*

=

* *

Cleavage of propeptides * Removes terminal amino acids * Enzyme-aminoproteinase and carboxyproteinase = Oxidative deamination * Nonenzymatic condensation reaction = Formation of covalent intra and inter molecular cross links = Formation of collagen fibrils

Purines are synthesized during the normal turnover of nucleic acids (RNA) Obtained from dietary sources Conversion of thus obtained purines to respective nucleotides is through the purine salvage pathway Phosphoribosyl pyrophosphate (PRPP) is the source of ribose in purine salvage pathway.

*

Adenine phosphoribosyltransferase Adenine y

Critical for the structure and biomechanical function

O

of collagen Gives strength to the collagen

PP,

Hypoxanthine-guanine phosphoribosyl transferase Guanine

Significance of Post-translational Modifications O

\

PRPP

* Enzyme lysyl oxidase * Cofactor-copper = Aggregation of collagen fibrils to form collagen fibers

Adenosine monophosphate

SNe PRPP

> GUanosine monophosphate

PP,

Hypoxanthine-guanine phosphate tarnsferase (HGPRT) Hypoxanthine 7

4.

(Ref: Biochemistry by Satyanarayana 5/e, pg. 598) ANSWER Introduction

O

Purines are nitrogenous bases

O

2 carbon purines

O

ring bases:

1 carbon rings: pyrimidines.

Adenine

Thymine

and and

guanine cytosine

\

PRPP

Discuss the purine salvage pathway. Add a note on the related disorders

Inosine monophosphate

PP,

Salvage pathway of purine metabolism

Significance of purine salvage pathway O

Important in brain, erythrocytes and bone marrow

O O

Saves energy Deficiency of syndrome

enzyme

HGPRT-Lesch-Nyhan

Or

are

Explain the cooperation between B-cell and T-cell in generation of immune response

are

(Ref: Biochemistry by Satyanarayana 5/e, pg. 699)

Purine Nucleotide Metabolism

*

Means

synthesis of purines from phosphor

¢-

-

@

ribose, one carbon units and amino acids

Main pathway Inosine monophosphate is the final product This acts as precursor for AMP and GMP Nucleoside monophosphates are converted into their respective di and tri phosphates by transfer of phosphate groups from ATP.

ANSWER

T cells and B Cells Cooperation in Immune Response O

O00 0

2 Pathways =" De Novo synthesis

-

O

T, cell receptors bind to class II histocompatibility complex (MHC)-antigen

major

Present on antigen presenting cells (APC) APC secretes interleukin-1, which activates T,, cells

Clones of T,, cells are produced T,, cells secrete interleukin-2-activation of cytotoxic T cells-cell mediated immunity is activated

> oO

D

=

Li

LL

O

OCO

PAPER-II O

Interleukin-2-activation of B_ cells-produces immunoglobulins-activation of humoral immunity

Structure of Immunoglobulin O O

.

O So _— _ |«—

Interleukin-1

Antigen fragment T-cell receptor

O

Interleukin-2 cytokines

2 identical heavy (H) and light (L) chains Each chain has constant (C,, and C,) and variable regions (V, and V, ) Types of light chains-differ C, regions " Kappak = Lambdaiz Types of heavy chains = y-IgG a-IgA 6-IgD u-IgM é-IgE

Junctional Diversity

Activation

O

Precise

position

are which

V, D,

J segments

are

fused-can lead to DNA recombination-changes in amino acids.

O.

This is known as junctional diversity

Somatic Hypermutation T, cell

B-cell

Cell-mediated immunity

Humoral immunity

'



O

Rearrangement hypermutation

Oo

Itsrateis more than that of mutation rate.

Explain the mechanisms involved development of antibody diversity

variable

gene

units-somatic

OR

Explain the application research and diagnosis

PART-III 5.

of

in_

PCR

in

biomedical

(Ref: Biochemistry by Satyanarayana 5/e, pg. 579)

the

(Ref: Biochemistry by Satyanarayana 5/e, pg. 189)

of

ANSWER Introduction

ANSWER Introduction

O

OO

O

O O

Human genome consists of

O

oO

D

=

LU

LL

O

OCO

O

Large number ofantibodies are formed by mutations and recombinations. Antigen diversification processes identified are: = Combination association of light and heavy chains =" Somatic hypermutation = Multiple germ line gene segments =" Combinatorial V-D-J joining = Junctional flexibility = P-and N-region nucleotide additions

Acell free amplification technique used to amplify specific segments of DNA Can generate billions of identical copies of specific DNA segments.

DNA polymerase synthesizes new DNA strand complimentary to the template strand. In PCR, dsDNA is denatured to separate the two strands These act as templates for DNA polymerase to produce two separate dsDNA and hence, multiple copies can be generated with this chain reaction.

Components of Polymerase Chain Reaction O O O

O O

Target dsDNA segment template Athermostable DNA polymerase Two primers (synthetic oligonucleotides containing about 30 base pairs) complimentary to the target DNA segments Deoxynucleotide triphosphates that provide energy for polymerization reaction Buffer system.

PAPER-II Technique of PCR

O

O

O

ANSWER

Denaturation: Heat (about 95 degrees Celsius) separates the DNA strands into two single stranded segments. Renaturation or annealing: The primers base pair with complimentary regions by forming hydrogen bonds as the temperature drops to 55 degree Celsius. Synthesis: The DNA polymerase adds nucleotides to 3’ end of each primer complimentary to the target DNA producing two dsDNA strands.

Note: This complete cycle (cycle 1) takes about 35 minutes to complete. Subsequent second cycle produces 4 molecules and so on.

Types of Blotting Techniques O

For DNA: Southern blotting

O

For RNA: Northern blotting

O

For both RNA/DNA: Dot blotting

O

For proteins: Western blotting

Southern Blotting O

Developed by Ed Southern in 1975

O

Blotting technique for separation of DNA

Technique Genomic DNA

Applications of PCR

O

Restriction endonuclease

Clinical Diagnosis =

=

Prenatal diagnosis of inherited diseases *

Can be detected by PCR in chorionic villus samples or cells from amniocentesis.

*

Such as sickle cell anemia, beta thalassemia

t

Separation of short + long DNA fragments

and phenylketonuria. Diagnosis and monitoring of infectious diseases * Retroviral infections such as HIV, by using PCR from cDNA *

=

Electrophoresis on agarose gel

| Denature by mild alkali Nitrocellulose membrane transfer

(Post transfer)

Bacterialinfections such malaria, tuberculosis,

etc. Diagnosis of altered oncogenes and monitoring of cancer therapy.

[cone probe

*

= O

Detection of virally-induced carcinomas such as cervical cancer by human papilloma virus * Cancers due to chromosomal translocations such as follicular lymphoma due_ to translocation of chromosome 14 and 18. Helps in determination of sex and sex-linked disorders in in vitro fertilized embryos.

Exact replication and annealing of DNA fragments

Research and molecular genetics = DNA sequencing =

In genomic studies-to compare genomes in two organisms.

"In

phylogenetic

analysis-DNA

analysis

in

Formation of hybridized bands on auto radiograph Fig. Technique of southern blotting

Application

O

Method of gene analysis

O

DNAcloning

O

In forensics thieves

O

Determination of RFLP (restriction fragment length polymorphisms) associated pathologies.

mummies, bones, hair etc.

=

O

b.

Gene expression analysis

rapists

and

Chaperones

Forensic Applications = As a tool in genetic fingerprinting with the help of a single molecule of DNA from any source such as

to identify parenthood,

(Ref: Biochemistry by Satyanarayana 5/e, pg. 556) > oO

D

ANSWER

hair, blood, semen etc.

=

Li

Introduction 6.

Write short notes on:

a.

Southern blotting (Ref: Biochemistry by Satyanarayana 5/e, pg. 576)

LL

O

Proteins are biologically quaternary structure

active

in

tertiary

and

O

Chaperones assist this protein to attain the 3-D conformation.

O

OCO

O

PAPER-II O O

Assist in protein folding. First chaperone to be known-histones-assist folding of DNA ATP dependent

O

the

Location

Mostly are heat shock proteins (HSP)

O

Expressed in response to high temperature cellular stress Mainly present in the endoplasmic reticulum

O

and

Types of Chaperones-2 Types O

Hsp 70 system

= Bind to protein individually = Assists in protein folding and helps protein attain its conformation = Consists of Hsp 70 and Hsp 40.

> oO

D

=

LU

LL

O

OCO

Functions O

0000

O

Chaperonin system = Oligomeric protein structure that encapsulate the folding substrate protein = Consists of Hsp 60 and Hsp 10. =" Work in association with Hsp 70 = Seen in both prokaryotes and eukaryotes

Reversibly bind to hydrophobic regions of unfolded proteins and folding intermediates Stabilize intermediate proteins Prevents formation of unwanted intermediates Assists in protein degradation Also involved in membrane transport

Clinical Significance O

Multisystem proteinopathy-due genes encoding chaperones

to

mutation

in

Your Roll No. ...............

Name of the Paper

:

Biochemistry Paper-!

Name of the Course

:

MBBS-2016

Semester

Annual M.M.: 50

Time: 3 Hours

INSTRUCTIONS Write your Roll No. on the top immediately on receipt of this question paper

Answers to Parts I, II and III should be written in separate answer sheets provided Attempt parts of a question in sequence

PART-I

[10]

Answer briefly: a.

Secondary active-transport across biological membranes b. Role of 2,3-bisphosphoglycerate in oxygen transport c. Role of cytochrome P-450 in xenobiotic metabolism d. Biochemical basis of renal osteodystrophy

[10]

Oral Rehydration Solution (ORS) is prescribed in diarrhoea Barbiturates administration precipitates attack of acute intermittent porphyria Excessive alcohol intake may damage liver Intake of fibers lowers serum cholesterol

2 FS

Explain why

ao

nol is

All questions are to be attempted

PART-II a.

Explain why obesity is considered a malnutrition state

b.

Define isoenzymes. Discuss significance of any two isoenzymes as diagnostic aid

What are trace elements. Discuss biochemical significance of copper and selenium.

[10] [5]

PART-III Describe the detoxification process of ammonia. Add a note on ammonia toxicity:

[10]

OR a. b.

Explain the reciprocal regulation of glycogenesis and glycogenolysis Discuss the molecular mechanism of action of insulin on target tissues

Answer in brief:

a. b.

Role of thermogenin in brown adipose tissue Biochemical basis of toxicity of lead

[S|

PAPER-I |

2016 PAPER-I

)

PART-I

Glucose

Na

Gut

1.

Answer briefly: a.

Secondary active-transport across biological membranes (Ref: Biochemistry by Satyanarayana 5/e, pg. 627) Intestinal mucosa

ANSWER

Symport system

Active Transport

O O O

Antiport System

Works against the concentration gradient Dependent on ATP Carrier mediated

O O O

2 Types O O

Primary active transport systemNa*-K* pump Secondary active transport symport and antiport system

the

= Proton pump in stomach

e.g. ion pumps-

system

Transport of 2 different molecules across membrane Both molecules move in the opposite direction E.g. movement of Cl and HCO, in RBCs Cl

e.g.

uniport,

Uniport System

Membrane

O

Movement of single molecule through the membrane

O

E.g. glucose transport in RBCs Glucose

ECM

Antiport system

b.

Role of 2,3-bisphosphoglycerate transport

in oxygen

(Ref: Biochemistry by Satyanarayana 5/e, pg. 199) RBCs

rrier Carrier

molecul molecule

ANSWER Mechanism of Action

Uniport system

O Symport System

O

Transport

of

2

different

molecules

across

the O

membrane

> oO

D =

Li

LL

O

OCO

O

Both molecules move in the same direction

O

E.g. movement of Na* intestinal mucosa

and

glucose

from

gut to

O

2, 3-BPG binds to deoxyhemoglobin and not oxyhemoglobin, decreasing the oxygen affinity to hemoglobin This facilitates release of oxygen, shifting the oxygen dissociation curve to right Central cavity of deoxyhemoglobin tetramer binds with one molecule of 2,3-BPG

PAPER-| O

Positively charged globin chains in the central pocket bonds with negatively charged phosphate groups of 2,3-BPG Cross linking of beta chains stabilizes the deoxy Hb

O

a storehouse of various enzymes catalyzing the oxidation-reduction reactions such as oxidases, cytochrome P 450.

*

Stripped Hb

(no 2,3-BPG) 100 -

O cS

s

/

Normal blood

(with 2,3-BPG)

O

= Cc

£ 50-

Blood of anemic patient

e

(2,3-BPG7)

Phase II includes conjugation reactions: Enzymes such as_ transferases, and conjugating agents such as glucuronic acid, glycine, cysteine, glutamine, methyl group, etc are present in liver. Oxidation is the major process for detoxification. Conjugation may occur either independently or following phase I reaction. Cytochrome P.,,,is the major enzyme family that is involved in oxidation reactions. d.

©



Biochemical basis of renal osteodystrophy (Ref: Biochemistry by Satyanarayana 5/e, pg. 410)

oS

0

ANSWER

rn 50 pO, (mm Hg)

100

Introduction

O

Also known as renal rickets

Clinical Significance

O

Seen in renal failure patients

O

O

Defective bone development

O

Alteration in bone morphology

O

Increase in 2, 3 BPG is seen in hypoxic conditions such high altitude, obstructive pulmonary diseases. Increased in conditions with increased oxygen demands such as severe anemia, fetal tissues.

O

During blood transfusion,

inosine is added to the

blood stored in acid citrate- dextrose medium prevent the decrease in 2, 3 BPG.

In the absence

Fetal hemoglobin binds weakly with 2,3-BPG, thus higher affinity for oxygen. c.

Role of cytochrome metabolism

P-450

in

O

Damage in renal tissue causes defective calcitriol synthesis

O

Also known as vitamin D resistant rickets

to

of 2,3BPG, transfused blood fails to supply oxygen immediately to tissues. O

Cause

Biochemical Basis

O

Failure of vitamin D to convert into calcitriol

O

Decreased calcitriol levels

O

Increased excretion through the kidney

O

Decreased reabsorption

O.

This causes hypocalcaemia

2.

Explain why:

xenobiotic

(Ref: Biochemistry by Satyanarayana 5/e, pg. 614)

of

calcium

and

phosphate

ANSWER

Role of Cytochrome P,._ in Xenobiotics

O. O O

Detoxification refers to the process of conversion of toxic compounds (xenobiotic) in the body to non-toxic or less toxic products that can be easily excreted. Liver is the main site for detoxification. Xenobiotic refers to the chemical/foreign substance that is toxic to the body. Mechanism of detoxification of xenobiotics = The mechanism of detoxification in liver occurs in two phases: *

Phase

I

includes’

reactions

such

as

oxidation, reaction and hydrolysis: Liver is

Oralrehydration solution (ORS) is prescribed in diarrhea (Ref: Biochemistry by Satyanarayana 5/e, pg. 410) >

ANSWER

Oral Rehydration Therapy O O00 9

O

a.

Given orally Cheap, simple, effective and noninvasive

Involves drinking water with salts and glucose Does not treat diarrhea Replenishes fluid and electrolyte balance

oO

D

=

Li

LL

O

OCO

PAPER-I O

Alcohol dehydegenase

Glucose in ORT enables intestine to absorb sodium

efficiently =

O

This is due to the presence of Glucose and Na* symport - Sodium Dependent Glucose Transporter - in the intestinal mucosal cells Should be taken every time the stool is passed

Acetaldehyde

ae

Alcohol

b.

Barbiturates administration precipitates attack of acute intermittent porphyria (Ref: Biochemistry by Satyanarayana 5/e, pg. 210)

NAD*

Fig. Metabolism of alcohol in liver

So increased amounts of alcohol = increased NADH = increased conversion of pyruvate to lactate = lactic acidosis O

Reduced gluconeogenesis-leads to hypoglycemia

O

Impaired citric acid cycle —> increased acetyl CoA — increased synthesis of TG fatty liver

O

Increased

ANSWER O

Acute intermittent porphyria

O

=

uric

acid:

As

a result

of lactic

Vitamin B and C deficiency Increased acetaldehyde leads to =" Headache =

Nausea

"

Hypotension

=" Tachycardia d.

Manifestations of Acute Intermittent Porphyria O O O O

serum

acidosis

O

Due to deficiency of uroporphyrinogen I synthase enzyme = Excretion of porphobilinogen and aminolevulinate in urine = Urine changes its colour on exposure to air = Oxidation of porphobilinogen to form porphyrin - black colour of urine

| NADH +H" |

Intake of fibers lowers serum cholesterol

(Ref: Biochemistry by Satyanarayana 5/e, pg. 507)

Neuropsychiatric disturbances Abdominal discomfort Cardiovascular abnormalities Photosensitivity

ANSWER Introduction

O

Barbiturates induces cytochrome P.,..

O00

Role of Barbiturates

Leads to increased aminolevulinate synthase enzyme Increased porphobilinogen and aminolevulinate Thus exaggerating/precipitating the manifestations of acute intermittent porphyrias. c.

O O

Complex carbohydrates Nondigestible by human enzymes

O

2types

Excessive alcohol intake may damage liver (Ref: Biochemistry by Satyanarayana 5/e, pg. 324)

ANSWER Introduction

O

O

O > oO

D

=

Excessive alcohol consumption causes liver disease Leads to metabolic impairment of liver Alcoholic liver diseases include: = Fatty liver = Alcoholic hepatitis = Liver cirrhosis

alcoholic

Biochemical Changes in Alcoholism

OCO

O

O

Most

of

the

metabolism

consumed

alcohol

enters

fibers

e.g.

Role in Maintaining Cholesterol Levels O

LU

LL

O

=" Soluble: * Present in legumes and fruits * Water soluble * Results in gel formation * E.g. Pectins =" Insoluble: * Found in vegetables and grains * Adsorb water * E.g. Cellulose and hemicellulose Intestinal bacterial can digest certain Pectins

liver

for

Reduces serum cholesterol levels by =" Decreasing intestinal absorption of dietary cholesterol = Decreases the enterohepatic circulation of bile salts by binding with them = Reduced degradation of cholesterol to bile salts ="

Increased excretion of cholesterol

PAPER-| b.

PART-II a.

Explain why obesity malnutrition state

is

considered

(Ref: Biochemistry by Satyanarayana 5/e, pg. 515)

Causes of Obesity

OOO

O

ANSWER

For answer refer 2017 paper-1, Q. 6(a), Pg. 365

ANSWER

O

(Ref: Biochemistry by Satyanarayana 5/e, pg. 114)

a

4.

What

Lack of physical activity Viral infection (adenovirus -36) is ob gene

Leptin defects - body weight regulatory hormone

O

=" Daily requirement: Less than 100mg =" Further divided into 3 subclasses

*

Body mass index (BMI) BMI (kg/m?) is calculated as weight in kgs divided by (height in meters)? =" Reference Range for BMI * 25-30-grade I obese * >30-grade II obese or clinical obesity * >40-grade III obese or morbid obesity Waist hip ratio-more effective = Ratio between waist size and hip size

Essential trace elements: e.g. iron, copper, iodine,

manganese,

zinc,

*

cobalt, selenium, chromium, fluorine Possibly essential: Nickel, vanadium, cadmium, barium

*

Nonessential: Aluminium, mercury, sliver, etc.

O

RDA: 2-3 mg/day-adults

O

0.5-2 mg/day: Infants and children

O

Sources: Liver, kidney, meat, egg yolk, cereals

=

6.6 mmol/L O

Leads to metabolic acidosis

Symptoms-non Specific

Palpitations

Write short notes on:

a.

Causes and consequences of hyperkalaemia

Compensatory

mechanism

to

0000 90

metabolic acidosis

Muscle pain Muscle weakness/numbness Abnormal heart rate Cardiac arrest Death in severe cases

Causes O

Renal failure Hypoaldosteronism Rhabdomyolysis Medications such as =" NSAIDs

=

Spironolactone

= Angiotensin converting enzyme inhibitors

b.

Lesch-Nyhan syndrome

ANSWER

For answer refer 2017 paper-1, Q. 2(b), Pg. 369

BIOCHEMISTRY

6.

Hyperventilation:

OO 0

Q

Qa AQ

Q

O

= Disadvantages * Need an adjuvant * Local reactions at site are common Latest types =" Recombinant vector * Made by combining an micro-organism and DNA of other * Example: RVSV- ZEBOV vaccine for ebola virus =" Dendritic cell vaccines * For treatment of brain tumors and malignant melanoma = DNA vaccine * Advantages 6 Stimulates either or both B and T cells Absence of any infective potential Improved vaccine stability Highly specific Easier to insert into the host body Low cost of production * Disadvantages 5 Limited to protein immunogens 6 May induce immunologic tolerance by antigens expressed inside host body © Poorimmunogenicity 6 Atypical processing of bacterial and parasite proteins oO May cause cancer = T cell receptor vaccine * Modulate cytokine production * Modulates cell mediated immunity * Examples 5 Valley fever 5 Stomatitis 5 Atopic dermatitis

Your Roll No. ...

Name ofthe Paper

:

Biochemistry Paper-I

Name ofthe Course

:

MBBS-2015

Semester

Annual M.M.: 50

Time: 3 Hours

INSTRUCTIONS Write your Roll No. on the top immediately on receipt of this question paper

Answers to Parts I, II and III should be written in separate answer sheets provided Attempt parts of a question in sequence

PART-I

[10]

Write briefly on

a b. c. d

Biochemical role of vitamin D in the body Glycemic index Reciprocal regulation of glycolysis and gluconeogenesis Active methionine

[10]

fF SP

Explain why:

ao

ee

All questions are to be attempted

A patient with pyruvate kinase deficiency presents with haemolytic anemia Free fatty acid levels rise in uncontrolled diabetes mellitus Antibiotic therapy leads to vitamin deficiency Hemoglobin is a better transporter of oxygen than myoglobin

PART-II a.

Explain how myocardial infarction in diagnosed in clinical lab

b.

Discuss the metabolic derangements in alcoholism

[10] [5]

Explain the mechanism of iron absorption

PART-III Discuss the breakdown of fatty acids in the body. Add a note on its regulation

[10]

Or Enumerate the common inborn errors of metabolism of amino acids. Discuss in detail any two of them Write short notes on:

a. b.

Principles and applications of ELISA Role of cytochrome P450 in the body

[S|

PAPER-|

yl

2015 PAPER:

= Example - glycemic index of ice cream is less due to the presence of high fat content resulting in reduced glucose absorption

Write briefly on

a.

Biochemical role of vitamin D in the body (Ref: Biochemistry by Satyanarayana 5/e, pg. 127)

Significance of Glycemic Index O

O

For answer refer 2017 paper-1, Q. 3(b), Pg. 360 Glycemic index

ANSWER

and

d.

Active methionine

(Ref: Biochemistry by Satyanarayana 5/e, pg. 361)

Introduction

There are variations in the increase and fall of blood glucose levels after the ingestion of different carbohydrate containing foods. These quantitative differences are assayed by glycemic index which measures the time course of post-prandial glucose concentrations from a graph. Glycemic index may be defined as the area under the blood glucose curve after the ingestion of a food compared with the area under the blood glucose curve after taking the same amount of carbohydrate as glucose. It is expressed as percentage. Area under the blood glucose curve after ingestion of test meal Area under the curve after ingestion of glucose

x 100

Factors Affecting Glycemic Index Complexity of carbohydrate =" Glyemic index of complex carbohydrate (e.g. starch) is lower than that of simple carbohydrate (e.g. glucose).

O O

Sulfur containing essential amino acids Needs to be activated for transmethylation

O

Active methionine - S-adenosyl methionine

(SAM)

O

Active methionine - donates during transmethylation

group

methyl

O

Occurs by transfer of adenosyl group from ATP to sulfur atom of methionine

O

Enzyme involved transferase

O

During activation, sulfur gets converted to sulfonium

-

methionine

S-adenosyl-

atom

O O

Active methionine - SAM -isasulfonium compound 3ATPs are consumed

Biochemical Functions of Active Methionine

O O

Presence of positive charge - makes it highly reactive Enzymes involved - methyl transferases

O

Active methionine transfers methyl group to an acceptor and gets converted to S-adenosyl homocysteine Irreversible reaction

OO

Association with proteins, fat and fiber

the

Synthesis of Active Methionine

= ‘This is based on slow digestion and absorption of complex carbohydrates O.

glycolysis

ANSWER

Expressed as percentage

Glycemic Index

O

of

For answer refer 2016 paper-1, Q. 5(or) part-(a), Pg. 360

Defined as area under the blood glucose curve after the ingestion of a food compared with the are under the same curve when the same of carbohydrate is taken as glucose.

Glycemic index =

Reciprocal regulation gluconeogenesis

ANSWER

Glycemic Index

O

Foods with low glycemic index and high fiber healthy foods c.

(Ref: Biochemistry by Satyanarayana 5/e, pg. 506)

O

Low glycemic index foods =" Higher satiety value = Lower calorie intake

ANSWER

b.

r

=" Reduced glycemic index

PART-I 1.

2015

Methionine is regenerated back in the process.

> oO

D

=

Li

LL

O

OCO

PAPER-I O

O O O

2.

Active methionine is useful in the synthesis of = Polyamines - spermidine, spermine =" Creatine = Epinephrine = N-Methylnicotinamide = Choline =" Methylated tRNA bases = Protein methylated amino bases Active methionine is required for the activation of certain inactive compounds Controls protein turnover - methylation prevents the protein degradation Precursor of plant hormone, ethylene in plants involved in ripening of fruits.

= Effect on Lipogenesis * Decreased synthesis of triacylglycerol from glucose * Decreased glycolysis and NADPH - from HMP shunt = Effect on lipolysis * Increases lipase activity * Increases release of fatty acids from stored fat in adipose tissue * Increased fatty acid mobilization from liver * Insulin regulates free fatty acids concentration = Effect on ketogenesis * Increased HMG CoA synthetase * Increased ketogenesis c.

Explain why:

a.

A patient with pyruvate kinase presents with hemolytic anemia

deficiency

Antibiotic therapy leads to vitamin deficiency

ANSWER

Antibiotics and Vitamin Deficiency O

ANSWER

Pyruvate Kinase Deficiency Leads to Hemolytic Anemia O

Halt in glycolysis leads to deficiency of ATP

O

With insufficient ATP in an RBC, all active processes

O

in the cell come to a halt. =

Na* K* ATPase pumps are the first to stop.

*

Since the cell membrane is more permeable to K* than Na*, potassium leaks out.

*

Intracellular fluid becomes hypotonic, osmosis occurs. * The cell shrinks and cellular death occurs, this is called ‘dehydration at cellular level? =" This leads to decrease in RBCs leading to hemolytic anemia =" Thus deficiency in pyruvate kinase results in hemolytic anemia - there is deficiency of RBCs as they are destroyed due to lack of ATP at a greater speed than they are being created. b.

Free fatty acid levels rise in uncontrolled diabetes mellitus (Ref: Biochemistry by Satyanarayana 5/e, pg. 644, 654)

BIOCHEMISTRY

ANSWER Uncontrolled Diabetes Mellitus Leads to Increase in

Effect on Vitamin C = There is a competition for binding sites on proteins between vitamin C and salicylate Effecton Vitamin K = Vitamin K deficiency is rare * Produced by intestinal bacteria * Also obtained from diet. * If the bacterial population of the intestine is decreased by antibiotics, endogenously formed vitamin decreases and can lead to hypoprothrombinemia in malnourished individual. *

O

Second generation cephalosporins antibiotics as cefoperazone, cefamandole and moxalactam cause hypoprothrombinemia, by warfarin-like mechanism. = Competition between dicoumarol and vitamin K for its binding sites = Salicylates also antagonizes Vitamin K Effect on Vitamin B Complex = Salicylates competitively antagonizes Vitamin B 5 = Several broad-spectrum antibiotics inhibit the absorption of Vitamin B 12 d.

Hemoglobin is a better transporter of oxygen than myoglobin

Free Fatty Acids

ANSWER

O O

Introduction

Adipose tissue - sensitive to insulin Absence/Deficiency of Insulin in uncontrolled diabetes mellitus = Reduces release of fatty acids from stored fat

The delivery of oxygen to tissue in the human follows the following path:

body

PAPER-| Oxygen is inhaled into the lungs Oxygen diffuses into the alveoli of the lungs Oxygen binds to hemoglobin in the blood in the alveoli The oxygenated hemoglobin travels through the arteries until it gets to the capillaries Hemoglobin releases oxygen in the capillaries according to the Hemoglobin Dissociation Curve Oxygen diffuses through the interstitial fluids and cell walls Oxygen is consumed by the mitochondria.

O O O O

PART-II 3.

Enzymes in myocardial infarction O

=" Oxygen reservoir/storage = Transports oxygen to rapidly respiring muscle cells Oxygen Dissociation Curve

When oxygen diffuses through interstitial fluids, the amount of oxygen dissolved in the fluid is called the Partial Pressure of Oxygen and is represented as PO.,. Oxygen diffuses from areas with high levels of PO, to areas with low levels of PO.,. Represents the oxygen binding capacity of hemoglobin and myoglobin Shows that myoglobin tightly binds oxygen as compared to Hb.

O O O

100-

Myoglobin

-

oO

Aspartate Transaminase (AST or SGOT)

O O O

Rises 2nd day after MI Peaks at 3rd or 4th day Returns to normal by 10-15 days Last enzyme to rise and last enzyme to return to normal

= Hemoglobin

£ 50_

-



0



pO, in tissues

O

|

|

|

|

|

|

50

pO, (mm Hg)

|

|

|

Rises after CK Reaches a peak after 2nd day of infarction Returns to normal by 4-5 days

Lactate Dehydrogenase (LDH)

O. O

2

First enzyme to be released into circulation Within 6-12 hours after infarction Reaches a peak within 24 - 30 hours Returns to normal by 2" or 3 day Earliest marker of MI

Cardiac Troponin (CT)

pO, in lungs

-

o = &

Three enzymes are important in the diagnosis of MI =" Creatine phosphokinase = Lactate dehydrogenase = Aspartate transaminase Non-enzymatic biomarkers in MI =" Cardiac troponins =" Myoglobin

Creatine Phosphokinase (CK)

Functions

O

is

Cardiac Markers in Myocardial Infarction (Ml)

0000 0

0 OO O

Heart

infarction

ANSWER

Oxygen binding hemoprotein Monomeric Resembles individual hemoglobin moieties structurally Found in =" Skeletal muscle ="

Explain how myocardial diagnosed in clinical lab

(Ref: Biochemistry by Satyanarayana 5/e, pg. 115)

Myoglobin O

a.

|

100

Early diagnosis of MI Troponin I - inhibitory element of actomyosin ATPase =" Released into circulation within 4 hours of MI =" Peaks at 12 - 24 hours = Returns to normal after a week =" Cardiac specificity Troponin T - Tropomyosin binding element = Also important in diagnosing MI = Also elevated in other degenerative diseases

Protein Myoglobin O O

Early marker of MI Not cardiac specific

BIOCHEMISTRY

OO

O

PAPER-I b.

Discuss the alcoholism

metabolic

derangements

in

Fatty Acid Oxidation O

ANSWER For answer refer 2016 paper-1, Q. 2(c), Pg. 378

Involves 3 stages = Activation of fatty acids - in cytosol = Transport of fatty acids to mitochondria = § Oxidation proper - in mitochondria

Fatty Acid Activation 4.

O

Explain the mechanism of iron absorption (Ref: Biochemistry by Satyanarayana 5/e, pg. 417)

O ANSWER

O O

For answer refer 2017 paper-1, Q. 3(a), Pg. 360 PART-III 5.

Transport of Acyl CoA into Mitochondria

Discuss the breakdown of fatty acids in the body. Add a note on its regulation (Ref: Biochemistry by Satyanarayana 5/e, pg. 288, 368)

O

Theinner mitochondrial membrane is impermeable to fatty acids.

O

They are transported through carnitine carrier system or carnitine shuttle. This process is completed in following steps: = Acyl group of acyl CoA is transferred to carnitine (B-hydroxy T-trimethyl aminobutyrate), which is catalysed by carnitine acyltransferase I (present on the outer surface ofinner mitochondrial membrane). = The acylcarnitine is transported across the membrane to mitochondrial matrix by carrier protein.

O. ANSWER Introduction

O O O

Activated by thiokinase or acyl CoA synthase to acyl CoA Occurs in 2 steps = Fatty acids + ATP to form acyladenylate = Acyladenylate + CoA to form acyl CoA Requires 2 high energy phosphates Irreversible reaction

Fatty acid breakdown takes place in body mainly by 8 oxidation $6 Oxidation is the oxidation of fatty acids on the B carbon Resulting in sequential removal of 2 carbon atoms =" Carnitine acyltransferase II (found on the inner acylcarnitine to acyl CoA. = The carnitine released returns to cytosol for reuse. Cytosol

surface

of inner

mitochondrial

Inner

Mitochondrial

mitochondrial

matrix

membrane)

converts

membrane

R-C-SCoA Acyl CoA

Carnitine acyltransferase |

Carrier protein

Carnitine acyltransferase II

O

BIOCHEMISTRY

CoASH

R-C-Carnitine Acyl-carnitine

R-C-SCoA Acyl CoA

Carnitine

Carnitine

ANSWER Introduction Oo

°

Structurally

Lipop roteins-lip ids + proteins

=" Risk

" Conjugate proteins = Function as transport vehicle for lipids in blood . , ; 5 types: Based on separation on electrophoresis

lipoprotein(a) above 30 mg/dL . Triggers heart attack. Metabolism of Lipoprotein a

2

of

CHD

increases

Blood Elastase |

apo(a)

Liverst nainein other tissue Degradation

Kidney

Disassemble f i

fe

p(a)



Excretion Fibrinogen

Lp(a) Apo (a)

Fibronectin

Fea

Deposition || ucigenin |

>



D

Collagen IV

Decarin

Fibulin 5

=

Lu

5

Lp(a) Apo (a)

O

Receutis

CO

co“:

Cellular uptake

Fig. Metabolism of Lipoprotein A

Hepatocytes Fibroblasts Macrophage

with

increase

in

PAPER-I Dietary Advice

Character

O

Diet low in lipids

O

Diet rich in antioxidants:

vitamins

E and

C or-

beta-carotene. Antioxidants, decrease the oxidation

of LDL, thereby reduces the risk of atherosclerosis and CHD

Biochemical e Defect

(Ref: Biochemistry by Satyanarayana 5/e, pg. 654) ANSWER Introduction Diabetes Mellitus O

Metabolic disorder of fuel metabolism

O

Characterized by hyperglycemia

O

Types of Diabetes Mellitus

e e e e

Plasmainsulin Auto antibodies Ketosis Acute complications

Also known

O

as type I diabetes

* *

or juvenile O

="

e Months to years e Found in 10-20% cases e Suitable for treatment e

Usually not

necessary

O

|Non-insulin dependent

diabetes mellitus

diabetes mellitus

(IDDM)

(NIDDM) * 80—90% of diabetic population e¢ Predominantly in adults (> 30 yrs) e Obese e¢ Very strong

Glucose excretion in urine

Glucose Tolerance Test

levels

(GTT)

Procedure Preferably done in early hours Fasting sample is taken 75 g glucose in 300 mL of water is given orally Samples taken every 30 minutes for 2 hours =" Interpretation * Fasting 110: 126 mg/dL * 2hours sample: 140-200 mg/dL

or adult-onset

Insulin-dependent

Normal: 126 mg/dL or 7 mmol/l

= Renal threshold of glucose: 180 mg/dL = Renal excretion occurs only at plasma above 180 mg/dL

Age of onset: Above 35 years Less severe than IDDM. Etiology: Due to genetic and environmental factors and obesity

e Body weight e Genetic predisposition

resistance of target cells to insulin © Decreasedorabsent ® Normalor increased e Frequently found e Rare e Very common e Rare e Ketoacidosis e Hyperosmolar coma

Glucosuria

Most common, accounting for 80-90%

e Age at onset

by B-cells and/or

¢-?-te&+ @

@ + © -

Also called type II diabetes diabetes

e 10-20% of diabetic population e Usually childhood (

Normal

Impaired glucose tolerance

Diabetes

oO

Xilla

Prothrombin Time-Differentiating Between Hepatic and Posthepatic Disease

O O O

Factors for blood clotting are synthesized in liver. Impairment of hepatic function also results in decrease in the concentration of plasma clotting factors. Laboratory assessment of clotting factors is done by measuring prothrombin time

D

=Li LL

O

OCO

PAPER-II O O

Prothrombin time is prolonged in patients with liver damage. Quick change in prothrombin time occurs as the half-lives of clotting factors are relatively short (5-72 hours). Hence, with prothrombin time test acute as

ANSWER

well as chronic liver damages can be assessed, besides

O

Microsatellites (Simple Tandem Repeats)

O

its help in the prognosis. c.

Explain the role of dendritic cells in the immune response. Howare follicular dendritic cells different from other dendritic cells?

ANSWER Dendritic Cells

O

Antigen presenting cells

O

Possess

O

increased surface area. Antigen uptake: Phagocytosis by tissue dendritic cell "

Dendrites-

O

O

MHC expression: Constitutive

Location:

O

O

Co-stimulator delivery: Constitutive, by mature non-phagocytic lymphoid dendritic cells Antigen presented: Peptides, viral antigens Lymphoid

tissue,

connective

Neurological disease in humans is associated with more than 20 unstable microsatellite repeats. Unstable repeats can be located on coding or noncoding region of the gene.

PART-II

tissue,

3.

epithelia Types of Dendritic Cells

a. Trace the sequence of steps in the initiation phase of translation of mRNAs in eukaryotes. Explains how regulation at this point of synthesis of key proteins of iron metabolism, helps to maintain iron homeostasis

O

Myeloid dendritic cells = 2types: mDC-1 and mDC-2 =" Secrete cytokine IL-12 and have TLR 2 and 4.

O

Plasmacytoid dendritic cells = Also known as interferon producing cells = Secrete IFN-alpha, TLR 7 and 9

ANSWER

Follicular dendritic cells

Process of formation of protein from mRNA

O

Follicular dendritic cells

= They are found in primary and secondary lymph follicles of B-cells of lymph tissue. Follicular dendritic cells are derived from mesenchyme. Not of hematopoietic origin as contrast to myeloid and plasmacytoid =" Do not express MHC-I

Introduction

Steps of Translation

O

Activation of amino acid: Amino acids in cytoplasm in the presence of enzyme amino acyl synthetase are activated by ATP

O

Transfer of amino

O

transferred to specific tRNA Initiation of polypeptide chain:

=

d.

BIOCHEMISTRY

Microsatellite instability may result a large number of complications including cancer of colon, endometrium, ovary, skin etc.

Increased in viral infection

O

O

Diseases Associated with Microsatellite Instability outgrowths:

numerous

Short repeat units (10-30 copies) usually composed of dinucleotide or tetranucleotide units. More popular than minisatellites (VNTRs) as DNA markers for two reasons. = Microsatellites are evenly distributed throughout the genome. = PCR can be effectively and conveniently used to identify the length of polymorphism. * By the use of PCR, the region surrounding the microsatellites is amplified, separated by agarose gel electrophoresis and identified.

What are ‘microsatellite repeat sequences’ in the genomic DNA? Why are they extensively used as genetic markers? Mention one example of microsatellite instability leading to a clinical disease (Ref: Satyanarayana 5/e, pg 585)

O O

acid to tRNA: Activated aa are

=

By start codons AUG

=

Requires

ribosomal

sub

unit,

mRNA,

aminoacyl tRNA and initiation factors Elongation of polypeptide chain: = By addition of amino acids at P site Termination of polypeptide chain:

GTP,

PAPER-II O

Eukaryotic Translation Initiation O O

Requires nearly 12 initiation factors Divided into 3 main steps = mRNA preparation: ¢ Eukaryotic initiation 4F (eIF4F) binds to 5’cap of mRNA * Poly A binding protein (PABP) binds to 3’ poly A tail of mRNA *

PABP

binds

to eIF4E,

5’ end

and

3’ end to

eIF4F and circularize mRNA * elF4Fis a protein complex consisting of 6 elIF4A: an ATP dependent RNA helicaseunwinds secondary _ structure in 5’noncoding region 6 elIF4E: cap binding protein-binds to 5’ cap of mRNA 6 elIF4G: scaffold protein-binding regions for other factors. =" 43S complex binding to mRNA *

40S

subunit

binds

met-tRNA,

and

lron Metabolism Regulation

O00

Byregulating dietary absorption by enterocytes Recycling by macrophages, and Storage in hepatocytes. Regulation is by 2 mechanisms = Systemic regulation * Mediated by hepcidin: A _liver-derived hormone =" Cellular regulation * Through the expression of the iron metabolism-related genes 6

IRP/IRE

O

O

b.

Discuss the mechanisms by which the vast diversity of immunoglobulins is generated in our body (Ref: Biochemistry by Satyanarayana 5/e, pg. 189)

ANSWER For answer refer 2017 paper-2, Q. 5, Pg. 372

4.

What are tumour suppressor genes? Mention two examples and explain the molecular mechanism of action of each (Ref: Biochemistry by Satyanarayana 5/e, pg. 631)

ANSWER Tumor Suppressor Gene

Post-transcriptional mechanisms, such as

O

Anti-oncogenic gene.

» » »

O

Prevents cells from converting into oncogenic cell and thus prevents chances of cancer. Has repressive effect on the regulation of the cell cycle or promote apoptosis.

Alternative splicing, MicroRNAs, TheIRP/IRE system andthe proteolytic cleavage.

Translation Initiation

O

O

other

initiation factors to form 43S complex. * When 43S complex binds to mRNA-start codon is scanned * Initiation factors are released = 80S ribosome formation * 43S and 60S subunits join to form 80S * This indicates finish of translation initiation and start of elongation.

O

O

IRP1 and IRP2 proteins =" Main regulators of cellular iron in humans. =" Bind to the IREs present at the 5’ and 3’-untranslated regions (UTRs) of mature mRNAs that code for proteins responsible for * Jron uptake (Dmtl and TfR1), * Storage (Ft-H and Ft-L) and * Export (Fpn1). IREs: 28-30 nucleotide sequences with a highly conserved secondary structure. During iron deficiency = IRP1 binds to IRE through a cavity on its structure During iron overload = The cavity is occupied by the 4Fe-4S cluster and unavailable to bind to the IRE. = IRP2 ubiquitination and consequent proteosomal degradation is increased, therefore preventing its binding to the IRE. This process is mediated by E3 ubiquitin ligase complex containing the F box and leucine-rich repeat protein 5 (FBXL5). FBXL5 stabilizes after the binding of iron and oxygen to its hemerythrin domain, promoting IRP2 ubiquitination.

system

O

Functions of Tumor Suppressor Gene (Jron regulatory proteins 1 and 2/

O

iron-responsive elements) - maintain the cellular iron content

O

Repression of genes essential for the continuing of the cell cycle. Coupling the cell cycle to DNA damage.

BIOCHEMISTRY

=" By terminating codons or stop codons: UAA, UAG, UGA

PAPER-II If the DNA

damage

cannot

be repaired,

the

should initiate apoptosis

deleterious, perhaps lethal, mutations.

Examples and their Mechanism of Action O

O

P53

= Molecular mass gives the gene its name: kilodalton fraction of protein - p53 =

O

O

DNA polymerase III act as proof reading enzyme. = Serves to check for correct matching of nucleotide bases. = If any base is wrongly matched, then it is checked and replaced with the help of DNA polymerase III.

53

Present on chromosome 17

=" =" = =" = =

O

Replication is carried out with high fidelity. Misreading of the template sequence could result in

cell OO

O

Codes for protein that regulates the cell cycle Regulates apoptosis Ithas a role in conserving the stability of gnome Prevents mutation Thus functions as tumor suppressor gene. Stress factors such as DNA damage stimulates the increase of p53 proteins = 3 major roles of p53 in such cases * Growth arrest: stops the progression of cell cycle preventing formation of more damaged DNA * DNA repair: p53 activates transcription factors involved in DNA repair * Apoptosis: Cell death Ras Proto-Oncogenes =" The Ras family of proto-oncogenes: Target for UV-B radiation damage = Proto-oncogenes when mutated (oncogenes) become oncogene, which produces a mutant protein = Role in the early event of skin carcinogenesis. PTCH Tumor Suppressor Gene = Implicated in BCC. = Mutations are UV-B specific C to T and CC to TT base substitutions. Other Genes

An incoming nucleoside triphosphate is correctly matched to its complementary base on the DNA template and is added as

DNA

| the monophosphate to the growing DNA chain.

polymerase

@A

COG-—S/ Pm).

3/ ’

DNA template

2

5'53'DNA

Pr-_

ts

polymerase activity |

pa

Newly synthesized strand

3

5

lf DNA polymerase mispairs a nucleotide with the template, it uses its 3'—5’ exonuclease activity to excise the mismatched nucleotide

=

Tumor suppressor gene p16

Enzyme

=

Cyclin dependent kinase gene CDK4 on chromosome 12q14, as well as, anumber of genes associated with the p53-related pathways.

retreats

3'5' Exo-

PART-III

BIOCHEMISTRY

5.

How is fidelity of DNA replication maintained during cell division? Discuss the different cellular DNA repair mechanisms for repair of damaged DNA. Add a note on diseases associated with defective DNA repair (Ref: Biochemistry by Satyanarayana 5/e, pg. 536)

ANSWER Introduction

O

DNAreplication is a process of formation of identical daughter molecules of DNA.

DNA Repair Mechanisms

O O.

Even after maintaining high accuracy in DNA synthesis, some errors may occur in the process. These errors in DNA replication can be corrected by following methods: =" Base excision-repair. = Nucleotide excision-repair. = Mismatch repair. =" Double-strand break repair.

PAPER-II Base Excision-Repair Certain

bases

can

undergo

= Affected patients are photo-sensitive. =" These patients are more susceptible to cancer skin cancer.

spontaneous

depurination O O O

For

example,

cytosine

to

uracil,

adenine

to

hypoxanthine and guanine to xanthine. Occurs due to spontaneous alteration or by chemical or radiation means. Repair is by removal of those base by N-glycosylase.

="

Damage

to a segment

of DNA

OR

Explain the clonal selection hypothesis of adaptive immune response. Discuss the mechanism of activation, proliferation and differentiation of naive B cells into plasma cell/memory cells, in response to thymus dependent antigens

by spontaneous,

chemical or radiation means. O

To correct the damage removal of the DNA fragment (30-nt length) and replacement occurs. The nucleotide excision repair (NER) pathway: Consists of five steps = Recognition of DNA lesion, = Incision of the damaged strand on both sides of the lesion,

O

= Removal of the damaged oligonucleotide, = Synthesis of a patch, and = Ligation of the patch. Example-Xeroderma pigmentosum

(Ref: Biochemistry by Satyanarayana 5/e, pg. 698)

ANSWER Introduction

Adaptive immune response O O

Mis-Match Repair O O

O O

Damage due to copying error. (1-5 base unpaired loops.) Template strand is methylated, new strand is nonmethylated: This difference recognizes the mis match Correction of damage is done by removal of the loops by exonuclease digestion and replacement. Example: Hereditary monopolyposis colon cancer

O

Genetic translocation

="

Broken chromosomes

=

Cell death

Damage caused by ionizing radiations, free radicals, chemotherapy etc. DNA repair occurs by homologous or nonhomologous end joinings This damage is repaired by unwinding, alignment and ligation.

OO 0

Results in "

Cancer can be Associated with Defective Gene O O

Alteration of genes that regulate normal cell division results in development of cancer. Defects in gene encoding proteins are linked to cancer. Example: Xeroderma pigmentosum:

Capable of specifically recognizing and eliminating the antigen or foreign substance. Four characteristics = Antigen specificity =" Recognition diversity = Immunological memory =" Discrimination of self and non-self

Clonal Selection Hypothesis

Double Strand Break Repair: O

Jt occurs due to a defect in the nucleotide excision

repair of damaged DNA.

Nucleotide Excision-Repair O

Itisarare autosomal recessive disease.

It hypothesizes = The function of lymphocytes in response o specific antigens invading the body. = Antibody diversity It states that in a pre-existing group of lymphocytes (specifically b-cells), a specific antigen only activates its counter-specific cell so that particular cell is induced to multiply for antibody production. This activation occurs in secondary lymphoid organs These organs include spleen and lymph nodes. The clonal selection theory can be summarised as = Each lymphocyte bears a single type of receptor with a unique specificity by V(D)J recombination. recognition is required for cell =" Receptor activation. = Activated lymphocytes have identical specific receptors. =" Those lymphocytes bearing receptors for self molecules will be destroyed at an early stage.

Naive B-cells O

O

B-cell not been exposed to an antigen is naive B cell Activated only after primary antigen encounter.

BIOCHEMISTRY

O

="

PAPER-II Produces antigen-specific immunoglobin IgM and later IgG followed by GC formation

ANSWER

Mechanism of activation: " Upon antigen exposure, Naive B cells are converted into memory B-cell or plasma cell. = This secrets antibodies specific to the antigen. = Plasma cells have short life where as memory cells have longer half-lives. Proliferation of Naive B-Cells: = Rate of proliferation of naive B-cells is slightly less than that of B-cells = Time taken for traversing subsequent reactions is similar in both type of cells.

For answer refer 2017 paper-2, Q. 2(d), Pg. 370

Differentiation Of Naive B-Cells: = On exposure to a T-cell dependent Ag, naive cells can differentiate into effector cells = This differentiation can occur in two ways. * Cells rapidly differentiate into short-lived Ig secreting cells producing low affinity IgM. * Germinal center is seeded. A high affinity Ag specific B-cells are selected and developed into memory cells.

Write short notes on:

BIOCHEMISTRY

a.

Tumour markers

b.

Signal hypothesis of protein targeting (Ref: Biochemistry by Satyanarayana 5/e, pg. 558)

ANSWER O

The smaller proteins can easily pass through nuclear pores, whereas for larger proteins, nuclear localization signals are needed to facilitate their entrv into nucleus. Protein transport in presence of these signals is postulated as signal hypothesis of protein targeting. Most of the proteins of mitochondria are synthesized in the cytosol, and their transport to mitochondria is a complex process. Majority of the proteins are synthesized as larger preproteins with N-terminal presequences for the entry of these proteins into mitochondria. The transport of unfolded proteins is often facilitated by chaperones. One protein namely mitochondrial matrix targeting signal involved in protein targeting has been identified - Signal Peptide This protein can recognize mitochondrial receptor and transport certain proteins from cytosol to mitochondria. This is an energy-dependent process

Your Roll No. ...............

Name ofthe Paper

:

Biochemistry Paper-I

Name ofthe Course

:

MBBS-2013

Semester

Annual

Time: 3 Hours

M.M.: 50

INSTRUCTIONS Write your Roll No. on the top immediately on receipt of this question paper 2.

All questions are to be attempted

3.

Answers to Parts I, II and III should be written in separate answer sheets provided

4.

Attempt parts of a question in sequence

PART-I 1.

Explain with suitable examples a. b. c. d.

2.

[10]

Uncouplers of oxidative phosphorylation Significance of rate limiting enzymes Down regulation of hormone receptors Significance of anaplerotic pathways

Explain why: a. b. c. d.

Myoglobin is not suitable as Vitamin B6 deficiency leads Muscle glycogenolysis does Triglycerides are suitable for

[10] a carrier of oxygen to anemia not contribute to glucose homeostasis in blood storing energy

PART-II 3.

Discuss the role of calcitriol and parathormone in calcium and phosphorus homeostasis:

4.

a.

[5]

Write salient features of glucose metabolism in erythrocytes and explain its biochemical and clinical significance.

[10] b.

Explain biochemical basis of use of: i. Statins in the treatment of hypercholesterolemia 11. Aspirin as anti-inflammatory drug

PART-III 5.

Trace the metabolic fate of tyrosine. Enumerate the biologically important compounds derived form it and explain one genetic disorder associated with it. [10] OR Describe the formation, composition and metabolism of chylomicrons:

6.

Write short notes on the following:

a. b.

Biochemical basis and features of galactosemia Significance of glutamate dehydrogenase

[5]

PAPER-I |

2013 PAPER-I

PART-I 1.

= = =" =

Explain with suitable examples a.

Uncouplers of oxidative phosphorylation

)

Newborn Adults exposed to cold Hibernating animals Thoracic region of adults

b.

Significance of rate limiting enzymes

ANSWER ANSWER Introduction

O

O 0.

Oxidative phosphorylation: ATP generation during the transport of electrons through the electron transport chain Site: Complex V of the inner mitochondrial membrane Inhibitors of oxidative phosphorylation = Uncouplers: Substances altering the permeability of inner nitochondrial membrane = Ionophores: Lipophilic substances that promote the transport ofions across biological membranes

O O

O

Present in low concentration

O

Amount of enzymes is directly proportional to the rate of reaction. Have short half-lives Determines the overall rate of the metabolic process

O

O

O

O O O O

Delinking of oxidative phosphorylation from electron transport chain is known as uncoupling Such compounds are known as uncouplers Increases the permeability of inner mitochondrial membrane to protons (H+). Resulting in dissipation of energy as heat No ATP synthesis occur Oxidation of substrates occurs through NADH or FADH2 without ATP formation. Examples of uncouplers: = 2,4-dinitrophenol (DNP)

=

Two Types O

Constitutive enzymes: Have constant concentration

O

Adaptive enzymes: body demands.

Their levels are regulated by

Some Rate Limiting Enzymes Metabolic Pathway

Rate Limiting Enzyme

e Glycolysis

e Phosphofructokinase

Glycogenesis

e Glycogen synthetase

*

Proton-carrier

Glycogenolysis

e Phosphorylase

*

Easily diffuses through the inner mitochon-

Gluconeogenesis

e Fructose 1,6 bisphosphate

Cholesterol synthesis

e HMG CoA reductase

Fatty acid synthesis

e Acetyl CoA carboxylase

Urea synthesis

e Carbonyl phosphate synthetase (Arginase)

Citric acid cycle

e lsocitrate dehydrogenase

Ketone bodies

e HMG CoA synthase

Porphyrin synthesis

e Aminolevulinate synthase

Bile acids

.

c.

Downregulation of hormone receptors

drial membrane. Dinitrocresol, pentachlorophenol,

trifluoro-car-

bonylcyanide phenylhydrazone (FCCP). = In high doses, aspirin also acts as an uncoupler. =" Physiological uncouplers * Thermogenin * Thyroxine * Long chain free fatty acids Significance of Uncoupling O

Maintenance of body heat Animals containing brown adipose tissue

O

Found in

O

Enzymes control the speed of the reaction catalyzed by that enzyme. Rate limiting enzyme: Slowest step in the reaction process that determines the overall rate of the metabolic process.

Significance

Uncouplers

O O

BIOCHEMISTRY

Introduction

7-alpha hydroxylase

PAPER-| ANSWER

O

Introduction O Receptors are proteins and are manufactured by cells. O Cells regulate their sensitivity to molecules/ cells/hormones by increasing (upregulating) or decreasing (downregulation) of its receptors. Down Regulation of Hormone Receptor a O Cell decreases its sensitivity to a hormone . O Decrease in the amount of available receptors.

O

Decreased production

O

Increased

O

©

9

Transamination is a process wherein an amino acid transfers its amino group toa Keto acid and itself gets convertedtoaketoacid. The formation ofa-ketoglutarate ; , and oxaloacetate occurs by this mechanism. a-Ketoglutarate can also be synthesized from

glutamate by glutamate dehydrogenase action.

degradation

or

inactivation

of

active

O

receptors Also results in ligand- induced desensitization or internalization of hormone receptors. ; Occurs when receptors are exposed to increased concentration of hormones for a long period of time

Glutamate + NADP)+

Significance of anaplerotic pathways

Center of metabolism,

= Major source

; ; Anaplerotic Reactions

. The important anaplerotic reactions are:

of ATP production

O

Socell needs to regulate concentrations of citric acid cycle intermediates in the mitochondria.

©

Anaplerotic reactions thus play an important role P play 'P

ANSWER

Non-essential ———— amino acids,

----a-Ketoglutarate +

NAD(P)H + H+ + NH; Significance of Anaplerotic Reactions oo, O The citric acid cycle: . ; level Crucial cycle ="

d.

+ H20

O

Anaplerotic flux balances cataplerotic flux in order to retain homeostasis of cellular metabolism

Aspartate

purines pyrimidines

Acetyl CoA

© Pyruvate

Pyruvate »

?

boxyl

eens

>

CHIE

Oxaloacetate /

Fatty acids, sterols

Citric acid cycle

Malic enzyme

Pyruvate ©

\

>

aN

o-Ketoglutarate Succinyl CoA

¥ Heme

Glutamate

¥

Non-essential amino acids purines

BIOCHEMISTRY

O

O

Synthesis of oxaloacetate from pyruvate under the influence of enzyme pyruvate carboxylase. = Pyruvate + CO, + ATP--> Oxaloacetate +ADP + Pi Conversion of Pyruvate to malate by NADP+ dependent malate dehydrogenase " Pyruvate + CO, + NADPH + H+ --— Malate + NADP + H,O

Explain why

O

2.

a.

O

PAPER-I Myoglobin is not suitable as a carrier of oxygen

ANSWER

O

(instead of fat) in a particular individual, the weight of average 70 kg individual would increase by approximately 50 kg.

For answer refer 2015 paper-1, Q. 2(d), Pg. 394

b.

Stored in pure anhydrous form 1 gram of fat stored in body yields 6 times more energy as compared to 1 gram of glycogen. So to store similar amount of energy in glycogen

Vitamin B6 deficiency leads to anemia (Ref: Biochemistry by Satyanarayana 5/e, pg. 149)

PART-II ANSWER 3.

Deficiency Symptoms

O

O

O

O

Neurological symptoms such as_ depression, confusion, irritability and nervousness (Due to deficiency of serotonin, GABA) Anemia (Due to reduction in heme production) Hyperhomocysteinemia Urinary stone formation (As Vitamin B6 prevents hyperoxaluria)

(Ref: Biochemistry by Satyanarayana 5/e, pg. 128, 409) ANSWER For answer refer 2017 paper-1, Q. 5(or), Pg. 364

4.

Features of Vitamin B6 Deficiency Anemia

OOO

O

ANSWER

c.

Introduction

Muscle glycogenolysis does not contribute to glucose homeostasis in blood

Muscle Glycogenolysis

O

Not a major contributor to glucose homeostasis in blood due to absence of glucose 6 phosphatase enzyme, hence free glucose 6 phosphate directly enters the glycolysis pathway without releasing free glucose. Small amount of glucose is produced in muscle by the action of alpha 1-6 glucosidase activity (debranching enzyme). d.

Triglycerides are Suitable for Storing Energy Because

BIOCHEMISTRY

(Ref: Biochemistry by Satyanarayana 5/e, pg. 199, 250)

Glycolysis (Embden-Meyerhof-Parnas Pathway EMP) O O O O

Fatty acids are stored in reduced form in TG yielding 9 cal/g in contrast to 4cal/g by carbohydrates.

Mostcommon pathway for glucose metabolism in cells. Breakdown of glucose into pyruvate or lactate to produce ATP In presence of oxygen: Pyruvate, that further breakdowns to carbon dioxide and water in Krebs cycle Inabsence of oxygen: Lactate

In Erythrocytes O

O O

Triglycerides are suitable for storing energy

ANSWER

O

a. Write salient features of glucose metabolism in erythrocytes and explain its biochemical and clinical significance.

Due to reduction in heme production Microcytic (small red blood cells) Hypochromic (low levels of hemoglobin) More common in pregnant and breastfeeding women.

ANSWER

O

Discuss the role of calcitriol and parathormone in calcium and phosphorus homeostasis

O

Mitochondria (centres of aerobic oxidation) is absent in mature erythrocytes. So glycolysis leads to the formation of lactate. In mature RBCs, alternative/supplementary pathway to glycolysis: Rapoport luebering cycle is active. Leads to the production of 2, 3-biphosphoglycerate (2,3 BPG)

PAPER-| Rapoport Luebering Cycle

O

Associated with atherosclerosis and coronary heart disease

{

O

Usually seen in diabetes mellitus, hypothyroidism, obstructive jaundice, nephrotic syndrome, etc

Glyceraldehyde 3-phosphate

O

Rate limiting enzyme in cholesterol synthesis is B-hydroxy B-methylglutaryl CoA (HMG-CoA) reductase.

Glucose

NAD Glyceraldehyde 3-phosphate dehydrogenase NADH +H 1-3-Bisphosphoglycerate 2-3-Bisphosphoglycerate mutase

CS

Biochemical Basis of Use of Statins

O

Cholesterol reducing drugs

O

Eg. lovastatin, atorvastatin, simvastatin, fluvastatin

O

Competitive

ATP Phosphoglycerate y kinase 2-3-Biphosphoglycerate phosphatase

of

enzyme

HMG

CoA

reductase

ADP

O

Almost 50-60% reduction in cholesterol levels are seen with these drugs

O

HMG

2-3-bisphosphoglycerate

4

inhibitors

CoA reductase activity is at peak at 6 hours

after dark, so these drugs are best taken at night for

maximum effectivity.

3-phosphoglycerate |

Acetyl CoA

Pyruvate

| Thiotase

Acetoacetyl CoA

Biochemical Significance

|HMc

O

Shunt pathway of glycolysis

O

Occurs without the production of ATP

O

End product: 2, 3 BPG-combines with hemoglobin

HMG CoA Rie

CoA synthase


«-Ketoglutarate

NAD(P)H + H” Fig. Regulation of GDH activity

BIOCHEMISTRY

O

Your Roll No. ...............

Name ofthe Paper

:

Biochemistry Paper-II

Name ofthe Course

:

MBBS-2013

Semester

Annual

Time: 3 Hours

M.M.: 50

INSTRUCTIONS Write your Roll No. on the top immediately on receipt of this question paper 2.

All questions are to be attempted

3.

Answers to Parts I, II and III should be written in separate answer sheets provided

4.

Attempt parts of a question in sequence

PART-I 1.

2.

Explain in brief:

[10]

a. b. c.

DNA-histone interaction plays a vital role in organization of DNA structure Certain tumor cells develop resistance to methotrexate Anemia and hypocalcemia occur in patients with chronic renal failure

d.

Telomeres is required for overcoming the end replication problem in eukaryotes

Write briefly about:

a. b. c. d.

[10]

Delayed hypersensitivity Role of MHC in organ transplant Anion gap RNA editing

PART-II 3.

4.

Outline the pathway of uric acid formation. Explain the biochemical basis of developing hyperuricaemia in the following conditions: [5] a. b.

Glucose 6-phosphatase deficiency Lesch-Nyhan syndrome

a.

Define the terms RFLP and VNTR. Explain their role in DNA fingerprinting

b.

Write the help of a diagram, explain various phases of cell-cycle. Discuss the molecular mechanism of transition from G1 to S-phase.

[10]

PART-II 5.

Describe the process of protein biosynthesis in eukaryotes. Enumerate discuss mechanism of action of any one in detail

various inhibitors of translation and [10]

OR a. b. 6.

What are multivalent subunit vaccines? Mention the advantages and disadvantages of these vaccines Mention the type of damage caused by UV radiations and discuss the repair process for the same

Write short notes on:

a. b.

Role of viruses in gene therapy Biochemical investigation in CSF in bacterial meningitis

[5]

PAPER-II

2013 2013 PAPER-II

=" Increase DHFR concentration in the cells: Hence in spite of the presence of drug, certain DHFR would be available to continue the pathway of de novo DNA synthesis. =" Decreased polyglutamation of MTX inside the cell. Polyglutamation enables longer retention of the drug by the cells, a decrease in that would result in rapid efflux of the drug and the enzyme, DHFR, would not be completely inhibited by the drug.

PART-I 1.

Explain in brief: a.

DNA-histone interaction plays a vital role in organization of DNA structure (Ref: Biochemistry by Satyanarayana 5/e, pg. 81)

ANSWER

=" Induction of another molecular form of dihydrofolate reductase exhibiting low affinity for methotrexate.

Histone Interactions with DNA

O

Each nucleosome

is composed

of DNA

wrapped

around 8 histone proteins, functions like a spool and

called a histone octamer. nm

nucleosomes

are

d. then

a 30

Telomeres is required for overcoming the end replication problem in eukaryotes

additional H1

(Ref: Biochemistry by Satyanarayana 5/e, pg. 530)

wrapped,

spiral, called a solenoid, where

into

histones are associated with each nucleosome maintain the chromosomal structure.

to ANSWER

Histone: DNA interactions

=" Hydrogen bonds: Between the backbone of the DNA and the amide group of histones. " Non-polar interactions: Between _ the deoxyribose sugars on DNA and histone proteins = Salt bridges and hydrogen bonds: between basic amino acids (especially lysine and arginine) and phosphate oxides on DNA. Highly active genes have less histone while nonactive genes have highly linked with histones during interphase. Histone proteins has a highly positively charge on N-terminus having lysine and arginine residues. b.

Certain tumor methotrexate

cells

develop

resistance

Telomeres O O O

O O

to

ANSWER

O

Resistance development: Due to _ continued administration to patients 5 mechanisms have been implicated =" Structural modifications in enzyme dihydrofolate reductase: Resulting in its loss of affinity for MTX. = Structural alteration in a transport protein for MTX which is located in the surface membrane of the cell- resulting in reduced uptake of MTX by cell.

O00 0

Mechanism of Resistance to Methotrexate O

Repeat sequences of six nucleotides (TTAGGG) present at the end of eukaryotic chromosome. Special structures that protect the chromosomes from continuous loss at the ends during cell cycle. Also prevents fusing of chromosomes with each other

Role of Telomeres

OO

The

)

As telomeres so not code for proteins, minor changes in its length is insignificant During the course of many replication cycles, there is progressive shortening of telomeres This shortening is prevented by telomeres Telomeres = Also known as telomere terminal transferase = Aids in maintenance of teleomeres = Contains both RNA and protein Serves as a template for extension of telomeres This leads to extension of DNA molecule Then translocation of telomeres occurs

DNA synthesis and translocation results in complete extension and formation of new DNA

Write briefly about: a.

Delayed hypersensitivity

BIOCHEMISTRY

|

441

PAPER-II ANSWER

* *

Temporal HLA mismatching effect. HLA-DR mismatch effectis the mostimportant in the first 6 months after transplantation,

*

HLA-B effect emerges in the first 2 years, and

*

HLA-A mismatches have a deleterious effect on long-term graft survival

For answer refer 2015 paper-2, Q. 5(or), Pg. 407 b.

Role of MHC in organ transplant

ANSWER c.

Anion gap

Transplantation Antigens O

O

O

Rejection response to grafted tissue is caused by =" MHC molecules: Organs transplanted between MHC identical individuals are readily accepted, whereas organs transplanted between MHC antigen-mismatched individuals are rejected in the absence of immunosuppressive therapy =" Minor histocompatibility antigens: processed peptides present by MHC =" ABO blood group antigens: Result in hyperacute rejection of vascularized tissue =" Monocytes/endothelial cell antigens. The phenomenon by which the recipient immune system reacts with donor antigens that are considered to be “non-self” is named allorecognition. The main and strongest responses to alloantigens are mediated by host T cells, which recognize peptide antigens presented in the context of MHC, by antigen-presenting cells (APCs).

ANSWER Anion Gap

O

Represented as A

O

Defined

Located on the short arm of chromosome

O

=

O

-G, -H loci

="

BIOCHEMISTRY

= = ="

HLA-C loci on chromosome 6;

=

Biologic function of the HLA molecules is presenting endogenous and exogenous antigens = They manifest high structural polymorphism. = In transplantation immunology * Major impact in graft loss is due to HLA-B and -DR antigens

total

136+4=100+25+A

A = 15 mEq/L Significance

O

Healthy individuals: 15 mEq/L (range-8-18 mEq/L)

O

Acid-base disorders: Associated with alteration in

anion gap

RNA editing (Ref: Lippincott’s 5/e, pg 457,458 )

ANSWER For answer refer 2015 paper-2, Q. 5, Pg. 405

PART-II

*

="

Involved in mother’s immunological tolerance to fetus MHC class II antigens: expressed on B lymphocytes, activated T cells, monocytes, macrophages * Encoded by genes of the HLA-D region Class land II genes encode codominantly expressed HLA cell surface antigens, recognized by CD8 and CD4 positive T cells, respectively Class III genes encode several components of the complement system Are present on all nucleated cells Encoded by genes of the HLA-A, HLA-B, or

the

Measured in mEq/L Nat + K*= Cl +HCO,+A

6 (HLA

Class I MHC antigens: encoded by the HLA-E, -F,

between

Calculation of Anion Gap

d.

Divided into three classes (I, I, and III) on the basis of their tissue distribution, structure, and function.

difference

Represents the unmeasured anions in plasma

complex) O

the

concentration of measured cations and anions

The Major Histocompatibility Complex O

as

3.

Outline the pathway of uric acid formation. Explain the biochemical basis of developing hyperuricaemia in the following conditions: a.

Glucose 6-phosphatase deficiency

b.

Lesch-Nyhan syndrome (Ref: Biochemistry by Satyanarayana 5/e, pg. 395)

ANSWER Introduction

O O O

Uric acid: Final excretory product of purine metabolism Serve as important antioxidant: By non enzymatically converting into allantoin Normal concentration: 3-7 mg/dL in serum

PAPER-II O.

Daily excretion of uric acid: 500-700 mg.

ANSWER

Pathway of Uric Acid Formation

Restriction Fragment Length Polymorphisms (RFLPs)

Glycogen

Glucose

J

O O

Glucose 6-phosphate

NADP

O

+

)

HNP Shunt

O

NADPH +H” Vv Ribose 5-phosphate | PRPP synthase

PRPP

O

A small stretch of DNA used as DNA marker for a specific gene Located randomly on chromosomes DNA exists in fragments of various lengths and acted upon by restriction endonucleases to produce polymorphic fragments. When certain gene is mutated, the number of fragments produced by the restriction endonuclease alters. Thus RFLPs are formed due to random mutations at the site of restriction endonuclease activity

Glutamate

|

R,

R,

R,

ae

DNA1

5- phophoriboxylamine Restriction endonuclease

HGPRT

>|

inosine monophosphate R,

GMP

AMP

Rs |

DNA2 Restriction endonuclease

spuotnne | Xanthine oxidase Xanthine

3 fragments

Application of RFLPs

| Xanthine oxidase

O

Uric acid

Glucose 6-phosphatase Deficiency

O

O00 0

O

In G6PD: Von Gierke’s glycogen storage diseaseGlucose 6 phosphate is not converted to glucose So increased G-6P in HMP shunt: Leading to increased formation of ribose 5 phosphate and PRPP So increased uric acid formation Von: Gierke’s disease-increased glycolysis Leading to lactic accumulation Resulting in impaired uric acid excretion through renal tubules.

Also, For answer refer 2017 paper-2, Q. 2 (b) Pg. 369

4.

a.

Define the terms RFLP and VNTR. their role in DNA fingerprinting

Variable Number Tandem Repeats (VNTRs)

O

Also known as minisatellites

O

Are DNA fragments of different base lengths (10-10 base pairs)

O

VNTRare formed due to difference in number of base sequences between two points of DNA molecule.

Applications of VNTR

O

Lesch-Nyhan Syndrome

Explain

(Ref: Biochemistry by Satyanarayana 5/e, pg. 383)

Has been useful in marking genes for = Sickle cell anemia: chromosome 11 =" Cystic fibrosis: Chromosome 7 = Huntington’s disease: Chromosome 4 = Alzheimer’s disease: Chromosome 21

Diagnosis of genetic disease related to degree of repeats of VNTRs e.g. Huntington’s chorea

Limitations of VNTR

O

Not evenly distributed throughout genome

O

More concentrated at telomeres.

Role in DNA Fingerprinting O

REFLPs and VNTR are unique to individual's genome - basis of DNA fingerprinting.

BIOCHEMISTRY

Hypoxanthine

PAPER-II O

Restriction fragment length polymorphisms caused by variations in the number of minisatellites between 2 restriction sites can be detected. DNA obtained from different individuals with VNTRs are acted upon by specific restriction endonucleases. Fragmented DNA are separated by electrophoresis, and identified after hybridization with a DNA probe complimentary to a specific sequence on the fragments. b.

Write the help of a diagram, explain various phases of cell-cycle. Discuss the molecular mechanism of transition from G1 to S-phase.

O

7) (/ye , G1

&

Ss

Period when the DNA is duplicated (that is, when

in daughter cell)

chromosomes

are duplicated)

G2 Period afer DNA is duplicated. Cell prepares

O

for division

Fig. Cell cycle

Various Phases of Cell Cycle The various phases of cell cycle are:

G,Phase "

O

Quiescent

and

senescent

state:

Post

mitotic

phase =" Cells stay in this phase for long periods of time. Interphase =" Preparatory phase: Prepares for division = Also known as intermitosis. = Interphase proceeds in a series of three stages, G,,

BIOCHEMISTRY

S, and G,.

*

G, 6 3 3 0

Phase First phase in the interphase. Gindicates gap. Also called growth phase. Marked production of proteins. ‘The G, phase is under the control of the p53 gene.

OO

O

G, phase

Gap between the DNA synthesis and mitosis. 0 The G, checkpoint mechanism controls to ensure that the cell is ready to enter the M (mitosis) phase and divides. Mitosis or M phase = The M phase consists of karyokinesis: Nuclear division. = The M phase is of several distinct phases, known as Prophase Metaphase Anaphase Telophase Cytokinesis

Molecular Mechanism of Transition from G1 to S-phase O

@

*

-?-ttef + ¢@

[Se

growth before the DNA is _ duplicated — (interphase begins

S§ phase 56 Commences with DNA replication. sister 56 Each chromosome has_ two chromatids. 5 Sensitive to external factors like drugs or mutagens. 5

(Ref: Biochemistry by Satyanarayana 5/e, pg. 530)

Period of cell

*

The GI/S transition is a stage in the cell cycle at the boundary between the G1 phase and the S phase, Cell cycle check point = DNA integrity is assessed = Cell cycle can pause at this point in response to improperly or partially replicated DNA.

= Itisa “point ofno return” beyond which the cell is committed to dividing = The G1/S transition occurs late in Gl. During this transition, Gl cyclin D-Cdk4/6 dimer phosphorylates retinoblastoma releasing transcription factor E2K, which then drives the transition from G1 to S phase. Regulation is by transcription factor p53 If a cell passes through the G1/S transition the cell will continue through the cell cycle regardless of incoming mitogenic factors due to the positive feedback loop of G1-S transcription. Positive feedback loops include G1 cyclins and accumulation of E2F.

PART-III Describe the process of protein biosynthesis in eukaryotes. Enumerate various inhibitors of translation and discuss mechanism of action of any one in detail

PAPER-II Elongation of Polypeptide Chain

Introduction

Translation-process of formation of protein from mRNA

O O O

Steps of Translation

O

Activation of amino acid: Amino acids in cytoplasm in the presence of enzyme amino acyl synthetase are activated by ATP

O

Transfer of amino acid to tRNA: Activated aa are transferred to specific tRNA

O

Initiation of polypeptide chain: =

By start codons AUG

="

Requires

ribosomal

sub

unit,

mRNA,

GTP,

aminoacyl tRNA and initiation factors O

*

Elongation of polypeptide chain: =

Binding of aminoacyl t-RNA to A -site

=

Peptide bond formation

=

= Translocation Termination of polypeptide chain: =" By terminating codons or stop codons: UAA, UAG, UGA

O

Eukaryotic Translation Initiation O O

Requires nearly 12 initiation factors Divided into 3 main steps = mRNA preparation: ¢ Eukaryotic initiation 4F (eIF4F) binds to 5’cap of mRNA * Poly A binding protein (PABP) binds to 3’ poly A tail of mRNA *

PABP

binds

to elIF4E, 5’ end

and

3’ end to

eIF4F and circularize mRNA * elF4Fis a protein complex consisting of 6 elIF4A: An ATP dependent RNA helicaseunwinds secondary _ structure in 5’noncoding region 6 elIF4E: Cap binding protein-binds to 5’ cap of mRNA 5 elIF4G: Scaffold protein-binding regions for other factors. =" 43S complex binding to mRNA

="

*

40S

subunit

binds

met-tRNA,

and

*

initiation factors to form 43S complex. When 43S complex binds to mRNA -

* *

O O. O

O

After the formation of specific polypeptide/protein, termination occurs Termination codons or stop codons: UAA, UAG and UCA) These act as stop signals: Do not have any specific tRNA to bind them. Release factor: eRF recognizes the stop codons eRF- GTP + peptidyltransferase cleaves the peptide bond between polypeptide and t-RNA at P- site. 80S ribosome splits to form 40S and 60S subunits

a.

What

are

multivalent

subunit

vaccines?

Mention the advantages and disadvantages of these vaccines

(Ref: Biochemistry by Satyanarayana 5/e, pg. 588) ANSWER For answer refer 2016 paper-2, Q. 5(or), Pg. 390

b.

start

43S and 60S subunits join to form 80S This indicates finish of translation initiation and start of elongation.

from

OR

other

codon is scanned * Initiation factors are released 80S ribosome formation

Movement of growing peptide chain A-site to P-site * Requires EF-2 and GTP Incorporation of amino acids * 6amino acids are added per second. * Has both speed and accuracy

Termination of Polypeptide Chain

O

O

Occurs by addition of amino acids Requires elongation factors Divided in 3 steps =" Binding of aminoacyl t-RNA to A-site * Aminoacyl tRNA binds to the A-site * Elongation factor 1a (EF-1a) is required * Energy in the form of GTP is used = Peptide bond formation * Occurs with the action of enzyme peptidyl transferase - ribozyme * At28S RNA of 60S ribosomal subunit. * Net result: Attachment of peptide chain to tRNA in A- site. = Translocation

Mention the type of damage caused by UV radiations and discuss the repair process for the same

ANSWER

Damage of UV Rays O

Specific mutations sequences include

induced

by

UV-B

in

DNA

BIOCHEMISTRY

ANSWER

PAPER-II =

O

Single-base substitutions thymine (T).

of

cytosine

(C)

for

ANSWER

" Double-base changes from CC to TT

CSF

UV-Aradiation

O

= T to guanine (G) transversions O

Repair

The type of DNA repaired by

O

and around the spinal cord.

" Double-base changes from TT to GG. UV-A radiation is known to be 10,000 times less mutagenic than UV-B.

DNA

damage

caused by UV radiation is

O O O

Origin: Choroid plexus Absorption: Arachnoid villi Daily production: 500 mL

O

Volume in brain: 120-150 mL

Replaced completely within 3 days Functions

pathway:

O

Hydraulic shock absorber: Prevents injury to brain

consists of five steps = Recognition of DNA lesion, = Incision of the damaged strand on both sides of the lesion = Removal of the damaged oligonucleotide

O O O O

Regulation of intracranial pressure Influences hunger sensation and eating behaviours Collection of CSF Spinal puncture in the lumbar region: L4-L5 region

"

Biochemical Investigation in CSF in Bacterial

nucleotide

The

excision

repair

(NER)

Synthesis of a patch, and

" Ligation of the patch.

6.

Clear, colourless liquid present in ventricles of brain

Write short notes on:

Meningitis

Colour and

Clear and colorless

Opalescent and turbid

Total cell count

O-S x 10°/L

Markedly increased

Protein

15-45 mg/dL

Markedly increased

Glucose

45-85 mg/dL

Markedly decreased

appearance

a.

Role of viruses in gene therapy

ANSWER

For answer refer 2016 paper-2, Q. 1(c), Pg. 384 b.

Biochemical investigation in CSF in bacterial meningitis

BIOCHEMISTRY

(Ref: Biochemistry by Satyanarayana 5/e, pg. 496)

Your Roll No. ...............

Name of the Paper

:

Biochemistry Paper-!

Name of the Course

:

MBBS-2012

Semester

:

Annual

Time: 3 Hours

M.M.: 50 INSTRUCTIONS

1.

Write your Roll No. on the top immediately on receipt of this question paper

2.

All questions are to be attempted

3.

Answers to Parts I, II and III should be written in separate answer sheets provided

4.

Attempt parts of a question in sequence

PART-I 1.

Answer briefly:

a. b. c. d. 2.

3.

4.

[10]

Therapeutic uses of prostaglandins Biochemical functions of Vitamin Bl Lipid peroxidation Discuss the importance of glycemic index

Answer in brief:

[10]

a. b. c. d.

Advantages and disadvantages of PUFA Function of cytochrome p450 in the body Role of Liposomes Wilsons disease

a.

Name the neurotransmitters derived from amino acids? Mention their physiological role.

b.

Explain the components of Electron transport chain and the flow of electrons in them

Explain the mechanism of action of peptide hormones:

[10]

[5]

PART-III 5.

Describe the synthesis and metabolism of HDL and LDL. Add a note on disorders of Lipoprotein metabolism

[10] OR Explain the biochemical changes in carbohydrate and lipid metabolism in Starvation 6.

Explain why: a. b.

NADPH plays a crucial role in phagocytosis Deficiency of Vitamin B12 results in anemia

[5]

PAPER-I |

2012 PAPER-I

PART-I 1.

Biochemical Functions

Coenzymethiamine pyrophosphateorcocarboxylase: Associated with carbohydrate metabolism. It is an important in the following reactions

Answer briefly: a.

|

Therapeutic uses of prostaglandins

O

(Ref: Biochemistry by Satyanarayana 5/e, pg. 618)

Conversion of pyruvate to acetyl CoA by enzyme pyruvate dehydrogenase =" Reaction: Oxidative decarboxylation

ANSWER

=

Introduction

O

Locally acting hormones

O

Derivatives

of

20-carbon

fatty

acid

namely

prostanoic acid, hence known as prostanoids

O

Precursor of prostaglandins is arachidonic acid

Therapeutic uses of Prostaglandins O

O

O

Enzyme: Alpha ketoglutarate dehydrogenase

O

=" Cycle: Citric acid cycle Enzyme transketolase: Dependent on TPP. =" Cycle: HMP pathway

O O

Decarboxylase: Catalyzes oxidative decarboxylation of branched chain amino acids Role in nerve impulse transmission. =

Required for acetylcholine synthesis

" PGE, PGA and PGI, - vasodilators

=

Jon translocation of neural tissues

= Lowers blood pressure by * Increased blood flow * Decreased peripheral resistance Inflammation PGE,

and

PGE,

cause

arteriolar

Cc.

Lipid peroxidation

ANSWER vasodilatation

and thus inflammation

O

O

Regulation of blood pressure

"

O

Cycle: Citric acid cycle

Introduction

Reproduction = PGE, and PGF, are used for medical termination of pregnancy and induction of labour Pain and fever = Pyrogens induce prostaglandin synthesis in hypothalamus = PGE, causes pain. = Aspirin blocks PG synthesis to reduce pain and fever Regulation of gastric secretion = PG inhibit gastric secretion - basis of use in treatment of gastric ulcers

O

b.

O

Biochemical functions of Vitamin B,

O O O

Peroxidation or auto-oxidation of lipids exposed to oxygen Responsible for tissue damage leading to cancer, inflammatory diseases, etc Source of free radicals Chain reaction, provides a continuous supply of free radicals that initiate further peroxidation

Stages of Lipid Peroxidation O

Initiation

ROOH+ Metal ‘“* >ROO*® + Mental")*H* X* +RH-R’ +XH Propagation

R* +0,ROO*

(Ref: Biochemistry by Satyanarayana 5/e, pg. 138)

ROO*+RH—-ROOH > oO

D

ANSWER

Vitamin B,

LL

O

O

OCO

ROO*+ R°>ROOR R°+R* RR

Also known as =" Thiamine

Marker of Lipid Peroxidation

=" Antiberiberi vitamin

O

=" Antineuritic vitamin

O

Termination ROO*+ ROO*>ROOR +0,

Introduction

=

LU

O

+R’, etc

Water soluble

O

Malondialdehyde (MDA): Biochemical marker of lipid peroxidation TBARS: Can be measured colorimetrically

PAPER-| Discuss the importance of glycemic index

ANSWER

(Ref: Biochemistry by Satyanarayana 5/e, pg. 506) ANSWER

Liposome

O.

Spherical vesicle having at least one lipid bilayer.

O

Can

O

nutrients and drugs. Prepared by disrupting biological membranes

For answer refer 2015 paper-1, Q. 1(b), Pg. 393

2.

Advantages and disadvantages of PUFA

O

2 families-omega represents the position of first double bond

Component acid,

OO

O

O

O

O

O

linolenic

of

essential acid,

fatty

arachidonic

acids:

Linoleic

acid-cannot

be

synthesized in body Component of membrane structure: Responsible for the flexibility of membrane Improves circulation and oxygen uptake by RBCs Required for cholesterol formation, transport and function Essential for the synthesis of eicosanoids: Inhibits inflammatory process and blood coagulation Reduces platelet aggregation Prevent fatty liver and coronary heart disease. Supports CVS, reproductive, immune and nervous systems.

O

Lipid peroxidation: PUFA-most susceptible to free radical damage Free radicals can damage DNA and RNA damage.

O

Causes inflammation when consumed in excess

b.

d.

(Ref: Biochemistry by Satyanarayana 5/e, pg. 420)

O O

Also known as hepatolenticular degeneration Abnormal copper metabolism

Manifestations of Wilson’s Disease

O

O O

O.

Deposition of copper in liver and lenticular nucleus of brain: Resulting in hepatic cirrhosis and brain necrosis Decreased plasma levels with increased excretion in urine Deposition of copper in kidney causes damage: Increased excretions of amino glucose in urine Increased absorption of copper

renal acids,

Causes of Wilson’s Disease

O O

Failure to synthesize ceruloplasmin or impairment in binding capacity of ceruloplasmin Mutation in gene encoding copper binding ATPase: Defect in it reduces intestinal excretion of copper through bile.

Function of cytochrome p450 in the body (Ref: Biochemistry by Satyanarayana 5/e, pg. 614)

3.

ANSWER

a.Name the neurotransmitters derived from amino acids? Mention their physiological role. (Ref: Biochemistry by Satyanarayana 5/e, pg. 357, 371)

For answer refer 2018 paper-2, Q. 2 (d), Pg. 349 c.

Wilsons disease

Introduction

Disadvantages O

of

ANSWER

Advantages of PUFA O

administration

Designed to deliver drugs in other ways - liposomes work to deliver drug by diffusion rather than by direct cell fusion. Biodetoxification of drugs by injecting empty liposomes with a transmembrane pH gradient. Liposomes activates endocytosis in other cell types.

PUFA

Omega6

for

O

O

O

a vehicle

Used as models for artificial cells.

ANSWER

Omega3

as

O

(Ref: Biochemistry by Satyanarayana 5/e, pg. 32)

O

used

Significance

Answer in brief:

a.

be

Role of Liposomes (Ref: Biochemistry by Satyanarayana 5/e, pg. 42)

ANSWER Introduction

O

Neurotransmitter: Endogenous enable neurotransmission

chemicals

that

BIOCHEMISTRY

d.

PAPER-I O

=" Mobile carriers: Co-enzyme Q and cytochrome C

Neurotransmitter derived from amino acids are

= Serotonin: From tryptophan = Dopamine, Norepinephrine: From tyrosine and phenylalanine = Glutamate, Aspartate: Themselves work as neurotransmitters

Components of ETC O

=

Physiological Role Vasoconstrictor:

Results

in

smooth

"

muscle

O00 0

contraction in bronchioles and arterioles

Regulation of cerebral activity Controls behavioural patterns Stimulates release of peptide hormones from GIT Necessary for GI motility

O

Dopamine

O O O O

Controls movement and posture. Modulates mood Plays a central role in positive reinforcement and dependency. Its deficiency causes muscle rigidity - Parkinson’s disease

O

O

Norepinephrine O

Important for attentiveness, dreaming, and learning.

O

Norepinephrine also acts as hormone into the blood- causes vasoconstriction and increased heart rate

emotions,

=

Excitatory neurotransmitter that is associated with learning and memory. O

Regulates activity of neurons Opens chloride channels Increases permeability of post-synaptic membranes a.

Explain the components of Electron transport chain and the flow of electrons in them

4.

(Ref: Biochemistry by Satyanarayana 5/e, pg. 225)

Explain the hormones

Introduction

Introduction

O O

O

O

to electron

Located in the inner mitochondrial membrane

Consists of

="

4 multiprotein complexes: |, II, III, IV

FMNH,

are transferred

to Fe-S,

O

iron

shuffles

= ATP is synthesized Cytochrome = Contains one heme prosthetic group = Cytochrome C transfers electrons from complex III to complex IV Complex IV or Cytochrome oxidase = Contains cyt a and cyt a3 =" 4 electrons are accepted from cyt c and passed to molecular oxygen = 1 ATP is synthesized

ANSWER

Also known as respiratory chain Electrons are transported from NADH carriers

from

During transfer of electrons, between Fe** and Fe* forms

ANSWER

O

BIOCHEMISTRY

Electrons

and cytochrome cl, both contain heme prosthetic group

O

Inhibitory Neurotransmitter-GABA-Gamma Aminobutyric Acid O O O.

reductase or NADH

and further to Coenzyme Q =" Energy released: 12kcal/mol-utilized to drive protons out of mitochondria and 1 ATP is generated Complex II or Succinate-Q-Reductase = Electrons from FADH, enter ETC at CoQ = No ATP is produced. " FADH, produces 2 ATPs while NADH, produces 3 ATPs Co-enzyme Q = Accepts a pair of electrons from NADH or FADH, through complex-I or II, respectively Complex III or Cytochrome Reductase = Cluster of iron-sulphur proteins, cytochrome b

sleeping,

Glutamate, Aspartate

O

Also known as NADH-CoQ dehydrogenase complex

= NADH: Donor of electrons = FMN: Acceptor of electrons and forms FMNH,

Serotonin O

ETC Complex-1

mechanism

of action

of peptide

Protein and peptide hormones - have cell surface receptors through which the hormone acts. Binding of hormone to receptor initiates a series of events that leads to generation of so-called second messengers within the cell (the hormone is the first messenger).

PAPER-| O

The second messengers then trigger a series of molecular interactions that alter the physiologic state of the cell.

O O

Structure of Cell Surface Receptors

Cell surface receptors are integral membrane proteins and have 3 domains: O

Transmembrane domains: Hydrophobic stretches of amino acids Cytoplasmic or intracellular domains: Tails or loops of the receptor within the cytoplasm - leading to generation of second messengers. Cytoplasmic residues of the receptor are thus the effector regions of the molecule.

Extracellular domains: Exposed to the membrane and bind the hormone - ligand-binding domain.

§-adrenergic receptor

Extracellular

space

RARRARARARARNN eteteYeYehofereyetoteYerefe Cytoplasm

Epidermal growth factor receptor

Insulin receptor

Fig. Cell surface receptors

themselves enzymes that are either activated or suppressed by being phosphorylated. Such changes in enzymatic activity within the cell clearly alter its state. Elevations in cAMP also have important effects on transcription of certain genes.

O O

Four second messenger systems are recognized in cells Cyclic AMP: Epinephrine and norepinephrine, glucagon, luteinizing hormone, follicle stimulating hormone, thyroid-stimulating hormone, calcitonin,

O

parathyroid hormone, antidiuretic hormone

Tyrosine Kinase Second Messenger Systems

Protein kinase activity: Insulin, growth hormone, prolactin, oxytocin, erythropoietin, several growth

O

factors Calcium and/or phosphoinositides: Epinephrine and norepinephrine, angiotensin II, antidiuretic hormone, — gonadotropin-releasing hormone, thyroid-releasing hormone. Cyclic GMP: Atrial natriuretic hormone, nitric oxide

Cyclic AMP Second Messenger Systems O O

Concentrations vary with increase or decrease of hormones. Results in activation of a cAMP-dependent protein kinase called protein kinase A. Upon activation, protein kinase A phosphorylates a number of other proteins, many of which are

O.

Certain receptors are protein kinases, that are activated by hormone binding The kinase activity associated with such receptors results in phosphorylation of tyrosine residues on other proteins. Insulin is an example of a hormone whose receptor is a tyrosine kinase.

PART-III Describe the synthesis and metabolism of HDL and LDL. Add a note on disorders of Lipoprotein metabolism (Ref: Biochemistry by Satyanarayana 5/e, pg. 326, 327)

BIOCHEMISTRY

Second Messenger Systems

PAPER-I Intracellular domain is responsible for the clustering of LDL receptors into regions of plasma membrane termed coated pits. Apo-B-100 binds to apo-B-100 receptor, receptor LDL complex is internalized by endocytosis. The endosome vesicles fuses with lysosomes. The receptor is recycles & returned to the cell surface. LDL particles, apoproteins & cholesterol esters are hydrolyzed by hydrolases, forming free amino acids & free cholesterol. 70% of LDL is degraded in the liver & remaining is in extra-hepatic tissues. plasma Free cholesterol incorporated into membrane or stored in cells.

ANSWER LDL

O00 0

Bad-cholesterol.

Transports cholesterol from liver to peripheral tissues. LDL contains only one lipoprotein Apo B-100. LDL particles are derived from VLDL. Small part is directly released from liver. Half-life is 2 days.

LDL Synthesis and Metabolism

O O O O

LDL is taken up by peripheral tissues by receptormediated endocytosis. LDL receptors are present in all tissues. But most abundant in hepatic cells. LDL receptors located in specialized regions called clathrin-coated pits. Binding of LDL to receptor is by apo-B-100. It is highly regulated.

LDL Receptors O O O

It is a polypeptide, consists of 839 amino acids. Contains extracellular & intracellular domains. Extracellular domain is responsible for binding of apo-B-100 & apo-E.

Functions of LDL O O

O00 0

O

75% of plasma cholesterol is incorporated into LDL particles. LDL transports cholesterol from liver to peripheral tissues. The transported cholesterol has following fates: For synthesis of steroids. May be incorporated into membranes. May be esterified to MUFA & stored.

Forward and Reverse Transport of Cholesterol Forward and reverse transport of cholesterol

Liver

oe

VLDL

H—>

LDL “Bad” cholesterol

deposit

“Good”

cholesterol (Excretion)

BIOCHEMISTRY

|

HDL

LDL & its Clinical Significance O O

LDL concentration is increased in cardiovascular diseases. Small fraction of cholesterol is taken up by macrophages. Increased levels of LDL or modified LDL or oxidized LDL increase the fraction of cholesterol taken by macrophages.

There is an LDL infiltration through arterial walls & taken up by macrophages or scavenger cells. This is starting event of atherosclerosis, leading to MI. These cells become engorged with cholesterol, foam cells are formed. These are deposited in sub-endothelial space triggering the formation of atheromatous plaque.

PAPER-| O

Leads to thrombosis & coronary artery disease. Defects in LDL receptor synthesis leads to familial hypercholesterolemia.

O

O

O

O O O O

O O

Lp(a) is associated with myocardial infarction & is called as “little rascal” Lp(a) is attached to apo-B-100 by a disulfide bond. In 40% population, there is no detectable level of Lp (a) in serum. Lp(a) levels >30 mg/dL is susceptible for heart attack at a younger age. Indians have higher levels of Lp(a) than western populations. Lp(a) interferes with plasminogen activation & impairs fibrinolysis. Leads to unopposed intravascular thrombosis & possible myocardial infarction.

O O O

O

into the interior of HDL

HDL becomes spherical shape with lot of cholesterol esters are formed. This is called as HDL-3. Mature HDLs are taken up by liver cells by apo A-1 mediated receptor mechanism. HDL is taken up by hepatic scavenger receptor B1. Hepatic lipase hydrolyzes HDL phospholipids & TAG, cholesterol esters are released into liver cells.

These cholesterol esters are used for the synthesis of bile acids or excreted as bile.

HDLisa good cholesterol. Transports cholesterol from peripheral tissues to liver. Synthesized in liver. Major apoproteins in HDL are Apo Al, with some Apo A2, Apo C & Apo E.

When HDL3 remains in circulation, cholesterol esters from HDL is transferred to VLDL, IDL & LDL by a cholesterol ester transfer protein (CETP). TAG from VLDL,IDL & LDL is transferred to HDL in

exchange for cholesterol esters. HDL particles rich in TAG & spherical are called as HDI-2 These particles are first acted upon by hepatic triglyceride lipase (HTGL) Efflux of cholesterol from peripheral cells to HDL is mediated by ABC transporter protein.

HODLis an plasma reservoir of Apo C & Apo E, which can be transferred to VLDL & chylomicrons.

HDL Synthesis and Metabolism O

esters moves

disc.

HDL O O

Second carbon of lecithin contains PUFA. This PUFA is transferred to 3rd OH group of cholesterol to form cholesterol esters. Cholesterol

OO

O

OO

Lipoprotein (A)

Free cholesterol is taken up by the HDL Apo A-1 of HDL activates LCAT. z LCAT binds to HDL disc. Cholesterol from cell is transferred to HDL by cholesterol efflux regular protein, which is an ABC protein. Lecithin is a component of lipid bilayer of HDL disc.

Intestinal cells synthesize components release into blood. Nascent HDL are discoid in shape.

of HDL

&

HDL Metabolism Intestine Liver

-

>

=

HDL Metabolism

PL,A1,C,E

Heart and

Discoid nascent HDL

At Excretion of

Ch + ChA

LCAT Mature HDL

Ch

BIOCHEMISTRY

ser PL CE

blood vessels

PAPER-I O

Functions of HDL

O O O O O

HDL is the transports cholesterol from peripheral tissues to liver, called as reverse cholesterol transport. Cholesterol is excreted through bile. Cholesterol excretion needs prior esterification with PUFA. PUFA reduces serum cholesterol levels. PUFAis anti-atherogenic.

O

Biochemical Changes in Carbohydrate Metabolism O

HDL and Its Clinical Significance O

O O O

Serum HDL levels are inversely related to the incidence of MI. HDLis “anti-atherogenic” or protective in nature. Itisa good cholesterol. rtij

O

O

Classification O

Primary disorders of elevated ApoB-Containing Lipoproteins = Lipid disorders associated with elevated LDL-C and Normal Triglycerides ¢ Familial hypercholesterolemia (FH) ¢ Familial defective ApoB-100 (FDB) * Autosomal dominant hypercholesterolemia due to mutations in PCSK9 (ADH-PCSK9 or

ADH3)

oO

O

O

O Or

Explain the biochemical changes in carbohydrate and lipid metabolism in Starvation >

6.

(Ref: Biochemistry by Satyanarayana 5/e, pg. 386)

oO

LU

by

liver

bodies

(mainly beta-

(due

to

hydroxybutyrate)

serve as fuel source for peripheral tissues and brain In adipose tissue = Increased breakdown of triacylglycerol = Increased release of free fatty acids and glycerol = Free fatty acids: Serve as fuel for various organs =" Glycerol: Serves as precursor for glucose synthesis by liver In skeletal muscle = Skeletal muscle uses fatty acids and ketone bodies as fuel in case of starvation = Incase of prolonged starvation, muscle uses only fatty acids as fuel = Leading to increased ketone bodies in blood. Inbrain =" Use ketone bodies as fuel

Explain why:

a.

D

=

Inliver = Increased fatty acid oxidation = Increased synthesis of ketone bodies excess production of acetyl CoA)

=" Ketone

*

O

= To meet the energy needs of brain In adipose tissue and skeletal muscle =" Reduced uptake of carbohydrates =" Decreased carbohydrate metabolism Inbrain =" Dependent on_ glucose supplied gluconeogenesis

Biochemical Changes in Lipid Metabolism

O

Autosomal recessive hypercholesterolemia (ARH) * Sitosterolemia * Polygenic hypercholesterolemia = Elevated plasma levels of lipoprotein(a) = Lipid disorders associated with elevated triglycerides Inherited causes of low levels of ApoB-Containing Lipoproteins Genetic disorders of HDL metabolism = Inherited causes of low levels of HDL-C = Inherited causes of high levels of HDL-C

Inliver = Acts as blood glucose buffering organ = Increased gluconeogenesis = Increased glycogenolysis =" Net result is increased blood _ glucose concentration

HDL levels 65 mg/dL reduces the risk of CAD.

Lipoprotein Disorders

Imposes certain metabolic compulsions on the organism Biochemical changes occur to maintain adequate supply of energy to the various tissues

NADPH plays a crucial role in phagocytosis (Ref: Biochemistry by Satyanarayana 5/e, pg. 274)

ANSWER

LL

O

OCO

Introduction Starvation

O O

State of metabolic stress Due to scarcity of food

ANSWER

Role in Phagocytosis O

NADPH oxidase plays a crucial role in host defence by neutrophils and macrophages

PAPER-| When cells ingest invading microbes, this enzyme becomes activated to reduce molecular oxygen to superoxide, a precursor of microbicidal oxidants, in the phagosome. NADPH oxidase (Nox) family contains the entire electron-transporting

apparatus

from

NADPH

to

molecular oxygen. In resting neutrophils, cytochrome b558 is mainly present in the membrane of the specific granule, an intracellular component, and is targeted to the phagosomal membrane during phagocytosis. NADPH oxidase (NOX) of phagocytic cells transfers electrons across the wall of the phagocytic vacuole, forming superoxide in the lumen. It promotes microbial killing through the generation of reactive oxygen species and through the activity of myeloperoxidase. Primary function of the oxidase is to produce the pH changes and ion fluxes, and the issues surrounding

these processes. b.

Deficiency of Vitamin B12 results in anemia (Ref: Biochemistry by Satyanarayana 5/e, pg. 157)

ANSWER Pernicious Anemia

Due to vitamin B 12 deficiency O

Characterized by triad of symptoms: =" Megaloblastic anemia * Anemiawithbonemarrowpromegaloblastosis * Due to the inhibition of DNA synthesis (specifically purines and thymidine) =" Gastrointestinal symptoms: * Alteration in bowel motility, such as mild diarrhea or constipation, and loss of bladder

or bowel control. * Due to defective DNA synthesis =" Neurological symptoms: * Sensory or motor deficiencies * Subacute combined degeneration of spinal cord * Seizures, or even symptoms of dementia and or other psychiatric symptoms may be present. * Deficiency symptoms in children include developmental delay, regression, irritability, involuntary movements and hypotonia.

BIOCHEMISTRY

O

Your Roll No. ...............

Name ofthe Paper

:

Biochemistry Paper-II

Name ofthe Course

:

MBBS-2012 Annual

Semester Time: 3 Hours

M.M.: 50

INSTRUCTIONS Write your Roll No. on the top immediately on receipt of this question paper 2.

All questions are to be attempted

3.

Answers to Parts I, II and III should be written in separate answer sheets provided

4.

Attempt parts of a question in sequence

PART-I 1.

Write short notes on:

a. b. c. d. 2.

[10]

Cell mediated immunity Creatinine clearance test Mechanisms of DNA repair (any two) Oncogenes

Write briefly on:

a. b. c. d.

[10]

Diagnostic importance of Anion-Gap determination RNA interference Blood buffers Retroviruses in gene therapy

PART-II 3.

4.

a.

What are histones? Discuss their role in formation of chromatin and regulation of gene expression.

b.

In the light of Operon Model, explain why glucose is preferentially utilized even in presence of lactose

Enumerate various biochemical tests for the functional assessment of pancreas. Discuss any two in details

[10]

[5]

PART-III 5.

Explain basic diseases.

concepts

of recombinant

DNA

technology.

Describe

its practical

application

in health

and [10]

OR Explain the basis of two important methods of separation of plasma proteins. State the biochemical functions of two plasma proteins-one a-globulin and one {$-globulin 6.

Compare and Contrast: i.

ii.

Kwashiorkor and Marasmus

Immunogenicity and Antigenicity

[5]

PAPER-II

2012

|

2012 PAPER-II

PART-I 1.

O

membrane,

=" Helper T cells proliferate and predominate. =" Helper T cells release interleukins (and other cytokines), which stimulate B cells to produce antibodies that bind to the antigens and stimulate nonspecific agents (NK and macrophages) to destroy the antigens.

Cell mediated immunity (Ref: Biochemistry by Satyanarayana 5/e, pg. 698)

ANSWER Introduction

b.

Cell-mediated Immunity

OO

O

O

O

Involves the activation of = Phagocytes, = Antigen-specific cytotoxic t-lymphocytes, = Cytokines in response to an antigen. = T cells Does not involve antibodies CD4 cells or helper T cells provide protection against different pathogens. Responds to any cell that displays non-self MHC markers, including cells invaded by pathogens, tumor cells, or transplanted cells The innate immune system and the adaptive immune system each comprise both humoral and cell-mediated components.

T cells

Activated Effector T cells can be placed into three functioning classes, detecting peptide antigens originating from various types of pathogen: O O

O

If MHC-I/ is recognized = Exogenous antigens are detected on the plasma

Write short notes on:

a.

Cytotoxic T cells: Kill infected target cells by apoptosis without using cytokines, THI cells: Activates macrophages, and TH2 cells: Stimulates B cells into producing antibodies.

)

Creatinine clearance test

(Ref: Biochemistry by Satyanarayana 5/e, pg. 462) ANSWER Creatinine Clearance

O

Volume (in mL) of plasms that would be completely cleared of creatinine per minute.

Procedure Traditional Method

Creatinine concentration of 24-hour urine and plasma concentration is estimated. Creatinine clearance in mg/dL or mmol/dl (C) = Urine concentration of creatine (U) x Urine output in ml/min (V)/ Plasma concentration of creatine (P) C=UxV/P

This formula is called the creatinine clearance and tends to exceed the true GFR by 10 to 20 percent or more depending upon the proportion of urinary creatinine that is derived from tubular secretion. Ifalaboratory is using standardized methods, creatinine clearance measurements will consistently be 10 to 20 percent higher than GFR in patients with a normal GFR and progressive higher as the GFR falls.

Mechanism The process by which cell mediated immunity works: O O O

Recognition of antigen presenting cells displaying foreign antigens and binding to T cells. Interleukins (secreted by APCs or helper T cells) costimulate activation of T cells. IfMHC-lis recognized = Endogenous antigens are detected on the plasma membrane, = Cytotoxic T cells proliferate and predominated. =" Cytotoxic T cells destroy cells displaying the antigens.

Modified Procedure

O

Instead of 24 hour urine collection,

O

sample is collected. Creatinine clearance formula.

is calculated

1 hour urine

by

the

same > oO

Referred Values

D

O

Li

O O

Normal range of creatinine clearance: 120-145 mL/min Lower in females Can be expressed in terms of body surface area.

=

LL

O

OCO

PAPER-II O

Diagnostic Importance Decrease

in creatinine

clearance:

= Removal of the damaged oligonucleotide, = Synthesis of a patch, and = Ligation of the patch. Example-Xeroderma pigmentosum d.

oO

D

O

(Ref: Biochemistry by Satyanarayana 5/e, pg. 659)

OCO

Growth factors

=

Stimulate normal growth and stimulation in healthy cells = Regulate cell division by transmembrane signal transduction =

O

2.

Play an important role in carcinogenesis

= Examples: Epidermal growth factor, nerve growth factor, tumor necrosis factor alpha = Cell proliferation is stimulated by growth factors Transforming growth factor alpha-protein required for epithelial cell growth and function =

Increased in psoriasis

=

Leading to excessive production of epithelial cells

Write briefly on: a.

Diagnostic importance determination

of

Anion-Gap

ANSWER For answer refer 2013 paper-2, Q. 2(c), Pg. 442

b.

RNA interference (Ref: Lippincott’s 5/e, pg. 459)

ANSWER For answer refer 2015 paper-2, Q. 5, Pg. 405

c.

Blood buffers

ANSWER For answer refer 2015 paper-2, Q. 4, Pg. 404

d.

LU

O

for oncoproteins-altered proteins. Some products of

Oncogenes

=

LL

Oncogenes code for certain proteins known as oncoproteins-altered proteins that alter the cell cycle.

Oncogenes encode’ counterparts of normal oncogenes are:

Base Excision-repair

O

Activation of proto oncogenes to oncogenes leads to carcinogenesis

Mechanism of Action of Oncogene

ANSWER

O

Viral oncogene similar to certain genes in normal host cells-proto oncogenes Proto oncogene code for growth regulating proteins

ANSWER Introduction O

Cancer causing genes are known as oncogene

Oo

First demonstrated in Rous sarcoma virus

Retroviruses in gene therapy (Ref: Biochemistry by Satyanarayana 5/e, pg. 667)

PAPER-II =" H65 histones are individual proteins involve in the packaging of specific region of DNA.

ANSWER Gene Therapy Using Retroviruses O

Gene therapy: Insertion of foreign genes into cells with genetic defects.

O

Most effective method: It is using viral vector for transfer of genes Retrovirus enters the cells through receptor mediated endocytosis

O O

O O O

Once

the

virus

enters

the

cells,

cDNA

incorporated into the host DNA forming a provirus Provirus gets integrated along with chromosomal DNA during cell division. Retroviral vectors are made such that the required genes take place of retroviral genes. Limitation of gene therapy using retrovirusesrandomness of integration-this can lead to activation of oncogenes.

a.

What are histones? Discuss formation of chromatin and gene expression.

O

O

O

O their role regulation

Each nucleosome is composed of DNA wrapped around 8 histone proteins, functions like a spool and called a histone octamer. The nucleosomes are then wrapped, into a 30 nm spiral, called a solenoid, where additional H1 histones are associated with each nucleosome to maintain the chromosomal structure.

gets

PART-II 3.

Histone Interactions with DNA

in of

Histone: DNA interactions

=" Hydrogen bonds: Between the backbone of the DNA and the amide group of histones. " Non-polar interactions: Between the deoxyribose sugars on DNA and histone proteins =" Salt bridges and hydrogen bonds: Between basic amino acids (especially lysine and arginine) and phosphate oxides on DNA. Highly active genes have less histone while nonactive genes have highly linked with histones during interphase. Histone proteins has a highly positively charge on N-terminus having lysine and arginine residues. b.

(Ref: Biochemistry by Satyanarayana 5/e, pg. 81) ANSWER

In the light of Operon Model, explain why glucose is preferentially utilized even in presence of lactose (Ref: Biochemistry by Satyanarayana 5/e, pg. 564)

O O O

Highly alkaline proteins found in eukaryotes DNA double helix is wrapped around the basic protein core-histones. Package and order the DNA into structural unitsnucleosomes

O

Condense DNA to form chromatin

O. O

The length of DNA is reduced considerably Act as gatekeepers: Determine which portions of the DNA is available for protein expression.

ANSWER Introduction

O O

Classes of Histones Two classes of Histones: O

Core Histones:

="

O

Includes H2A, H2B, H3, H4

=" Two of each of these core histone proteins assembles to form one octameric nucleosome core particle, and 147 base pairs of DNA wrap around this core particle. Linker Histones: =" H1,H5 =" H1 binds the nucleosome at the starting and ending sites of the DNA, thus locking the DNA into place and help in the formation of higher order structure.

Operon concept: Coordinated unit of genetic expression in bacteria Structure of lac operon system = Consists of * Regulatory gene (I) * Operator gene (O) * ‘Three structural genes 5 Z-codes_ for betagalactosidase: Hydrolyzes lactose to galactose and glucose 5 Y-codes for galactoside permease: Transport of lactose into the cell 5 A-codes for galactoside acetylase: Function unknown * Promotor site: Next to operator gene-binds RNA polymerase

Glucose is Preferentially Utilized in Presence if Lactose—because O O

E.coli utilizes glucose in preference to lactose. As glucose interferes with the induction of lac operon

BIOCHEMISTRY

Introduction

PAPER-II Catabolite gene activator protein (CAP) is required for the attachment of RNA polymerase to the promoter site

O

CAP is bound to cAMP

O

Glucose inactivates enzyme adenylyl Thus reducing cAMP concentration

cyclase:

Fecal enzyme assays = Chymotrypsin = Elastase-1 Serum enzyme assays

Indirect Test

Formation of CAP-cAMP is decreased

O

In the absence of CAP-cAMP, binding of RNA polymerase to DNA is diminished Thus transcription is absent in the presence of glucose Therefore glucose interferes with the activation of lac operon by depleting cAMP

O O

Fat absorption test = C13-MTG test = Fluorescein dilauryl test BI-PABA Test Pancreatic Schilling test

Endocrine Function O

Oral glucose tolerance test

Pancreatic Damage O

Serum amylase

Secretin Stimulation Test ATP

Adenylyl cyclase

O

oar (0)

O

CAP

(4)

z

Second duodenal tube is used for the infusion of a nonabsorbable marker, such as polyethylene glycol or bromsulphalein For the measurement of volume and bicarbonate production, pure secretin is used The usual dose of CCK is 50 microg per kg of body weight per hour administered by continuous infusion. Duodenal juice is collected by gravity drainage in iced tubes at 15-minute intervals for 11/2 hours after the stimulants are given.

CAP-cAMP ( ZY )

y |

P

O

Z

lac operon

¥

A

|

sage

el

Polycistronic mRNA

4.

Requires the passage of a triple lumen tube Its distal opening usually is placed near the angle of Treitz by fluoroscopic guidance.

Enumerate various biochemical tests for the functional assessment of pancreas. Discuss any two in details.

Drawbacks of Secretin test = The length of the procedure, = The necessity for nasogastric intubation

(Ref: Biochemistry by Satyanarayana 5/e, pg. 466)

Glucose Tolerance Test (GTT)

ANSWER

O

Procedure = Preferably done in early hours =" Fasting sample is taken = 75 g glucose in 300 mL of water is given orally = Samples taken every 30 minutes for 2 hours

O

Interpretation

Functional Assessment of Pancreas Indications for Pancreatic Function Tests O O > oO

O

D

=

LU

LL

O

OCO

O

Characterization of pancreatic function in suspected pancreatic disease Differential diagnosis in malabsorption Assessment of adequacy of pancreatic replacement therapy Pediatric disorder of exocrine pancreas

Pancreatic Function Test

Direct Test O

Secretin stimulation test

O

Lundh standardized meal test

enzyme

= =

Fasting 110-126 mg/dL 2-hour sample-140-200 mg/dL

PART-III Explain basic concepts of recombinant DNA technology. Describe its practical application in health and diseases. (Ref: Biochemistry by Satyanarayana 5/e, pg. 571)

PAPER-II ANSWER

Practical Application in Health and Diseases

Introduction

Gene Mapping

Also known as genetic engineering Involves manipulation of DNA to achieve the desired goal in pre-determined way

O

O Basic Concept of Recombinant DNA Technology O

Isolation and manipulation of DNA is the object of recombinant DNA research.

O

Steps Involved in Recombinant DNA Technology O

O

O

O O

Generation of DNA fragments and selection of desired piece of DNA Specific restriction endonuclease is obtained Restriction endonuclease cuts the specific DNA at the two ends which becomes the restriction fragment Insertion of restriction fragment into the cloning vector-e.g. a plasmid

O

OO

(Cloning vector)

J

0.

of a of DNA a gene) organism

Plasmid vector cut with restriction endonuclease

J

CITTTTTTT Tr rrr rity)

O

Foreign DNA

are

This technology has two prominent merits: = Produce large amounts of proteins that could not be obtained by conventional purification methods. =" Provides human material. For example: * Recombinant Insulin 6 For the treatment of diabetes mellitus 6 Technique involves insertion of human insulin gene and promoter gene of lac operon on the plasmid of E coli * Recombinant Vaccine oO New generation vaccine First synthetic Oo Hepatitis B_ vaccine: vaccine

Determining molecular basis of number of diseases e.g., familial hypercholesterolemia, sickle cell disease, the thalassemia, cystic fibrosis, Huntington’s

O Recombinant plasmid

conditions,

Disease Diagnosis

Carrvi (Carrying a gene)

Foreign DNA ligate joined with plasmid vector DNA

chromosome-linked

Protein Production

Host bacterium divides to give multiple copies of the recombinant DNA

like plant

X

really located at specific sites.

Introduction of rDNA into the host cells

asmid vector Plasmi

Genes that code for proteins with similar functions can be located on separate chromosomes. The genes involved in many hereditary disorders known to be due to specific protein deficiencies, including

The plasmid DNA now contains foreign DNA and its DNA is called recombinant DNA (1r-DNA)

Isolation fragment (carrying from any

Specific genes to distinct chromosomes are localized by this technique and thus to define a map of the human genome. Techniques used: = Somatic cell hybridization = In situ hybridization

O

chorea Diagnosis of existing disease and risk prediction of certain diseases is possible Genetherapyforsickle cell disease, the thalassemia’s, adenosine deaminase deficiency, and other diseases

may be devised. |

Recombinant DNA Mobilization of recombinant plasmid DNA into host

organisms like E cos by transformation Bacterial

|

choromosome

Recombinant DNA

Multiplication (cloning) of recombinant plasmid DNA in the host organisms following cell divisions (cell multiplication)

Fig. Recombinant DNA technology

OR

Explain the basis of two important methods of separation of plasma proteins. State the biochemical functions of two plasma proteinsone a-globulin and one #-globulin (Ref: Biochemistry by Satyanarayana 5/e, pg. 13, 185) ANSWER For answer refer 2015 paper-2, Q. 1(c), Pg. 401

BIOCHEMISTRY

O O

PAPER-II 6.

Compare and Contrast: a.

Kwashiorkor and Marasmus

ANSWER

Marasmus

Kwashiorkor

Bloating of abdomen

No

Yes

Biochemical findings

Serum albumin 2-3 mg/dL Increased serum cortisol

Serum albumin less than 2 mg/dL Decreased serum cortisol

Treatment

Providing a nutritious, wellbalanced diet with lots of fresh fruits and vegetables, grains, and protein. Especially adding Vitamin B to the diet.

Providing carbohydrates followed by high protein foods. Dried milk specially

Introduction

and

Marasmus

__ Kwashiorkor-protein

energy

malnutrition O

Nutritional disorder

O

Seen in developing countries

O

Mostly seen in young children and infants

Introduction

Etiology

Marasmus

Kwashiorkor

Marasmus — meaning to waste

Protein calorie

Deficiency of calories, insufficient intake of proteins, fats and carbohydrates due to failed breastfeeding, feeding inadequate infant formula or suffering from some disease

malnutrition

Insufficient intake of proteins, due to weaned from mother’s milk to a diet low in protein

b.

ANSWER Also, For answer refer 2016 paper-2, Q. 3(b), Pg. 387 Immunogenicity O

Characteristic of the host

O

Ability to respond to an immunogen " Influenced by:

like diarrhea

Symptoms

Age of onset Muscle

BIOCHEMISTRY

wastage

Large belly sticking out, diarrhea, change in skin pigment, muscle wasting, diarrhea, growth retardation, fatigue, hair changes, etc.

Peeling and alternately pigmented skin, hair loss, edema or swelling, skin folds are formed, etc.

Generally children under age of 1

Generally children of age 1-5 years

Yes

No

Immunogenicity and Antigenicity

*

Genetics

* * *

Route of administration Dosage Tolerance

Antigenicity O

Characteristic of a molecule

O

Ability to trigger an immune response = Factors affecting antigenicity-ability to bind to TCR antibody

Your Roll No. ...............

Name ofthe Paper

:

Biochemistry Paper-I

Name of the Course

:

MBBS-2011

Semester

Annual M.M.: 50

Time: 3 Hours

INSTRUCTIONS Write your Roll No. on the top immediately on receipt of this question paper All questions are to be attempted

Attempt parts of a question in sequence

PART-I

[10]

aoSF PS

Write briefly: Role of HDL in Reverse-cholesterol metabolism

Glycated Haemoglobin and its clinical significance Allosteric enzymes with an example Biochemical roles of NADPH

[10]

fF PS

Explain why:

ao

a

Answers to Parts I, II and III should be written in separate answer sheets provided

Vitamin — B12 deficiency leads to Neuronal symptoms High levels of Lp ‘a’ (Lipoprotein-‘a’) makes a person prone to CAD Photosensitivity occurs in Congenital erythropoietic porphyria Mucopolysaccharides have gel like texture

PART-II a.

What is the role of Glutamate in Ammonia metabolism?

b.

What is the role of PIP3 (Phosphatidyl Inositol triphosphate) in Hormone action

Give an overview of synthesis of various Neurotransmitters from different amino acids

[10] [5]

PART-III What is the diagnostic criteria of Diabetes Mellitus? Discuss different complications of Uncontrolled Diabetes Mellitus [10] Or Discuss various types of jaundice and their Biochemical findings Write mechanism of action of following drugs: a. b.

Cholestyramine Statins

[5]

PAPER-I |

2011 PAPER-I

b.

PART-1 1.

|

Glycated haemoglobin and its clinical significance

Write briefly: a.

Role

of

(Ref: Biochemistry by Satyanarayana 5/e, pg. 655) HDL

in_

Reverse-cholesterol

metabolism

ANSWER Introduction

ANSWER Reverse Cholesterol Transport

O O O

O

Multi-step process Results in movement of cholesterol from peripheral tissues back to the liver. Process = Cholesterol is transferred to HDL from non hepatic tissues by: * ABCAI1: ATP-binding cassette transporteracts as a Carrier * Apolipoprotein Al: Protein of HDL-acts as acceptor * Phospholipid component of HDL: Acts as sink for mobilized cholesterol. = Cholesterol is esterified to form cholesteryl esters by LCAT = These are transferred by cholesteryl ester transfer protein (CETP) in exchange for triglycerides = Cholesterol esters form a part of HDL. =" Uptake of HDL by liver is through lipoprotein lipase (hepatic lipase). = Cholesterol is metabolized and degraded for excretion in liver. Hepatic lipase activity = Increased by androgens and = Decreased by estrogens.

BIOCHEMISTRY

Transport of Cholesterol O

Dependent on enzyme lecithincholesterol acyltransferase (LCAT) and high density lipoproteins

O

LCAT

= = =" =

Plasma enzyme Synthesized by liver Substrate: HDL-cholesterol Transfers the fatty acid on second position of lecithin (phosphatidyl choline) to hydroxyl group of cholesterol = Reversible reaction = Associated with apo-A, protein of HDL

O

Glycated or glycosylated hemoglobin: Glucose derived products of normal adult hemoglobin HbA

O

Glycation:

O

addition of sugar residue to amino acids HBA,: Most abundant glycated hemoglobin - produced by condensation of glucose with

Post-translational,

non

enzymatic

N-terminal valine of each f- chain of HbA Clinical Significance O O O O

HbA,,: Indication of blood glucose concentration over a period of 6-8 weeks (half life of RBC) Rate of synthesis of HbA, . is directly proportional to exposure of RBC to glucose. HbA, concentrationis about3-5% oftotal hemoglobin. Determination of HbA, ,: For monitoring of diabetes control

Normal Range O O O

Inhealthy non diabetic: 4-5.6% Prediabetic: 5.7%-6.4% Diabetic: 6.5% or higher c.

Allosteric enzymes with an example (Ref: Biochemistry by Satyanarayana 5/e, pg. 103)

ANSWER

Regulation of Enzyme Allosteric Enzymes O

Allosteric enzymes bind activators or inhibitors at sites other than the active site (Fig).

+ Activator

*No Vv

activator or inhibitor

+|nhibitor

(Substrate)

PAPER-| Sigmoidal curves are generated by plots of v versus [S]. = An allosteric enzyme has two or more subunits each with substrate-binding sites that exhibit cooperativity. Binding of a substrate molecule at one site facilitates binding of other substrate molecules at other sites. * Allosteric activators cause the enzyme to bind substrate more readily (shift the kinetic curve to the left, thereby decreasing the apparent Km). * Allosteric inhibitors cause the enzyme to bind substrate less readily (shift the curve to the right, increasing the apparent Km). " Similar effects occur during O, binding to hemoglobin.

=" Glycogen phosphorylase = Key enzymes of gluconeogenesis =" Citrate Lyase =" Phosphorylase b kinase "

Enzymes Active in Dephosphorylated State

O O

O ALA synthase Aspartate transcarbamoylase

HMG CoA Reductase Phosphofructokinase

Inhibitor

Heme ATP

CTP

O

Cholesterol Fructose 2,6

Citrate

Pyruvate carboxylase

Acetyl CoA

ADP

Acetyl CoA carboxylase

Citrate

Acetyl CoA

N Acetyl glutamate (NAG)

Carbamoyl phosphate synthetase II

ATP

O O

Proteins can bind to enzymes, altering their activity. For example, regulatory subunits inhibit the activity of protein kinase A. When these regulatory subunits bind cyclic AMP (cAMP) and are released from the enzyme, the catalytic subunits become active. Biochemical roles of NADPH

(Ref: Biochemistry by Satyanarayana 5/e, pg. 272) ANSWER Biochemical Role of NADPH

OUTP

Regulation of Enzyme Activity by Covalent Modification

O

state

ATP

Carbamoyl phosphate synthetase |

O

Usually enzymes are in dephosphorylated when body is in well fed state. Under the influence of Insulin =" Glycogen synthase =" Key enzyme of Glycolysis = Acetyl CoA carboxylase = Pyruvate dehydrogenase =" HMG CoA reductase

b.

bisphosphate

Citrate synthase

CoA reductase kinase

Regulation by Protein-protein Interactions

Examples of Allosteric Enzymes Activator

HMG

Enzyme activity may increase or decrease after the covalent addition of a chemical group. Phosphorylation affects many enzymes. = Pyruvate dehydrogenase and glycogen synthase are inhibited by phosphorylation. =" Glycogen’ phosphorylase is activated by phosphorylation. Phosphatases that remove the phosphate groups alter the activities of these enzymes. Phosphorylation introduces negative charges to the protein, which may alter the secondary and tertiary

O

O-

Role in synthesis =" Required for the biosynthesis of fatty acids and steroids. That’s why HMP shunt is more active in adipose tissues, liver. = Required for the synthesis of certain amino acids such as enzyme glutamate dehydrogenase Antioxidant reactions involving NADPH prevents H,0, induced DNA damage. Glutathione detoxifies peroxides, peroxidase catalyses the _ reaction. NADPH plays an important role in the regeneration of reduced glutathione from its oxidized state. Role in xenobiotics = Cytochrome P450 is NADPH dependent. Cyt P 450 is responsible for the detoxification of various drugs

Also, For answer refer 2012 paper-1, Q. 6(a), Pg. 454

structures.

Enzymes Active in Phosphorylated State O O

Usually enzymes are in Phosphorylated state when body is fasting. Under the influence of glucagon

2.

Explain why a.

Vitamin-Bl2 symptoms

deficiency

leads

to Neuronal

(Ref: Biochemistry by Satyanarayana 5/e, pg. 157)

BIOCHEMISTRY

O

PAPER-| Sigmoidal curves are generated by plots of v versus [S]. = An allosteric enzyme has two or more subunits each with substrate-binding sites that exhibit cooperativity. Binding of a substrate molecule at one site facilitates binding of other substrate molecules at other sites. * Allosteric activators cause the enzyme to bind substrate more readily (shift the kinetic curve to the left, thereby decreasing the apparent Km). * Allosteric inhibitors cause the enzyme to bind substrate less readily (shift the curve to the right, increasing the apparent Km). " Similar effects occur during O, binding to hemoglobin.

=" Glycogen phosphorylase = Key enzymes of gluconeogenesis =" Citrate Lyase =" Phosphorylase b kinase "

Enzymes Active in Dephosphorylated State

O O

O ALA synthase Aspartate transcarbamoylase

HMG CoA Reductase Phosphofructokinase

Inhibitor

Heme ATP

CTP

O

Cholesterol Fructose 2,6

Citrate

Pyruvate carboxylase

Acetyl CoA

ADP

Acetyl CoA carboxylase

Citrate

Acetyl CoA

N Acetyl glutamate (NAG)

Carbamoyl phosphate synthetase II

ATP

O O

Proteins can bind to enzymes, altering their activity. For example, regulatory subunits inhibit the activity of protein kinase A. When these regulatory subunits bind cyclic AMP (cAMP) and are released from the enzyme, the catalytic subunits become active. Biochemical roles of NADPH

(Ref: Biochemistry by Satyanarayana 5/e, pg. 272) ANSWER Biochemical Role of NADPH

OUTP

Regulation of Enzyme Activity by Covalent Modification

O

state

ATP

Carbamoyl phosphate synthetase |

O

Usually enzymes are in dephosphorylated when body is in well fed state. Under the influence of Insulin =" Glycogen synthase =" Key enzyme of Glycolysis = Acetyl CoA carboxylase = Pyruvate dehydrogenase =" HMG CoA reductase

b.

bisphosphate

Citrate synthase

CoA reductase kinase

Regulation by Protein-protein Interactions

Examples of Allosteric Enzymes Activator

HMG

Enzyme activity may increase or decrease after the covalent addition of a chemical group. Phosphorylation affects many enzymes. = Pyruvate dehydrogenase and glycogen synthase are inhibited by phosphorylation. =" Glycogen’ phosphorylase is activated by phosphorylation. Phosphatases that remove the phosphate groups alter the activities of these enzymes. Phosphorylation introduces negative charges to the protein, which may alter the secondary and tertiary

O

O-

Role in synthesis =" Required for the biosynthesis of fatty acids and steroids. That’s why HMP shunt is more active in adipose tissues, liver. = Required for the synthesis of certain amino acids such as enzyme glutamate dehydrogenase Antioxidant reactions involving NADPH prevents H,0, induced DNA damage. Glutathione detoxifies peroxides, peroxidase catalyses the _ reaction. NADPH plays an important role in the regeneration of reduced glutathione from its oxidized state. Role in xenobiotics = Cytochrome P450 is NADPH dependent. Cyt P 450 is responsible for the detoxification of various drugs

Also, For answer refer 2012 paper-1, Q. 6(a), Pg. 454

structures.

Enzymes Active in Phosphorylated State O O

Usually enzymes are in Phosphorylated state when body is fasting. Under the influence of glucagon

2.

Explain why a.

Vitamin-Bl2 symptoms

deficiency

leads

to Neuronal

(Ref: Biochemistry by Satyanarayana 5/e, pg. 157)

BIOCHEMISTRY

O

PAPER-I ANSWER

O

Protoporphyrin overproduction erythroid tissue.

Neuronal Symptoms due to Vitamin B12 Deficiency

O

It is released in dark, but production increases on

O O O

O

Peripheral sensory-motor symptoms Subacute combined degeneration of spinal cord Dementia and depression due to the underproduction of methionine because of the inability to convert homocysteine into this product. Methionine is a necessary cofactor in the production of several neurotransmitters. The neurological complex, defined as myelosis funicularis, consists of the following symptoms: = Impaired perception of deep touch, pressure and vibration, loss of sense of touch, very annoying and persistent paresthesias = Ataxia of dorsal column type = Decrease or loss of deep muscle-tendon reflexes ="

O O

O

Protoporphyrin-sensitized photodamage endothelial cells is due to the presence protoporphyrin in lipid structures.

O

Pathogenesis,

O

of of

=

Invasion of neutrophils into interstitial tissue and

=

Complement activation

=

Endothelial cell injury

Clinical symptoms are probably evoked by uroporphyrin and coproporphyrin present in the interstitial tissue. =

others, also severe paresis Severe and irreversible brain damage. These symptoms of neuronal damage may not reverse after Correction of hematological

b.

in

=" Degranulation of mast cells,

Photodamage to intercellular structures probably represents the initial event.

= Activation of complement may contribute to the final expression of the cutaneous symptoms. d.

Mucopolysaccharides have gel like texture

abnormalities, and the chance of complete reversal

(Ref: Biochemistry by Satyanarayana 5/e, pg. 23) ANSWER

Gel Like Structure of Mucopolysaccharides is Due to

O

O

O

O

mainly

exposure to light

Pathological reflexes: Babinski, Rossolimo and

decreases with the length of time the neurological symptoms have been present. Ininfants = Poor growth, = Apathy = Developmental regression. =" While most symptoms resolve with supplementation some developmental and cognitive problems may persist = Tinnitus may be associated with vitamin B12 deficiency

occurs

O

High levels of Lp ‘a’ (Lipoprotein-‘a’) makes a person prone to CAD

Macromolecular aggregate of protein- polysaccharide complex Property of self-aggregation Forces involved are hydrogen bonds, Van der Waals forces, dipole-dipole interactions Influenced by changes in = Tonic strength = Jon binding = Lyotropic effects

(Ref: Biochemistry by Satyanarayana 5/e, pg. 326, 327)

PART-II ANSWER

3.

a.

For answer refer 2012 paper-1, Q. 5, Pg. 453

c.

Photosensitivity occurs erythropoietic porphyria

in

Congenital

What is the role of Glutamate in Ammonia metabolism? (Ref: Biochemistry by Satyanarayana 4/e, pg. 333,336; Harper’s 16/e pg. 109)

BIOCHEMISTRY

(Ref: Biochemistry by Satyanarayana 5/e, pg. 210)

ANSWER ANSWER

For answer refer 2014 paper-1, Q. 3(a), Pg. 415 Photosensitivity in Congenital Erythropoietic Porphyrias O O

Photosensitivity is due to protoporphyrin Protoporphyrin is found in plasma either to albumin or to lipoproteins

b.

Whatis the role of PIP, (Phosphatidyl Inositol triphosphate) in Hormone action (Ref: Biochemistry by Satyanarayana 5/e, pg. 431)

PAPER-| ANSWER

ANSWER

Signaling Pathway

Introduction

O

Increases in the intracellular Ca** concentrations are

Diabetes Mellitus

often a result of IP3 activation.

O

Metabolic disorder of fuel metabolism

When aligand binds toa G protein-coupled receptor (GPCR) that is coupled to a Gq heterotrimeric G protein, the a-subunit of Gq can bind to and induce activity in the PLC isozyme PLC-f, which results in the cleavage of PIP2 into IP3 and DAG.

O O

Characterized by hyperglycemia Types of Diabetes Mellitus = Insulin dependent DM (IDDM) * Also known as type I diabetes onset diabetes,

Ifa receptor tyrosine kinase (RTK) is involved in activating the pathway, the isozyme PLC-y has tyrosine residues that can become phosphorylated

*

Age of onset-12-15yrs age

*

Less common, accounts for about 10 to 20% of the known diabetics.

upon

*

and this will activate

PLC-y and allow it to cleave PIP2 into DAG and IP3. O.

This occurs in cells that are capable of responding to growth factors such as insulin.

O

PIP3is asoluble molecule and is capable of diffusing through the cytoplasm to the ER

©

Once

at the ER, IP3 is able to bind to the Ins3P

receptor on a ligand-gated Ca* channel that is found on the surface of the ER. This causes the release of Ca** into the cytoplasm.

@

Etiology: Almost total deficiency of insulin due to destruction of B-cells of pancreas =" Non: Insulin dependent DM (NIDDM) * Also called type II diabetes or adult-onset diabetes Most common,

¢

activation of an RTK,

Age of onset: Above 35 years

¢-

O

Less severe than IDDM.

Etiology: Due to genetic and environmental factors and obesity

Function

Diagnostic Criteria

O O

O

O O

4.

Mobilize Ca* from storage organelles Regulates cell proliferation and other cellular reactions that require free calcium. Plays an important role in the induction of plasticity in cerebellar Purkinje cells. Increase in IP3-mediated Ca* release from the ER in several animal models has been implicated in Alzheimer’s disease.

Give

an

overview

of

synthesis

of

various

O

O

Neurotransmitters from different amino acids

(Ref: Biochemistry by Satyanarayana 5/e, pg. 357, 371) ANSWER O. Also, For answer refer 2012 paper-1, Q. 3(a), Pg. 449

What is the diagnostic criteria of Diabetes Mellitus? Discuss different complications of Uncontrolled Diabetes Mellitus (Ref: Biochemistry by Satyanarayana 5/e, pg. 654)

accounting for 80-90%

Blood Glucose levels " Fasting: * Normal-126 mg/dL or 7 mmol/L Glucosuria = Glucose excretion in urine = Renal threshold of glucose-180 mg/dL = Renal excretion occurs only at plasma levels above 180 mg/dL Glucose Tolerance Test (GTT) = Procedure * Preferably done in early hours * Fasting sample is taken * 75g glucose in 300 ml of water is given orally * Samples taken every 30 minutes for 2 hours Interpretation = Fasting 110-126mg/dL

= 2hour sample-140 -200 mg/dL

PART-III 5.

or juvenile

¢-

O

Diagnostic criteria for oral glucose tolerance test (WHO 1999)

>

Condition

oO

Plasma glucose concentration as mmol/L (mg/dL) Normal Impaired glucose tolerance Diabetes

Fasting

2 hours after glucose

Fluorescently labeled DNA



Test

Hybridization

molecule

Mechanism

O O O

O O.

Bcells produce both IgM and IgD After activation by antigen, B cells proliferate. On encountering specific signalling molecules (CD40, TH cells), B cells undergo antibody class switching to produce IgG, IgA or IgE antibodies. Knownas class switch recombination (CSR) binding The order of the heavy chain exons are: = u-IgM = 6-IgD =" y3-IgG3 = yl-IgGl

| contoca microscopy

Hybridizing signals

=" al-IgAl

d.

Hybridizing

TCCCTTGG

oligonucleotides (8)

CCCTTGGC

DNAsequence

--- AGTCCCTTGGC ---

Applications of DNA Chips

O

O

O

O

= y2-IgG2 =" y4-IgG4 = ¢-Igk =" o2-IgA2 During CSR, portions of the antibody heavy chain are removed, and the gene segments surrounding the deleted portion are rejoined to produce antibody of a different isotype.

sale

Microarray technique

Detection of genes responsible for = Development of nervous system = Inflammatory diseases Detection of micro-organisms for monitoring Detection and screening of single nucleotide polymorphisms

(Ref: Biochemistry by Satyanarayana 5/e, pg. 578) ANSWER

3.

a.

Role of stem cells in therapeutic cloning. (Ref: Biochemistry by Satyanarayana 5/e, pg. 575)

Introduction

O O

DNA chips or microarrays: Method used for DNA sequencing DNA probes are immobilized at different positions on the nylon or glass surface DNA probes: Short DNA molecules-cDNA or synthetic oligonucleotides

Technique

O O

O.

Fluorescently labeled DNA test molecule is applied to the chip. Hybridization occurs between the complementary sequences of the test DNA molecule and oligonucleotide of the chip. The position of hybridization is determined confocal microscopy.

ANSWER Stem Cells

O

OO0 O

O

Undifferentiated cells which can mature to form any other cell of the body. Properties Self-renewal Relocation and differentiation

Have the potential to replace damaged cell

Stem Cells and Therapeutic Cloning O O

In therapeutic cloning, the blastocyst is not transferred to a womb. Instead, embryonic stem cells are isolated from the cloned blastocyst.

> oO

D

=

Li

LL

O

OCO

PAPER-II O

These stem cells are genetically matched donor organism.

to the *

Benefits of Therapeutic Cloning

O00

O

and is considered a non-specific immune response, igg] class, can also “fix” complement The mannose-binding lectin pathway. Antigen

Organ transplant alternatives Leukemia Degenerative diseases Skin grafts b.

Describe diagram

Humoral

t { { { { {

B-cell

Immunity

using

Presented to antigen presenting cell

a flow

Major histocompatibility complex-ll

(Ref: Biochemistry by Satyanarayana 5/e, pg. 698)

T-helper cell activation

ANSWER

Cytokines release

Introduction

Humoral Response O O

Antibody-mediated response Mediated by macromolecules found in extracellular fluids

OO

B-cell proliferation

such

as

secreted

antibodies,

:

complement

proteins, and certain antimicrobial peptides. Involves B cells Recognize antigens or pathogens that are circulating in the lymph or blood

O

B = = =

cell activation: When an antigen is encountered by B cell, Binds to receptor and endocytosis occur. The antigen is processed and presented on its surface again with MHC-II molecule. B cell proliferation: =" TH cell binds to the APC and releases cytokines = Induction of B cells by cytokines to replicate and become plasma cells or memory cells. =

O

O

Memory cells

Release antibodies

Future immunity

f f f f

Its

activation

opsonization, > oO

D

=

LU

LL

O

OCO

leads

to

immune

{

Antigen + antibody complex

Activate complement system

Phagocytosis of antigen

4.

What are various causes of Hyperuricaemias? Mention one drug to control the problem

ANSWER

Plasma cells release antibodies

Antibody-antigen reaction: = Antibody binds to antigen = Chemical interaction attracts macrophages killer cells to attack and phagocytose them. Complement system: =

:

Plasma cells

Mechanism O

|

cytolysis,

Introduction

or

chemotaxis,

clearance,

and

inflammation, as well as the marking of pathogens for phagocytosis. =" Three biochemical pathways activate the complement system: * Theclassical complement pathway: Requires antibodies for activation and is a specific immune response * The’ alternate complement pathway: Activated without the presence of antibodies

Hyperuricemia is an abnormally high level of uric acid in the blood Causes

Causes of hyperuricemia can be classified into three types O

O

Increased production of uric acid = High levels of purine in the diet * Foods high in the purines hypoxanthine = Increased purine metabolism * Tumor lysis syndrome * Lesch-Nyhan syndrome Decreased excretion of uric acid = Kidney disease *

Impaired kidney function

adenine

and

OO

PAPER-II =" Drugs

"

* Antiuricosurics * Diuretics =" Ketogenic diet

=" Blocks uric acid production

PART-III

Mixed type High levels of alcohol = Increased production of lactic acid, hence lactic acidosis. = Increases the plasma _ concentrations of hypoxanthine and xanthine. = Promote dehydration and clinical ketoacidosis Increased fructose in diet = Interferes with purine metabolism = Inhibits excretion of uric acid by competing with its transport protein Starvation = Increases the amount of purine converted to uric acid = Impairs the ability of the kidney to excrete uric acid

Allopurinol O

Decreases uric acid formation

O

Inhibits purine synthesis

Elevated levels of xanthine

5.

Describe the role of MHC I and II molecules in antigen presentation and Organ Transplantation

ANSWER

For answer refer 2013 paper-2, Q. 2(b), Pg. 442 OR Describe the process of Protein synthesis and write various post-translational modifications (Ref: Biochemistry by Satyanarayana 5/e, pg. 555) ANSWER For answer refer 2014 paper-2, Q. 3(a), Pg. 428

6.

Write briefly on: a.

DNAvaccine

(Ref: Biochemistry by Satyanarayana 5/e, pg. 588)

Mechanism of Action

OO

0

O

Purine analogues Structural isomer of hypoxanthine Inhibits enzyme xanthine oxidase. Xanthine oxidase’ catalyzes

ANSWER

For answer refer 2017 paper-2, Q. 2(a), Pg. 369 oxidation’

of

O

hypoxanthine and xanthine, to produce uric acid

Inhibition of xanthine oxidase causes =" Increased hypoxanthine: Salvaged to purine ribonucleotides-AMP and GMP * Increased AMP and GMP causes feedback inhibition of Amidophosphoribosyltransferase * This is first and rate limiting step of purine biosynthesis

b.

Creatinine clearance

(Ref: Biochemistry by Satyanarayana 5/e, pg. 462)

ANSWER

For answer refer 2012 paper-2, Q. 1(b), Pg. 457

> oO

D

=

Li

LL

O

OCO

Your Roll No. ...............

Name of the Paper

:

Biochemistry Paper-!

Name of the Course

:

MBBS-2010 Annual

Semester Time: 3 Hours

M.M.:

50

INSTRUCTIONS Write your Roll No. on the top immediately on receipt of this question paper All questions are to be attempted

Attempt parts of a question in sequence

PART-I Answer briefly:

a b. c. d

[10]

How can chronic alcoholism lead to fatty liver? Toxicity of reactive oxygen species. Inhibitors of prostaglandin synthesis and their clinical significance. Pivotal role of citric acid cycle in metabolism.

fF SP

Explain in brief:

ao

a

Answers to Parts I, II and III should be written in separate answer sheets provided

[10]

Role of Vitamin D in maintenance of blood calcium level. The importance of HMP shunt pathway. Substrate level phosphorylation.

Calculation and interpretation of creatinine clearance test.

PART-II a.

Discuss the formation and utilization of ketone bodies. Why are they harmful when present in excess?

b.

Explain the mechanism of glycogenolysis in muscle. Can it, to a very small extent, contribute to blood glucose levels?

Biochemically

explain

the

major

symptoms

of different

types

of porphyrias.

How

regulated?

is porphyrin

[10]

synthesis

[5] PART- Ill

What are the different types of lipoproteins? Describe their role in cholesterol transport. Explain the role of G-proteins in hormone action.

[10] [5]

PAPER-|

2010

|

2010 PAPER-I

PART - | 1.

c.

)

Inhibitors of prostaglandin their clinical significance.

How can liver?

and

(Ref: Biochemistry by Satyanarayana 5/e, pg. 618)

Answer briefly: a.

synthesis

chronic

alcoholism

lead to fatty

ANSWER Also For answer refer 2012 paper-1, Q. 1(a), Pg. 448

ANSWER

Introduction

For answer refer 2017 paper-1, Q. 1(b), Pg. 356

b.

Toxicity of reactive oxygen species.

O

ANSWER

One-electron peroxidase oxidations to form cation radicals Cytochrome p450 metabolism to free radical products Stabilisation of the ROS-generator, cyp2e1 Futile cycling of other cytochromes p450.

Harmful Effects O Damage the biomolecules =" Causes oxidation of sulfhydryl groups of proteins, causes fragmentation and crosslinking of amino acids: Loss of protein function =" Causes. lipid peroxidation: |. PUFA-most susceptible to free radical damage = Cause DNA breakage, fragmentation of bases.

Substrate analogues Aspirin-like drugs d.

Pivotal role of citric acid cycle in metabolism (Ref: Biochemistry by Satyanarayana 5/e, pg. 256)

ANSWER Introduction

O O O

Involves the oxidation of acetyl CoA to carbon dioxide and water Utilizes about 2/3rds of the oxygen consumed by the body Also known as =" Tricarboxylic acid cycle =" Krebs cycle

Disease Association

=" Cardiovascular disease: Oxidized LDL promote atherosclerosis =" Cancer: Due to effect on nucleic acid-causes mutagenicity and cytotoxicity-role in carcinogenesis

@

=" Inflammatory diseases: Rheumatoid arthritis =" Diabetes mellitus: Glycation ofproteins increases its susceptibility to free radical damage-basis of diabetic microangiopathy, diabetic nephropathy. = Other disease Male infertility Aging Cataract Parkinson’s Alzheimer’s Infertility “ret? © &

O

O O

Liver cirrhosis, etc

Role of Citric Acid in Metabolism

O

Amphibolic nature

O

Provides intermediates for the synthesis of various compounds Both catabolic and anabolic in nature

0000 0

O O O

Locally acting hormones Derivatives of 20-carbon fatty acid namely prostanoic acid, hence known as prostanoids Precursor of prostaglandins is arachidonic acid

Prostaglandin Synthesis Inhibitors

ROS are produced in the metabolism of drugs and industrial chemicals by: O

O O

Involved in

Gluconeogenesis Transamination Deamination >

Synthetic (Anabolic) Reactions O

O

Oxaloacetate and a: Ketoglutarate-serve as precursor for synthesis of aspartate and glutamate, used in the synthesis of non-essential amino acids, purine and pyrimidines. Succinyl CoA: Synthesis of porphyrins and heme

oO

D

=

Li

LL

O

OCO

PAPER-I O

Acetyl CoA: Biosynthesis of fatty acids, sterols

O

Provides Ribose-5-Phosphate: = Required for Purine biosynthesis. = Ribose-5-Phosphate is the precursor molecule for nucleotide synthesis. = The enzyme Glucose-6-Phopshate dehydrogenase in HMP shunt optimizes the concentration of Ribose-5-Phosphate.

O

Producing Glycolytic Intermediate: =" Glycolytic intermediates: Glyceraldehyde3-Phosphate and _ Fructose-6-Phosphate are produced during HMP pathway.

Metabolic (Catabolic) Reactions O

Central metabolic hub of the cell.

O O

Aerobic metabolism of carbohydrates Provides ATP and NADH, FADH2: Provides energy for various metabolic pathways.

2.

Explain in brief: a.

Role of Vitamin

D in maintenance

of blood

calcium level

c.

(Ref: Biochemistry by Satyanarayana 5/e, pg. 127)

Substrate level phosphorylation

ANSWER

ANSWER

Substrate Level Phosphorylation

For answer refer 2017 paper-1, Q. 3(b), Pg. 360 b.

The importance of HMP shunt pathway.

O

Quicker, less efficient source of ATP

O

Unlike oxidative phosphorylation, oxidation and phosphorylation are not coupled in the process of substrate-level phosphorylation, and reactive intermediates are most often gained in the course of oxidation processes in catabolism. Occurs in the cytoplasm of cells during glycolysis and in mitochondria during the Krebs cycle under both aerobic and anaerobic conditions. Results in the formation of ATP or GTP by the direct transfer of a phosphoryl (PO3) group to ADP or GDP from another phosphorylated compound. Involved organic compound itself serves as both electron donor (becomes oxidized) and electron acceptor (becomes reduced) and its high energy phosphate bonds are transferred to ADP to form ATP

(Ref: Biochemistry by Satyanarayana 5/e, pg. 270) ANSWER Importance of HMP Shunt is

O

HMP pathway: Major source of NADPH in body =" Required for the biosynthesis of fatty acids, steroids and amino acids = Antioxidant reactions involving NADPH prevents HO, induced DNA damage. Glutathione detoxifies peroxides, peroxidase catalyses the reaction. NADPH plays an important role in the regeneration of reduced glutathione from its oxidized

=

="

="

> oO

D

=

LU

LL

O

OCO

=

state. Cytochrome P450 is NADPH dependent. Cyt P 450 is responsible for the detoxification of various drugs NADPH oxidase playsacrucialrolein host defence by neutrophils and macrophages. It promotes microbial killing through the generation of reactive oxygen species and through the activity of myeloperoxidase. Role in RBCs * Regulates the concentration of glutathione * Prevents the integrity of RBC membrane * Maintains the ferrous of hemoglobin in ferrous state, preventing the formation of methemoglobin in RBC Preserves the transparency of lenses of eye by maintaining a high concentration of NADPH in lens.

O

O

O

Importance of Substrate-Level Phosphorylation in Anoxia O

During anoxia, provision of ATP by substrate-level phosphorylation is important =" As energy source = Prevent mitochondria from straining glycolytic ATP reserves by maintaining the adenine nucleotide translocator in ‘forward mode’ carrying ATP towards the cytosol d.

Calculation and interpretation of creatinine clearance test. (Ref: Biochemistry by Satyanarayana 5/e, pg. 462)

ANSWER

For answer refer 2012 paper-2, Q. 1(b), Pg. 457

PAPER-| PART - Il

Utilization of Ketone Bodies

O 3.

a.

Discuss the formation and utilization of ketone bodies. Why are they harmful when present in excess?

O

(Ref: Biochemistry by Satyanarayana 5/e, pg. 295)

O

ANSWER

skeletal muscle

O

Introduction O O O

Ketone bodies: Acetone, acetoacetate, h-hydroxyl butyrate Water soluble Energy yielding

Ketogenesis

O00 0

O

During prolonged starvation, = Fuel source for brain-meets 50-70% of its energy needs. More important during periods of starvation and diabetes mellitus

Ketosis

O

Normal serum ketone body concentration-lmg/dL

O

Excessive accumulation of ketone bodies

O

Ketonemia: Increased concentration in blood

O

Ketonuria: Increased excretion of ketone bodies in urine

Site: Liver

Enzymes are present in mitochondrial matrix Acetyl CoA: Precursor of ketone bodies

O

Ketonemia + ketonuria = ketosis

O

Causes of ketosis "

8-oxidation of fatty acids yields acetyl CoA

Results in increased breakdown of fatty acids Increased production of acetyl CoA Accumulation of acetyl CoA leads to increased ketogenesis =" Uncontrolled diabetes mellitus * Impaired carbohydrate metabolism and increased lipolysis as a effect of insulin deficiency * Leads to accumulation of acetyl CoA * Increased acetyl CoA = increased ketone bodies = Non pathologic ketosis * After high fat meal * Weight loss programs

=

Condensation of two acetyl CoA to form acetoacetyl CoA under the influence of enzyme thiolase = Acetoacetyl CoA combines with another acetyl

to

form

f{-hydroxy-B-methylglutaryl

CoA (HMG CoA). Enzyme involved-HMG CoA synthase. Rate limiting step in ketogenesis =" HMG CoA in the presence of HMG CoA lyase splits to form acetoacetate and acetyl CoA =" Acetoacetate undergoes decarboxylation to form acetone and reduction to form £ hydroxyl butyrate Acetyl CoA |

Acetyl CoA

| Acetoacetyl

| 6-ketothiolase

HMG CoA

Acetyl CoA wy

HMG CoAL

al a

daa

Acetoacetate

Spontaneous CO,

Acetone

Regulation of Ketogenesis O

CoA

Acetyl CoA ~ \ HMG CoA Synthase

NADH +H* NAD

+

8-hydroxybutyrate dehydrogenase

6-hydroxy butyrate

Fig. Ketogenesis

Starvation

* * *

Process

CoA

Easily transported to various tissues from liver as it is water soluble Source of energy for peripheral tissues such as

O

Hormonal regulation = Glucagon stimulates ketogenesis = Insulin inhibits ketogenesis Regulation takes places at 3 stages mainly =" Mobilization of free fatty acids from adipose tissue =" Carnitine palmitoyltransferase: | activity is low in fed state and increases in starvation, which results in elevated fatty acid oxidation = Low Oxaloacetate concentration impairs Krebs cycle leading to accumulation of acetyl CoA b.

Explain the mechanism of glycogenolysis in muscle. Can it, to a very small extent, contribute to blood glucose levels?

> oO

D

=

Li

LL

O

OCO

PAPER-I ANSWER

ANSWER

Introduction

Introduction

O

O

O O

Glycogenolysis is the breakdown/degradation of glycogen. Irreversible process Degraded by breakdown of alpha 1-4 and alpha 1 -6 glycosidic bonds

Glycogenolysis

Porphyrias are the metabolic disorders of heme synthesis = Characterized by the increased excretion of porphyrins or porphyrin precursors. =" Porphyrias are either inherited or acquired. = They are broadly classified into two categories *

Glycogen

*

Pw (—)

Glycogen phosphate

Glu 1-

Erythropoietic Protoporphyria

phosphate

O Limit dextrin

| Debranching enzyme

Glucose 1-phosphate Phosphoglucomutase v Glucose-6-phosphate

O

Fate of Glucose-6-phosphate

O

Depends on glucose-6- phosphatase enzyme Glycose-6-phosphate

Types

= Acute intermittent porphyria =" Porphyria cutanea tarda =" Hereditary coproporphyria = Variegate porphyria Characteristic features = Photosensitivity is due to protoporphyrin =" Protoporphyrin is found in plasma either to albumin or to lipoproteins = Protoporphyrin overproduction occurs mainly in erythroid tissue. =

Absence of enzyme

Glucose

O

Present in Liver, kidney and intestine

O

Absentin muscle and brain.

Muscle Glycogenolysis O

O

4. > oO

D

=

LU

LL

O

OCO

Not a major contributor to glucose homeostasis in blood due to absence of glucose 6 phosphatase enzyme, hence free glucose 6 phosphate directly enters the glycolysis pathway without releasing free glucose. Small amount of glucose is produced in muscle by the action of alpha 1-6 glucosidase activity (debranching enzyme).

Biochemically explain the major symptoms of different types of porphyrias. How is porphyrin synthesis regulated? (Ref: Biochemistry by Satyanarayana 5/e, pg. 210)

Itis released in dark, but production increases on

exposure to light =" Protoporphyrin: Sensitized photodamage of endothelial cells is due to the presence of protoporphyrin in lipid structures. =" Pathogenesis, * Degranulation of mast cells, * Invasion of neutrophils into interstitial tissue and * Complement activation * Endothelial cell injury

Glucose 6-phosphate

Glycolysis

Erythropoietic: Enzyme deficiency occurs in the erythrocytes. Hepatic: Enzyme defect lies in the liver.

Hepatic Porphyrias O

O

Types

=" Congenital erythropoietic porphyria = Protoporphyria Characteristic features = The clinical symptoms are probably evoked by uroporphyrin and coproporphyrin present in the interstitial tissue. = Photodamage to intercellular structures probably represents the initial event. = Activation of complement may contribute to the final expression of the cutaneous symptoms.

PAPER-| O

Acute Intermittent Porphyria

O

O

O

O

Due to deficiency of uroporphyrinogen I synthase enzyme Excretion of porphobilinogen and amino levulinate in urine Urine changes its colour on exposure to air Oxidation of porphobilinogen to form porphyrinblack colour of urine Manifestations of acute intermittent porphyria =" Neuropsychiatric disturbances

Variegate Porphyria O O

= Abdominal discomfort =" Cardiovascular abnormalities = Absence of photosensitivity

Congenital Erythropoietic Porphyria O

Porphyria Cutaneous Tarda O O

Due to deficiency of enzyme uroporphyrinogen decarboxylase Characteristic features = Photosensitivity = Increased excretion of uroporphyrins i and ii and rarely porphobilinogen ="

O

Due to deficiency of enzyme uroporphyrinogen III cosynthase Characteristic features = Photosensitivity = Increased hemolysis

Protoporphyria O O

Liver exhibits fluorescence due to accumulation

of porphyrins Hereditary Coproporphyria

O

Due to deficiency of protoporphyrinogen oxidase Characteristic features =" Abdominal pain = Photosensitivity =" Neuropsychiatric symptoms

Defect in enzyme coproporphyrinogen oxidase

Due to deficiency of enzyme ferrochelatase Characteristic feature = Photosensitivity = Protoporphyrin IX is excreted in urine = Reticulocytes and skin biopsy exhibit fluorescence.

Regulation of Porphyrin Synthesis Glycine + Succinyl CoA lron-IRE

| 5-aminolevulinate

synthase

=

|

Amino acids

:

5-aminolevulinate Lead inhibits Porphobilinogen Hydroxymethylbilane

qe

Amino acids

synthase

(+)

Hydroxymethylbilane

'

|

4

Erythropoietin {polypeptide hormone from kidney}

Uroporphyrinogen III

Protoporphyrin IX

Lead inhibits

Fe Protoheme

Hemoglobin

2+

02 Protohemin

BIOCHEMISTRY

O

Characteristic features = Excretion of ALA and PBG in urine = Photosensitivity =" Abdominal pain =" Neuropsychiatric symptoms

PAPER-I PART - Ill

Affects =" Intracellular Ca2+ mobilization,

5.

=" Arachidonic acid release

What are the different types of lipoproteins? Describe their role in cholesterol transport.

= Membrane potential. The mechanism which G activates proteins provides

(Ref: Biochemistry by Satyanarayana 5/e, pg. 316) ANSWER

GPCR

For answer refer 2012 paper-1, Q. 5, Pg. 52

6.

for amplification, reversal of action, and continued monitoring of incoming signals.

O

Explain the role of G-proteins in hormone action. (Ref: Lippincott’s 5/e, pg. 430)

These receptors, known as G protein-coupled receptors (GPCR), are characterized by an extracellular ligand-binding region, seven transmembrane

helices,

and

an_

intracellular

domain that interacts with G proteins The extracellular domain contains

the binding site for a ligand (a hormone or neurotransmitter)

ANSWER

membrane

Functions of G- Proteins

O

as

neurotransmitters,

and

autocrine

and

paracrine factors to regulate cellular functions. =" The chemical signals are most often hormones or neurotransmitters,

BIOCHEMISTRY

helices

Signal transduction: Mechanism by which many hormones,

O

Seen anes

each of which binds to a

membrane receptor. = Therefore, tissues that respond to more than one chemical signal must have several different receptors, each of which can be linked to adenylyl cyclase. Modulates cAMP formation: Adenylyl cyclase is a membrane-bound enzyme that converts ATP to 3}5’-adenosine monophosphate (also called cyclic AMP or cAMP).

o'=s0 se

modification

Position of the removed

PART-II 3.

Chemical (covalent)

(Ref: Biochemistry by Satyanarayana 5/e, pg. 659) For answer refer 2017 paper-2, Q. 4, Pg. 371

b_

Explain the principle of hybridoma technology and state some applications (Ref: Biochemistry by Satyanarayana 5/e, pg. 696)

ANSWER

For answer refer 2018 paper-2, Q. 2(d), Pg. 349

PAPER-II

2010 O

O

PART-II 5.

Whatis delayed type of hypersensitivity? Describe the mechanism of its development

diarrhea, weight loss, and anemia.

Tests of fat malabsorption =" Qualitative tests include the acid steatocrit test and Sudan

ANSWER

III stain of stool - test for intestinal

malabsorption

For answer refer 2015 paper-2, Q. 5(or), Pg. 406

6.

May be intestinal or pancreatic malabsorption Malabsorption is suspected in a patient with chronic

Howwill you distinguish between pancreatic and intestinal malabsorption? (Ref: Biochemistry by Satyanarayana 5/e, pg. 466)

ANSWER

= D-xylose test - tests for pancreatic insufficiency Schilling test = Malabsorption of vitamin B-12 may occur as a consequence of deficiency of intrinsic factor (eg, pernicious anemia, gastric resection), pancreatic insufficiency, bacterial overgrowth, ileal resection, or disease.

Analysis of serum pancreatic lipase and amylase enzymes

Introduction

O

Malabsorption - impaired absorption ofall nutrients or many nutrients

> oO

D

=

Li

LL

O

OCO

Your Roll No. ...............

Name of the Paper

:

Biochemistry Paper-!

Name of the Course

:

MBBS-2009 Annual

Semester Time: 3 Hours

M.M.:

50

INSTRUCTIONS Write your Roll No. on the top immediately on receipt of this question paper 2.

All questions are to be attempted

3.

Answers to Parts I, II and III should be written in separate answer sheets provided

4.

Attempt parts of a question in sequence

PART-I 1.

Answer the following questions briefly: i. ii. iii. iv.

2.

[10]

Illustrate the process by which long chain fatty acids are transported into the mitochondria. Explain the role of different vitamins in reactions catalyzed by pyruvate dehydrogenase complex. Describe allosteric regulation of enzyme activity with the help of a suitable example. Explain how hypoxia leads to hemolytic crisis in sickle cell anemia.

a.

A patient diagnosed for malaria was administered primaquine. His condition worsened with increased hemolysis. On biochemical laboratory investigation of his blood sample, this patient was found to have an enzyme deficiency. (1) Name the enzyme and the related pathway (11) Explain the biochemical basis of primaquine induced crisis in this patient. [10]

b.

During glucose Write a Explain

c. d.

prolonged starvation, brain gets its energy mainly from the ketone bodies. However, a minimum level of is always needed for energy metabolism of brain. Explain. note on the absorption of dietary iron. the mechanism of action of the oral anticoagulant, warfarin.

PART-II 3. 4.

a.

Explain the role of insulin in the regulation of fat metabolism in the adipose tissue.

[10]

b.

With the help of a diagram illustrate the pathway of HDL metabolism. Explain the role of LCAT in HDL metabolism.

a.

Compare and contrast the tissue distribution, catalytic properties and kinetic properties of hexokinase and glucokinase.

b.

With the help of a diagram show the sites of ATP synthesis in the electron transport chain.

PART-III 5.

Describe the sequence of reactions involved in the removal of amino groups from the amino acids leading to the formation of ammonia. [10]

OR Describe the reaction involved in the synthesis of glycogen. Describe the activation of protein kinase by glucagon. 6.

Write short notes on:

1. ii.

Role of haemoglobin as a blood buffer. Role of parathormone in calcium homeostasis.

[5]

PAPER-I

2009

|

2009 PAPER-I

PART |

1.

O

This process is completed in following steps:

= Acyl group of acyl CoA is transferred to carnitine (B-hydroxy T-trimethyl aminobutyrate), which is catalysed by carnitine acyltransferase I (present

Answer the following questions briefly:

a.

|

Illustrate the process by which long chain fatty

on the outer surface of inner mitochondrial

acids are transported into the mitochondria.

membrane).

(Ref: Biochemistry by Satyanarayana 5/e, pg. 288, 368)

=" The

acylcarnitine

membrane ANSWER

transported

to mitochondrial

across

the

matrix by carrier

protein.

=" Carnitine acyltransferase II (found on the inner surface of inner mitochondrial membrane)

Role of Carnitine

O

is

The inner mitochondrial membrane is impermeable

converts acylcarnitine to acyl CoA.

to fatty acids. They are transported through carnitine

= ‘The carnitine released returns to cytosol for reuse.

carrier system or carnitine shuttle. Cytosol

Inner mitichondrial membrane

O

Mitochondrial matrix

>.

|

R — C—SCoA

CaniticCe

OO

O

————————

dS.

lI

Carnitine

R—C—SCoA

Acyl CoA

Acyl CoA

Carnitine acyltransferase |

CoASH

Carrier

Carnitine acyl-

protein

transferase II

O O | | R — C — Carnitine -————————————————+> R — C — Carnitine

Acyl-carnitine

NY

CoASH

Acyl-carnitine

Fig. Carnitine shuttle

b.

Explain the role of different in reactions catalyzed by

vitamins pyruvate

dehydrogenase complex.

O

Pyruvate Dehydrogenase is a Multi-enzyme Complex

ANSWER

that uses three Enzymes

>

O

oO

Introduction

The pyruvate dehydrogenase complex O. O

Links glycolysis to the TCA cycle Large multi-enzyme complex composed enzymes involving five cofactors.

The pyruvate dehydrogenase complex oxidizes pyruvate to generate acetyl-coA in the mitochondria.

O of three

Pyruvate dehydrogenase = Uses thiamine pyrophosphate (TPP) as prosthetic group. Dihydrolipoyl transacetylase = Uses lipoamide and coenzyme A (also known as coASH) as prosthetic groups.

D

=

Li

LL

O

OCO

PAPER-I O

Dihydrolipoyl dehydrogenase =" Uses flavin adenine dinucleotide (FAD) and nicotinamide adenine dinucleotide (NAD+) as its cofactors.

O

HbS solubility unaffected.

O

In HbS, a sticky patch forms on the outer surface of beta chains. There is a receptor complementary to sticky patch on deoxyHbS Leading to formation of long aggregates of deoxyHbS molecules that are stiff: Sickling of HbS Repeated episodes of sickling damage the cell membrane and decrease the cell’s elasticity. Stiff red blood cells are unable to deform as they pass through narrow capillaries, leading to vessel occlusion and ischaemia Hypoxia: Low oxygen tension-increases deoxy HbS -so more sickling - more lysis - hemolytic crisis. OxyHbS: complementary receptors are masked - so no aggregation.

O.

Thiamine Pyrophosphate

O O O

O

Thiamine (vitamin B1) derivative Produced bythe enzyme thiamine diphosphokinase. Acts asa cofactor that is present in all living systems, in which it catalyzes several biochemical reactions. Also known as = Thiamine diphosphate (ThDP) =" Cocarboxylase

O O O

The Net Reaction of Converting Pyruvate into Acetyl CoA and CO. is

O

2pyruvate+2NAD++2CoA>2acetyl CoA+2NADH+2CO,

O

in

oxygenated

form

remains

Five Step Process

O O

O

O

O

Pyruvate is decarboxylated by pyruvate dehydrogenase with help from TPP. The reactive carbon of the TPP is oxidized and transferred as the acetyl group to lipoamide (which is the prosthetic group of the dihydrolipoyl transacetylase). This forms hydroxyethyl-TPP. An H+ ion is required for the intermediate to give off CO2. E2 (dihydrolipoyl transacetylase with cofactor lipoamide) oxidizes hydroxyethyl- to acetyl- and then transfers acetyl- to CoA, forming acetyl-CoA. Acetyl CoA was made in the previous the process is incomplete. The E2is to the acetyl CoA molecule. So, E3 dehydrogenase) oxidizes the thiol dihydrolipoamide back to lipoamide. As

a

side

reaction,

NAD+

step. However, still attached (dihydrolipoyl groups of the

becomes

reduced

to

2.

Describe allosteric regulation of enzyme activity with the help of a suitable example

A patient diagnosed for malaria was administered primaquine. His condition worsened with increased hemolysis. On biochemical laboratory investigation of his blood sample, this patient was found to have an enzyme deficiency. i. ii.

Name the enzyme and the related pathway Explain the biochemical basis _ of primaquin induced crisis in this patient. (Ref: Biochemistry by Satyanarayana 5/e, pg. 275)

ANSWER Introduction

O

NADH. c.

a.

Enzyme involved: Glucose-6-phosphate genase (G6PD) deficiency

dehydro-

Primaquin Induced Crisis O

Primaquine: Antimalarial drug-inhibits liver stages of P. falciparum, P. vivax, and P. ovalae parasites that

ANSWER

O

For answer refer 2011 paper-1, Q. 1(c), Pg. 464

d.

Explain how hypoxia leads to hemolytic crisis in sickle cell anemia

BIOCHEMISTRY

(Ref: Biochemistry by Satyanarayana 5/e, pg. 204)

ANSWER

Role of Hypoxia in Sickle Cell Anemia O

Replacement of glutamate by valine causes decrease in HbS solubility in deoxygenated form.

O

lead to relapses of malaria In G6PD deficiency, deficiency of GSH prevents the defense of erythrocytes against oxidative attack Primaquine causes severe hemolysis and oxidative damage by =" Increasing oxyhemoglobin generation: Forms hydrogen peroxide = GSH depletion = Stimulation of the HMP shunt "

Generation

of

Heinz

bodies:

Insoluble

aggregates that attach on RBC

b.

During prolonged starvation, brain gets its energy mainly from the ketone bodies.

PAPER-| However, a minimum level of glucose is always needed for energy metabolism of brain. Explain.

Mechanism of Action

O

Vitamin K dependent clotting factors require gamma carboxylation of their glutamic acid residues. This carboxylation is essential for the proper functioning of the clotting factors.

O

Gamma carboxylation is catalyzed by vitamin KH2, hydroquinone form of Vitamin K (enzyme Vitamin K reductase).

O

The Gla residues chelate strongly and selectively with Ca2+ (factor IV) and the latter forms ion bridges to anionic phosphate head groups of phospholipid membrane surfaces. Binding at the membrane surface is a critical step in the process of activation of the clotting factors.

O

Warfarin is a vitamin K antagonist. It produces its anticoagulant effect by interfering with the Vitamin K reductase enzyme so that vitamin KO cannot be recycled back to vitamin K. This leads to a depletion of vitamin KH2, thereby limiting the y- carboxylation of the coagulation factors. Factors like prothrombin are not carboxylated, and cannot effectively bind to phospholipid membranes. Its activation by Factor Xa is not effected. Thus blood coagulation is limited.

(Ref: Biochemistry by Satyanarayana 5/e, pg. 386)

ANSWER Starvation

O O O O

State of metabolic stress Due to scarcity of food Imposes certain metabolic compulsions on the organism Biochemical changes occur to maintain adequate supply of energy to the various tissues

Ketone Bodies

O

Acetone, acetoacetate, §-hydroxyl butyrate

O O

Water soluble Energy yielding

During Prolonged Starvation O

O

O

O

Ketone bodies: Fuel source for brain-meets 50-70%

of its energy needs Rest of the demands is met by glucose Glucose during _ starvation is provide by gluconeogenesis in liver Ketone bodies: More important during periods of starvation and diabetes mellitus c.

PART-II 3.

a. Explain the role of insulin in the regulation of fat metabolism in the adipose tissue.

Write a note on the absorption of dietary iron

(Ref: Biochemistry by Satyanarayana 5/e, pg. 644, 654)

(Ref: Biochemistry by Satyanarayana 5/e, pg. 417)

ANSWER ANSWER Introduction

d.

Explain the mechanism of action of the oral anticoagulant, warfarin. (Ref: Biochemistry by Satyanarayana 5/e, pg. 192)

O

Hormone produced by langerhans of pancreas

beta

cells

of

islets

O

Important role in maintain glucose levels in blood.

Effects of Insulin on Fat Metabolism

O

Adipose tissue: Sensitive to insulin

O

Reduces release of fatty acids from stored fat

Introduction

O

Decreases ketogenesis

O

Warfarin: Coumarin derivative

O

Effect on Lipogenesis

O

Vitamin K antagonist

O

Vitamin K dependent clotting factors are = Factor II (Prothrombin)

ANSWER

of

="

Factor VII

"

Factor IX

" Increased synthesis of triacylglycerol from glucose: Increased glycolysis and NADPH-from HMP shunt Effect on lipolysis = Decreases lipase activity

"

Factor X

=

O

=" Protein C

Reduces release of fatty acids from stored fat in adipose tissue

=

Protein S

=" Decreased fatty acid mobilization from liver

="

Protein Z

=

Insulin regulates free fatty acids concentration

BIOCHEMISTRY

For answer refer 2017 paper-1, Q. 3(a), Pg. 360

PAPER-I O

Effect on ketogenesis = Decreased HMG CoA synthetase = Decreased ketogenesis b.

O

Feedback Inhibition

=" Hexokinase feedback is inhibited by glucose 6 phosphate. While glucokinase has no direct feedback inhibition.

With the help of a diagram illustrate the pathway of HDL metabolism. Explain the role of LCAT in HDL metabolism.

b.

With the help of a diagram show the sites of ATP synthesis in the electron transport chain. (Ref: Biochemistry by Satyanarayana 5/e, pg. 225)

ANSWER ANSWER

For answer refer 2012 paper-1, Q. 5, Pg. 453

Introduction

a.

Compare and contrast the tissue distribution, catalytic properties and kinetic properties of hexokinase and glucokinase.

O O

OO

4.

ANSWER

Also known as respiratory chain Electrons are transported from NADH to electron Carriers Located in the inner mitochondrial membrane Consists of "

=" Mobile carriers: co-enzyme Q and cytochrome C

Difference Between Hexokinase and Glucokinase

O.

Site of Action =" Glucokinase is present only in liver while hexokinase is present in all tissues of the body including liver. = Normal cells do glycolysis to get the energy so they are having hexokinase which is having lower value of Km which results in higher affinity of the enzyme towards glucose to do faster and efficient glycolysis. =

On the other hand liver cells have avery important function which is to first convert glucose in glycogen first and to store it. To make glycogen efficiently liver cells have to slow down glycolysis so they have an enzyme glucokinase with higher value of Km which results into slower and lesser glycolysis.

Activity

=" The activity of glucokinase depends upon the nutritional stat of glucose. Means it only becomes active when quantity of glucose is present in liver.

BIOCHEMISTRY

=" While activity of hexokinase doesn’t depend on nutritional state of glucose. Means hexokinase remains active even at low glucose levels. Km value =" Glucokinase has a high km value while hexokinase has a low km value. Allosteric Enzyme =" Hexokinase is an allosteric enzyme _ while glucokinase is not an allosteric enzyme. Induction by Insulin =" Hexokinase is not induced by insulin glucokinase is induced by insulin.

4 multiprotein complexes: |, II, III, IV

while

Components of ETC O

ETC Complex-1 = Also known as NADH-CoQ dehydrogenase complex =" NADH: Donor of electrons

reductase or NADH

" FMN- acceptor of electrons and forms FMNH, "

Electrons from FMNH,

are transferred to Fe-S,

and further to Coenzyme Q Energy released: 12kcal/mol-utilized to drive protons out of mitochondria and 1 ATP is generated Complex II or Succinate-Q-Reductase = Electrons from FADH, enter ETC at CoQ = No ATP is produced. " FADH, produces 2 ATPs while NADH, produces 3 ATPs Co-enzyme Q = Accepts a pair of electrons from NADH or FADH, through complex-I or II, respectively Complex III or Cytochrome Reductase = Cluster of iron-sulphur proteins, cytochrome b and cytochrome cl, both contain heme prosthetic group =

During

transfer

of

electrons,

iron

shuffles

between Fe* and Fe* forms = ATP is synthesized Cytochrome = Contains one heme prosthetic group = Cytochrome C transfers electrons from complex III to complex IV Complex IV or Cytochrome oxidase = Contains cyt a and cyt a3 =" 4 electrons are accepted from cyt c and passed to molecular oxygen =

1 ATP is synthesized

PAPER-|

2009

Substrate



NAD

—>

FMN

>

CoQ

—>

Cytb

—>

u

J

Cyta, uy

ATP (site 1)

ATP (site 2)

ATP (site 3)

PART-III 5.

O

Describe the sequence of reactions involved in the removal of amino groups from the amino acids leading to the formation of ammonia. (Ref: Biochemistry by Satyanarayana 5/e, pg. 335)

ANSWER Introduction

O O

Removal of amino groups from amino acids leading to the formation of ammonia - Deamination Liberated ammonia is used for urea synthesis

"

Cytc,

>

Cytc

>

Cyta

>

>

O,

2types ="

Oxidative deamination

="

Non oxidative deamination

Oxidative Deamination

O O O

Coupled with oxidation Site: Liver and kidney Glutamate dehydrogenase = Glutamate: Collection centre for amino groups =" Amino group is_ transferred to alpha: Ketoglutarate to form glutamate = Glutamate undergoes oxidative deamination to liberate ammonia =" Enzyme: Glutamate dehydrogenase

Oxidative deamination occurs primarily on glutamic acid because glutamic acid was the end product of many transamination reactions. Glutamic acid and oxidative deamination

NADH +H

NAD

ue)

HO Oxidative deamination

O

OO

II

lll

O NH, O I| II OH-C-CH,CH,-CH - C-OH

R-C-CH,CH,-C -C-OH Alpha-ketoglutaric acid

Glutamic acid ~

a

NH, O |

O

Transamination

I

ene ll

il

R-CH-C-OH

R-C-C-OH

Any amino acid

Keto acid

Oxidative deamination by amino acid oxidases = An amino acid is converted into the corresponding keto acid by the removal of the amine functional group as ammonia

= The amine functional group is replaced by the ketone group. = The ammonia eventually goes into the urea cycle

> oO

D

=

Li

LL

O

OCO

PAPER-I L-amino acid oxidase

a-keto acid + NH,

LN

L-amino acid

ANSWER Introduction

FMN

FMNH,

O O

Glycogenesis: Synthesis of glycogen from glucose Site: Cytosol

Pathway of Glycogenesis

H,0,

O

1/2 O,

Synthesis of UDP-Glucose =

Glucose is Phosphate.

phosphorylated

into

Glucose-6-

=" This reaction is catalyzed by Hexokinase Muscle and Glucokinase in Liver. =

Glucose: 6-P is converted to Glc-1-Phosphate in a _ reaction catalyzed by the enzyme Phosphoglucomutase

=

Glucose: 1-P reacts with Uridine triphosphate (UTP) to form the active nucleotide Uridine diphosphate Glucose (UDP-Glc). The reaction is catalyzed by the enzyme UDPGlc Pyrophosphorylase

Non Oxidative Deamination

O

Without oxidation.

O

Amino acid dehydrases "

Serine,

threonine

hydroxy amino deamination

homoserine

homoserine

acids - undergo

- by =" Enzyme (dehydratases). Serine threonine

and

PLP-dependent

are

the

non oxidative

dehydrases O

Dehydratase

>

YY

— , .

.

=

a-keto acids

In absence of pre-existing glycogen glycogenin-acts as primer

fragment,

= Primer: Accepts glucose from UDPG

Amino acid desulfhydrases =" The sulfuamino acids, namely cysteine and homocysteine undergo deamination coupled with desulfhydration to give keto acids. Cysteine

Requirement of primer to initiate glycogenesis =" Pre-existing glycogen fragment: Acts as primer

NH, O

in

O

Glycogen synthesis by glycogen synthase =

Glycogen synthase, the enzyme transfers the Glucose from UDP-Glc to the non-reducing end of Glycogen to form alpha 1,4-linkages.

=" Glycogen

Desulfhydrases ~~

>

Pyruvate

linear

synthase

unbranched

catalyze the synthesis of a molecule

with

alpha-1,4-

glycosidic linkages. NH, + H,S

O

O

Deamination of histidine = The enzyme histidase acts on histidine to liberate NHS3 by a non-oxidative deamination



2.

a.

Eggs

="

Milk

="

Beans

Biochemical Functions

O

O O O O O

O

Essential component of several enzymes carbonic anhydrase, alkaline phosphatase, alcohol dehydrogenase Antioxidant - zinc containing enzyme superoxide mutase protects against free radical damage Storage and secretion of insulin Maintenance of normal serum vitamin A levels Wound healing - zinc enhances cell growth and stabilizes cell membranes Taste sensation - zinc containing protein - gusten responsible for taste sensation Reproduction b.

What is BMI? Give its clinical importance.

ANSWER

Clinical obesity is measured by O O

Significance of Uncoupling O O O

="

O

Give the biochemical significance of zinc in the body (Ref: Biochemistry by Satyanarayana 5/e, pg. 421)

Body mass index (BMI) BMI (kg/m‘*) is calculated as weight in kgs divided by (height in meters)? =" Reference Range for BMI * 18.5-24.9 kg/m?’- healthy * 25-30 - grade I obese * > 30- grade II obese or clinical obesity * >40 - grade III obese or morbid obesity Waist hip ratio - more effective =" Ratio between waist size and hip size =

Malate + NADP

ANSWER

O

of

Aspartate

Acetyl CoA

Pyruvate carboxylase

Citrate bene eeeeeeee > Fatty acids, sterols

Oxaloacetate

Citric acid

cycle Malic enzyme

Pyruvate *



Malate

a-Ketoglutarate

\ Succinyl CoA

>

v Heme

oO

Glutamate

D

=

Li

v Non-essential amino acids purines

LL

O

OCO

PAPER-I Significance of Anaplerotic Reactions O.

O

The citric acid cycle = Crucial cycle ="

Center of metabolism,

= Major source of ATP production O Socell needs to regulate concentrations of citric acid cycle intermediates in the mitochondria. O Anaplerotic reactions thus play an important role Anaplerotic flux balances cataplerotic flux in order to

="

retain homeostasis of cellular metabolism

Serum

creatinine

and

serum

creatine

are

diagnostic test for assessing the renal function.

PART-II 3.

Biosynthesis of creatine = Glycine, arginine and methionine are required for creatine synthesis = Arginine transfers guanidine group to glycine to form guanidinoacetate (glycocyamine) under the effect of enzyme arginine-glycine transamidinase = Active methionine donates its methyl group to glycocyamine to form creatine = Creatinine is an anhydride of creatine.

b.

a. Enumerate the biochemical functions of glycine in the body and explain any two in detail.

With the help of a suitable example describe the mechanism of action of a hormone acting through phosphatidyl inositol (PI) pathway hormone.

(Ref: Biochemistry by Satyanarayana 5/e, pg. 343)

(Ref: Biochemistry by Satyanarayana 5/e, pg. 4321)

ANSWER

ANSWER

Introduction

For answer refer 2011 paper-1, Q. 3(b), Pg. 467

00000 0

Glycine Non essential amino acid Optically inactive Glycogenic amino acid Most abundant Excreted in urine Commonest amino acid in proteins Synthesized from = Serine by enzyme serine hydroxymethyl transferase = Threonine by enzyme threonine aldolase

4.

(Ref: Biochemistry by Satyanarayana 5/e, pg. 632)

ANSWER Introduction Antioxidant

O

Any substance that delays or inhibits oxidation of another substance.

O

Oxidation leads to the formation of free radicals that

Biochemical Functions Of Glycine

O

BIOCHEMISTRY

O

O

Formation of purine ring =" Glycine is utilized for the position 4 and 5 of carbon and position 7 of nitrogen of purines Synthesis of Glutathione = Glutathione - tripeptide - requires glutamyl, cysteinyl and glycine for its formation Synthesis of heme = Glycine combines with succinyl CoA to form delta-amino levulinate - serves as a precursor for heme synthesis. = Enzyme - ALA synthase Conjugation reactions =" Glycine acts as conjugating agent in two important reactions * Glycine conjugates bile acids - cholic acid and Chenodeoxycholic acid to form glycocholic acid and glycochenodeoxy cholic acid respectively * Glycine conjugates benzoic acid to form hippuric acid - important in detoxification

O O

are harmful to the biological tissues. So to prevent the free radical damage, antioxidants are required. These are scavengers of free radicals

Vitamin Antioxidant Vitamin E

0000 0

O

What are antioxidant vitamins? Briefly explain the biochemical role of any one.

Tocopherol - alpha - tocopherol Fat soluble vitamin Biologically most active Present in all cellular membranes Protection against lipid peroxidation Directly acts on oxyradicals and serves as important chain breaking antioxidant

Vitamin C

O

Water soluble vitamin

O00 0

PAPER-| Present in biological fluids Efficiently scavenges free radicals Inhibits lipid peroxidation Promotes alpha tocopherol regeneration (from alpha toco-pheroxyl radical produced during

scavenging of ROS) Carotenoids

ANSWER Introduction

O O O O

Also known as renal rickets Seen in renal failure patients Defective bone development Alteration in bone morphology

O

Provitamin A

Cause

O O

Has lipophilic compounds Beta - carotene - most important = Acts as antioxidant under low partial pressure

O O

= Acts in association with vitamin C and vitamin E

Biochemical Basis

O

Lycopene

= Fat soluble pigment = Has antioxidant property

O

Failure of vitamin D to convert into calcitriol

O

Decreased calcitriol levels

O

Increased excretion of calcium through the kidney Decreased reabsorption This causes hypocalcaemia

Alpha-Lipoic Acid Vitamin like compound Plays a role in recycling other antioxidants such as ascorbic acid, alpha - tocopherol and glutathione

PART-III 5.

Whatare ketone bodies? Describe their synthesis. How is ketogenesis regulated in the body? Mention the common causes of ketosis.

O O.

and

phosphate

Signs and Symptoms O O00

O O

Damage in renal tissue causes defective calcitriol synthesis Also known as vitamin D resistant rickets

Multiple bone fracture Bone pains Joint pains Deformity b.

Active transport

(Ref: Biochemistry by Satyanarayana 5/e, pg. 296) ANSWER ANSWER

For answer refer 2016 paper-1, Q. 1(a), Pg. 376

For answer refer 2010 paper-1, Q. 3(a), Pg. 479

6.

Write short notes on: a.

Renal rickets

(Ref: Biochemistry by Satyanarayana 5/e, pg. 410)

> oO

D

=

Li

LL

O

OCO

Your Roll No. ...............

Name of the Paper

:

Biochemistry Paper-II

Name of the Course

:

MBBS-2007

Semester

Annual

Time: 3 Hours

M.M.:

50

INSTRUCTIONS Write your Roll No. on the top immediately on receipt of this question paper All questions are to be attempted

Attempt parts of a question in sequence

PART-I a.

Compare and contrast the following structural and functional difference between mitochondrial and nuclear DNA.

[10] b. c. d.

Apoptosis and Necrosis Laboratory tests which aid in differentiating hepatic and post-hepatic disease. TandB cells with respect to antigen recognition.

PS

Write briefly on:

aoSF

a

Answers to Parts I, II and III should be written in separate answer sheets provided

[10]

DNA binding motifs Post-translational modifications

Tumor markers and their significance Regulation of gene expression by RNA editing, with the help of an example.

PART-II a.

Enumerate the different modes of transport across biological membrane. Discuss glucose transport in detail.

b.

Discuss the biological consequence of complement activation which aid in elimination of foreign pathogen.

What is RFLP? How can it be used in molecular diagnosis of disease?

[10]

[5]

PART-III How is fidelity of DNA replication maintained? Enumerate mechanisms in the body? Explain any one in detail.

the causes of DNA damage. What are the repair [10]

OR What are vaccines? Discuss the different types of vaccines, giving their advantages and disadvantages. Write short notes on:

a.

Molecular chaperons

b.

Renal function tests

[5]

PAPER-II

2007

|

2007 PAPER-II

PART-I 1. a. Compare structural

and_ and

contrast functional

the

following

difference

between

mitochondrial and nuclear DNA.

Parameter

Hemolytic jaundice | Obstructive jaundice (prehepatic jaundice) | (posthepatic jaundice)

Urobilinogen in urine

Excretion tT

Alkaline phosphatase; 7: Inc

ANSWER DNA

Nuclear DNA

Location

Mitochondria

Cell nucleus

Structure

Circular and closed

Linear and open ended

Genome size

Gene encoding

1 chromosome with

46 chromosomes

16,569 base pairs

with 3.3 billion base pairs

37 genes

20,000 — 25,000

Some codons donot _ follow universal codon pattern

Follows universal codon pattern

Transcription

Polycistronic

Monocistronic

Genome inheritance

Maternal

Maternal and paternal

Mutation rate

Higher than nDNA

process

O

O

O O O

=

Apoptosis and Necrosis (Ref: Biochemistry by Satyanarayana 5/e, pg. 8)

ANSWER

=

For answer refer 2009 paper-2, Q. 1(c), Pg. 497

c.

Laboratory tests which aid in differentiating hepatic and post-hepatic disease. (Ref: Biochemistry by Satyanarayana 5/e, pg. 457)

ANSWER Hemolytic jaundice | Obstructive jaundice (prehepatic jaundice) | (posthepatic jaundice)

Serum bilirubin

Unconjugated bilirubin 7S

Conjugated bilirubin7

van den Bergh reaction

Indirect positive

Direct positive

Serum enzymes

ALT, ASTand ALP>

Bilinubin in urine

Not excreted

ALP 7%, ALT and AST margin 7 Exceted

to

antigen

Class I * Antigen is usually endogenous (e.g. viral proteins). * CD8* cytotoxic T lymphocytes (CTLs) recognize antigen in association with class I MHC gene product on APC. Class II Molecules * Antigen is usually extracellular. * CD4* Helper T Lymphocytes recognize antigen in association with class II MHC gene product on APC.

B Cell Antigen Recognition O

Parameter

respect

Recognize antigen peptide fragments bound to specialize cell surface molecules on antigen presenting cells (APC) Molecules are encoded by major histocompatibility complex Peptides are displayed to T cells as peptide: MHC complexes T cell antigen receptors recognize peptide: MHC complexes Each MHC molecule can bind numerous different peptides Two classes of MHC molecules

orl

ALT: Alanine transaminase: AST: Aspartate transaminase: ALP:

d.

Mitochondrial

|

O O

2.

Cell surface immunoglobulin receptor or B-cell receptor (IgM and IgD) Antigen contact initiates B-cell activation, clonal expansion, maturation to plasma cell Antigen receptor is identical to immunoglobulin that will ultimately be produced Write briefly on: a.

DNA binding motifs (Ref: Biochemistry by Satyanarayana 5/e, pg. 566)

> oO

D =

Li

LL

O

OCO

PAPER-II O O

ANSWER

This segment forms helix: Loop-helix motif Binds to DNA

Introduction

OO

O O

O

Motif: Dominant element DNA binding motif: Specific control of transcription occurs when specific regulatory proteins bind them. Part of protein Protein DNA interactions occur by 4 unique motifs =" Helix-turn-helix (HTH) = Zinc finger = Leucine zipper = Helix-loop-helix (HLH) The DNA binding motifs are maintained by = Hydrogen bonds = Van der Waals forces

b.

Post-translational modifications

(Ref: Biochemistry by Satyanarayana 5/e, pg. 557) ANSWER

For answer refer 2010 paper-2, Q. 2(d), Pg. 486 c.

Tumor markers and their significance

ANSWER

For answer refer 2017 paper-2, Q. 2(d), Pg. 370 d.

Regulation of gene expression by RNA editing, with the help of an example.

Helix-turn-helix Motif

O O O

A small domain part of large protein that interacts with DNA About 20 amino acids long Examples = Lactose repressor =" Cyclic AMP catabolite activator protein

Zinc Finger Motif O O O O O

Transcription factor TFIUIA Contains zinc ions Complex is formed by closely spaced cysteinecysteine pair followed by histidine-histidine pair Binds to major groove of DNA, contacting 5bp of DNA Used by steroid hormone receptors transcription factors

Leucine Zipper Motif O

O O O

Rich in amino position

acid: Leucine-present at every 7"

Allowing two identical monomers or heterodimers to zip and form dimeric complex Dimer interacts with DNA Example: Enhancer binding proteins-fos and jun

Helix-loop-helix Motif

BIOCHEMISTRY

O

Formed by two amphipathic alpha helical segments of proteins

(Ref: Lippincott’s 5/e, pg. 457, 458) ANSWER For answer refer 2015 paper-2, Q. 5, Pg. 405

PART-II 3.

a.

Enumerate the different modes of transport across biological membrane. Discuss glucose transport in detail.

ANSWER

Introduction Biological membranes are: O O O

Composed of hydrophobic bilayers Semi- permeable Lipid soluble readily pass through the membranes

O

Impermeable to water soluble molecules

Transport Across Membranes

O

Occurs by 3 mechanisms =" Passive diffusion: depends on concentration gradient = Facilitated diffusion = Active transport-with the use of ATP

PAPER-II High Carrier protein

Concentration

Membrane

gradient

Low

Passive diffusion

Facilitated diffusion

Active transport

f Passive transport

Mechanism of transport across membrane

Mechanism of Facilitated Diffusion—PING PONG

Facilitated Diffusion

Diffusion is facilitated by carrier/transport proteins Occurs along the concentration gradient Solutes moves from higher concentration to lower concentration No energy is used

O O

OO

O

Carrier or transport proteins

used

in facilitated

MODEL

O

Transport

protein

occurs

in two

forms-ping

and

O

pong Inpong conformation: It is exposed to the side with higher solute concentration.

Othe solute binds to the specific sites at the carrier Upon

binding with the solute, the carrier protein

diffusion are

O

= Glucose =" Galactose

undergoes conformational changes (ping form) Othe solute is thus released on the other side of the

= Phenylalanine etc.

0

Po Oo

Oo

0

3

O

—_——_———>

UO

00 Pong

Ping Diagrammatic representation of ‘Ping-Pong’ model of facilitated diffusion

Example of Facilitated Diffusion

2 Types

O O

e)

Primary active transport system- e.g. ion pumps

O

Secondary active transport symport and antiport system

Hormone play a role in facilitated dil ‘fusion Insulin = Increases glucose transport in muscle and adipose tissue = Amino acid transport in liver.

Works against the concentration gradient

O O

Dependent on ATP Carrier mediated

e.g.

uniport,

D

Li

O

P High intracellular K* concentration concentration is required in cells

O

High K* concentration is needed for glycolysis and protein biosynthesis

oO

=

Na*-K* Pum

Active Transport

O

system

>

LL

and low Nat

O

OCO

PAPER-II

OO

O

O

Nat and K* transmission

gradients

are

required

for

nerve

Na* K* pump maintains this gradient. Enzyme: Na’* K* ATPase Structure of Na* K* pump = 2 alpha (qa) units = 2 beta (fs) units

the repair mechanisms in the body? Explain any one in detail. (Ref: Biochemistry by Satyanarayana 5/e, pg. 536)

ANSWER For answer refer 2014 paper-2, Q. 5, Pg. 430

= Represented as (a),

O O

OR What are vaccines? Discuss the different types of vaccines, giving their advantages and disadvantages.

=" Pumps 2K‘ ions inside the cell and 3Na* ions outside the cell Inhibited by steroid derivative: Ouabain and steroid glycoside Digoxin Na* cotransport system: Amino acids and sugars are co transported with the movement of Na* K*

(Ref: Biochemistry by Satyanarayana 5/e, pg. 588) ANSWER

pump 3Na

+

2K

+

For answer refer 2016 paper-2, Q. 5(or), Pg. 390 Extracellular

f

Na-K’ pump

6.

Write short notes on:

a.

Membrane

Molecular chaperons (Ref: Biochemistry by Satyanarayana 5/e, pg. 556)

Intracellular

ADP + Pi

ATP 3Na

+

2K

ANSWER For answer refer 2017 paper-2, Q. 6(b), Pg. 373

b. Na-K"

b.

pump

Discuss the biological consequence of complement activation which aid in elimination of foreign pathogen. (Ref: Biochemistry by Satyanarayana 5/e, pg. 699)

ANSWER

(Ref: Biochemistry by Satyanarayana 5/e, pg. 461) ANSWER Introduction O

Toassess the renal function

O

2/3rds of the renal tissue needs to be damaged to show any abnormality in tests. 4 groups of tests =" Glomerular function tests =" Tubular function tests = Analysis of blood/serum

O

For answer refer 2014 paper-2, Q. 1(b), Pg. 425

4.

What

is RFLP?

How

can it be used in molecular

=" Urine examination

diagnosis of disease?

BIOCHEMISTRY

(Ref: Biochemistry by Satyanarayana 5/e, pg. 583)

ANSWER For answer refer 2013 paper-2, Q. 4(a), Pg. 443

Glomerular Function Tests

O

Measures the glomerular filtration rate

O O

Most useful Concept: Quantity of substance excreted can be measured quantitatively Clearance: Refers to volume of plasma required to completely eliminate the substance per minute.

O

PART-III O

5.

Renal function tests

How is fidelity of DNA replication maintained? Enumerate the causes of DNA damage. What are

Measured as: C=UxV/P

C = clearance

PAPER-II U = concentration of substance in urine

Urine Examination

V= volume of urine excreted per minute P = concentration of substance in serum

O

Volume

O

pH

Clearance tests:

O O

Specific gravity Osmolarity

="

Creatinine clearance test

=" Urea clearance tests O

Inulin: Is used to measure clearance tests

Choice of Renal Function Tests Starts with routine urine examination

Tubular Function Tests O

Measures the specific gravity of urine

O

Urine concentration measurement

O

Normal: 1.020 in early morning sample.

Analysis of Blood

OO

0

O

To assess overall kidney function Serum urea Measurement Serum creatinine measurement: Also assess GFR Cystatin C: Biomarker of kidney function

L Serum creatinine examination

y Blood examination

L GFR/tubular tests

> oO

D

=

Li

LL

O

OCO

ory

orl

noe

Extra Edge Subject-wise Spotters, Important Tables, Important MCQs, and Clinical Bridge with Answer Keys

& ) Important Tables

Important MCQs

(8)

Clinical Bridge

‘a



Ss

This whole section is the compilation of the important contents from CBS Exam books as follows:

K Raviraj, VD Agrawal New SARP Series for NEET/NBE/Al Anatomy

S Krishna Kumar New SARP Series for NEET/NBE/AI

CRISP Complete Review of Integrated System

Physiology Lomoereeninre Ucioen’ Bent ot Bx hesne ect Mow ee

ianc=erve eer

/Conceptual

Sgpme | Cosening Concepe, Tory ~~ yee om amy Roky

Review

of

Biochemistry

Smily Pruthi Pahwa Conceptual Review of Biochemistry

Along

With

High

Yield

f

nevieg €

Ranjan Kumar Patel & Sudhir Kumar Singh Review of NEET & DNB Pattern Qs 2018; 1st Edition

rs

at

tor

I ot

Be

nute

Revis

&

NHN S58 Covering 600 Qs with Explanations 6 800+ High Yield Tables for Late we’

Anatomy Spotters MUSCLE Types of muscles and their features

Location

Mostly in limbs, trunks

In the wall of viscera and blood

Heart

vessels Connective tissue

Encloses the muscle as epimysium, perimysium and endomysium

Organized as only endomysium

Organized as only endomysium

Fiber length

Very long

Up to 15-200 nm

50-100 nm

Width

Wide fiber

Small

Small

Striations

Transverse striations prominent

Not seen

Faint transverse striations

Nucleus

Peripheral and multinucleated

Central and single

Central and single

Shape

Cylindrical

Spindle shaped

Short cylinders with branches

Junctional complexes

Nil

Gap junctions

Intercalated disc with desmosome

Nerve supply

Cranial and spinal nerves

Autonomic system

Autonomic system

Activity

Voluntary contraction strong, discontinuous Quick voluntary contractions

Slow weak involuntary contractions

Quick, strong continuous involuntary contractions

SKELETAL MUSCLE Multiple peripheral nuclei Peripheral nuclei

Capillary Endomysium

Dark and light bands

| band with Z line A band with H line | band with Z line

Sarcomere @ INSET

@

ELASTIC CARTILAGE Anatomy

Longitudinal section

“3

Transverse section

Elastic fibers

al

Chondrocytes

Longitudinal

section Fibrous layer of perichondrium Cellular layer of perichondrium

AOE —k

TA

DAY 52 Gas

Zona eh) Osan i \o/

Transverse section Chondrocytes in lacuna Elastic fibers

O

CARDIAC MUSCLE

Perinuclear space

Itis present in pinna (external ear), epiglottis, external auditory meatus, part of arytenoid, corniculate and cuneiform cartilages.

O

Chondrocytes are larger than those ofhyaline cartilage and present singly or in small groups.

O

The ground substance contains abundance branching and anastomosing elastic fibres.

O

Perichondrium is present and is composed of an outer fibrous and an inner chondrogenic and vascular layer.

O

Cells: Chondroblasts and chondrocytes

Intercalated disc

Ground substance: Glycoprotein—the chondromucoprotein

Branching muscle fibers Perinuclear

space

Capillary

Intercalated disc

Cross striations

This type of cartilage has elasticity i.e. it comes back to its natural size after being stretched, i.e. it is highly flexible.

O

O00 9

Awojeuy |¥/39N/LIIN 404 Salas duYVS MeN ‘Femessy GA ‘fesaey >

Perichondrium

Calcification: Does not occur Perichondrium seen all around Ground substance appears homogeneous Chondrocytes in lacuna lie in groups of 2-4 cells

of

O O

O O

Bone is covered by periosteum consisting of outer fibrous layer and inner osteogenic and vascular layers

O

Trabeculae extend into the substance of the spleen.

O

COMPACT BONE

Parenchyma of spleen consists of red pulp and white |

pulp O

Collagen fibers from this layer penetrate into outermost lamellae of bones nailing the two together termed sharpey’s fibers Characteristic feature of compact bone is haversian system or osteon Each haversian system comprises: =

Oo

=

bundles of collagen, elastic fibers and reticular fibers with

=

fibroblast,

reticular

cells

and

Splenic sinusoids are surrounded by flattened, elongated modified endothelial cells called as stave cells—banana shaped cell

and

lymphatics.

The

Interstitial lamellae lie in the angles adjoining haversian lamellae

=

lymphocytes,

vessels,

nerves

haversian

O

White pulp: Aggregations of T and B lymphocytes that surrounding the arteries (periarteriolar lymphocytic sheath). White pulp also contains some antigen presenting cells.

Malpighian corpuscle outlined

=

oO

2 > Arteriole

White pulp

with

Trabecula Germinal center

Red pulp Splenic cords Venous

Periosteum

B

sinus

A

ossers)

;

Haversian

lamellae

THYMUS O

O

Septum arising from capsule incompletely divides the parenchyma into lobules Connective tissue septum separates the lobules.

O

Interstitial lamellae

SPLEEN

Each lobule contains outer darkly stained cortex, in which densely packed lymphocytes and inner lightly

O

stained medulla, in which the cells are diffuse.

Covered by capsule and trabeculae arising from the capsule.

© = Wn > 0 ao) % oO) _ ©a)



malpighian corpuscle

~—}+~——~ Haversian | canal

=

m m

Capsule

y

>

ga o) = a

=

lamellae

Circumferential lamellae

Oo

mn"

— Haversian

lamellae




between

Interstitial

~ rs) of)

=

Bone marrow absent

reversian -

=

2.

macrophages

Canaliculi from adjacent lacunae communicate with each other Haversian canals are connected with one another and communicate with marrow cavity via Volkmann’s canal

=

Splenic cords of billroth: Consists of anastomosing

Centrally situated haversian canal containing fine

Haversian lamella or matrix is composed of collagen fibers and deposited calcium salts Between lamellae or on the lamellae are spaces/ lacunae imprisoning the osteocytes.

=

Red pulp: Consists of splenic cords of billroths and sponge like sinusoidal space, through which red cells are passed. These phagocytic cells lining the sinusoid engulf the broken red cells and other debris.

canal is surrounded by 6-12 concentric lamellae =

Anatomy

O

Medulla consists of lymphocyte and more reticular cells and also Hassall’s corpuscles.

>

S ®ot fe) 3


non-keratinized

thymic

barrier:

Protects

the

newly

generating lymphocytes from antigen present O

Six types of epithelioreticular cells present in thymus

named from I to VI O O

Lymphatic nodule with germinal center

Capsule with

_Hassall's corpuscles contain type VI epithelioreticular

B |

Sos

trabecula

cells

O

Blood thymic barrier contains type 1 epithelioreticular cells

Underlying lamina lymphatic nodules

O

These lymphatic nodules contains centre surrounded by peripheral lymphoid aggregates

O

Beneath the capsule few skeletal muscle fibers seen

Capsule Interruptd cortex of lobule Hassll’s corpuscle

Medulla

propria

contains

numerous

pale germinal dark staining

(belongs to superior constrictor) O

Tonsil have no afferent lymphatic vessel only efferent vessel drain into deep cervical nodes

PAROTID GLAND

Capillary

Cortex

Le)

O

The acini of the gland secrete enzymes.

O

The acinus is rounded and is lined by pyramidal cells surrounding a very small lumen.

Interrupted cortex

O

The cells show basal basophilia and lighter apical portion.

Connective tissue of trabecula

O

The nuclei are rounded and basal in position.

O

With higher magnification, the active cells show basal striations and apical eosinophilic zymogen granules.

ee

Medulla continous between lobules “f)]

Thymic/ Hassall’s corpuscle

rHassall’s corpuscle

Fat cells

Serous acinus

©@ INSET @

Interlobular connective

TONSIL O

Mucous

tissue

membrane

covered

by _ nonkeratinized

stratified squamous epithelium O

Epithelial lining invaginates into underlying connective tissue forming numerous grooves called tonsillar crypts Interlobular

Stratified squamous non-keratinized epithelium Crypt

Lymphatic nodule

cues

Intercalated

duct

aiadieaaeaainian Intralobular

Serous acini

Fe eD

®& \—

interlobular

\ Se

connective tissue

Cm

&

®@ INSET &

SUBMANDIBULAR GLAND O

These glands consist of both serous and mucous acini.

O

The mucous acinus is lined by truncated columnar cells. The size of the acinus is larger than the serous one and shows a bigger lumen.

O O

O

O

The

nucleus

is

flattened

against

the

(PAS) stain, the cytoplasm takes up magenta colour. O

Demilunes of Giannuzzi are present Compares the serous and mucous acini.

here

A

basement

pe]

Light stained

Between mucous cells and the basement membrane are half-moon shaped polyhedral granular serous cells. These cells are known as demilunes of Giannuzzi.

mucous acinus

cells

=

ga =

pe8)

Interlobular connective tissue with ducts

Fine canaliculi pass between mucous secreting cells from the demilunes to reach the lumen of the acinus. The secretions of the demilunes make mucous less viscid. or basket

= A

membrane.

A few myoepithelial

also.

Anatomy

= —_ =

M

=

7a)

>

2) uU 72)

ie?)

are present

= MD 7)

between the basement membrane and the acinar cells

-—>

Demilune of

which help to squeeze the secretion from the acinus.

Giannuzzi

Striated duct

12) mn

= m Mm



=

Mucous acinus Intralobular

Serous scinus

duct

Intercalated duct

Adipose cells in interlobular connective tissue Interlobular duct

Demilune of Giannuzzi

16)

a > > >

ry)

ot

Oo

3






«536.

epithelium with microvilli

Lamina propria

Subserous coat

Cut section of fold of mucous membrane Smooth muscle and elastic fibers Venule

B | Serous coat

Arteriole

Fibromuscular coat

The lamina propria has loose connective tissue with diffuse lymphocytes as well as cut sections of folded epithelium. The muscularis mucosae and glands are absent. Outside the lamina propria is fibromuscular coat with a few smooth muscle fibres and collagen fibres. This layer rests on a thin fibroareolar coat containing numerous small blood vessels. The serous coat covers it on its inferior surface whereas thin adventitia envelopes the rest of the surface. At the neck one of the folds is prominent which is spiral in shape.

Crypt of Lieberkunn

PANCREAS O

Lamina propria

Capillary i

the villus Transverse section of

Transverse section of villus Goblet cell in villus

O

Consists of Exocrine gland and Endocrine gland It looks like serous salivary gland and differentiated from the latter by presence of islets of Langerhans.

(Secreting unit is called Acini)

Crypt of lieberkGhn Lamina propria

Lymph nodule Muscularis mucosae

Centroacinar Cells O O

Present at the junction of acini and intercalated duct Function: They add bicarbonate ion to pancreatic juice.

GALLBLADDER O O O O O

The gallbladder lies on the undersurface of the liver. Its capacity is 30 to 60 millilitres and it concentrates bile to one-tenth of its amount. Itsmucous membrane is thrown into temporary folds. The epithelium consists of a single layer of tall columnar cells. Each cell has basal oval shaped nucleus, and an apical part having a brush border due to the presence of microvilli. Columnar epithelium with microvilli

Islet of langerhans

Serous acini

Serous acini Islet of

Striated duct

Langerhans Adipose tissue

Fibromuscular coat Small duct Subserous coat

A! Serous coat

ANATOMY aa

PROSTATE

Medulla

O O

Prostate is fibro muscular glandular structure Glandular part consists of 30-50 tubuloalveolar

O

glands.

O

Medullary

O

between the medullary cords. Each medullary cord contains lymphocytes, macrophages, dendritic and plasma cells.

O

Alveoli of the gland shows large, irregular lumen with simple columnar epithelium Lumen of the alveoli contains secretion of prostate which becomes calcified known as corpora amylacea a f t e component fibromuscular of Stroma consists Fibrous part made up of collagen fibers and muscular part consists of smooth muscle fibers

O O O

Medulla contains lymphocytes arranged in the form of anastomosing cords called medullary cords. sinuses

occupy

the

spaces

present

in|

Capsule Caitical Wodules

< 7

Medullary Acinus of

x P*) pe) Ss. = &.

=~

in cortex cords in medulla

cae @= |

Hilum

the gland

< = e) a

uU

Fibromuscular

o =

stroma

cN

3 wn

Z m m

—~ A Germinal

centre

Smooth

fibers

Acinus lined by columnar epithelium

Glandular

. Cortical

epithelium

nodule

stroma

ee)

Subcapsular

>—

Trabecular sinus

Hilum Duct

'B |

~~ Capsule

LYMPH NODES Subcapsular—

O

Covered by fibrous capsule from which trabeculae ; extend into cortex

O

Subcapsular sinus present below the capsule separates the cortex from the capsule

O

Parenchyma of the lymph node is then divided into outer cortex and inner medulla.

sinus

Germinal centre

Kea

Cortex O O

Lymphocytes are arranged in the form of lymphatic follicles . Lymphatic follicle consists of pale germinal center contains large immature cells called lymphoblast and surrounded by thick peripheral cells contain mature

;

Germinal centre wnt ei a

|

Capsule

is ‘

Trabeculum |

with capillary

+= —

lymphocytes. Deep part of cortex contains diffuse

Medullary cord

lymphatic tissue not arranged as nodules. This portion of cortex is known a as paracortex

= Medullary sinus 'D|

m

> mw

=>

fe) 3




Tunica

Ww vu

albuginea

~ Seminiferous tubule

Fa)

©

—, om

Capillary

Ww

Seminiferous

—h

tubule

©

es

Spermatogenic+—ASe

az m m

633 ||

yee

ety oe Interstitial

al az

Sertoli cells ——

Ww

A >

cell of Leydig

Interstitial cells of Leydig

p> > Ww ot

©

3

,

Fine focus

Ww ©) Fa)

fa?)

—, fa) Ww —h

‘@)

om }

az m m

a

az w

au > =)

rS) Fa)

me)

(=) Le)

3 sok fap) ot

fa) Ww

fa?)

3.

wn

ot


wine color

O

Positive test indicate acetoacetate and not acetone

«

Fasting (zero sample)

7)

3.

< vu =ot = v ) > = Le 2) O° 5 oO . Oor c

30 Minutes

60 Minutes

90 Minutes

120 Minutes

150 Minutes

180 Minutes

Blood glucose (mg/dl)

90

100

150

120

110

80

70

Urinary glucose

Nil

Nil

Nil

Nil

Nil

Nil

Nil

Normal glucose tolerance curve

=

»

2D D Ss. © < °“bh 2 °2) > D

160 140 120 100 ©

150

©

Blood glucose (mg/dl)

—-»>

Glucose tolerance curve

wee

0 Mins

30 Mins

60 Mins

90 Mins

120 Mins

»>

3.

Fig.



150 Mins

Glucose

180 Mins

Time (minutes)

Normal GTT

ot


= & a) fe) S Oo 0) Ooct c =

50 ML single-dose

50% Dextrose ® Injection, USP

25 grams/50 ml

a

(0.5 g/mL) Mespira. inc Late Forest, iL 60045 USA

The cataract shown is oil characteristic of galactosemia.

drop

cataract,

These clinical manifestations of liver milk is the diet, are typical of classical Bilateral cataract rules out the possibility disease and hereditary fructose intolerance

which

is

affected when Galactosemia. of Von Gierke’s although other

Solution of glucose is known as DEXTROSE. ‘D’ means structural Isomer i.e. Enantiomer. Abundant form of a carbohydrate is always D- form. (+) means dextrorotatory (d). Glucose is always dextrorotatory (d)(+).

a

p) © Ss. © = fe} ow e)2) > oD)

-—>

3.

~ ot




Quadrilateral Strap-like

Sternohyoid, Sartorius

WN

Strap-like with tendinous

Rectus abdominis

—, fa?)

Saddle

Thumb, sternoclavicular, calcaneocuboidal, incudo-malleus

joint

Ball and socket

Thyrohyoid

Ww ©) fa?)

Knee, TM joints of jaw

Example

az




Triangular

Temporalis, adductor longus

p>

Unipennate

Flexor pollicis longus, Extensor digitorum longus

Bipennate

Rectus femoris, dorsal interossei

Multipennate

Deltoid, Subscapularis

Circumpennate

Tibialis anterior

> Ww ct

‘e)

3

ga = w

Mesoderm

= a

Head (Neural crest)

=

| Paraxial

@

e

Cranium

e

e

Connective tissue

e Striated skeletal muscle (trunk, limbs) e Skeleton except cranium e Dermis of skin, and connective tissue

of head e

Dentin

Muscles of head,

=

7) > y) ae) 7)

@ — io) re)

-—>

12-14

-

Embryonic dis, villi, and yolk sac appear

15-16

-

Primitive streak appears

17-18

-

Neural folds elevate

19-20

1-3

Head fold appears

21-23

4-12

Neural fold fusion begins, heart begins to

e Urogenital system, including gonads, ducts, and accessory glands

e Connective tissue and muscle of viscera e Serous membranes of pleura, pericardium, And peritoneum e Primordial heart e Blood and lymphatic cells e Spleen e Suprarenal (adrenal) cortex

beat

23-25

13-20

21-29

21-29

Arm buds appear, neural tube closed, optic cup

28-30

40

Leg buds appear, heart chambers, lung buds, metanephric bud

30-32

-

Lens invagination, septum primum, gonadal ridge

32-34

-

Lens vesicle closed, external ears becoming recognizable

35-36

-

Eye pigment appears, hand plate, hypophysis, liver

37-40

-

Finger rays, foot plate, ear defined, somites less apparent superficially

40-42

-

Eyelid, finger rays notched, toe rays, nerve plexuses

42-44

-

Head more erect, limbs extend forward,

muscles developing, duodenum closed 45-46 46-48

-

Finger, scalp plexus present, optic nerve, septum secundum Hands meet over heart region, corpus striatum, thalamus, heart valves

48-50

-

Fingers overlap those of opposite hand, duodenum re-opened

50-52

-

Head erect and rounded, scalp plexus reaching head vertex ossification begins

= m m

2

=

@

x

> >

=) pe)

ond Oo

3

Two branchial arches, foregut, hindgut, optic evagination

Oo >




Ww vu WN

fa?)

—, fa?) Fa)

Structures Derived from Pharyngeal Arch Arch artery

Arch skeletal elements

Muscles

Cranial nerve

Terminal branch of

Derived from arch cartilages (originating from neural crest cells): from maxillary cartilage: alisphenoid, Inas From Meckel’s cartilage: malleus Derived by direct ossification from arch dermal mesenchyme: maxilla, zygomatic, squamous portion of temporal bone, mandible) (orinate from neural crest cells)

Muscles of mastication (temporalis, masseter, medial and lateral

Maxillary and mandibular divisions of trigeminal nerve (Vv)

maxillary artery

bh

'e) es

az m m

al az Ww

A >

p> > Ww cot

pterygoids), mylohyoid, anterior belly of the digastric, tensor tympani, tensor veli palatini (originate from head mesoderm)

Stapedial artery (embryonic), caroticotympanic artery (adult)

Stapes, styloid process, lesser horns and part of body of hyoid (derived from the secondarch [Reichert’s] cartilage; originate from neural crest cells)

Muscles of facial expression (orbicularis oculi, orbicularis oris risorius, platysma, auricularis, frontalis, and buccinator), posterior belly of the digastric, stylohyoid, stapedius (originate from head mesoderm)

Facial nerve (VII)

Common carotid artery,

Lower rim and part of body of hyoid (derived from the third arch cartilage; originate from neural crest)

Stylopharyngeus (originates from head mesoderm)

Glossopharyngeal nerve (IX)

Arch of aorta (left side), right subclavian artery (right side); Original sprouts of pulmonary arteries

Thyroid and epiglottal laryngeal cartilages (derived from the fourth- arch cartilage; originate from neural crest cells)

Constrictors of pharynx, cricothyroid, levator veli palatini (originate from occipital somites)

Superior laryngial branch of vagus nerve (X)

Ductus arteriosus; roots

Remaining laryngeal cartilages (derived from the sixth-arch cartilage; uncertain whether they originate from neural crest or mesoderm)

Intrinsic muscles of larynx (except the cricothyroid; originate from occipital somites)

Recurrent

'@)

3

|

System

Stratified squamous

Cranial Nerves —- Components and Function | Components

| Function

Opening in skull

Olfactory nerve

Sensory (SVA)

Smell sensation

Openings in the cribriform plate of ethmoid

Optic nerve

Sensory (SSA)

Vision

Optic canal

Oculomotor

Motor (GSE,GVE)

Turns eyeball upward, downward and medially, Raises upper eyelid, also Constricts the pupil; accommodates eye

Superior orbital fissure

Trochlear nerve

Motor (GSE)

It helps in turning eyeball downward and laterally

Superior orbital fissure

Maxillary division

Sensory (GSA) only

Supplies dura mater of anterior part of middle cranial fossa, conjunctiva of inferior eyelid, Skin of face over maxilla; teeth of upper jaw; mucous membrane of nose, the maxillary sinus, and palate

Passes through Foramen Rotundum to enter Pterygopalatine fossa.

Ophthalmic division

Sensory (GSA) only

Supplies Cornea, Superior conjunctiva, skin of dorsum of — Passes through Superior external nose, forehead, scalp, superior eyelids, and; also _ orbital fissure mucous of nasal cavity, ethmoid, frontal and sphenoid sinuses.

Mandibular division

Sensory

Skin of lower lip, buccal, parotid and temporal regions of face;, external ear (Auricle, tympanic membrane and acoustic meatus); mucous membrane of mouth and anterior two-third part of tongue

Trigeminal

Contd...

553

|

Components

| Function

Openingin skull

Motor (SVE)

Supplies 4 muscles of mastication, mylohyoid, anterior belly of digastric, tensor tympani and tensor veli palatini

Abducent

Motor (GSE)

Lateral rectus muscle turns eyeball laterally

Facial

Sensory (SVA)

Taste from anterior two-thirds of tongue, from floor of mouth and palate

Motor (SVE)

Muscles of face and scalp, stapedius muscle, posterior belly of digastric and stylohyoid muscles

ga =

Secretomotor (GVE)

Submandibular and sublingual salivary glands, the lacrimal gland, and glands of nose and palate



parasympa-

z

thetic

x a pe)

S.

2.


= > x Wn = O = >


(GVE)

Parotid salivary gland

Motor (GVE, SVE) Sensory (GVA, SVA, GSA)

Heart and great thoracic blood vessels; larynx, trachea, bronchi, and lungs; alimentary tract from pharynx to splenic flexure of colon; liver, kidneys, and pancreas

5 So 3

Vagus

Spinal accessory

Hypoglossal

Stylopharyngeus muscle—assists swallowing

Jugular foramen

Jugular foramen

Spinalroot

Motor(SVE)

Sternocleidomastoid and trapezius muscles

Cranialroot

Motor(SVE)

Muscles of soft palate (except tensor veli palatini), pharynx (except stylopharyngeal, and larynx (except cricothyroid in branches of vagus

Jugular foramen

Motor (GSE)

Muscles of tongue (except palatoglossus) controlling its shape and movement

Hypoglossal canal

Important MCQs 1. Which of the following is multipennate muscle:

a. b. c. d.

FPL EPL Deltoid FHL

2. The Haversian system is found in:

a. b. c. d.

Diaphysis of long bones Cancellous bone Epiphysis Spongy bones of children

3. Which of the following is aberrant epiphysis?

a. b. c. d.

Coracoid process Greater tubercle of humerus Base of lst metacarpal Base of 2nd metacarpal

4. Bone which is pneumatic:

a. b. c. d. e.

Maxillary Parietal Temporal Frontal Ethmoidal

ANATOMY . Enchondral ossification is/are seen in: a. Long bones b. Flat bones of skull c. Clavicle d. Mandible e. Nasal bones . Median atlantoaxial joint is:

a. c.

Condylar Fibrous

b. d.

Cartilaginous Synovial Joint

. Intracapsular articular disc is present in which joint: a. Sternoclavicular joint b. Elbow c. Hip joint d. Knee joint . Which of the following is a traction epiphysis? a. b. c. d.

Tibial condyles Trochanter of femur Coracoid process of scapula Head of femur

17. Length of the mature spermatozoon is: a.

50-60 microns

b.

10-40 microns

c.

100-200 microns

d.

300-500 microns

18. Independent assortment of chromosomes

occurs

at which level: a. b. c. d.

Primordial germ cell to spermatogonia Spermatogonia to primary spermatocyte Primary spermatocyte to secondary spermatocyte Secondary spermatocyte to spermatids

aA aw wy

19. Germ cells derived from:

ga =

a.

Epiblast

b.

Hypoblast

c.

Endodermal sinus

d.

Neural crest cells

mw

= a az

20. All are seen in mitosis except?

a. b. c. d.

fa?)

=

Cytokinesis Separation of chromatids DNA replication Kinetochore formation

—r

>

Ww vu Fa)

Mm

ms @ Ww -—>

. Pisiform is which type of bone: a. c.

Pneumatic bone Accessory bone

b. d.

Sesamoid bone Long bone

10. Muscles with parallel fibers are all, except:

a. c.

Sartorius Sternohyoid

b. d.

Rectus abdominis Tibialis anterior

11. Thoracic duct opens into:

a. b. c. d.

Subclavian vein Right internal jugular vein Right brachiocephalic vein Left brachiocephalic vein

12. Which of the following is not epiphysis:

a. c.

Lesser trochanter Deltoidtuberosity

b. dd.

Greater tubercle Coracoid process

13. Long bones have:

a. b. c. d.

One One One Only

epiphysis and two diaphysis diaphysis and two epiphysis diaphysis and one epiphysis one diaphysis

14. Cell lining of common bile duct is-

a. b. c. d.

Stratified Columnar Stratified Squamous Simple Cuboidal Simple Columnar

15. ‘Pecten’ of Anal Canal is lined by:-

a. b. c. d. 16

Stratified Columnar Epithelium Keratinized stratified - squamous epithelium Simple columnar epithelium Non Keratinized stratified squamous epithelium

Osteoclasts are derived from?

a. c.

Osteoprogenitor cells b. Osteoblasts d.

Monocytes Osteocytes

21. Cloacal

membrane gestation age:

a. c.

ruptures

at

what

week

of

mt

az m rm

6 weeks 10 week

b. d.

al

8 weeks 12 weeks

az Ww

cau >

22. Closure of umbilical vessels occurs by:

a. c.

[@)

3-5 day 2-3 months

b. d.

> > Ww

7-10 th day 2-3 weeks

ot

[@)

a

Ww vu WN

fa?)

—, fa?)

32. Superficial branch of ulnar nerve supplies:

a. c.

Abductor pollicis b. Palmaris brevis Abductor digiti minimid. Oppenens pollicis

33. Posterior wall of axilla is formed by all, except:

Fa) —h

(e) es

az m m

al

a. c.

Teres major Supraspinatus

p> > Ww ct

‘e)

3

Subscapularis

34. Muscles used in climbing tree:

az Ww

b.

Quadriceps tendon and medial side of patella Lateral side of patella On the tibia On the femur

a. b. c.

Inversion of ankle joint Dorsiflexion of foot Plantarflexion of foot d.

Eversion of ankle joint

44. All are branches of lumbar plexus, except:

a.

Iliohypogastric nerve b.

Ilioinguinal nerve

c.

Obturator nerve

Subcostal nerve

d.

45. Middle lobe of right lung contains segments

a. c.

Superior and inferior b. Anterior and posterior Medialandlateral dd. Apical and basal

46. Thoracic duct crosses at: a.

Tl

b.

C7

c.

T5

d.

T8

47. Which of the following not comes under chest wall

proper? a. b.

External intercostals muscle Internal intercostals muscle

c. d.

Pectoralis major Sternocostalis

48. Aortic orifice lies at the level of:

a. b. c. d.

2™intercostal cartilage 3™ costal cartilage 3™intercoastal space 2" Intercoastal space

49. Sternocostal surface of heart not formed by: a.

RV

b.

LV

c.

RA

d.

LA

50. Cardiac dominance is based on:

a. b. c. d.

Cicumflex artery Posterior interventricular artery Anterior interventricular artery None of the above

51. Aortic

opening, thoracic duct and opening of azygos vein in diaphragm is situated at the level of:

a.

T8

b.

T10

c.

T12

d.

T6

52. Posterior to transverse pericardial sinus is: a.

Aorta

c.

SVC

b. d.

Pulmonary trunk Left atrium

53. Superior vena cava opens into right atrium at the

level of: a.

Tl

b.

T3

c.

T4

d.

T5

34. Transpyloric plane separates:

a. aoe

cho St

a.

43. Action of tibialis anterior:

Hypogastrium from lumbar region Hypochondrium from lumbar region Iliac fossa from lumbar region Umbilical region from lumbar region

ANATOMY 55. Pyramidalis supplied by: a. c.

69. Whitnall’s ligament refers to:

Subcostal nerve b. Iliohypogastric nerve d.

Ilioinguinal nerve Genitofemoral nerve

a. b.

56. Liver segment which is physiologically independent: Segment!

b.

Segment II

c.

Segment IV

d.

Segment III

57. Spleen projects into peritoneal cavity? a.

Paracolic gutter

the b.

c. Left sub hepatic spaced.

following

b. d.

a.

space

Greater sac Sigmoid colon Rectum

Left subphrenic space

b. Hepatic flexure

c. Adrenal

d.

61. Artery to vas arise from: a. Aorta b.

Duodenum (4th part) Inferior epigastric

d. Internal pudendal

62. Spleen develops from: Dorsal mesogastrium

b. Ventral mesogastrium c. d.

63. Duct of bellini is present in: a. c.

Gallbladder Anal canal

b. d.

Kidney Lymph nodes

65. Nerve supply of scalp: a. Auriculo temporal nerve

d.

Infratrochlear nerve

Inferior to superior turbinate

4

71. Tensor tympani muscle is attached to:

2

a. Tympanic membrane b. Malleus 4 - Incus c. Stapes 72. Part of larynx above the vocal cord is known as:

c.

Prosencephalon

Rhombencephalon

None

a.

LI

b.

12

c.

L3

d.

L4

c.

Medulla ; , Midbrain

c. Occipital nerve e. Frontal nerve

a. c.

Medullaoblongata Mid brain

b.

Cerebellum

d.

Pons

a. b.

Nucleus ambiguous Fastigii nucleus

c.

Dentate nucleus

79. Broca’s speech area is located in:

d.

Cingulate sulcus

Choroid

d.

Retina

68. Nasal septum is formed by: a. Perpendicular plate of ethmoid

Pons Spinal cord

d. Globose nucleus

67. Lamina cribrosa is a modification of: b.

b. d.

78. Spherical shaped nucleus in cerebellum:

Superior frontal gyrus Inferior frontal gyrus Insula

c. Opticnerve sheath

d.

Mesencephalon

75. Spinal cord in infants ends at the level of:

a. b. c.

Sclera

b.

a. Medial striate branch of anterior cerebral artery b. Striate branch of middle cerebral artery c. Fronto- polar artery d. Calloso- marginal artery

a.

= Z2 2 wv 2

® o =

74. Charcot’s artery is:

66. All are paired venous sinuses, except: a. Superior petrosal b. Inferior petrosal c. Superior sagittal d. Transverse

a.

b. Infraglottis d. Saccule of larynx

77. Chemoreceptor Trigger Zone is related to:

64. The layer of scalp which is vascular: a. Skin b. Pericranium d. Subcutaneous tissue c. Aponeurosis

b. Zygomatic nerve

d.

76. Olive is seen in which part of brain:

Mesoderm of septum transversum All of above

a

2,


rm >

° 3

w

ga

5.

a

7.

a

8.

b

9.

b

10.

d

d

15.

d

17.

a

18.

Cc

19.

a

20.

Cc

24

d

25.

C

27.

c

28

c

29.

b

30.

c

c

34

d

35.

a

37.

b

38.

C

39.

a

40.

d

43.

b

44

d

45.

Cc

47.

c

48

c

49.

d

50.

b

c

53.

d

54

b

55.

a

57.

d

58.

C

59.

b

60.

d

62

a

63.

b

64

d

65.

a

a

A a

67.

a

68.

a,b,c,d,

69.

c

70.

Cc

72.

a

73.

c

74

b

75. = C

jo

Answer Keys Cc

2.

a

3.

d

4.

d

12

c

13

b

14

b

22

a

23.

b

a

32

b

33.

b

42.

a

c

52

c

b

ade

77.

a

78

d

79.

b

80.

Cc

36

w

= a az fa)




Ww ©) WN

fa?)

Clinical Bridge to Anatomy

—, fa?) Fa) —h

(e) es

az m m

a

az Ww

a >

p> > Ww ct

‘e)

3

e) ae) v,) ie?)

=.

MD a)

-—>

Oo mn"

=

m m

—~ =

es)

a

>

> S @=>

3




©

—,

The tensor fasciae latae can flex and medially rotate the thigh, so this is the muscle most likely damaged. The hamstring muscles (semitendinosus, semimembranosus, and biceps femoris) can extend the thigh and flex the leg. The sartorius can flex the thigh and leg. The rectus femoris can flex the thigh and extend the leg. The vastus intermedius can extend the leg.

om Ww bh

© es

az m m

al az Ww

a >

7. Ans. (d)

Lesser saphenous vein

The lesser (small) saphenous vein ascends on the back of the leg in company with the sural nerve and terminates in the popliteal vein. The peroneal vein empties into the posterior tibial vein. The anterior and posterior tibial veins are deep veins and join to form the popliteal vein. The great saphenous vein drains into the femoral vein.

p> > w ot

©

3

Excitation contraction coupling

Mechanical coupling between DHPR and RyR

‘@)

rs)

AP Duration

3-5 ms

200-400 ms

10-15 ms may be 1 sec

Fa)

Twitch Duration

20-200 ms

200-400 ms

200 ms-sustained

Phospholamban

Present

Present

Present

Calsequestrin

Present

Present

Present

my

=z m rm

a

az

©)

‘@) [@)

3 sok fap) ct

ra)

= Electromechanical coupling between DHPR and RyR

Electromechanical and pharmacological coupling

Table 7: Types of EPSP and IPSP

Ww fap)

mn

m

a

a m

> 2.

a

of line-skilled voluntary movements

5 e Feeding/hunger center

Lateral

«

=



:

,

5

clonus se 4 ypertonicity of the “rude Clasp-knife reaction

e Rage center

Loss of reflexes

©

3 . Hypertonicity

o O a

Reaction of degeneration

= = o

Muscular Fasciculation

=]

ga mm 1

Table 11:

Hormone

4

Gastrointestinal Hormones

| Source

Stimulus

Actions

© fo’ LD wn

Gastrin

G cells of gastric antrum

e Small peptides, amino acids, Ca?* in lumen of stomach e Vagus (via GRP) e Stomach distension e Inhibited by: H* in lumen of antrum

e e e e

Tt HCl secretion by parietal cells Trophic effects on GI mucosa 1 pepsinogen secretion by chief cells T histamine secretion by ECL cells

CCK

| cells of duodenum and jejunum

e Fatty acids, monoglycerides e Small peptides and amino acids

e Stimulates gallbladder contraction and relaxes sphincter of Oddi e tT pancreatic enzyme secretion e Augments secretin-induced stimulation of pancreatic HCO, e Inhibits gastric emptying e Trophic effect on exocrine pancreas/gallbladder

Secretin

S cells of duodenum

el DH in duodenal lumen e Fatty acids in duodenal lumen

e tT pancreatic HCO, secretion (neutralizes H’) e Trophic effect on exocrine pancreas e 1 bile production et gastric H* secretion

GIP

K cells of duodenum and jejunum

Glucose, fatty acids, amino acids

e 7 insulin release el gastric H* secretion

Motilin

Enterochromaffin cellsin duodenum and jejunum

Absence of food for>2 hours

Initiates MMC motility pattern in stomach and small intestine

a J = a
S pe)

x c 3 pe) = = S

Early diastole/ joint diastole

5 o S, D o 3

Ventricular contraction

Z

entricular

> O i a fe) 3

)

tr ove ume ric

Closed

Closed

Closed ose

ventricles are depolarized.

Due to active contraction of

It is isoelectric.

Whole heart is relaxed at that period. Ventricles are expanding and passive filling occurs, approx. ~80% of volume filled at this phase.

Open

Closed

Closed ose

relaxation

Ventricular ejection

Table

13:

ECG

Open

P wave

60. Carbon monoxide poisoning is a type of:

a. c.

Anemic hypoxia Hypoxic hypoxia

b. d.

Histotoxic hypoxia Stagnant hypoxia

6l. What is the effect of moderate exercise on cerebral

Ww ©)

blood flow?

WN

a. b.

Does not change Increases

c. d.

Decreases Initially decreases then increases

fap)

—. fa?) Ww —h

(e) my

az m rm

al az Ww

ot > (=)

pS) WN

vu

(=) ‘e)

3 To

Peptide Steroid Insulin Thyroxin

fa?)

a.

Membrane

Ww fa)

c.

Nuclear

Ww ©) Fa)

fa?)

—, fa) Ww bh

‘@) my

az m m

a

az w

au > (=)

rS)

Fa)

me)

(=) Le)

3 sok ra?) ot

rap) Ww fa?)

Homogentisic oxidase

MMCoA

=

a

Alkaptonuria

Methylmalonic acidemia and

< a) cor — a) y) > = & a) fe) 5 Oo fo) ke)ct c

~ ct

Hyperactivity; Tremors; Mousy odour urine

dehydrogenase

3.

Metabolic acidosis; Developmental retardation

amino acid in blood

Propionaturia; Methylmalonuria

Table 10: Glycogen storage disorders Glycogen storage disorder

Enzyme defect

Manifestation

Von Gierke disease

Glucose 6 phosphatase

Hypoglycemia, Hyperlipemia, Hypercholesterolemia, Lactic acidemia, Hyperuricemia

Pompe disease

Acid maltase

Muscle hypotonia, Myopathy, Muscular dystrophy

Forbes or Cori disease

De-branching enzyme

Hepatomegaly, Myopathy Fasting hypoglycemia

Anderson’s disease

Branching enzyme

Hepatosplenomegaly, Myoglobin in urine

McArdle's disease

Muscle phosphorylase

Exercise induced cramps, Low lactate levels in serum, Myoglobin in urine

Hers’ disease

Liver phosphorylase

Hepatomegaly, Mild hypoglycemia

Tauri’s disease

Phosphofructo kinase —1

Exercise induced cramps, Hemolytic anemia, Low lactate levels in serum, Myoglobin in urine



tissues convert lactate EXCEPT:

a

c.

Galactose

b. d.

20. All

Ethanol Cholesterol

a. Glycosaminoglycans (GAG) Protein

d.

many ATPs are used in energy investment phase of glycolysis?

12. Heparin is a: b.

Glucose

19. How

64 8

11. The monosaccharide with absorption in intestine is:

b.

fe) 5 Oo to) ke)ot c

is-

10. Parent alcohol in carbohydrates is:

a. c.

M

18. The major role of 2,3 bisphosphoglycerate in RBCs

. Number of isomers possible for glucose are: a. c.

e.

a. b. c. d. e.

. Dextrose is:

a. c.

result in

17. Hyaluronic acid is composed of:

. Insulin is not broken due to beta anomers at:

a. c.

Fructose

i

NADP FADP

. Cellulose is not broken due to beta anomers at:

a. c.

c.

a ce

High protein diet High carbohydrate diet High fat diet It does not depend on the macronutrients NAD FAD

Glucose

a.

. Which is required in Anabolic reactions: a. c.

can

16. All are true about glycosaminoglycans except:

. Thermogenic food is which of the following a. b. c. d.

of the following

Sugar alcohol Sugar amines

a.

< a) cor — a) Q) > = & =

15. Which of the following is a keto sugar?

. Hormone sensitive lipase is not activated by-

a. c.

3.

Fructose polymer Nonstarch polysaccharide Starch polysaccharide Glycosaminoglycan

14. Excess of which cataract?

. Insulin promotes lipogenesis by all except

%)

13. Cellulose is a:

This is a Link between TCA and ETC This is oxidative deamination of pyruvate This is oxidative decarboxylation of acetyl CoA This reaction requires Lipoic acid and four Bcomplex Vitamins

=

~ ot


uty

H,O

to Cis

Sd

a.

Citrate converted

a

| 22. What is liberated when Aconitate?

henylene ll of these

BIOCHEMISTRY 36. N1

of purine ring is derived from-

a.

Asparagine

a.

c.

Glycine

aot

enzyme?

Fumarylacetoacetate hydrolase p-hydroxy phenyl pyruvate dehydrogenase Tyrosine transaminase Tyrosine ligase

45. Which intermediate of citric acid cycle is used in detoxification of ammonia in brain?

a. Citrate c. Alpha-ketoglutarate

b. d.

Succinate Oxalo-acetate

46. A peptide bond:

aot

a.

Has partial double bond character Ionized at physiological pH Occurs in cis configuration All

47. Which of the following is most essential fatty acid ?

a. b. c. d.

Alpha Linolenic acid Gamma Linolenic acid Lauric acid Linoleic acid

a.

Corticosteroids

b.

Linolenic acid

c.

Oleic acid

d.

Arachidonic acid

49. HDL is synthesized and secreted from :

Liver Adipose tissue

b. d.

Peripheral tissue Muscle

50. Which lipoprotein is largest in size?

a. HDL c. VLDL

b. d.

LDL Chylomicrons

ol. Which

of the following lipoproteins would contribute to a measurement of plasma cholesterol in a normal person following a 12-hour fast:

a. c.

Chylomicrons HDL

b. d.

VLDL LDL

52. Major Product of fatty acid synthesis is:

a. c.

Acetyl CoA Citrate

b. d.

ATP Palmitate

53. Which of the following enzymes is the most

97. By which method foreign DNA is introduced into a @¢

cell by a virus or viral vector? a. c.

Transduction b. Transcription Lysogenic conversion d. Transformation

58. Denaturation of DNA breaks:

a. b. c. d.

Hydrogen bonds 3’ 5’ Phosphodiester bonds Beta-N-glycosidic bonds O-Glycosidic bonds

39. Function of endonucleases:-

a. b. c.

Cut DNA at specific DNA sequences Enhancers To find out antibiotic resistances

d.

To point out the coding regions

a. b. c. d.

Cyclic GMP Ligand membranes Binding of regulatory proteins to DNA Membrane attack complexes

61. The base sequence of the strand of DNA used as the

template for transcription has the base sequence GATCTAC. What is the base sequence of RNA product?

a. c.

CTAGATG GAUCUAC

b. d.

a. RER (Rough endoplasmic reticulum) b. SER (smooth endoplasmic reticulum) c. Bothaandb d. Nucleus 63. Western blot detects?

a. c.

DNA m-RNA

b. d.

assess function of a gene?

important enzyme regulating lipogenesis:

c.

Gene knockout animals

a. c.

d.

Transgenic animals

Acetyl-CoA carboxylase Acetyl transacetylase

54. a-oxidation of fatty acid takes place in a.

Mitochondria

c.

ER

b. d.

Golgi apparatus Peroxisomes

55. The enzyme deficient in Lysch Nyhan syndrome is:

aot

a.

RNA Protein

64. Which of the following methods is most suited to Southern blot

b. d.

GTAGATC GUAGAUC

62. Protein synthesis occurs in:

a.

Hydratase Enoyl reductase

Aspartate Glutamine

60. Leucine zipper motif is a mediator for:

48. Prostaglandins are derived from

a. c.

b. d.

b.

Northern blot

65. Most common enzymatic tool in genetic engineering:

a.

DNA ligase

b.

c. d.

Topoisomerase Restriction endonuclease

Halicase

66. Epigenetics is a:

Adenosine Deaminase

a.

Chemical modification of DNA

PRPP synthetase

b.

Normal variation of nucleotides

HGPRTase

c. d.

Change in nucleotide sequence Irreversible modification of DNA

Xanthine oxidase

Asjsiwaysolg JO MalAay jeN}daduo;D ‘emyed !yINJd AjIws

44. Tyrosinosis is caused due to deficiency of which



69. Vitamin deficiency causing dementia:

67. Vitamin which is excreted in urine is

a. c.

Vitamin A Vitamin D

b.

Vitamin C

a.

Vitamin B12

b.

Pyridoxine

d.

Vitamin K

c.

Thiamine

d.

Biotin

68. Excess of avidin causes deficiency of: W

3.

a.

Folate

b.

Choline

c.

Vitamin B12

d.

Biotin

< a) cot =. a”) ) > = 2 me

70. Which amino acid does not take part in one carbon transfer reactions:

a. Glycine c. Threonine

b. d.

Serine Tyrosine

|

Answer Keys

‘@)

fe) S oO fo) xe)et c

a, d,e b

2.

PS) 2) i © = fe)“bh oe fe)2) > oO)

2.

Cc

3.

a

4.

a

5.

12.

a

13.

b

14

a

15

b cede

6.

a

7.

b

8.

a

9.

10.

a

16.

Cc

17.

a

18

c

19.

20.

a

28

a _—

29.

30.

b

38.

c

39.

40.

c

Cc 24.

d

25.

bd

26.

b,dve

27.

b

34

a

35.

b

36.

a

37.

43.

b

44.

a

45.

c

46.

a

47.

d

48

d_

49.

50.

d

d

53.

b

54.

d

55.

c

56.

b

57.

a

58

a

59.

60.

c

a

63.

d

64

c

65.

d

66.

a

67.

b

68

d_

69.

70.

d

d

22.

a

PGP

a

32.

ac, d,e

33.

b

42.

a

3.

Cc

52.


= & =

2)

fe) 5 oO to) Ooot c

2.

2) © S. © = fe) we fe)2) > ©

—h

=

oa)

ot


= Le 2) O° 5 Oo . Oor c =

Thyroid Hypothalamus

Adrenal Parathyroid

A16-year-old female hasaconstant sense ofhunger, obesity, almond-shaped eyes, strabismus, mental retardation, and retarded puberty development. Which of the following best describes the reason for this chromosomal abnormality? a. c.

Nondisjunction Translocation

b.

Point mutation

d.

Deletion

Lo)

ot


D

b. d.

20.

and would be considered acidosis, instead of alkalosis.

An example of a metabolic alkalosis is hypokalemia, a reduction in normal potassium values. Owing to low serum potassium levels, potassium leaves the cells and is replaced by protons from the circulation. The loss of protons from the blood leads to the alkalosis. 4. Ans. (b) Ketone bodies Ketone bodies are weak acids. In diabetic ketoacidosis, the liver produces ketone bodies, which will reduce the brain’s

2. Ans. (d) Fatty acids are being released adipose triacylglycerol stores

from

During fasting, fatty acids are released from adipose tissue and oxidized by other cells. Liver glycogen is not depleted until about 30 hours of fasting. After an overnight fast, both glycogenolysis and gluconeogenesis by the liver help maintain blood glucose levels. Muscle glycogen stores are not used to maintain blood glucose levels. The liver produces ketone bodies but does not oxidize them, but under the conditions described in this question, ketone body formation would be minimal.

dependency on glucose as its sole energy source. This is due to the lack of insulin, and the liver switching to starvation mode owing to the constant signaling by glucagon. Hemoglobin in the red blood cells and bicarbonate, both in the red blood cells and the plasma, are two of the body's major buffers, and their overproduction would not lead to an acidosis. HCl overproduction within the stomach might lead to duodenal ulcers or gastroesophageal reflux, but not to an overall metabolic acidosis, as the protons do not find their way into the circulation. A loss of chloride, if severe enough, could produce a metabolic alkalosis, but not an acidosis

3. Ans. (b) 7.55 The patient is having a panic attack tunnels) and is hyperventilating, respiratory alkalosis. The loss of carbonic anhydrase reaction in the production,

which

reduces

(due to driving in causing an acute CO2 pushes the direction of CO2

the proton

concentration

(and thereby raising the pH). The patient would lose consciousness with a more severe attack. A respiratory alkalosis is usually mild as compared to a metabolic alkalosis. For this reason, the pH increase is smaller (7.55 is more likely than a pH of 8.10, which could occur

5. Ans. (c) Western blot The patient has Kaposi's sarcoma and AIDS. The causative agent is HIV, an RNA virus. The Western blot technique is used to identify whether a specific blood sample contains antibodies that will bind to HIVspecific proteins. The HIV proteins are run through a gel, transferred to filter paper, and probed using the sera from the patient. If the patient has antibodies to the HIV proteins, then a positive result will be obtained. A Southern blot is used to identify the DNA, and in

BIOCHEMISTRY this case, it is easier to check for the presence of anti-HIV antibodies in the patient’s sera. A Northern blot would check for viral RNA, but itis more efficient, and reliable, due

to the low levels of viral RNA, to check for anti-HIV proteins instead. A Southwestern blot is used to detect DNA binding to proteins, and would not be applicable for AIDS testing.

of glycerol. The treatment for a GLUT1 deficiency is a ketogenic diet-one high in fats such that ketone bodies are continuously generated to provide fuel for the brain. 10. Ans. (d)

6. Ans. (b) Insertion The patient has Fragile X syndrome, caused by the expansion of a triplet nucleotide repeat (CGG) within the FMR1 gene on the X chromosome. The expansion interferes with the normal functioning of this gene product in the brain, leading to the symptoms observed. This is an extreme example of an insertion mutation. A point mutation occurs when only one base is substituted for another, and the change in base results in an amino

acid change in the protein. Deletion is removal of one or more nucleotides from the gene. Mismatch (answer D) repair is a DNA repair process that is utilized when a mismatch is found in the DNA. A silent mutation is one in which a base change in the DNA leads to no change in the corresponding amino acid in the protein (due to degeneracy in the genetic code).

7. Ans. (a) RNA polymerase Two drugs are utilized for latent tuberculosis: isoniazid and rifampin. Isoniazid works by blocking the synthesis of mycolic acid, a necessary component of the cell wall of the bacteria that leads to tuberculosis. Rifampin works by inhibiting bacterial RNA polymerase, and blocking the synthesis of new proteins. Neither drug affects DNA polymerase or peptidyl transferase (chloramphenicol is the antibiotic that inhibits bacterial peptidyl transferase activity). 8. Ans. (a) Proteoglycan The patient has osteoarthritis, and wants to use glucosamine/chondroitin sulfate to provide cushioning in the joint. These molecules are proteoglycans, which consist of long, linear chains of glycosaminoglycans attached to a core protein. Each chain is composed

Lactate, glycerol, and oxaloacetate

Ethanol metabolism (which produces high NADH levels) does not prevent glycogen degradation. In fact, glycogen stores would be rapidly depleted under these conditions because of decreased gluconeogenesis. Lactate is converted to pyruvate during gluconeogenesis. The pyruvate-lactate equilibrium greatly favors lactate when NADH is high. Thus, alanine and lactate are prevented from producing glucose. Lactate levels are elevated, and a lactic acidosis can result. Glycerol normally enters gluconeogenesis by forming glycerol- 3-phosphate, which is oxidized to DHAP. High NADH levels prevent this oxidation. Aspartic acid is converted to oxaloacetate (via transamination), as do other amino acid degradation products that enter the TCA cycle (a-ketoglutarate, succinyl-CoA, fumarate). However, the high NADH levels favor malate formation from oxaloacetate, reducing the amount of oxaloacetate available for gluconeogenesis (through the phosphoenolpyruvate carboxykinase reaction). Thus, the three major gluconeogenic precursors (alanine, glycerol, and lactate) do not form glucose because of the high NADH levels, and as glycogen stores are depleted, hypoglycemia results.

%)

3.

< in

©

cor =. a) ) > = & 2) fe) S Oo 0) Ooct c =

a

p) © Ss. © = fe} ow e)‘2 > oD)

-—>

3.

wn

ct m1


= 2 a) fe) 5 oO 0) Ooor c

13. Ans. (a)

The synthesis of bile salts in the liver

into the blood in an adenosine deaminase deficiency.

Bacteria in the intestine deconjugate and dehydroxylate bile salts, converting them to secondary bile salts.

=

2

PS) oO) Ss. © = fe)“bh fe) 2) > )

3.

Therefore, the bile salts become less water-soluble and

The patient has Cushing syndrome, and is overproducing

less effective as detergents, less readily absorbed, and more likely to be excreted in the feces than recycled by the liver. Fewer micelles would be produced, so less dietary lipid (including the fat- soluble vitamins) would be absorbed. Because fewer bile salts would return to the liver, more bile salts would be synthesized. Bile salts inhibit the 7a-hydroxylase that is involved in their synthesis. In addition, the person’s food intake might decrease, which would augment some of the effects noted above. Charges on bile salts will not affect the uptake of water-soluble vitamins. Since the bacteria in the gut deconjugate and dehydroxylate the bile salts, the amount of conjugated bile salts in the intestine will decrease.

the

Lo)

ot


Hest amd second bembekals Occasionally: Used hambrical

»

Dxtreembryee

*

Deaa

»

m Palm

*

= Penctional column ne uctel of facial nerve 7

©

Clinical linical Pattern

by:

M

Iti U

| Ipte

Choi

(

ue

ice

s

&® ti on

Important MCQs

:

= y

. Whack fteet 2 —_—e

s

vhaned

1 Cotatiome te ae —

ae

was

Types

of

cartilaginous

joint

with

one ime,

mae

||

;

are

es

Pre

06

Guens

6

wes

6

Ge

Cienega

ese

i i

TAe



Extra edge section with fully colored=

Scere

— spotters, important tables, MCQs and — -Clinical bridge along with clinical pattern MCQs= —— for quick revision during practical exams —

2 > 4 2

Aietes pcan | seed Pompton Meter ramon L streal romptom

A pees wee ce OFT peeptyieme AD ee meager bel twtee te the dow p te rtee ere pe

bo 4

4:

»

SS

of bewmet wert © of ewe! ower wuteh et bens of Wen coer wate ed femer > Obes over mabaksed treper

al

of Ge

®

NON

Rabin bmguanel begermered Lowes hugh

Denienst wet

b

Nate

a

dane

——

naw

aggre wube

Pn

tas

ee

en

©

C eBebee

Sper

MM.

Altemnpt

_

bb pee

be wken

o

a! parte.

Bach part hax to be

attempted on separate

AIP

©

nu

caching

UAvooernwese

—— -

phosphate wreTetase

oo

*

anewer

sheets

Draw

neat

and ecstetie

leteled Gagram:

wherever

eae

*

2

predece

Pate “2

of “glucose

6 oF

1.

& 1

An

6 phonphate

phosphate wate

isb

tise

further urtheT

t tseu e

{10}

refer

2015

paper-l,

Q.

1(b),

Py.

48

Stractures pierced by

parctid duct

mae

oe canal

ery

f

4

drer

'

eraterese

— Ghycager

2

ave

.

=